Download as pdf or txt
Download as pdf or txt
You are on page 1of 213

ANCIENT INDIAN HISTORY

PRE HISTORY

Q.1 Bhimbetka caves shows the continuity of human evolution from the Lower Palaeolithic Period to
the Mesolithic period to the chalcolithic period. Why?
1. It contains evidence of the use of tools and implements from all these periods.
2. There are a large number of Shanka Lipi inscriptions in the Bhimbetka cluster of rock shelters.
Which of the above is/are correct?
(a) 1 only
(b) 2 only
(c) Both 1 and 2
(d) None
Solution: (a)
Justification: Statement 1: In the caves, the continuity of human evolution from the Lower Palaeolithic Period is
noticed by the smaller size of stone tools in the following Middle Palaeolithic Period besides new tools like scrapers.
During the Upper Palaeolithic Period newer tool types like: blades, borers and burins had also emerged.
However, it is in the Mesolithic Period that there is a clear change in the materials and tool typology.
Earlier, the tools were largely made of quartzite and sandstone, whereas the tools being made in the Mesolithic
Period were most often of chalcedony.
The Mesolithic culture at Bhimbetka continued much longer as understood by the presence of Chalcolithic
potteries in otherwise Mesolithic contexts.
By the Early Historic times it appears that interaction with the surrounding cultures became more pronounced.
This is evidenced by the presence of rock-cut beds in a rock shelter on the top portion of an inselberg like outcrop
not far from the later built temple at this site.
Statement 2: Shankhalipi or "shell-script" is a term used by scholars to describe ornate spiral Brahmi characters
that resemble conch shells (or shankhas). They are found in inscriptions across various parts of India except the far
south and date to between the 4th and 8th centuries CE.

Q.2 The metal central to this age finds frequent mention in the Vedas and the age itself follows after
the Chalcolithic age. It can be
(a) Palaeolithic Age
(b) Iron Age
(c) Copper-stone
(d) Later Stone Age
Solution: (b)
Learning: The Chalcolithic age is followed by Iron Age. Iron is frequently referred to in the Vedas.
The Iron Age of the southern peninsula is often related to Megalithic Burials. Megalith means Large Stone.
The burial pits were covered with these stones. Such graves are extensively found in South India.
The early period of the age is characterized by the widespread use of iron or steel. The adoption of these materials
coincided with other changes in society, including differing agricultural practices, religious beliefs and artistic styles.

Q.3 Consider the following major archaeological sites ranging from the Mesolithic age to the iron age in
the Indian subcontinent. Match them with their respective regions in present day India:
1. Koldihwa: Madhya Pradesh
2. Mehrgarh: Haryana
3. Paiyampalli: Tamil Nadu
Select the correct answer using the codes below.
(a) 1 and 2 only
(b) 3 only
(c) 2 and 3 only
(d) 1 only
Solution: (b)
Justification: Statement 1: Located in present day UP, this site represents three occupational levels: the Neolithic,
Chalcolithic, and Iron Age phases.
Statement 2: Mehrgarh is located near the Bolan Pass, to the west of the Indus River valley and between the now
Pakistani cities of Quetta, Kalat and Sibi.
Mehrgarh is supposedly the most sophisticated, ingenuous and best planned ancient farm villages of ancient India.
Statement 3: It is known for the excavation remains of neolithic and megalithic periods.

Q.4 Palaeolithic or Old Stone Age sites are widely found in various parts of the Indian subcontinent.These
sites are generally located near
(a) Water sources
(b) Deserts
(c) Major urban centres
(d) Mines
Solution: (a)
Learning: Several rock shelters and caves used by the Paleolithic people are scattered across the subcontinent.
Some of the famous sites of Old Stone Age in India are:
The Soan valley and Potwar Plateau on the northwest India.
The Siwalik hills on the north India.
Bhimpetka in Madhya Pradesh.
Adamgarh hill in Narmada valley.
Kurnool in Andhra Pradesh and
Attirampakkam near Chennai.
If you notice, most of these sites are near water sources, for e.g. Soan valley, Siwalik hills (starting from Indus),
Adamgarh hill in Narmada valley.
A reason can be that water formed a subsistence base for Palaeolithic civilization, which is why major art works are
found near these water sources.

Q.5 The Neolithic period is followed by Chalcolithic period. In the chalcolithic period, the use of which of the
following started?
1. Copper and bronze
2. Technology of smelting metal ore
3. Crafting metal artifacts
Select the correct answer using the codes below.
(a) 1 only
(b) 2 and 3 only
(c) 3 only
(d) 1, 2 and 3
Solution: (d)
Justification: Despite the use of copper and bronze, the use of stone tools was not given up. Some of the micro-
lithic tools continued to be essential items.
People began to travel for a long distance to obtain metal ores.
This led to a network of Chalcolithic cultures and the Chalcolithic cultures were found in many parts of India.

Q.6 Which of these are the chief difference(s) between Mesolithic and Neolithic cultures?
1. Hunting-gathering pattern of life began not before the Neolithic period.
2. Mesolithic cultures did not practice domestication of animals unlike Neolithic people.
3. The use of bow and arrow cannot be found in Mesolithic culture, unlike Neolithic culture.
4. Evidence of pottery is absent from the Neolithic culture but is markedly found in Mesolithic culture.
Select the correct answer using the codes below.
(a) 3 and 4 only
(b) 2 only
(c) 1 and 3 only
(d) None of the above
Solution: (d)
Justification:
Statement 1 and 3: Hunting-gathering pattern of life was prevalent since Old Stone Age to Mesolithic and Neolithic.
However, there seems to have been a shift from big animal hunting to small animal hunting and Fishing. This is how
bows and arrows were used in this period. Both 1 and 3 are thus wrong.
Statement 2: Domestication of animals, horticulture and primitive cultivation started during Mesolithic period
itself. However, during Neolithic period, domestication of sheep, goats and cattle was widely prevalent. Cattle were
used for cultivation and for transport.
Statement 4: During Neolithic age, wheels were used to make pottery. Pottery was used for cooking as well as
storage of food grains. So, 4 is wrong.
Learning: During the Neolithic phase, the cultivation of plants and domestication of animals led to the emergence
of village communities based on sedentary life. There was a great improvement in technology of making tools and
other equipments used by man. Stone tools were now polished. Mud brick houses were built instead of grass huts.
Large urns were used as coffins for the burial of the dead. Wheat, barely, rice, millets were cultivated in different
areas at different points of time. Rice cultivation was extensive in eastern India. The people of Neolithic Age used
clothes made of cotton and wool.
Q Source: Page 13-14: TN 11th Standard History Textbook

Q.7 The beginning of agriculture can be ascribed most suitably to which of these times?
1. 2,500 years ago with the onset of Magadha empire
2. 4,700 years ago with the appearance of the first cities on the Indus
3. Nearly 8,000-10,000 years ago
4. About 25,000 years ago in the first Penistone glacial period
Solution: (c)
Justification & Learning:Option D: People evolved into their current form some 200,000 years ago (see human evolution), yet
they did not begin to engage in agriculture until about 15,000–10,000 years before thepresent.
Wild grains were collected and eaten from at least 20,000 BC. So, D is wrong
Option C: The first agriculture appears to have developed at the closing of the last Pleistocene glacial period, or Ice Age
(about 11,700 years ago).
From around 9,500 BC, the Neolithic founder crops such as emmer wheat, einkorn wheat, hulled barley, peas etc were
cultivated in the Levant. Rice was domesticated in China between 11,500 and 6,200 BC.

Q.8 Consider the following matches of periods with the beginning of major activities in the history
of human civilization:
1. Building of megaliths: 3000 years ago
2. Cotton cultivation: 2000 years ago
3. Domestication of animals: 12000 years ago
Select the correct answer using the codes below.
(a) 1 and 2 only
(b) 1 and 3 only
(c) 2 only
(d) 1, 2 and 3
Solution: (b)
Justification: Statement 1: Beginning of cities started about 4700 years ago. Settlement at Inamgaon
began between 3600 and 2700 years ago.

Statement 2: This started at Mehrgarh about 7000 years ago.


Statement 3: Domestication was a gradual process that took place in many parts of the world.
It began about 12,000 years ago. Some of the earliest plants to be domesticated were wheat and
barley.
The earliest domesticated animals include sheep and goat.
Q.9 Among the earliest chalcolithic cultures in India, the Ahar or Banas culture was discovered in the
(a) Mewar region of Rajasthan
(b) Gwalior region of Madhya Pradesh
(c) Bundelkhand region of Uttar Pradesh
(d) Kangra region of Himachal Pradesh
Solution: (a)
Learning: Nearly one hundred sites of the culture have been located along its principal axis, i.e., the
valleys of river Banas and its tributaries and subtributaries in Banswara, Udaipur etc.
The technology at Ahar was based mainly on copper and very few microblades and microliths have
been discovered.
Ahars were a separate culture from GJs. Located in northeast Rajasthan, the Ganeshwar-Jodhpura
complex which was an early centre of agriculture and copper metallurgy in the subcontinent.

Q.10 With reference to Chalcolithic cultures in Indian subcontinent, consider the following statements.
1. Ochre-coloured pottery sites have been usually found in the Gangetic doab of India.
2. Anthropomorphic figures of worship have been found in the copper hoard culture.
Which of the above is/are correct?
(a) 1 only
(b) 2 only
(c) Both 1 and 2
(d) None
Solution: (c)
Concept: The Harappan culture is generally supposed to have been followed by non-urban Chalcolithic culture
characterized by the use of copper and stone. The differences between these cultures were not fundamental but
were primarily confined to pottery.
Justification: Copper Hoards describe find-complexes which occur in the northern part of India.
These occur mostly in hoards large and small and are believed to date to the later 2nd millennium BCE, although
very few derive from controlled and dateable excavation contexts.
A fragment of an anthropomorph came to light in controlled excavations at Lothal and a second one at Saipai
Lichchwai, Etawah district. The doab hoards are associated with the so-called Ochre Coloured Pottery (OCP)
which appears to be closely associated with the Late Harappan (or Posturban) phase.

PROTO HISTORY

Q.11 The Indus Valley Civilisation (IVC) is named the Harappan civilisation because
1. Harappa was the largest settlement of the IVC.
2. Harappa was the first site to be excavated in the IVC.
Which of the above is/are correct?
(a) 1 only
(b) 2 only
(c) Both 1 and 2
(d) None
Solution: (b)

Justification: Statement 1: Mohenjo-daro, Harappa and Ganweriwala (all in Pakistan) and Rakhigarhi and Dholavira
(both in India) are ranked as the first to the fifth biggest Harappan sites.
Statement 2: The Indus Valley Civilisation is also named the Harappan civilisation after Harappa, the first of its sites
to be excavated in the 1920s, in what was then the Punjab province of British India.
The discovery of Harappa, and soon afterwards Mohenjo-daro, was the culmination of work beginning in 1861 with
the founding of the Archaeological Survey of India in the British Raj.
Along with Ancient Egypt and Mesopotamia, it was one of three early cradles of civilisations of the Old World, and
of the three, the most widespread.
The Indus cities are noted for their urban planning, baked brick houses, elaborate drainage systems, water supply
systems, and clusters of large non-residential buildings

Q.12 Which of the following facts is NOT associated with Indus Valley civilization?
(a) Pottery and bead-making
(b) Worship of natural forces as deities
(c) Water storage structures
(d) Domestication of animals.
Solution: (b)
Explanation: Option A was a common feature.
The evidence of domesticating animals has been shown in a previous explanation. Bones of dog, cattle, sheep were
found in Harappan sites in Kashmir.
Great bath and other storage structures were used by the community.
We do not have any Harappan scripture which can testify the claim of animism being practiced in Indus valley
civilization. However, they did have deities like Pashupati which they even inscribed on seals.

Q.13 Which of the following is/are NOT mature Harappan sites?


1. Balakot
2. Rangpur
3. Ganweriwala
4. Digona
Select the correct answer using the codes below.
(a) 1 and 2 only
(b) 4 only
(c) 3 and 4 only
(d) 1, 2 and 3 only
Solution: (b)
Q.14 Consider the following statements.
1. Harappans practiced metal craft and bead making.
2. No evidence of the practice of pottery can be found in the Harappan civilization.
Which of the above is/are correct?
(a) 1 only
(b) 2 only
(c) Both 1 and 2
(d) None
Solution: (a)
Justification: Seal-making and teracotta manufacture were important crafts.
The Harappans were also experts in beads making.
The potter’s wheel was in full use, and the Harappans produced their own characteristic pottery, which was made
glossy and shining pottery. So, clearly both A and R are wrong.
The Harappans also practised boat-making.
They practised navigation on the coast of the Arabian Sea. They knew the use of wheel, and carts with solid wheels
were in use in Harappa.
The goldsmiths made jewellery of silver, gold and precious stones; the first two may have been obtained from
Afghanistan and the last from south India.

Q.15 Consider the following about metal trade and metallurgy in Harrapan civilization.
1. The Harappans produced enough copper ore to export it to Western India and even West Asia.
2. No evidence of the use of an alloy can be found in the civilization.
3. There is evidence of use of Gold and precious metals in the civilization that was sourced from Southern India and
even abroad.
Select the correct answer using the codes below.
(a) 1 only
(b) 2 and 3 only
(c) 3 only
(d) 1 and 2 only
Solution: (c)
Justification: Statement 1: The Harappans probably got copper from present-day Rajasthan, and even from Oman
in West Asia.
Statement 2: Tin, which was mixed with copper to produce bronze, may have been brought from present-day
Afghanistan and Iran. So, 2 is wrong.
Statement 3: Gold could have come all the way from present-day Karnataka, and precious stones from present-day
Gujarat, Iran and Afghanistan.

Q.16 Our knowledge of the Harappan civilization is based on the archaeological evidence alone
because
1. Harappans maintained all official works in palmscripts that are subject to organic decay.
2. No written scripts from the Harappans have been obtained in excavations.
Which of the above is/are correct?
(a) 1 only
(b) 2 only
(c) Both 1 and 2
(d) None
Solution: (d)
Justification: Harappans knew how to write and most of their seals contain some form of script. But,
unfortunately no one has yet been able to decipher that script.
Various types of utensils, toys, seals, figurines have been excavated from different sites that often contain some
signs of a written script.
Since Harappans had an expanded trade network, they could not have merely survived writing palmscripts. They
must have had a robust system of writing and communication, especially to contact other countries by trade.
Q.17 Depictions of which of the following cannot be found in Harappan Seals?
(a) One-horned rhinoceros or the Unicorn
(b) Peepal leaves
(c) Humped bull and elephant
(d) Horses
Solution: (d)
Justification: Numerous seals carrying the images of the one-horned rhinocerous known as unicorn, peepal leaves
and a male god throw light on the religious beliefs of the Harappans.
It appears that they worshipped plants and animals and the forces of nature.
They worshipped a male god resembling Lord Shiva (Pashupati) of later times and a mother goddess among others.
They probably believed in life after death and also in charms and spells. Seals engraved with animal figures like the
humped bull, elephant and rhinocerous suggest that these animals were considered sacred.

Q.18 Which of these earliest cities was located to the east of Indus river?
1. Dholavira
2. Rakhi Garhi
3. Kalibangan
4. Sotkakoh
Select the correct answer using the codes below.
(a) 1, 2 and 3 only
(b) 2 and 4 only
(c) 1 and 3 only
(d) 1, 2, 3 and 4
Solution: (a)
Justification:

Learning: These cities were found in the Punjab and Sind in Pakistan, and in Gujarat, Rajasthan, Haryana and the
Punjab in India. Archaeologists have found a set of unique objects in almost all these cities: red pottery painted
with designs in black, stone weights, seals, special beads, copper tools, and long stone blades. Cities such as
Kalibangan and Lothal had fire altars, where sacrifices may have been performed. In some cities, like Mohenjodaro,
a very special tank, which archaeologists call the Great Bath, was found.
Q Source: Page 33: NCERT Class VI Our Pasts – I

Q.19 With reference to the Indus Valley civilization, consider the following statements:
1. Indus valley civilization was the biggest exporter of semi-precious stones and a major importer of agricultural
products.
2. Fishing was a regular occupation while hunting and bull fighting were banned.
3. Linga worship was prevalent and even natural figures like trees were worshipped.
4. Lothal was an important point of trade between the Harappan civilization and the remaining part of India as well
as Mesopotamia.
Select the correct answer using the codes below.
(a) 1 and 2 only
(b) 3 and 4 only
(c) 1, 2 and 3 only
(d) 2, 3 and 4 only
Solution: (b)
Justification: Statement 1: Gold, copper, tin and several semi-precious stones were imported. Main exports were
several agricultural products such as wheat, barely, peas, oil seeds etc.
Statement 2: Hunting and bull fighting were common pastimes, not banned. Marbles, balls and dice were also used
for games.
Statement 3: The chief female deity was the Mother Goddess represented in terracotta figurines. In latter times,
Linga worship was prevalent. Trees and animals were also worshipped by the Harappans. They also believed in
ghosts and evil forces and used amulets as protection against them.
Statement 4: Lothal is an important Harappan centre, you should read more about it.
Q Source: Page 21-23: TN 11th Standard History Textbook

Q.20 This archaeological site consist of a number of low archaeological mounds created by generations of
superimposed mudbrick structures. Located close to the mouth of the Bolan Pass, it was abandoned by the time
of the emergence of the literate urbanised phase of the Indus Civilisation. An entry in the UNESCO tentative
list it is?
(a) Lothal
(b) Mehrgarh
(c) Inamgaon
(d) Surkotada
Solution: (b)
Learning: It was a small farming village. It is one of the earliest sites with evidence of farming and herding in South Asia.
Mehrgarh is now seen as a precursor to the Indus Valley Civilization, displaying the whole sequence from earliest settlement
and the start of agriculture, to the mature Harappan Civilisation (based on the above Q description of site abandonment).
Most of its archaeological deposits are buried deep beneath accumulations of alluvium.

Q.21 Harappans were aware of and practiced which of these number systems in their weights
and measures?
(a) Binary system
(b) Decimal system
(c) Hexadecimal system
(d) Both (a) and (b)
Solution: (d)
Learning: Exchanges were regulated by a precise system of weights, usually made of a stone called
chert and generally cubical with no markings.
The lower denominations of weights were binary (1, 2, 4, 8, 16, 32, etc). While the higher
denominations followed the decimal system, the smaller weights were probably used for weighing
jewellery and beads and bigger weights were used for food grains.

Q.22 The first city that was discovered in the Harappan civilization was
(a) Lothal
(b) Inamgaon
(c) Harappa
(d) Mohenjodaro
Solution: (c)
Learning: In 1860s, when railway lines were being laid down for the first time in the Punjab, engineers stumbled
upon the site of Harappa in present-day Pakistan.
Then, in 1930s archaeologists found the site, and realised that this was one of the oldest cities in the subcontinent.
As this was the first city to be discovered, all other sites from where similar buildings (and other things) were found
were described as Harappan. These cities developed about 4700 years ago.

Q.23 In the context of Indus Valley civilization, which of these statements is the most appropriate?
(a) Indus valley people were fashion conscious.
(b) Indus Valley civilization was a closed economy.
(c) It was an atheist civilization.
(d) The civilization did not have access to urban amenities.
Solution: (a)
Justification: Option A: Jewellery has been excavated. This tells us that Indus valley people were concerned with
fashion.
Option B: International trade was in practice in the civilization. This is because stone weights have been found
which were used because there was a market where goods were traded.
Option C: There is evidence of images of gods and goddesses like pashupati, who was considered the chief deity
in the civilization.
Option D: A number of urban amenities like bath structures, pukka houses, drainage system etc
Were present in these cities.

VEDIC PERIODS
ADVENT OF ARYANS AND RIG VEDA

Q.24 In Vedic socieities, Gahapatis were


(a) Big land-owner or big farmer involved in farming and trade
(b) Householders who made regular donations to religious sects
(c) Preserver of the law and order in social congregations such as festivals
(d) The priest who performed daily rituals for the royal household
Solution: (a)
Learning: The institutional development within the oligarchies of Vedic societies suggests a stabilized
agrarian economy. In such a stable economy, there existed a hierarchy based on land ownership and
distribution.
Sources mention wealthy householders (gahapatis) employing slaves and hired labourers to work on their
lands. The existence of gahapatis suggests the breaking up of clan ownership of land and the emergence of
individual holdings.
In the Pali canon, various Pali words have been translated into the English word "householder", including
agārika, gahapati, gahattha and gihi

Q.25 Consider the following statements about the Rig Vedic society.

1. The Rig Vedic society was matriarchal.


2. Women were allowed to participate in literary activities.
3. The basic unit of society was the family whose head was called as Gahapati.
Select the correct answer using the codes below.
(a) 1 only
(b) 2 and 3 only
(c) 1 and 3 only
(d) 1, 2 and 3
Solution: (b)
Justification: We have had poets like Apala, Viswavara, Ghosa and Lopmudra during the Rig Vedic period.
Women could even attend the popular assemblies. There was no child marriage and the practice of sati was absent.
However, the society was patriarchal. The basic unit of society was family or graham. The head of the family was
known as grahapathi.
Monogamy was generally practiced while polygamy was prevalent among the royal and noble families. The wife
took care of the household and participated in all the major ceremonies

Q.26 The basic material or mantra text of each of the Vedas is called
(a) Samhita
(b) Brahmanas
(c) Aranyakas
(d) Paujanas
Solution: (a)
Learning: Samhita literally means "put together, joined, union", "collection", and "a methodically, rule-based
combination of text or verses".
Samhita also refers to the most ancient layer of text in the Vedas, consisting of mantras, hymns, prayers, litanies
and benedictions.
Parts of Vedic Samhitas constitute the oldest living part of Hindu tradition
Some post Vedic texts are also known as “Samhitas” such as Ashtavakra Gita, Bhrigu Samhita, Brahma Samhita,
Deva Samhita, Garga Samhita, Kashyap Samhita, Shiva Samhita and Yogayajnavalkya Samhita.

Q.27 The Vedic literature is broadly divided into two categories viz. Shruti and Smriti. What is the
difference between them?
1. Shruti is considered eternal whereas Smriti is subject to change.
2. Smriti philosophy stands in direct opposition or contrast to Shruti philosophy.
Which of the above is/are correct?
(a) 1 only
(b) 2 only
(c) Both 1 and 2
(d) None
Solution: (a)
Justification: Shruti describes the sacred texts comprising the central canon of Hinduism viz. Vedas, Brahmanas,
Aranyakas, & Upanishads.
Smiriti literally means “that which is remembered,” and it is entire body of the post Vedic
Classical Sanskrit literature. It comprises Vedanga, Shad darsana, Puranas, Itihasa, Upveda, Tantras, Agamas,
Upangs. There is another post Vedic class of Sanskrit literature called Epics which includes Ramayana and
Mahabharta.
Shruti is “that which has been heard” and is canonical, consisting of revelation and unquestionable truth, and is
considered eternal. It refers mainly to the Vedas themselves.
Smriti is “that which has been remembered” supplementary and may change over time. It is authoritative only to
the extent that it conforms to the bedrock of Shruti.
However, there is not a distinct divide between Shruti and Smriti. Both Shruti and Smriti can be represented as a
continuum, with some texts more canonical than others
Q.28 Vedic (Shrauta) yajnas are typically performed by four priests of the Vedic priesthood: the hotar, the
adhvaryu, the udgatar and the Brahmin. What are their respective roles?
1. The Hotri recites invocations and litanies drawn from the Rigveda.
2. The Udgatri is the superintendent of the entire performance.
3. The Brahmin is responsible for building the altar explained in the Yajurveda.
4. The Adhvaryu chants the introductory, accompanying and benediction hymns.
Select the correct answer using the codes below.
(a) 1 only
(b) 2, 3 and 4 only
(c) 2 and 3 only
(d) D. 1, 2, 3 and 4
Solution: (a)
Justification: The functions associated with the priests were:
The Adhvaryu is the priest's assistant and is in charge of the physical details of the ritual like measuring the ground,
building the altar explained in the Yajurveda. The adhvaryu offers oblations.
The Udgatri is the chanter of hymns set to melodies and music (sāman) drawn from the Samaveda. The udgatar,
like the hotar, chants the introductory, accompanying and benediction hymns.
The Brahmin is the superintendent of the entire performance, and is responsible for correcting mistakes by means
of supplementary verses.
The Hotri recites invocations and litanies drawn from the Rigveda.

Q.29 Consider the following with reference to the ‘Painted Grey Ware’.
1. PGW is a predecessor of the Black and red ware culture.
2. These were high quality metalworks produced by local craftsmen.
3. PGW culture is associated with both village and town settlements.
4. PGW sites can be found in Ganges-Basin.
Select the correct answer using the codes below.
(a) 1 and 4 only
(b) 2 and 3 only
(c) 3 and 4 only
(d) 1 and 2 only
Solution: (c)
Justification: Statement 1 and 2: The Painted Grey Ware culture (PGW) is an Iron Age culture of the Gangetic plain
and the Ghaggar-Hakra valley, lasting from roughly 1200 BCE to 600 BCE.
It is contemporary to, and a successor of the Black and red ware culture.
These were high quality earthenware.
Statement 3: Characterized by a style of fine, grey pottery painted with geometric patterns in black,
this culture is associated with village and town settlements (but without large cities like those of the Harappans),
domesticated horses, ivory-working, and the advent of iron metallurgy
Statement 4: While the main concentration of sites is in Indian Punjab, Haryana, north-eastern Rajasthan and the
upper Ganges-Jamuna basin in Uttar Pradesh, the occurrence of some sites has been reported from as far west as
in Sind and Harappa in southern Punjab, both in Pakistan.

Q.30 The Baudhayana sutras are a group of Vedic Sanskrit texts which cover
1. Statecraft
2. Dharma
3. Daily ritual
4. Mathematics
Select the correct answer using the codes below.
(a) 1, 2 and 3 only
(b) 2, 3 and 4 only
(c) 1 and 4 only
(d) D. 1, 2, 3 and 4
Solution: (b)
Justification: These sutras belong to the Taittiriya branch of the Krishna Yajurveda school and are among the
earliest texts of the sutra genre, perhaps compiled in the 8th to 7th centuries BCE.
It contains dharma, daily ritual, mathematics, etc.
The Baudhayana sutras consist of six texts: For e.g. Sulbasutra is noted for containing several early mathematical
results, including an approximation of the square root of 2 and the statement of a version of the Pythagorean
theorem.

Q.31 With reference to the Vedic civilization, Rajasuya, Asvamedha and Vajpeya were
(a) Rituals and sacrifices performed by the King to strengthen his position
(b) Mercenary army contingents that were owned by the local landlords
(c) Land tax imposed on areas inside the jurisdiction of the Empire
(d) Body of officials who maintained a record of the extent of empire, its defeats and successions
Solution: (a)
Context: Larger kingdoms were formed during the later Vedic period as many jana or tribes were amalgamated to
form janapadas or rashtras in the later Vedic period.
Hence the royal power had increased along with the increase in the size of kingdom.
Learning: So, the king performed various rituals and sacrifices to strengthen his position. They include Rajasuya
(consecration ceremony), Asvamedha (horse sacrifice) and Vajpeya (chariot race). The kings also assumed titles like
Rajavisvajanan, Ahilabhuvanapathi, (lord of all earth), Ekrat and Samrat (sole ruler) to showcase his power.

Q.32 The river(s) that do NOT find mention in Rigveda is?


(a) Ganga and Yamuna
(b) Indus and its other tributaries
(c) Saraswati
(d) All of the above findmention.
Solution: (d)
Learning: Option B: There is a hymn in the form of a dialogue between sage Vishvamitra, and two rivers, (Beas and Sutlej) that
were worshipped as goddesses.
Option C: The most prominent river of the Rigveda is the Sarasvati, next to the Indus. Option A: The Ganga and Yamuna are
named only once.

Q.33 The credit of bringing the upper Ganges - Jamuna valley under large-scale cultivation goes to the Painted
Grey Ware people. They could achieve this mainly because of
1. Their iron technology
2. The exclusion of resource intensive rice from the basket of crops
Which of the above is/are correct?
(a) 1 only
(b) 2 only
(c) Both 1 and 2
(d) None
Solution: (a)
Justification: Statement 1: They cleared the heavy jungle with the help of iron axes. Of the ploughshare no
example has so far been found from this period, though the subsequent period has yielded examples.
Statement 2: The excavations at Hastinapura brought to light the remains of rice.

Q.34 The Vajapeya sacrifice lays down some rituals. Consider the following about it with refence to the
early Vedic period.
1. These rituals are to be followed in chariot races.
2. The Vajapeya was the most important political ritual of the time, even greater than the Rajasuya.
3. Under the sacrifice, oblations were offered by the king to the Ratnins who were the important courtiers and
officials.
Select the correct answer using the codes below.
(a) 1 and 3 only
(b) 2 only
(c) 2 and 3 only
(d) 1, 2 and 3
Solution: (a)
Answer Justification :
Justification: Under the Vajapeya sacrifice, the king was expected to win the chariot race and was then
proclaimed and installed to the high office.
It is however important to note that in the Vedic period the Väjapeya was of lesser political importance than the
Räjasüya.
In the later Vedic period the Väjapeya however obtained the higher rank than the Rajasuya because it was believed
that by performing the former, one secures the imperial position, while by performing the latter, one attains to
mere royalty.
Another important feature of the sacrifice was the Ratnin oblations, which were offered by the king to the Ratnins,
(important courtiers and officials) by repairing to their houses.
In the epic period, the royal oath formed an important part in the coronation ceremony.

LATER VEDIC PERIODS

Q.35 The northern India of 6th century B.C. consisted of a large number of independent kingdoms,
some of which were republics which consisted of only one tribe. Which of the following was NOT
one of them?
(a) Sakyas
(b) Licchavis
(c) Mallas
(d) Vakatakas
Solution: (d)
Learning: Some of the republics consisted of only one tribe like the Sakyas, Licchavis and Mallas. In these
republics the power of decision vested with the Public Assembly which was composed of the tribal representatives
or head of families
All decisions were by a majority vote.
Apart from republics, there were also monarchical forms of government. The Buddhist literature Anguttara Nikaya
gives a list of sixteen great kingdoms called ‘Sixteen Mahajanapadas’.
In course of time, the small and weak kingdoms either submitted to the stronger rulers or gradually got eliminated.
Finally in the mid 6th century B.C., only four kingdoms – Vatsa, Avanti, Kosala and Magadha survived

Q.36 With reference to ratnahavimshi ceremony in Ancient India, consider the following statements.
1. It was a part of Ashwamedha sacrifice where oblations to certain gods were also offered.
2. It involved the King going on successive days to the homes of certain people called ‘ratnins’.
Which of the above is/are correct?
(a) 1 only
(b) 2 only
(c) Both 1 and 2
(d) None
Solution: (b)
Justification: Statement 1: The ratnahavimshi (ceremony of the jewel offering) was a part of the rajasuya sacrifice.
It involved the rajan going on successive days to the homes of certain people—the ratnins (literally, 'jewels')—and
offering oblations to certain gods.
There is some variation in the names and order of the list of ratnins in different texts. They included, inter alia, the
following:
the Brahmana or purohita (he usually heads the list)
the rajanya (nobles)
mahishi (chief queen)
parvrikti (the discarded queen; it is necessary to visit her to ward off evil)
senani (commander of the army)
suta (charioteer or bard)
gramani (village headman)
Learning: The ceremony indicates the importance of ratnins and the King’s dependence on them. Some ratnins
were related to the king through kinship, whereas others were functionaries with whom he had no kinship
relations. This illustrates the transitional nature of the later Vedic polity—it was in between a polity in which
kinship was still an important factor and one marked by an elaborate military and administrative machinery.

Q.37 Matsya were one of the Indo-Aryan tribes of Vedic India. By the late Vedic period, they
ruled a kingdom located south of the Kurus. It roughly corresponds to the former state of
(a) Rajasthan
(b) Madhya Pradesh
(c) Gujarat
(d) Punjab
Solution: (a)
Learning: It was to the west of the Yamuna River which separated it from the kingdom of the Panchalas.
It roughly corresponded to the former state of Jaipur in Rajasthan. The capital of Matsya was at
Viratanagari (present-day Bairat).
Alwar has been a part of Matsya region.

Q.38 Which of these developments can be attributed to Later Vedic age, as against the practice in
the Vedic age?
1. Women lost their political rights of attending assemblies.
2. Child marriage were banned and the practice of sati was absent.
3. Women were not allowed to wear garments made of cotton.
Select the correct answer using the codes below.
(a) 1 only
(b) 2 and 3 only
(c) 3 only
(d) 2 only
Solution: (a)
Background: The Rig Vedic society was patriarchal. But, the condition of women was much better in this
period that in the Later Vedic age.
Women were given equal opportunities as men for their spiritual and intellectual development.
There were women poets like Apala, Viswavara, Ghosa and Lopamudra during the Rig Vedic period.
Women could even attend the popular assemblies. There was no child marriage and the practice of
sati was absent.
Justification: Statement 1: In the family, the power of the father increased during the Later Vedic
period.
There was no improvement in the status of women. They were still considered inferior and subordinate
to men.
Women also lost their political rights of attending assemblies.
Statement 2: Child marriages had become common during the later vedic period.
Statement 3: In both periods, men and women wore upper and lower garments made of cotton and
wool. A variety of ornaments were used by both men and women.

SHAMAN TRADITIONS
JAINISM

Q.39 Jains celebrate five major events in the life of a Tirthankar. They are called Kalyanak
(auspicious events). Consider the following with reference to them.
1. Chyavana Kalyanak is the event when the Tirthankar's soul is born.
2. Diksha Kalyanak is the event when the Tirthankar's soul attains true knowledge.
3. Kevaljnana Kalyanak is the event when Tirthankar’s soul leaves the physical existence and becomes a siddha.
Select the correct answer using the codes below.
(a) 1 only
(b) 2 and 3 only
(c) 1 and 3 only
(d) None of the above
Solution: (d)
Justification: Chyavana Kalyanak - This is the event when the Tirthankar's soul departs from its last life, and is
conceived in the mother’s womb.
Janma Kalyanak - This is the event when the Tirthankar's soul is born.
Diksha Kalyanak - This is the event when the Tirthankar's soul gives up all his/her worldly
possessions and becames a monk/nun. (Digambar sect does not believe that women can become Tirthankar or be
liberated.)
Kevaljnana Kalyanak - This is event when Tirthankar’s soul destroys the four ghati karmas completely and attains
the Kevaljnana (absolute knowledge). Celestial angels set Samavsaran for Tirthankars from where he/she delivers
the first sermon. This is the most important event for the entire Jain order as the Tirthankar reinstates Jain Sangh
and preaches the Jain path of purification and liberation.
Nirvana Kalyanak - This event is when a Tirthankar’s soul is liberated from this worldly physical existence forever
and becomes a Siddha. On this day, the Tirthankar’s soul destroys the four aghati karmas completely, and attains
salvation, the state of eternal bliss.

Q.40 Jainism postulates that time has no beginning or end. It moves like the wheel of a cart. In this
context what are Avsarpani and utsarpini?
(a) Eternal cycles of moral rise and decline
(b) Time of the birth and death of a Tirthanakara respectively
(c) Two auspicious time portals when one can attain true Kaivalya
(d) Timeline of the birth of first and last Tirthankara respectively
Solution: (a)
Learning: Time rolls along in eternal cycles of rise and decline. Utsarpini is a "rising" era in which human morale and
natural conditions improve over time.
At the end of Utsarpini, begins Avasarpini, a "declining" era of the same length, in which human morale and virtues
deteriorate.
During the middle of every rising and declining era twenty-four souls become Tirthankaras.
They are the humans like us who rise to that level. While accumulating different karmas, they also accumulate get a
special karma called Tirthankar Nam Karma in the last 3rd of their life by performing one or more of the 20 special
austerities.
Tirthankar Nam Karma matures in the final life and leads the person to become a Tirthankara. After attaining
omniscience, Tirthankara reorganize Jain religion to suit the changing times.

Q.41 Consider the following about the spread of Jainism.


1. Mahavira allowed both men and women to join Jain Sanghas.
2. The then rulers of Southern Indian patronized Jainism.
3. Chandragupta Maurya is said to have persecuted Jainas on account of their severe ascetic practices.
Select the correct answer using the codes below.
(a) 1 and 2 only
(b) 1 and 3 only
(c) 2 only
(d) None of the above
Solution: (a)
Justification: Mahavira organised the Sangha to spread his teachings. He admitted both men and women in the
Sangha, which consisted of both monks and lay followers.
The rapid spread of Jainism was due to the dedicated work of the members of the Sangha. It spread rapidly in
Western India and Karnataka.
Statement 3: Chandragupta Maurya, Kharavela of Kalinga and the royal dynasties of south India such as the Gangas,
the Kadambas, the Chalukyas and the Rashtrakutas patronized Jainism.
By the end of the fourth century B.C., there was a serious famine in the Ganges valley. Many Jain monks led by
Bhadrabagu and Chandragupta Maurya came to Sravana Belgola in Karnataka.
Those who stayed back in north India were led by a monk named Sthulabahu who changed the code of conduct for
the monks. This led to the division of Jainism into two sects Svetambaras and Digambaras.

Q.42 The Jainas believe that the natural and supernatural things of the universe can be traced back
to seven fundamental elements. They include
1. Jiva
2. Vinijja
3. Kshati
4. Samvara
5. Nirjana
Select the correct answer using the codes below.
(a) 2 and 3 only
(b) 1, 4 and 5 only
(c) 1, 2 and 3 only
(d) D. 1, 2, 3, 4 and 5
Solution: (b)
Justification: These are jiva, ajivaa, astikaya, bandha, samvara, nirjana, and moksa. Substances like body which
exist and envelope (like a cover) are astïkaya.
Anastikayas like ‘time’ have no body at all. The substance is the basis of attributes (qualities).
The attributes that we find in a substance are known as dharmas. The Jainas believe that things or substance have
attributes.
These attributes also change with the change of kala (time). From their point of view, the attributes of a substance
are essential, and eternal or unchangeable.
Without essential attributes, a thing cannot exist. So, they are always present in everything.
For example, consciousness (chetana) is the essence of the soul; desire, happiness and sorrow are its changeable
attributes.
Q.43 The Principle of “Right Faith” as propounded by Mahavira Jaina is that
(a) That all sentient beings are destined to salvation
(b) All must understand their role in spiritual life and strictly follow the doctrine of ahimsa
(c) Unshaken belief in the teachings and wisdom of Mahavira
(d) Not believing in the theory of a God or a creator
Solution: (c)
Justification: The three principles or Triratnas of Jainism are right faith, right knowledge and right conduct.
Triratnas—3 Gems of Jainism
Samyak Shradha/ Vishwas (Right Faith) — Belief of Tirthankaras
Samyak man or Gyan (Right Knowledge) — Knowledge of Jam belief
Samyak Karma/Acharana (Right Action/Conduct) — Practise of 5 vows of Jainism
Learning: Mahavira regarded all objects, both animate and inanimate, have souls and various degrees of
consciousness. They possess life and feel pain when they are injured.
Even the practice of agriculture was considered sinful as it causes injury to the earth, worms and
animals. Similarly the doctrine of asceticism and renunciation was also carried to extreme lengths by the practice of
starvation, nudity and other forms of self-torture.

Q.44 Consider the following statements about certain beliefs of sects of Jainism.
1. Digambaras believe that women cannot achieve liberation without first being reborn as a man.
2. Digambara monks are not allowed any possessions, not even personal begging bowls.
Which of the above is/are correct?
(a) 1 only
(b) 2 only
(c) Both 1 and 2
(d) None
Solution: (c)
Answer Justification :
Concept: Jains are divided into two major sects; the Digambara (meaning sky clad) sect and the Svetambara
(meaning white clad) sect. Each of these sects is also divided into subgroups.
The two sects agree on the basics of Jainism, but disagree on:
 details of the life of Mahavira
 the spiritual status of women
 whether monks should wear clothes
 rituals
 which texts should be accepted as scripture
Justification: Statement 1: As per them, this is because women cannot live a truly ascetic life, because they have
to possess clothes. They believe true monks must live without clothes.
Statement 2: They believe that one can become a monk only
by having no worldly possessions
by demonstrating indifference to earthly emotions such as shame
Digambara monks are not allowed any possessions, not even begging bowls and so can only receive gifts in their
cupped hands.

Q.45 Jainism advocates three principles known as Triratnas (three gems) which are right faith, right
knowledge and right conduct. On the other hand, Buddhism preaches four noble truths and the eightfold
path. What is/are common between the teachings of both religions?
1. Acceptance of the theory that God exists and it has multiple aspects
2. Understanding that no object possesses a soul and is inanimate in the larger flow of life
3. To practice ahimsa and abstain from acquiring precious metals like Gold and Silver
4. Penance and extreme austerity to punish the body
Select the correct answer using the codes below.
(a) 1 and 2 only
(b) 3 only
(c) 2 and 3 only
(d) 1 and 4 only
Solution: (b)
Justification: We will explain it in the light of teachings of Mahavira.
Statement 1 and 2: Right Knowledge (as per Mahavira) is the acceptance of the theory that there is no God and that
the world has been existing without a creator and that all objects possess a soul. Buddhism sheds any discussion on
God and soul as futile. Buddha also taught that the soul does not exist, so there is no meeting ground in this case.
So, 1 and 2 are wrong.
Statement 3: Right conduct (Jainism) refers to the observance of the five great vows such as not to injure life, not to
lie, not to steal, not to acquire property and not to lead an immoral life. Buddhism also forbids its monks from
acquiring property (that may bring a sense of ego and pride to its monks and nuns) and injuring life. So, 3 is a
meeting ground.
Statement 4: Only Jainism advocates extreme penance, Buddhism preaches moderation. So, 4 is wrong.

Q.46 Adipurana written by Gunabhadra is a work on the


(a) Commentary of esoteric aspects found in the Vedas
(b) Life stories of various Jain saints
(c) Sanskrit Grammar
(d) Verses used in sacrificial ceremony
Solution: (b)
Justification: It is a 9th century Sanskrit poem composed by Jinasena, a Digambara monk. It deals
with the life of Rishabhanatha, the first Tirthankara, and other saints.
Learning: Amogavarsha I, who was a Jain patronized many Jain scholars.
His teacher Jinasena composed Parsvabhudaya, a biography of Parsva in verses.
Sakatayana wrote the grammer work called Amogavritti. And, the great mathematician of this period,
Viracharya was the author ofGanitasaram.

Q.47 Consider the following statements with reference to Jaina sects.


1. The Svetambara tradition of Jainism trace their lineage through Sthulabhadra.
2. As per the Digambara sect of Jainism, Bhadrabahu was the last Shruta Kevalin.
Which of the above is/are correct?
(a) 1 only
(b) 2 only
(c) Both 1 and 2
(d) None
Solution: (c)
Justification & Learning: Statement 1: By the end of the fourth century B.C., there was a serious famine in
the Ganges valley. Many Jain monks led by Bhadrabagu and Chandragupta Maurya came to Sravana Belgola in
Karnataka.
Those who stayed back in north India were led by a monk named Sthulabahu who changed the code of conduct
for the monks.
This led to the division of Jainism into two sects Svetambaras (whiteclad) and Digambaras (Sky-clad or Naked).
The first Jain Council was convened at Pataliputra by Sthulabahuin the beginning of the 3rd century B.C.
Statement 2: According to the Digambara sect of Jainism, there were five Shruta Kevalins in Jainism
Govarddhana Mahamuni, Vishnu, Nandimitra, Aparajita and Bhadrabahu.
Shrutakevalin is a term used in Jainism for those ascetics who have complete knowledge of Jain Agama (texts).

BUDDHISM

Q.48 Consider the following about Abhidhamma Pitaka.


1. It is a scripture of Mahayana Buddhism.
2. It contains Buddhist philosophy.
3. It was first compiled in Sanskrit and then translated to Prakrit.
Select the correct answer using the codes below.
(a) 1 only
(b) 2 and 3 only
(c) 2 only
(d) 1 and 2 only
Solution: (c)
Justification: Gautama’s three main disciples known as Upali, Ananda and Mahakashyap remembered his
teachings and passed them on to his followers.
It is believed that soon after the Buddha’s death a council was called at Rajagriha where Upali recited the Vinaya
Pitaka (rules of the order) and Ananda recited the Sutta Pitaka (Buddha’s sermons or doctrines and ethics).
Sometime later the Abhidhamma Pitaka consisting of the Buddhist philosophy came into existence.
It is the last of the three pitakas constituting the Pali Canon, the scriptures of Theravāda Buddhism.
It is a detailed scholastic reworking of material appearing in the Suttas, according to schematic classifications. It
does not contain systematic philosophical treatises, but summaries or enumerated lists

Q.49 The famous Buddhist Diamond Sutra is a/an


(a) Old Japanese text
(b) Translation of Abhidhamma pitaka compiled in India
(c) Recent work of the Gompa monastery, Spiti
(d) Chinese version of the Sutta pitakas
Solution: (a)
Learning: Buddhist missionaries from China introduced hand-printing technology into Japan
around AD 768-770.
The oldest Japanese book, printed in AD 868, is the Buddhist Diamond Sutra, containing six sheets of text and
woodcut illustrations.
Diamond Sutra is one of the most influential Mahayana sutras in East Asia, and is a key object of devotion and study
in Zen Buddhism.

Q.50 Vajrayana Buddhism


1. Includes various Buddhist traditions of Tantra
2. Developed parallelly with Mahayana Buddhism in India
3. According to Vajrayana scriptures, Vajrayana is the only vehicle to enlightenment among the three schools of
Buddhism
Select the correct answer using the codes below.
(a) 1 only
(b) 2 and 3 only
(c) 1 and 2 only
(d) 1 and 3 only
Solution: (a)
Justification: Vajrayāna, Mantrayana, Esoteric Buddhism and Tantric Buddhism refer to the various
Buddhist traditions of Tantra and "Secret Mantra", which are systems of beliefs and practices that
developed in medieval India and spread to Tibet and East Asia under varying names and forms.
It subscribes to the literature known as the Buddhist Tantras. It includes practices that make use of mantras,
dharanis, mudras, mandalas and the visualization of deities and Buddhas.
According to its scriptures, the term Vajrayana refers to one of three vehicles or routes to enlightenment, the other
two being the Sravakayana (also known as the Hinayana) and Mahayana.

Q.51 How do the Jatakas relate to Buddhism?


1. In Theravada Buddhism, they are considered a textual division of the Pali Canon.
2. They are literature native to India concerning the previous births of Gautama Buddha.
Which of the above is/are correct?
(a) 1 only
(b) 2 only
(c) Both 1 and 2
(d) None
Solution: (c)
Justification: The Jataka tales are a voluminous body of literature native to India concerning the previous births of
Gautama Buddha in both human and animal form. The future Buddha may appear as a king, an outcast, a god, an
elephant—but, in whatever form, he exhibits some virtue that the tale thereby inculcates.
In Theravada Buddhism, the Jatakas are a textual division of the Pali Canon, included in the Khuddaka Nikaya of the
Sutta Pitaka. The term Jataka may also refer to a traditional commentary on this book.
Many stupas in northern India are said to mark locations from the Jataka tales; the Chinese pilgrim Xuanzang reported
several of these.

Q.52 The term Boddhisattvas have been subject to multiple interpretations in Buddhism. Which of
the following is NOT one of them?
(a) Bodhisattva was primarily used to refer to Gautama Buddha in his former life.
(b) Bodhisattva meant someone on the path to liberation.
(c) Bodhisattva was someone who delayed his final liberation to help other sentient beings.
(d) Bodhisattva was someone who became enlightened by the good karma of the Sangha.
Solution: (d)
Justification: In early Indian Buddhism, the term bodhisattva was primarily used to refer specifically to Gautama
Buddha in his former life.
From this Jataka tales, Bodhisattva originally meant the Buddhism practitioner of austerities.
In later Theravada literature, the term "bodhisatta" is used fairly frequently in the sense of someone on the path to
liberation.
According to some Mahayana sources a bodhisattva is someone on the path to full Buddhahood. Others speak of
bodhisattvas renouncing Buddhahood.
However, under the new tradition most accepted definition is the one who aspires to become buddha as soon as
possible and then help sentient beings.

Q.53 Abhidhamma Pitakas contain


(a) Contemporary research into the fundamental Buddhist ideas available in different languages
(b) Teachings attributed to the Buddha or his close companions
(c) Philosophical and psychological discourse and interpretation of Buddhist doctrine
(d) Rules and regulations of monastic life ranging from dress code and dietary rules to prohibitions of certain
personal conducts
Solution: (c)
Justification: The Tripitakas of Buddhism are known as the Sutta, the Vinaya and the Abhidhamma Pitakas.
Out of them, the Abhidhamma Pitaka is a detailed scholastic reworking (interpretation and comment) of material
appearing in the Suttas, according to schematic classifications. It does not contain systematic philosophical treatises,
but summaries or enumerated lists.
Abhidhamma has been variously described as philosophy, psychology, and metaphysics.
Tradition holds that the Buddha thought out the Abhidhamma immediately after his enlightenment then taught it to
the gods some years later. Later the Buddha repeated it to Sariputta who then handed it on to his disciples.
Scholars, however, generally date the Abhidhamma works to originating sometime around the third century BCE, 100
to 200 years after the death of the Buddha. Therefore, the seven Abhidhamma works are generally claimed by
scholars not to represent the words of the Buddha himself, but those of disciples and scholars

Q.54 Which of the following texts deals with Buddhism?


(a) Swayambhu Purana
(b) Markandey Purana
(c) Bhagavata Purana
(d) Skanda Purana
Solution: (a)
Learning: Swayambhu Purana is a Buddhist scripture about the origin and development of Kathmandu valley.
Swayambhu Purana gives details of all the Buddhas who came to Kathmandu. It also provides information about the
first and the second Buddhas in Buddhism.
The Swayambhu Purana is one of the oldest texts of Nepal's Newar Buddhist cult.

Q.55 Nyngma, Kagyud, Shakya and Geluk are sects/schools of


(a) Mahayana Buddhism
(b) Theravada Buddhism
(c) Vajrayana Buddhism
(d) None of the above
Solution: (c)
Learning: This is a lateral explanation.
Inscribed in 2012 on the Representative List of the Intangible Cultural Heritage of Humanity, Buddhist chanting of
Ladakh is an important part of monastic life.
In the monasteries and villages of the Ladakh region, Buddhist lamas (priests) chant sacred texts representing the
spirit, philosophy and teachings of the Buddha.
Two forms of Buddhism are practised in Ladakh – Mahayana and Vajrayana – and there are four major sects, namely
Nyngma, Kagyud, Shakya and Geluk. Each sect has several forms of chanting, practised during life-cycle rituals and on
important days in the Buddhist and agrarian calendars.
Chanting is undertaken for the spiritual and moral well-being of the people, for purification and peace of mind, to
appease the wrath of evil spirits or to invoke the blessing of various Buddhas, Bodhisattvas, deities and rinpoches.
The chanting is performed in groups, either sitting indoors or accompanied by dance in monastery courtyards or
private houses.

Q.56 The first Buddhist Council was held at Rajagraha under the chairmanship of
Mahakasapa immediately after the
(a) Mahaparinibbana of the Buddha
(b) First sermon of the Buddha
(c) Initiation of Rahula, Buddha’s son
(d) Separation of Buddhism into Mahayana and Theravada
Solution: (a)
Learning: Its purpose was to maintain the purity of the teachings of the Buddha. The second Buddhist Council was
convened at Vaisali around 383 B.C.
The third Buddhist Council was held at Pataliputra under the patronage of Asoka. Moggaliputta Tissa presided over it.
The final version of Tripitakas was completed in this council. The fourth Buddhist Council was convened in Kashmir by
Kanishka under the chairmanship of Vasumitra. Asvagosha participated in this council.

Q.57 In the context of Buddhism, the term Boddhisattvas can be used to refer to
1. Gautam Buddha in his former lives
2. Compassionate beings who accumulated merit through their efforts not to attain nibbana but to help others
3. Jataka story tellers who were appointed by the Sangha
4. Bodhi masters who renounce their physical bodies for the benefit of sentient beings
Select the correct answer using the codes below.
(a) 1 and 2 only
(b) 2, 3 and 4 only
(c) 1 and 4 only
(d) 1, 2, 3 and 4
Solution: (a)
Justification: Bodhisattva is the Sanskrit term for anyone who, motivated by great compassion, has generated
Bodhicitta, which is a spontaneous wish and a compassionate mind to attain Buddhahood for the benefit of all
sentient beings.
In early Indian Buddhism, the term bodhisattva was primarily used to refer specifically to Gautama Buddha in his
former life.
The Jataka tales, which are the stories of the Buddha's past lives, depict the various attempts of the bodhisattva to
embrace qualities like self-sacrifice and morality.
According to the Jataka tales, the term "bodhisattva" originally referred to the pre- enlightened practitioner of
austerities
The worship of images of the Buddha and Bodhisattas became an important part of Mahayana tradition.

Q.58 In the context of Buddhist history, what is common between Jetavana and Venuvana?
(a) Both were Buddhist Viharas.
(b) Both were donated to the Buddha by Anathapindika.
(c) Both are very close to Griddhakuta where Buddha gave majority of his teachings.
(d) All of the above
Solution: (a)
Learning: Jetavana Grove, one of the eight major sites relating to the life of the Buddha, has long served as
one of the major destinations for Buddhist pilgrims. The grove is located inside the ancient city of Śrāvastī
(Savatthi, Pāli) in the Indian state now known as Uttar Pradesh.
During his ministry, the Buddha was said to have spent 25 out of the 45 rainy seasons at or around this grove; he
also delivered a wide range of teachings that are transmitted in canonical sūtras and performed a multitude of
miracles.
As the legend is told, the grove was given to the Buddha gift from both Anāthapiṇḍika, a wealthy merchant,
and Prince Jetakumāra after a boastful challenge to cover the entirety of the grounds withgold was met.
It was the second vihara donated to Gautama Buddha after the Venuvana in Rajgir.

Q.59 Earliest Viharas were built for Buddhist monks and nuns because
1. It served as a permanent shelter for the wandering monks and nuns during rainy season.
2. It was planned to serve as a centre for propagation of Buddhism.
Which of the above is/are correct?
(a) 1 only
(b) 2 only
(c) Both 1 and 2
(d) None
Solution: (a)
Justification: Statement 1: Buddhist monks went from place to place throughout the year, teaching people. The
only time they stayed in one place was during the rainy season, when it was very difficult to travel. Then, their
supporters built temporary shelters for them in gardens, or they lived in natural caves in hilly areas.
As time went on, many supporters of the monks and nuns, and they themselves, felt the need for more permanent
shelters and so monasteries were built. These were known as viharas.
Statement 2: There was no Buddhism when the earliest Viharas were made. Buddhism, as an organized religion,
came into being after the death of the Buddha.
So, the purpose of preaching dhamma was a later purpose, and not the reason for establishing viharas.

Q.60 In Ancient India, Kutagarashala was


(a) A place where intellectual debates among Buddhist mendicants took place
(b) A place housing artefacts of high value
(c) An open place where royal religious processions took place
(d) A subsidized public homestay for travellers around the kingdom
Solution: (a)
Learning: There were many sects or schools of thought in that period (especially Buddhist period).
Lively discussions and debates took place between the teachers of these schools of thought.
Teachers like Buddha and Mahavira travelled from place to place, trying to convince one another as well
as laypersons, about the validity of their philosophy or the way they understood the world.
Debates took place in the kutagarashala (a hut with a pointed roof) and in groves where travelling
mendicants halted.
If a philosopher succeeded in convincing one of his rivals, the followers of the latter also became his
disciples. So support for any particular sect could grow and shrink over time.

Q.61 Who among the following is/are associated with Buddhist Councils?
1. Ashoka
2. Kanishka
3. Ajatashatru
4. Mahapadma Nanda
Select the correct answer using the codes below.
(a) 1, 2 and 3 only
(b) 1 and 2 only
(c) 3 and 4 only
(d) 1, 2, 3 and 4
Solution: (a)
Justification: The third Buddhist Council was held at Pataliputra under the patronage of Asoka Moggaliputta Tissa
presided over it. The final version of Tripitakas was completed in this council. So, 1 is correct.
The fourth Buddhist Council was convened in Kashmir by Kanishka under the chairmanship of Vasumitra. Asvagosha
participated in this council. So, 2 is correct.
The first Buddhist Council was held at Rajagraha under the chairmanship of Mahakasapa immediately after the
death of Buddha under the patronage of king Ajatashatru. Its purpose was to maintain the purity of the teachings
of the Buddha. So, 3 is correct.

Q.62 The Buddha taught that suffering and unhappiness is caused because
1. We have cravings and desires which often cannot be fulfilled.
2. The basic condition of the existence of nature is suffering.
3. We do not practice immobile asceticism.
Select the correct answer using the codes below.
(a) 2 only
(b) 1 and 3 only
(c) 1 only
(d) 2 and 3 only
Solution: (c)
Justification: Statement 1: Buddha taught that sometimes, even if we get what we want, we are not satisfied, and
want even more (or want other things). The Buddha described this as thirst or tanha.
He taught that this constant craving could be removed by following moderation in everything.
Statement 3: Buddha was against the passive dying out of karma by practicing immobile asceticism. He said one
cannot be liberated by this practice that involves sitting motionlessly for extended periods of time.

Q.63 Consider the following with reference to the Madhyamaka school of Buddhism.
1. It was founded by Nagarjuna.
2. All phenomena are empty, as per the school.
3. It does not believe in dependent co-arising.
Select the correct answer using the codes below.
(a) 1 and 2 only
(b) 3 only
(c) 1 only
(d) 2 and 3 only
Solution: (a)
Justification: It refers primarily to the later schools of Buddhism philosophy founded by Nagarjuna.
According to Madhyamaka all phenomena (dharmas) are empty (sunya) of "nature," a "substance" or "essence"
(svabhava) which gives them "solid and independent existence," because they are dependently co-arisen.
But this "emptiness" itself is also "empty": it does not have an existence on its own, nor does it refer to a
transcendental reality beyond or above phenomenal reality.

Q.64 What would be the parallel in Jainism to viharas in Buddhism?


(a) Sthanakas
(b) Vishuddha
(c) Nikaya
(d) Vanjana
Solution: (a)
Learning: Sthanakvasi is a sect of Svetambara Jainism founded by a merchant named Lavaji in 1653 AD.
The Sthanakvasi do not believe in idol-worship at all. As such they do not have temples but only sthanakas, that is,
prayer halls, where they carry on their religious fasts, festivals, practices, prayers, discourses, etc.
This is because this sect believes that idol worship is not essential in the path of soul purification and attainment of
Nirvana/Moksha.

Q.65 Consider the following stupas that were built in the places associated with Buddha’s life.
1. Lumbini: Where he died
2. Bodh Gaya: Where he attained mahaparinibbana
3. Sarnath: Where he built his first vihara
4. Kusinagara: Where he ordained his first disciple
Select the correct answer using the codes below.
(a) 1 and 2 only
(b) 2 and 3 only
(c) 3 and 4 only
(d) None of the above
Solution: (d)
Justification: Lumbini is where he was born.
Bodh Gaya is where he attained enlightenment.
Sarnath is where he gave his first sermon (public speech).
Kusinagara is where he attained nibbana (Death) gradually, each of these places came to be
regarded as sacred.
Learning: By the second century BCE a number of stupas, including those at Bharhut, Sanchi and
Sarnath had been built.
These were built because:
The tradition of erecting stupas may have been pre-Buddhist, but they came to be associated with
Buddhism.
Stupas were built because relics of the Buddha such as his bodily remains or objects used by him were
buried there.

Q.66 Consider the following matches of Buddhist sites with their characterization.
1. Kushinagar: Mahaparinirvana
2. Shravasti : Jetavana monastery
3. Vaishali : Third Buddhist Council
4. Piprahwa : Believed to be Ancient Kapilavastu
Select the correct answer using the codes below.
(a) 1, 2 and 4 only
(b) 1 and 3 only
(c) 2 only
(d) 1, 2, 3 and 4
Solution: (a)
Justification: Statement 1: Known as Kushavati in the ancient times, Kushinagar is where Gautama Buddha attained
Parinirvana, which occurs upon death of a body of someone who has attained nirvana during their lifetime. Many
of the ruins of stupas here are dated as early as 3rd and 5th century B.C.
Statement 2: The third turning of the Wheel of Dharma was delivered to an audience of bodhisattvas in the ancient
city of Shravasti or Savatthi, one of the six largest cities in India during Gautama Buddha’s lifetime. Jetavana
monastery, the most famous Buddhist monastery in India where Buddha gave majority of his teachings and
discourses, is also located here.
Statement 3: The ancient city of Bihar, which is now an archeological site, is where Gautama Buddha preached his
last sermon before his death in 483 B.C. The second Buddhist council was also held here in 383 B.C. and the city
contains one of the best preserved Pillars of Ashoka, topped by a single Asiatic lion.
Statement 4: Piprahwa is a village near Birdpur in Siddharthnagar district of Uttar Pradesh. Piprahwa and its
surroundings are of great significance to the Buddhist religion. It is one of the sites that is linked directly to the early
life of the Buddha as it is thought to be the location of the ancient city of Kapilavastu.
According to Pali texts and ancient Buddhist traditions Piprahwa is also the site of one of eight stupas constructed
over the remains of Lord Buddha. Piprahwa is to be found about nine kilometres north of Birdpur close to the
border that separates India from Nepal.

Q.67 In Tibetan Buddhism, the word ‘Terma’ means


(a) Boddhisattvas
(b) Religious texts written by monks and nuns
(c) Various forms of hidden teachings
(d) Life stories of the Buddha
Solution: (c)
Learning: These hidden teachings are key to Vajrayana or Tibetan Buddhist and Bon religious traditions. Termas are a part of
tantric literature.
The belief is that these teachings were originally esoterically hidden by various adepts such as Padmasambhava during the 8th
century, for future discovery at auspicious times by other adepts.
As such, terma represent a tradition of continuous revelation in Vajrayana or Tibetan Buddhism.
Termas are not always made public right away. The conditions may not be right; people may not yet be ready for them; and
further instructions may need to be revealed to clarify their meaning.

Q.68 This council held near Srinagar prepared an authoritative commentary on the Buddhist Tripitakas and
gave the Mahayana doctrine a final shape. The council was convened by
(a) Kanishka
(b) Samprati
(c) Bindusara Amitraghata
(d) Devavarman
Solution: (a)
Learning: He convened this Fourth Buddhist Council that discussed matters relating to Buddhist theology and doctrine.
It was held at the Kundalavana monastery near Srinagar in Kashmir under the presidentship of Vasumitra. About 500 monks
attended the Council.
Learning:Asvagosha was a great philosopher, poet and dramatist and was particularly close to Kanishka.He becamehis
religious advisor in his later years and composed Buddhacharita.
Nagarjuna from south India also adorned the court of Kanishka. The famous physician of ancient India Charaka was also
patronized by him.

Q.69 Buddhist Viharas are


(a) A monument to meditate and chant for the entire Sangha
(b) Permanent shelters or monasteries for dwelling and rest
(c) Divine extensions of stupas
(d) Sites containing relics of Buddha
Solution: (b)
Learning: To begin with, both Jaina and Buddhist monks went from place to place throughout the year, teaching people.
The only time they stayed in one place was during the rainy season, when it was very difficult to travel.
Then, their supporters built temporary shelters for them in gardens, or they lived in natural caves in hilly areas.
As time went on, many supporters of the monks and nuns, and they themselves, felt the need for more permanent shelters
and so monasteries were built.
These were known as viharas.

Q.70 Popularly known Jatakas were


(a) Stories of previous lives of the Buddha
(b) Objects of concentration
(c) Donation passed from student to teacher
(d) Arahants who sacrificed their lives for the service of Sangha
Solution: (a)
Learning: They are a voluminous body of literature native to India concerning the previous births of Gautama Buddha in both
human and animal form.
The future Buddha may appear as a king, an outcast, a god, an elephant—but, in whatever form, he exhibits some virtue that
the tale thereby inculcates.
In Theravada Buddhism, the Jatakas are a textual division of the Pali Canon, included in the Khuddaka Nikaya of the Sutta
Pitaka.
The term Jataka may also refer to a traditional commentary on this book.

Q.71 Which of these schools was the first to make faces and figures of the Buddha?
(a) Gandhara
(b) Mathura
(c) Pahari
(d) Malwa
Solution: (b)
Justification: While the earlier Buddhists had used only symbols to depict the Buddha, the Mathura school
became the first to make faces and figures of the Buddha.
Folklores such as the Jatakas were drawn out in long panels on rock faces. Besides the images of Buddha, which
were made in large numbers, statues of Mahavira were also produced.
Learning: Mathura, which was the centre of the indigenous school of art, was also influenced by the invasions. A
number of images from here of terracotta and red sandstone, which have definite Saka-Kushan influence, have
survived. The most famous is the headless statue of Kanishka from Mathura.
Q.72 Consider the following statements.
1. The Anguttara Nikaya, a part of Abhidhamma Pitaka, contains thematically linked discourses of Buddha.
2. The Majjhima Nikaya, a segment of Vinaya Pitaka, contains the smallest discourses given by the Buddha to his close
disciples.
Which of the above is/arecorrect?
(a) 1 only
(b) 2 only
(c) Both 1 and 2
(d) None
Solution: (d)
Justification: Both are a part of Sutta Pitaka. The word Nikaya is most commonly used in reference to the Buddhist texts of the
Sutta Piṭaka.
Statement 1: This is incorrect because Abhidhamma Pitaka is a detailed scholastic reworking of material appearing in the
Suttas, not a collection of original discourses by the Buddha himself.
Statement 2: This is incorrect because Vinaya pitaka contains rules for the Sangha, it isn’t known for hosting discourses.
Ladakh in Jammu & Kashmir.
They are raised for ultra-fine cashmere wool Pashmina, which literally translates to "Soft Gold" in Kashmiri.
The Changthangi goats have revitalized the poor economy of Changthang, Leh and Ladakh region.
Q.73 With reference to Buddhist disciples, consider the following.
1. Sariputta was a chief female disciple of Gautama Buddha.
2. Khema, who joined the Buddhist Sangha, was one of the queens of King Bimbisara.
3. Moggallana, a disciple of Buddha was known for his psychic powers.
4. King Ajatasatru of Magadha and King Prasenajit of Kosala became Buddha’s disciples.
Select the correct answer using the codes below.
(a) 1and2only
(b) 3and4only
(c) 2, 3 and 4 only
(d) 1 and 3 only
Solution: (c)
Justification:Statement 1: Sariputta was one of two chief male disciples of Gautama Buddha along with Moggallana,
counterparts to the bhikkhunis Khema and Uppalavanna, his two chief female disciples. They were to maintain the order
of monks and nuns.
Statement 2: The conversion of Khema was one of the rare cases where the Buddha used his psychic powers to make a change
in the heart of another.
Statement 3: Moggallana attained enlightenment shortly after joining the Sangha. As a teacher, he became known for his
psychic powers, which he used extensively in his teaching methods.
Statement 4: Kings like Prasenajit of Kosala and Bimbisara and Ajatasatru of Magadha too accepted the doctrines of the
Buddha and became his disciples.

Q.74 Buddhamitra was known for


1. Writing virasoliyam which is a work on Tamil Grammar
2. Erecting inscriptions on images of bodhisattvas and the Buddha near Ganges river
Which of the above is/are correct?
(a) 1 only
(b) 2 only
(c) Both 1 and 2
(d) None
Solution: (c)
Justification: Buddhamitra was a Buddhist nun from India during the Kushan Empire.
Statement 1: She is remembered because of dated inscriptions on images of bodhisattvas and the
Buddha that she erected in three cities near the Ganges river.
They mark her success in attracting money and patronage to the Sarvastivada, the sect of Buddhism
to which she belonged.
Statement 2: Virasoliyam attempts to find synthesis between Sanskrit and Tamil grammar.
There were other books written on Tamil grammar as well like Yapperungalam and
Yapperungalakkarigai by the Jain ascetic Amirtasagara.
Q.75 What was important about Shravasti in Buddhist history?
1. It was ruled by Pasenadi who was a disciple of Buddha.
2. The famous Jetavana Vihara is situated here.
3. It hosted the first sermon of the Buddha.
Select the correct answer using the codes below.
(a) 1 only
(b) 1 and 2 only
(c) 2 and 3 only
(d) 1, 2 and 3
Solution: (b)
Justification: Statement 1: About 2,500 years ago, it was one of the six largest cities of India and served as the
capital of the Kosala Kingdom, and its king was called Pasenadi, who was a disciple of Buddha.
The Buddha passed the greater part of his monastic life in Shravasti.
Statement 2: It was also a prominent trading centre and center of religious activity by Buddha.
According to the history of Buddhism, The Buddha first came to Shravasti at the urging of Sudatta, a rich
merchant who met the Buddha in Rajagir.
Sudatta invited the Buddha to Shravasti and began to look for a suitable place to build a vihara.

Q.76 According to Buddhist tradition, Maha Sammata was


(a) The first monarch of the world
(b) A character of Gautam Buddha’s previous life
(c) A compassionate Bodhisattva who renounced his life for the Sangha
(d) The priest who consecrated Buddha’s relics
Solution: (a)
Learning: Literally, the Great Elect, Maha Sammata was the first monarch of the world.
The chronicles of Theravada Buddhist tradition such as Mahavamsa and Maha Yazawin state that he
was the founder of the Sakya dynasty, to which the historical Buddha belonged.
He was the first of the eleven world monarchs named Maha Sammata, each of whom founded the
eleven dynasties that existed from the beginning to the day of the Buddha.

Q.77 In the context of Buddhist history, Mahapajapati Gotami was


(a) Composer of Buddhacharita
(b) The queen who extended the highest financial patronage to Buddhism
(c) Author of Abhidhamma Pitaka
(d) First woman to be ordained as a bhikkhuni
Solution: (d)
Learning: Initially, only men were allowed into the sangha, but later women also came to be
admitted. The Buddha’s foster mother, Mahapajapati Gotami was the first woman to be ordained as a
bhikkhuni. Many women who entered the sangha became teachers of dhamma.
The Buddha’s followers came from many social groups. They included kings, wealthy men, gahapatis,
workers, slaves and craftspeople.
Once persons get into the sangha, all were regarded as equal, having shed their earlier social
identities on becoming bhikkhus and bhikkhunis

OTHER SHAMAN TRADITIONS

Q.78 With reference to the Shramaṇa tradition, consider the following statements.
1. It was a Vedic movement in ancient India.
2. Ajivikas and Carvakas were a part of the movement.
3. Bhakti cult originated from this movement.
Select the correct answer using the codes below.
(a) 1 and 3 only
(b) 1 and 2 only
(c) 2 only
(d) 2 and 3 only
Solution: (c)
Justification: Statement 1 and 2: The term refers to several Indian religious movements parallel to but separate
from the historical Vedic religion. The shramaṇa tradition includes Jainism, Buddhism, and others such as the
Ajivikas, Ajnanas and Carvakas.
Sramaṇas held a view of samsara as full of suffering (Dukka). They practiced Ahimsa and rigorous ascetism. They
believed in Karma and Moksa and viewed rebirth as undesirable.
Vedics, on the contrary believe in the efficacy of rituals and sacrifices, performed by a privileged group of people,
who could improve their life by pleasing certain Gods.
Statement 3: Sramanism, emphasizing thought, hard work and discipline, was one of the three strands of Hindu
philosophy. The other two included Brahmanism, which drew its philosophical essence from Mimamsa.
The third and most popular strand of Indian philosophical thought revolves around the concept of Bhakti or Theism,
based on the idea of God, as understood in most parts of the world. So, Bhakti was not a part of the Sramana
tradition.

Q.79 With reference to Ajivikas, consider the following statements.


1. It is known for its Niyati doctrine of absolute determinism.
2. Followers of the sect were atheists and rejected the authority of the Vedas.
3. They did not believe in the idea of a self or Atman.
Select the correct answer using the codes below.
(a) 1 only
(b) 1 and 2 only
(c) 2 and 3 only
(d) 1 and 3 only
Solution: (b)
Justification: It was founded in the 5th century BCE by Makkhali Gosala.
S2: It is known for its Niyati doctrine of absolute determinism, the premise that there is no free will, that everything
that has happened, is happening and will happen is entirely preordained and a function of cosmic principles.
It considered the karma doctrine as a fallacy. Ajivika metaphysics included a theory of atoms similar to the
Vaisheshika school, where everything was composed of atoms, qualities emerged from aggregates of atoms, but
the aggregation and nature of these atoms was predetermined by cosmic forces.

Q.80 Lokayatas believe that


1. Everything is predetermined
2. A human being is made up of six elements
3. Death is unreal and human beings continue to survive in ethereal form after death
Select the correct answer using the codes below.
(a) 2 only
(b) 1 and 3 only
(c) 1, 2 and 3
(d) None of the above
Solution: (d)
Justification: Materialists or Lokayatas are those who believe that everything is not predetermined.
Materialist teacher Ajita Kesakambalin says that a human being is made up of the four elements.
When he dies the earthy in him returns to the earth, the fluid to water, the heat to fire, the windy to air,
and his senses pass into space.
The talk of gifts is a doctrine of fools, an empty lie. Fools and wise alike are cut off and perish. They do
not survive after death.

TERRITORIAL STATES
MAHAJANAPADAS

Q.81 Early Buddhist and Jaina texts mention certain states that were very large and
known as mahajanapadas. Consider the following statements about them.
1. Sanghas, which were smaller mahajanapadas, were forbidden to control land resources.
2. In smaller mahajanapadas, sanghas, power was shared by a number of men.
Which of the above is/are correct?
(a) 1 only
(b) 2 only
(c) Both 1 and 2
(d) None
Solution: (b)
Justification: Statement 1: While most mahajanapadas were ruled by kings, some, known as ganas or
sanghas, were oligarchies, where power was shared by a number of men, often collectively called
rajas.
Both Mahavira and the Buddha belonged to such ganas.
Statement 2: In some instances, as in the case of the Vajji sangha, the rajas probably controlled
resources such as land collectively.

Q.82 With reference to Ancient India, mahajanapada rulers imposed taxes on


(a) Crops and crafts persons only
(b) Goods, crops, herders, hunter gatherers and crafts persons
(c) Goods and royal services only
(d) Hunter gatherers and crafts persons only
Solution: (b)
Background: As they were building huge forts and maintaining big armies, they needed more
resources.
Instead of depending on occasional gifts brought by people, as in the case of the raja of the
janapadas, they started collecting regular taxes.
Learning: Taxes on crops were the most important. This was because most people were farmers.
There were taxes on crafts persons as well. These could have been in the form of labour. For example, a
weaver or a smith may have had to work for a day every month for the king.
Herders were also expected to pay taxes in the form of animals and animal produce.
There were also taxes on goods that were bought and sold, through trade.
And hunters and gatherers also had to provide forest produce to the raja.

Q.83 With reference to Matsays, one of the early states, consider the following statements.
1. They were one of the Indo-Aryan tribes of Vedic India.
2. Their rule extended to the whole of eastern and central India.
3. They were an appendage of Ashmaka dynasty.
Select the correct answer using the codes below.
(a) 1 only
(b) 2 and 3 only
(c) 3 only
(d) 1, 2 and 3
Solution: (a)
Justification: Statement 1 and 2: By the late Vedic period, they ruled a kingdom located south of the
Kurus, and west of the Yamuna River which separated it from the kingdom of the Panchalas.
It roughly corresponded to the former state of Jaipur in Rajasthan.
Statement 3: The capital of Matsya was at Viratanagari (present-day Bairat). Alwar has been a part
of Matsya region. Ahmakas were further down south, see map below.
MAGADHAN EMPIRE
Q.84 Dakshinapatha and Uttarapatha were
1. Two great highways that have connected different parts of the sub-continent since the Iron Age
2. Pali transcriptions of two different sects of Buddhism
3. Extensions of Chola Empire in Sri Lanka and Northern India respectively
4. The largest gopurams of Brihadeshwara temple made under Rajaraja I
Solution: (a)
Learning: Dakshinapatha is a historical which has been used to describe the great southern highway in
India, traveling from Magadha to Pratishthana.
Uttarapatha or the great northern road ran from Taxila in Afganisthan, through the modern Punjab up
to the western coast of Yamuna.
Following the course of Yamuna, it went southwards up to Mathura, from there it passed on to Ujjain in
Malwa and to Broach on western coast.
The most important ruler of the Satavahanas, Gautamiputra Shri Satakarni and other Satavahana
rulers were known as lords of the dakshinapatha.

IRANIAN AND MACEDONIAN INVASIONS

Q.85 Consider the following about the religious impact of the rule of the Greeks, Kushanas, Shakas
and Parthians who were called Yavanas.
1. They patronized religions like Buddhism.
2. Their coins contained images of religious Hindu figures.
Which of the above is/are correct?
(a) 1 only
(b) 2 only
(c) Both 1 and 2
(d) None
Solution: (c)
Justification: We have covered a bit on them in previous test.
They soon merged with the Indian society and adopted Indian names and inter-married.
Even their coins started carrying the images of Indian gods like Vishnu, Ganesha and Mahesha.
The fact that they had adapted to the Indian society easily may explain why foreign rulers patronised Buddhism.

Q.86 In Ancient India, a means of claiming high status for the Kings was to identify with a
variety of deities. Consider the following with reference to it.
1. Kushana rulers installed god-like colossal statues of them.
2. Some Kushanas adopted the title devaputra which implied the “son of the God”.
3. Vijayanagara kings claimed to rule on behalf of the god Virupaksha.
4. Vijayanagara rulers adopted the title ‘brahma-varna” to segment themselves as the highest castes
equivalent to God.
Select the correct answer using the codes below.
(a) 1, 2 and 3 only
(b) 2 and 4 only
(c) 3 only
(d) 1, 2, 3 and 4
Solution: (a)
Justification: Statement 1: Huge statues of Kushanas have been found installed in a shrine at Mat
near Mathura (Uttar Pradesh). Similar statues have been found in a shrine in Afghanistan as well.
Statement 2: The title devaputra was possibly inspired by Chinese rulers who called themselves sons
of heaven.
Statement 3: They indicated their close links with the gods by using the title “Hindu Suratrana”
meaning Hindu Sultan.
Q.87 In the Indian subcontinent, the first coins to bear the names and images of rulers were issued
by the
(a) Indo - Greeks
(b) Mauryas
(c) Parthians
(d) Yaudheyas
Solution: (a)
Learning: Mauryas issued punch marked coins made of silver and copper.
Indo-greeks used names of rulers on coins. First gold coins were issued by Kushanas (some sources
dispute this fact).
Coins were also issued by tribal republics of Punjab and Haryana called the Yaudheyas.
Archaeologists have unearthed several thousand copper coins issued by the Yaudheyas.
Some of the most spectacular gold coins were issued by the Gupta rulers.

Q.88 Consider the following statements.


1. Kanishka sent missionaries to Central Asia and China for the propagation of Mahayana Buddhism.
2. Kanishka patronised not only Buddhist scholars like Vasumitra, but also patronised the famous physician of
ancient India, Charaka.
Which of the above is/are correct?
(a) 1 only
(b) 2 only
(c) Both 1 and 2
(d) None
Solution: (c)
Justification: Statement 1: This is because Mahayana Buddhism came in vogue during Kanishka’s rule. It was
established in the Fourth Buddhist Council convened by him where matters relating to Buddhist theology and
doctrine were discussed.
In Mahayana Buddhism, the Buddha came to be worshipped with flowers, garments, perfumes and lamps.
Statement 2: Asvagosha and Nagarjuna were other important scholars patronized by him.

Q.89 He was a staunch follower of Brahmanism. Buddhist sources often refer him as a persecutor of Buddhism. He
is?
(a) Pushyamitra
(b) Menander
(c) Somapura
(d) Dharmaraksita
Solution: (a)
Learning: He belonged to the Sunga dynasty. The rule of the Sungas was important because they defended the
Gangetic valley from foreign invasions.
In the cultural sphere, the Sungas revived Brahmanism and horse sacrifice. They also promoted the growth of
Vaishnavism and the Sanskrit language.
“In short, the Sunga rule was a brilliant anticipation of the golden age of the Guptas” - This statement can be asked as
a question by UPSC in Mains.
Clarification: While Buddhist sources refer him as a persecutor of Buddhism. But there is enough evidence to
show that Pushyamitra patronised Buddhist art. During his reign, the Buddhist monuments at Bharhut and
Sanchi were renovated and further improved.

AGE OF MAURYAS

Q.90 Consider the following statements.


1. Asoka’s Dhamma contained the ideal of a ‘welfare state’.
2. Ashoka propagated Dhamma with the help of officials known as Dhamma Mahamattas.
3. Prohibition of animal sacrifices was a part of Asoka’s Dhamma.
Select the correct answer using the codes below.
(a) 1 and 2 only
(b) 2 and 3 only
(c) 1 and 3 only
(d) 1, 2 and 3 only
Solution: (d)
Justification: Statement 2: Efficient organization of administration in the direction of social welfare and
maintenance of constant contact with people through the system of Dhammayatras was an important feature of
Ashoka’s dhamma.
The Dhamma Mahamatras were asked by Asoka to take steps against unjust imprisonment. Remission of sentences
is also mentioned in Asoka’s inscriptions. But, he did not advocate non- punishment. So, 3 is wrong.
Other features of Ashoka’s dhamma were:
Service to father and mother, practice of ahimsa, love of truth, reverence to teachers and good treatment of
relatives.
Prohibition of animal sacrifices and festive gatherings and avoiding expensive and meaningless ceremonies and
rituals.
Humane treatment of servants by masters and prisoners by government officials.
Consideration and non-violence to animals and courtesy to relations and liberality to Brahmins.

Q.91 Among the following, which of these sites were related to the Mauryan Kingdom?
1. Sopara
2. Meerut
3. Sanchi
4. Dhauli
Select the correct answer using the codes below.
(a) 1, 2 and 3 only
(b) 2 and 4 only
(c) 1 and 3 only
(d) 1, 2, 3 and 4
Solution: (d)
Justification: Originating from the kingdom of Magadha in the Indo-Gangetic Plain in the eastern side of the Indian
subcontinent, the empire had its capital city at Pataliputra (modern Patna).
The empire was the largest to have ever existed in the Indian subcontinent, spanning over 5 million square
kilometres
At its greatest extent, the empire stretched to the north along the natural boundaries of the Himalayas, to the east
into Assam, to the west into Balochistan (southwest Pakistan and southeast Iran) and the Hindu Kush mountains of
what is now Afghanistan.
Q.92 Ashoka built several chaityas, stupas and pillars at Dhauli, Odisha because
1. He converted the largest number of subjects to Buddhism at Dhauli.
2. He wanted to overcome the remorse of the Kalinga bloodshed.
3. A second Ashokan capital was established here.
4. It was home to the largest number of Buddhist viharas in India.
Select the correct answer using the codes below.
(a) 1, 3 and 4 only
(b) 2 only
(c) 2 and 4 only
(d) 1 and 3 only
Solution: (b)
Justification: Dhauli hill is presumed to be the area where the Kalinga War was fought.
The rock-cut elephant above the Edicts is the earliest Buddhist sculpture of Odisha.
Ashoka had a special weakness for Dhauli, where the battle was fought.
The Daya River here is said to have turned red with the blood of the many deceased after the battle, and enabled
Ashoka to realize the magnitude of horror associated with war.
Thereafter Ashoka repented of the violence which he had done, and converted to Buddhism.
He expressed his remorse, and his intention to govern the kingdom according to the principles of his new faith, in a series
of rock-cut edicts that he caused to be inscribed on over 100 monuments throughout his vast kingdom.

Q.93 Consider the following statements about muventars.


1. They were the dhamma officials appointed by the crowned monarch of historical South India.
2. Ashokan edicts mention the muventars.
Which of the above is/are correct?
(a) 1 only
(b) 2 only
(c) Both 1 and 2
(d) None
Solution: (b)
Justification: The ruling monarchs of three chiefdoms of the Cheras, Cholas and Pandyas together were called
muventars.
The Cholas ruled over the fertile Kaveri basin with Uraiyur as its capital and the important port was Puhar or
Kaveripattinam. The most important ruler of this kingdom was Karikal.
The Pandyas ruled over the pastoral and littoral parts with Madurai as capital and Korkai as important port.
Nedunjeliyan was the most famous king.
The Cheras controlled the hilly region in the west with Vanji or Karur as its capital and Muciris as the well known
port.
The important ruler of the kingdom was Udeyinjiral. The king was called the ventan, and this was not only the time
of great kings but also great chieftains who were subordinates of the kings. These chieftains were divided into
two—velir and non-velir. There were three kinds of chiefdoms in Tamilakam— Velir or bigger chiefs, Vedar or the
biggest chiefs and Kizar who were headman of a small village (ur) bound the kingship.
Ashokan edicts also mention the muventars.
Q.94 Kulasangha, mentioned by Kautilya in Arthashastra, is
(a) Recruitment manual for new officers
(b) A system of government
(c) A forum of the members of royal family
(d) Procession led by a group of Kingdoms to assert authority over a certain region
Solution: (b)
Background and learning: North Indian royal titles (e.g. adhiraja) gained more and more currency in the south in
this period but the early south Indian kings seem to have derived their
legitimation from tribal loyalties and the network of their respective clan.
This sometimes implied the division of power among many members of the clan. The Chera kingdom of the
southwest coast (Kerala) must have been such a large-scale family enterprise.
Kautilya has referred to this system of government in his Arthashastra; he called it “kulasangha” and thought that it
was quite efficient. Among the Pandyas and Cholas the monarch seems to have played a more important role.
This was particularly true of the Chola king, Karikala, who ruled over a relatively large area around AD 190 after he
had vanquished a federation of the Pandyas and Cheras. Even about 1,000 years later the Chola rulers still referred
to this great ancestor and they attributed to him the building of dikes along the banks of the Kaveri and the
decoration of Kanchipuram with gold.
Karikala’s policy was obviously aimed at extending the territorial base of the Cholas at the expense of the other
tribal principalities, but this policy

Q.95 The three tribal principalities which are mentioned in Ashoka’s inscriptions of the third century BC and in
some Kharavela’s inscription of the first century BC can be
(a) Pachyatanas, Nibhikas and Rualias
(b) Rashtrakutas, Pallavas and Dundas
(c) Cholas, Pandyas and Cheras
(d) Mundakas, Niralayas and Senas
Solution: (c)
Learning: The early history of the ‘far south’ is the history of the these three tribal principalities which are
mentioned in Ashoka’s inscriptions of the third century BC, in some brief Tamil inscriptions of the second century
BC and in Kharavela’s inscription of the first century BC.
About this period in South India: The chronicles of Sri Lanka contain many references to the fights between the
kings of Sri Lanka and the kings of southern India.
Sangam literature was named after the ‘academies’ (sangam) of Madural and its environs where poets worked
under the patronage of the Pandya kings.
The famous Tarnil grammar, Tolkappivam, is considered to belong to the beginning of this whole period and the
great Tamil epic poem.
Shilappatikaram, to its very end, perhaps even to the fifth or sixth centuries AD.
The Chera kingdom of the southwest coast (Kerala) must have been such a large-scale family enterprise gaining
legitimacy from tribal loyalties and the network of their respective clan.

Q.96 Hathigumpha Inscription in a Jain cave at Udaigiri hills can potentially give us the details of the reign
of
(a) Kharavela
(b) Vasudeva
(c) Chintakara
(d) Nandivela
Solution: (a)
Learning: A kingdom which rose to a position of importance after the Mauryas was Kalinga.
Kalinga included modern Orissa and parts of Northern Andhra. Its most important ruler was Kharavela.
The Hathigumpha Inscription in a Jain cave at Udaigiri hills would give us a detailed account of his reign, but
unfortunately it is not easily decipherable.
It is definitely known that he was a great administrator as well as a brave warrior. He carried out works of piety and
public utility, like building roads and gardens.
Q.97 Ashokan inscriptions remain valuable sources for the study of Asoka and the Mauryan Empire. This is
because
1. His wooden pillars heralded a new architectural era because most of the monuments before his period were
made of stone.
2. Pillar edicts give a summary of his efforts to promote the Dhamma within his kingdom and outside.
3. They often deal with instructions given to his officials which is informative of Mauryan polity.
Select the correct answer using the codes below.
(a) 2 and 3 only
(b) 1 and 3 only
(c) 1 and 2 only
(d) 2 only
Solution: (a)
Answer Justification:
Justification: Statement 1: The monuments before the period of Asoka were mostly made of wood and therefore
perished. The use of stone started from the time of Asoka. Even of the numerous monuments of Asoka, only a few
have remained.
His palace and monasteries and most of his stupas have disappeared. The only remaining stupa is at Sanchi.
Statement 2: The state and spread of Dhamma gives crucial information about the social life at the time of Ashoka.
Statement 3: Ashoka often gave instructions to his Dhamma mahamahattas to spread Dhamma. These instructions,
their style etc were recorded on the edicts.
Learning: Ashokan inscriptions (first deciphered by James Princep) are written in Pali language and in some places
Prakrit was used. The Brahmi script was employed for writing. In the northwestern India Asokan inscriptions were
found in Karoshti script. There are fourteen Major Rock Edicts. The two Kaling Edicts are found in the newly
conquered territory. The major pillar Edicts were erected in important cities. There are minor Rock Edicts and minor
pillar Edicts.

Q.98 The Mauryan state had a well-organized civil service, this is evident from
1. Civilservants called Amatyas who looked after the day-to-day administration were selected based on merit.
2. Adyakshas officers helped control the retail and wholesale prices of goods and ensured their steady supply in the state.
3. All officials were interviewed and appointed directly by the emperor.
Select the correct answer using the codes below.
(a) 1and2only
(b) 2and3only
(c) 3 only
(d) 2 only
Solution: (a)
Justification: A council of ministers called Mantriparishad assisted the king in administrative matters.
It consisted of Purohita, Mahamantri, Senapati and Yuvaraja.
Statement 1: Amatyas were similar to the IAS officers of independent India. The method of selection of Amatyas was
elaborately given by Kautilya.
Statement 2: Department of Commerce and Industry had controlled the retail and wholesale prices of goods and tried to ensure
their steady supply through its officers called Adyakshas. It also controlled weights and measures, levied custom duties
and regulated foreign trade.
Statement 3: Selection process was different for different officials.
Learning:Asoka also appointed DhammaMahamatras tosupervise the spreadof Dhamma.Even the military service was
well organized. The Mauryan army was well organized and it was under the control of Senapati. The salaries were paid in cash.
Kautilya refers to the salaries of different ranks of military officers.

Q.99 Which of these ancient scripts generally writes in left to right direction?
(a) Brahmi
(b) Kharosthi
(c) Harappan
(d) All of the above
Solution: (a)
Learning: Brahmi: is one of the oldest writing systems used in South and Central Asia from the 1st millennium BCE.
MostmodernIndianscriptshavedevelopedfromtheBrahmiscriptoverhundredsofyears. Harappan:Iravatham
MahadevanestablishedthattheHarappanscriptisfromrighttoleft.
Kharosthi: is an ancient script used in ancient Gandhara (primarily modern-day Afghanistan and Pakistan) to write the
Gandhari Prakrit and Sanskrit

Q.100 With reference to Ancient India, in the northern part of the country, the village headman was
known as the
(a) Gramabhojaka
(b) Agrahara
(c) Dhammsurna
(d) Sentokwara
Solution: (a)
Learning: Usually, men from the same family held the position for generations or via hereditary. The
grama bhojaka was often the largest landowner. Generally, he had slaves and hired
workers to cultivate the land. Besides, as he was powerful, the king often used him to collect taxes
from the village. He also functioned as a judge, and sometimes as a policeman.
Apart from the gramabhojaka, there were other independent farmers, known as grihapatis, most of
whom were smaller landowners.

Q.101 Consider the following statements.


1. Most Ashokan inscriptions were in the Greek language while those in the northwest of India
subcontinent were in Aramaic and Prakrit.
2. Ashokan inscriptions were written in both Prakrit and Brahmi scripts.
Which of the above is/are correct?
(a) 1 only
(b) 2 only
(c) Both 1 and 2
(d) None
Solution: (b)
Justification: Statement 1: A large number of them were in Prakrit, but in the NW direction one could
find Aramaic and Greek languages.
Statement 2: The Aramaic and Greek scripts were used for inscriptions in Afghanistan.
James Prinsep, an officer in the mint of the East India Company, deciphered Brahmi and Kharosthi,
two scripts used in the earliest inscriptions and coins. This gave a new direction to investigations
into early Indian political history.

Q.102 The taking of Census was regular during the Mauryan period. Consider the following with reference to it.
1. Village officials were to record the caste and occupation of people.
2. Livestock population in each house wastracked.
3. Municipal officials tracked the movement of population both foreign and indigenous.
4. The data collected from official census were cross checked by the spies.
Select the correct answer using the codes below.
(a) 1 and 2 only
(b) 2, 3 and 4 only
(c) 3 and 4 only
(d) 1, 2, 3 and 4
Solution: (d)
Justification: This is a lateral explanation, since the details of census have already been covered in the question.
We will touch upon other important aspects of Mauryan administration.
Other aspects: Kautilya mentions the existence of both civil and criminal courts.
The chief justice of the Supreme Court at the capital was called Dharmathikarin. There were also subordinate courts
at the provincial capitals and districts under Amatyas.
Different kinds of punishment such as fines, imprisonment, mutilation and death were given to the offenders.
Torture was employed to extract truth. Police stations were found in all principal centres.
Both Kautilya and Asokan Edicts mention about jails and jail officials. Remission of sentences is also mentioned in
Asoka’s inscriptions.

Q.103 Lauria Nandangarh is famous for


1. A large excavated Stupa belonging to the Ashokan period
2. A monolithic iron pillar of the Mauryan period that was used to propagate Dharma to Sri Lanka
Which of the above is/are correct?
(a) 1 only
(b) 2 only
(c) Both 1 and 2
(d) None
Solution: (a)
Justification: Lauriya Nandangarh is a historical site located in West Champaran district of Bihar. Remains of
Mauryan period have been found here.
On excavation, Nandangarh turned out to be stupendous Stupa with a polygonal or cruciform base; with its missing
dome which must have been proportionately tall, the Stupa must have been one of the highest in India
This thirty-two-foot-tall column has an almost fifty-ton seated lion capital placed on its top, an engineering feat
worth admiring.
The bull capital from Rampura is also another fine example of Mauryan sculpture.
The village draws its name from a pillar (laur) of Ashoka standing there and the stupa mound Nandangarh (variant
Nanadgarh) about 2 km south-west of the pillar.

Q.104 Ashoka’s policy of “Dhamma Vijaya” meant that


1. Dhamma cannot be spread to another country except by voluntary submission and acceptance of its subjects
2. Every empire should fund welfare projects in neighbouring empires.
3. No empire should try to conquest another empire.
Select the correct answer using the codes below.
(a) 2 only
(b) 1 and 3 only
(c) 3 only
(d) None of the above
Solution: (d)
Justification: The mindless destruction of life and property in Kalinga war shattered Ashoka so greatly that he
vowed never to wage any war again.
Instead he adopted the policy of Dhamma Vijaya that is conquest through dhamma.
In his thirteenth major Rock Edict, Asoka states that true conquest is by piety and virtue.
Such a decision taken by a king, who lived in an era where military might was the measure of power, earned him a
unique place in history.
According to a Buddhist tradition, Asoka sent Buddhist missions to regions such as Sri Lanka and Central Asia.
Buddhism spread to different parts of the world and although it is no longer a major force in India today, yet it
continues to be popular in Sri Lanka and the Far Eastern countries.

Q.105 Which of these features is found absent from the Mauryan Empire?
1. Appointment of provincial governors purely based on meritocracy
2. Presence of a village administration
3. Municipal administration
4. Committees that looked after the manufacture of goods
Solution: (a)
Justification: Option A: The provincial governors were mostly appointed from the members of royal
family. They were responsible the maintenance of law and order and collection of taxes for the
empire.
Option B: Village administration was in the hands of Gramani and his official superior was called
Gopa who was in charge of ten or fifteen villages.
Option C: Both Kautilya and Megasthanes provided the system of Municipal administration.
Arthasastra contains a full chapter on the role of Nagarika or city superintendent. His chief duty was
to maintain law and order.
Option D: Megasthenes refers to the six committees of five members each to look after the
administration of Pataliputra. These committees looked after: 1. Industries 2. Foreigners 3.
Registration of birth and deaths 4. Trade 5. Manufacture and sale of goods 6. Collection of sales tax.

Q.106 How was the Kharoshthi script written?


1. Right to left
2. Left to right
3. Upside down
4. In capital
Select the correct answer using the codes below.
(a) 1 and 3 only
(b) 1 only
(c) 2 and 4 only
(d) 3 and 4 only
Solution: (b)
Justification: Kharosthi is a syllabic alphabet - each letter has an inherent vowel /a/. Other vowels are indicated
using diacritics.
It was written from right to left in horizontal lines.
The Kharosthi alphabet was invented sometime during the 3rd century BC and was possibly derived from the
Aramaic alphabet. It was widely used in northwest India and central Asia until the 4th century AD.
Unlike the Brahmi script, which was invented at around the same time and spawned many of the modern scripts
of India and South East Asia, Kharosthi had no descendants.
Kharoshti was deciphered by James Prinsep and others around the middle of the 19th century. Since then further
material has been found and the script is now better understood.

Q.107 With reference to Ashokavadana, consider the following statements.


1. It is a text written in Pali language that espouses Ashokan dhamma.
2. It was written by Ashoka himself to praise the city of Mathura and his associated initiatives.
Which of the above is/are correct?
(a) 1 only
(b) 2 only
(c) Both 1 and 2
(d) None
Solution: (d)
Justification: Statement 1: It is an Indian Sanskrit-language text that describes the birth and reign
of the Maurya Emperor Ashoka.
It is one of the avadana texts contained in the Divyavadana, an anthology of several Buddhist
legends and narratives.
Statement 2: According to Jean Przyluski, the text was composed by the Buddhist monks of the
Mathura region, as it highly praises the city of Mathura, its monasteries and its monks
Learning: According to Ashokavadana, Asoka distributed portions of the Buddha’s relics to every
important town and ordered the construction of stupas over them.
It contains legends as well as historical narratives, and glorifies Ashoka as a Buddhist emperor
whose only ambition was to spread Buddhism far and wide.
POST MAURYANS

Q.108 Consider the following statements.


Assertion (A): Kushanas banned the use of gold coins taken from their empire on the silk route.
Reason (R): Kushanas could not control the Silk route and were at a loss from its trade.
In the context of the above, which of these is correct?
(a) A is correct, and R is an appropriate explanation of A.
(b) A is correct, but R is not an appropriate explanation of A.
(c) A is correct, but R is incorrect.
(d) Both A and R are incorrect.
Solution: (d)
Justification: The best-known of the rulers who controlled the Silk Route were the Kushanas, who ruled over
central Asia and north-west India around 2000 years ago.
Their two major centres of power were Peshawar and Mathura. During their rule, a branch of the Silk Route
extended from Central Asia down to the seaports at the mouth of the river Indus, from where silk was shipped
westwards to the Roman Empire.
The Kushanas were amongst the earliest rulers of the subcontinent to issue gold. So, both A and R are wrong.

Q.109 “Kanishka must have conquered the greater part of the Gangetic plain”, the evidence of this
statement comes from
1. Coins of Kanishka are found in places like Mathura, Sravasti and Benares.
2. His coins exhibit only Buddhist images, and not Hindu gods, a cult that was popular then in the Gangetic plain.
Which of the above is/are correct?
(a) 1 only
(b) 2 only
(c) Both 1 and 2
(d) None
Solution: (a)
Justification: Statement 1: At the time of his accession his empire included Afghanistan, Gandhara, Sind and
Punjab. Subsequently he conquered Magadha and extended his power as far as Pataliputra and Bodh Gaya. This
evidence comes from his coins that are found in the gangetic plains.
Statement 2: Kanishka embraced Buddhism in the early part of his reign. However, his coins exhibit the images of
not only Buddha but also Greek and Hindu gods. So, 2 is wrong.

Q.110 An important item of trade from Ancient India, it was highly valuable in Roman Empire so much that it
was known as ‘black gold’. What does the item refer to?
(a) Crude oil
(b) Coal
(c) Dark Garnet stones
(d) Pepper
Solution: (d)
Learning: Frequently referred to as "black gold" in ancient India, on account of its demand and trade value
mainly due to its high trade value, the Zamorin ruler of Malabar coast had a flourishing trade centre for export of
pepper and other spices in Kerala. The main mercantile traders were Arabs and people from Middle East. The
early Roman Empire got direct access to the Malabar Coast in India and its range of exotic spices after their
conquest of Egypt in 30 BC. The prices of pepper were extremely high in the Middle Ages and the trade was
completely dominated by the Romans.
SOUTHERN KINGDOMS

Q.111 The history of the Vakatakas is largely known from inscriptions and texts like the Puranas. They
became a major power in which of these regions of India?
(a) Deccan region
(b) Eastern region
(c) Kurukshetra region
(d) Kashmir region
Solution: (a)
Learning: They initially established themselves in the Vindhyan region, and later extended their power southwards,
eventually becoming a major political power in the Deccan. Vakataka rule lasted from the mid-3rd to the late 5th/
early 6thcenturies AD.
The founder of the Vakataka dynasty was Vindhyashakti I. The second Vakataka king was Pravarasena I who seems
to have extended the empire southwards into Vidarbha and adjoining areas of the Deccan.
His capital was Kanchanaka (modern Nachna). Pravarasena I cemented an important political alliance by marrying
his son, Gautamiputra, to the daughter of the Naga king, Bhavanaga of the Bharashiva family, which was prominent
in eastern UP and adjoining areas.
The political gains of this alliance were immense. Pravarasena is mentioned in the Puranas and inscriptions as
performing several vajapeya, vajimedha and ashvamedha sacrifices, accompanied by the distribution of lavish
gifts. He was the only Vakataka king with the imperial title of samrat.

Q.112 Consider the following matches:


1. Anantavarman: Kerala
2. Jagannatha: Bengal
3. Mahodayapuram: Orissa
4. Lilatilakam: Kangra
Select the correct answer using the codes below.
(a) 1, 2 and 3 only
(b) 2 and 4 only
(c) 1 and 3 only
(d) None of the above
Solution: (d)
Justification: Statement 1: Anantavarman Chodaganga Deva was a ruler of the Eastern Ganga
dynasty which ruled southern part of Kalinga.
Statement 2: Literally meaning "Lord of the Universe", it is a deity worshipped in regional traditions
of Hinduism and Buddhism in India and Bangladesh. Correct match is Odisha.
Statement 3: Correct match is Tamil Nadu. The Later Cheras (the Kulasekharas) ruled from
Mahodayapuram (now Kodungallur) on the banks of River Periyar and fought numerous wars with
their powerful neighbours such as the Cholas and Rashtrakutas.
Statement 4: Lilatilakam is a text in Manipravalam dealing with grammar and poetics written in
Manipravalam. The correct match would be Kerala.

Q.113 In ancient Southern India, kadaisiyar and adimai were used to refer to
(a) Ordinary ploughmen
(b) Landless labourers, including slaves
(c) Tax officials
(d) Large landowners
Solution: (b)
Learning: Society was segregated even in ancient India.
There were at least three different kinds of people living in most villages in the southern and northern parts of the ancient
Indian subcontinent.
In the Tamil region, large landowners were known as vellalar, ordinary ploughmen were known as uzhavar, and landless
labourers, including slaves, were known as kadaisiyar and adimai.
In the northern region, this distinction was made in terms of land, political position and assets held.

SATAVAHANAS

Q.114 Which of these were Kingdoms that ruled in the Deccan region of India?
1. Vakatakas
2. Chalukyas
3. Dwamathas
4. Chamangalas
Select the correct answer using the codes below.
(a) 1 and 2 only
(b) 2, 3 and 4 only
(c) 1 and 4 only
(d) 2 and 4 only
Solution: (a)
Justification: Satavahanas controlled the Deccan for a long time. After their decline, many small
kingdoms came up in the Deccan.
The first one among them was that of the Vakatakas, who tried to build a strong state, but they did not
last long.
After the Vakatakas came the Chalukyas of Vatapi and Kalyani. Pulakesin was a powerful ruler of the
Chalukya dynasty.
The Chalukyas kept fighting with the Rashtrakutas (towards the north) and the Pallavas (towards the
south). The Chalukya rule came to an end in 753 A.D. when the Rashtrakutas defeated them

Q.115 Ptolemy, a Greek writer, mentions many important ports in the Deccan. Among them, the greatest port of
the Satavahanas in Western Deccan was
(a) Kalyani
(b) Ganjam
(c) Satakarni
(d) Pulamayi
Solution: (a)
Learning: Vashishtaputra Pulamayi extended the Satavahana power up to the mouth of the Krishna river.
He issued coins on which the image of ships was inscribed. They reveal the naval power and maritime trade of the
Satavahanas.
Their greatest port was Kalyani on the west Deccan. Gandakasela and Ganjam on the east coast were the other
important seaports.
The last great ruler of Satavahanas was Yajna Sri Satakarni

Q.116 The Satavahanas patronized Buddhism and Brahmanism. The evidence of this is clear from
the fact that
1. They built several chaityas and viharas.
2. They prohibited Buddhist monks from acquiring land to help in their spiritual pursuit.
3. They persecuted Brahmanism.
4. They patronized the Prakrit language and literature.
Select the correct answer using the codes below.
(a) 1, 2 and 3 only
(b) 1 and 4 only
(c) 1, 3 and 4 only
(d) 2 and 3 only
Solution: (b)
Justification: Statement 1: Vashishtaputra Pulamayi repaired the old Amaravathi stupa. Their
architecture in Nagarjunakonda was also notable.
Statement 2: They also made grants of villages and lands to Buddhist monks.
Statement 3: Brahmanism was revived by the Satavahanas along with the performance of
asvamedha and rajasuya sacrifices.
Statement 4: Hala’s Sattasai is an excellent piece of Prakrit literature.

Q.117 The earliest inscriptions recording royal land grants were issued during
1. Guptas
2. Satvahanas
3. Pallavas
4. Hoysalas
Solution: (b)
Learning: In early India, land possessed great value and the gift of an estate was a marker of status.
While the earliest inscriptions recording royal land grants were issued during Satavahana rule, the
practice truly grew in scope from the fourth century CE.
By the 5th-6th centuries, ruling dynasties across the subcontinent, as well as their subordinates
and feudatories were all engaged in making such grants.
Land grant charters were usually engraved on sheets of hammered copper. They are described as
tamrapatta, a ‘plate of copper’ or tamrashasana, ‘an order in copper’.

THREE EARLY KINGDOMS (CHOLA, PANDYA AND CHERAS)

Q.118 The Chola ruler Karikala was known for


1. Allying with the Cheras and the Pandays to create the “Great Southern Empire”
2. Getting the Grand Anicut constructed which is still in use as one of the oldest water structures in the world
Which of the above is/are correct?
(a) 1 only
(b) 2 only
(c) Both 1 and 2
(d) None
Solution: (b)
Justification: Statement 1: He was the most important ruler of this kingdom. According to the
Porunararruppaṭai, Karikala Chola fought a great Battle of Venni in which both Pandyan and Cheran kings suffered
a defeat.
Statement 2: The Grand Anicut, also known as the Kallanai was built by Karikala and is considered one of the
oldest water-diversion or water-regulator structures in the world which is still in use
Karikala has been credited with many welfare activities. He got may canals dug so that water from the river
Cauvery could be used for irrigation purposes.
Karikala patronised works of literature and art. He was a follower of the Vedic religion

Q.119 In Medieval India, Mahamandaleshvara was a term often used for


(a) Samantas who gained power and wealth to overpowered their Lords
(b) Kings who undertook an overseas expedition
(c) Priests who performed the most Ashvamedha sacrifices
(d) Patrons of temples with several deities
Solution: (a)
Justification: By the seventh century there were big landlords or warrior chiefs in different regions of the subcontinent.
Existing kings often acknowledged them as their subordinates or samantas.They were expected to bring gifts for their
kings or overlords, be present at their courts and provide them with military support.
As samantas gained power and wealth, they declared themselves to be maha-samanta, mahamandaleshvara (the great
lord of a “circle” or region) and so on.
Sometimes they asserted their independence from their overlords.

Q.120 Which of these justify the robust and benevolent system of administration that Cholas had?
1. Chola kings undertook royal tours to increase the efficiency of the administration.
2. All legal professions were exempt from tax liabilities.
3. All lands were carefully surveyed and classified for assessment of revenue.
4. During the hard times, there were remission of taxes.
Select the correct answer using the codes below.
(a) 1 and 4 only
(b) 1, 3 and 4 only
(c) 2 and 3 only
(d) 2 and 4 only
Solution: (b)
Justification: Statement 1: There were regular drills with officials.
Statement 2 and 3: There were tolls and customs on goods taken from one place to another, various kinds of
professional taxes, dues levied on ceremonial occasions like marriages and judicial fines. Moreover, the land
revenue department was well organized. It was called as
puravuvarithinaikkalam. The residential portion of the village was called ur nattam. These and other lands such as
the lands belonging to temples were exempted from tax.
Statement 4: During the hard times, there were remission of taxes and Kulottunga I became famous by abolishing
tolls and earned the title – Sungam Tavirtta Cholan. The main items of government expenditure were the king and
his court, army and navy, roads, irrigation tanks and canals.

Q.121 If you compare the socio-economic life of Cholas and Sangam age, what is/are the chief
similarities in these periods with respect to the position of women in society?
1. The practice of Sati was present.
2. Devadasi system was abolished.
3. Women were not allowed to write or study literature.
4. Love marriage was forbidden.
Select the correct answer using the codes below.
(a) 1 only
(b) 2, 3 and 4 only
(c) 3 and 4 only
(d) 1, 2 and 4 only
Solution: (a)
Justification: Statement 1 and 4: Love marriage was a common practice in the Sangam age.
Women were allowed to choose their life partners.
However, the life of widows was miserable. The practice of Sati was prevalent in the higher strata of
society.
Even in Cholas, practice of ‘sati’ was prevalent among the royal families.
Statement 2: The devadasi system or dancing girls attached to temples emerged during the Chola
period. There is evidence that they were often subjected to harassment by priets and officials.
Statement 3: Women poets like Avvaiyar, Nachchellaiyar, and Kakkaipadiniyar flourished in the
sangam period and contributed to Tamil literature.

Q.122 With reference to Chola Education and Literature, consider the following:
1. Temples and mathas served as the only educational centres.
2. Not only Vedas and Epics, but subjects like mathematics and medicine were also taught.
3. Endowment of lands was made to run educational centres.
Select the correct answer using the codes below.
(a) 1 and 2 only
(b) 2 and 3 only
(c) 3 only
(d) 1, 2 and 3
Solution: (b)
Justification: Statement 1: During this period, besides the temples and mathas as educational
centres, several educational institutions also flourished. The inscription at Ennayiram,
Thirumukkudal and Thirubhuvanai provide details of the colleges existed in these places.
Statement 2 and 3: The development of Tamil literature reached its peak during the Chola period. And
a lot of it came from the contribution of education and literature to the enrichment of this culture.
Mathematics and medicine studies showed that Kings encouraged logical and intellectual pursuits
and were not centred entirely on religious studies.

Q.123 Vanavaramban, Vanavan, Kuttuvan, Irumporai and Villavar were famous titles assumed by
(a) Chera kings
(b) Sangam poets
(c) Thiruvalluvar, a tamil philosopher and poet
(d) Temple priests in Southern Indian dynasties
Solution: (a)
Learning: Hereditary monarchy was the form of government during the Sangam period.
The Chera kings assumed titles like Vanavaramban, Vanavan, Kuttuvan, Irumporai and Villavar, the
Chola kings like Senni, Valavan and Killi and the Pandya kings Thennavar and Minavar.
Each of the Sangam dynasties had a royal emblem – carp for the Pandyas, tiger for the Cholas and
bow for the Cheras.

Q.124 Consider the following statements with reference to the South Indian dynasties.
1. Arikkamedu was a seaport during the Sangam period.
2. Uraiyur was famous for pearls.
3. Lord Murugan was the primary deity of the Sangam age.
Select the correct answer using the codes below.
(a) 1 only
(b) 2 and 3 only
(c) 3 only
(d) 1, 2 and 3
Solution: (d)
Justification: Statement 1: External trade was carried between South India and the Greek kingdoms.
The port city of Puhar became an emporium of foreign trade, as big ships entered this port with
precious goods.
Other ports of commercial activity include Tondi, Musiri, Korkai, Arikkamedu and Marakkanam.
Statement 2: Uraiyur was an ancient Chola city with a fortress and city wall on the southern banks of
the river Kaveri. It was known for pearls.
Statement 3: The worship of Murugan was having an ancient origin and the festivals relating to
God Murugan was mentioned in the Sangam literature. He was honoured with six abodes known
as Arupadai Veedu.

SANGAM LITERATURE

Q.125 Consider the following statements.


1. The form of government during the Sangam period in South India was a Federal republic.
2. Each of the Sangam dynasties had a royal Emblem.
3. The head of the Government in the Sangam age was assisted by a Council of Ministers.
Select the correct answer using the codes below.
(a) 1 and 2 only
(b) 2 and 3 only
(c) 1 and 3 only
(d) 1, 2 and 3
Solution: (b)
Justification: Statement 1: Hereditary monarchy was the form of government during the Sangam period. The
king also used to take the advice of his minister, court-poet and the imperial court or avai.
S2: Each of the Sangam dynasties had a royal Emblem - carp for the Pandyas, tiger for the Cholas and bow for the
Cheras.
The king was assisted by a large body of officials who were divided into five councils. They were ministers
(amaichar), priests (anthanar), military commanders (senapathi), envoys (thuthar) and spies (orrar). The military
administration was also efficiently organized during the Sangam Age.

Q.126 Which of these is NOT one of the so called ‘Five Epics’ (‘the five great poems’) attributed to
the Sangam age?
(a) Jivakachintamani
(b) Silappadikaram
(c) Valayapathi
(d) Pattupattu
Solution: (d)
Learning: Modern scholarship use the term ‘Sangam Literature’ for only those works in verse (prose is of much
later origin), which are comprised in the Ettutogai (Eight collections), Pattupattu (Ten songs) and
Patinenkilkanakku (The Eighteen Minor Works), which are judged to have been produced in that order during the
period A.D 150-250.
The so called ‘Five Epics’ (‘the five great poems’) include Jivakachintamani, Silappadikaram, Manimekalai,
Valayapathi and Kundalakesi. These are assigned much later dates.
These poetical works describe about the social, religious, economic and political conditions of Tamilakam with the
focus on the cities like Madurai, Puhar (Poompuhar/ Kaveripattinam), Vanji (Karur) and Kanchi.

Q.127 In Sanskrit sources, the usage of the words "Yona", "Yauna", "Yonaka", "Yavana" or "Javana" appears
repeatedly, and particularly in relation to the
(a) The southern tip of India that in present day borders the Mannar coast near Sri Lanka
(b) Vedic Ritual sacrifices that were supposed to be performed only by the Emperor
(c) The art of building esoteric mandalas in places of spiritual importance
(d) Greek kingdoms which neighboured and sometimes occupied the Punjab region
Solution: (d)
Learning: These terms appears repeatedly, and particularly in relation to the Greek kingdoms which neighboured
or sometimes occupied the Punjab region over a period of several centuries from the 4th century BCE to the first
century CE.
Examples are the Seleucid Empire, the Greco-Bactrian Kingdom and the Indo-Greek Kingdom.
The Yavanas are mentioned in detail in Sangam literature epics such as Pattinappalai, describing their brisk trade
with the Early Cholas in the Sangam period.

Q.128 Among the following, the Sangam texts most prominently mention which of the following ports?
(a) Comari (Kanyakumari)
(b) Puhar (Kaveripattinam)
(c) Poduke (Arikamedu)
(d) Sopatma
Solution: (b)
Learning: Sangam texts prominently mention only the ports of Musiri, Puhar (Kaveripattinam) and Korkai, the
three great ports of the three great rulers of the times.
However, the Periplus refers to the ports of Tondi, Musiri and Comari (Cape Comorin / Kanyakumari), Colchi
(Korkai), Poduke (Arikamedu) and Sopatma.
According to Periplus there were three types of vessels in use in South India. These included small coasting
vessels, large coasting vessels and ocean-going ships. There is also the mention of large vessels called Colandia
sailing from the Tamil Coast to the Ganges.
Q.129 What is the relationship between these two Tamil Epics - Silappatikaram and Manimekalai?
(a) Manimekalai is a poetic composition of the prosaic content of Silappatikaram.
(b) Both bear the same author and the same date of compilation.
(c) Silappatikaram is a commentary on Manimekalai.
(d) Manimekalai is a sequel to Silappatikaram
Solution: (d)
Learning: These are two of the five epics of Tamil Literature that are considered in high regard by the Tamil
community.
The Silappatikaram is a poetic rendition with details of Tamil culture; its varied religions; its town plans and city
types; the mingling of different people; and the arts of dance and music. A poet- prince from Kodungallur near
Kochi (part of ancient Tamilakam, now in modern Kerala), referred to by the pseudonym Ilango Adigal, is credited
with this work.
The epic revolves around Kannagi, who having lost her husband to a miscarriage of justice at the court of the
Pandyan Dynasty, wreaks her revenge on his kingdom.
Manimekalai, by the poet Chithalai Chathanar, is a poem in 30 cantos. Its story is a sequel to another of the Five
Great Epics, Silappatikaram, and tells the story of the daughter of Kovalan and Madhavi, who became a Buddhist
Bikkuni.

Q.130 The theory of the three Sangams establishes that, chronologically, these were
(a) Successive
(b) Contemporary
(c) Held at a Single meeting
(d) Held at a Single court organized by a Tamil monarch
Solution: (a)
Learning: The Sangam period extended from roughly 400 BC to 200 AD (early Chola period before the
interregnum), when the earliest extant works of Tamil literature were written.
The traditional accounts of Iraiyanar Ahapporul mention that there were three Sangams (I, II and III) held, which
flourished for 9990 years at frequent intervals.
These were attended by over 8598 scholars. Sage Agastyar was the founding father. The Ahapporul commentary
also mentions about their successive order and the deluges occurring during the intervals between them.
These Sangams or academies were patronized by 197 Pandyan kings. According to the tradition, of the three
successive Sangams the first two belong to prehistory. All the three were held in the capital of the Pandyas.
As the capital was shifted from time to time, old Madurai was the headquarters of the first Sangam, and the
second academy was held at Kapatapuram. Both these centres were washed away by the sea during successive
deluges. The third Sangam was located in modern Madurai.

Q.131 The Sangam texts mention prominently the ports of


1. Musiri
2. Puhar (Kaveripattinam)
3. Sopatma
4. Korkai
Select the correct answer using the codes below.
(a) 1, 2 and 4 only
(b) 2 and 3 only
(c) 2 only
(d) 1, 3 and 4 only
Solution: (a)
Background: The Tamils of the Sangam age had trading contacts with the Mediterranean world
(Greece and Rome), Egypt, China, Southeast Asia and Sri Lanka.
The literary works like Silappadikaram, Manimekalai and Pattinappalai frequently refer to the contact with the
Greek and Roman traders. This period marked the height of the Indo-Roman trade.
The discovery of coin hoards at many places also attests this fact.
Justification: The Sangam texts mention prominently only the ports of Musiri, Puhar (Kaveripattinam) and Korkai,
the three great ports of the three great rulers of the times.
However, the Periplus (foreign accounts) refers to the ports of Tondi, Musiri and Comari (Cape Comorin /
Kanyakumari), Colchi (Korkai), Poduke (Arikamedu) and Sopatma.
According to Periplus there were three types of vessels in use in South India. These included small coasting
vessels, large coasting vessels and ocean-going ships. There is also the mention of large vessels called Colandia
sailing from the Tamil Coast to the Ganges.

Q.132 The ‘Aimperunkulu’ mentioned in Silappadikaram was a/an


(a) Assembly of elders
(b) Land grant given to the underprivileged classes
(c) Council of Ministers
(d) Religious shrine of Kannagi
Solution: (c)
Learning: The administrative machinery of the age was described by the Sangam texts.
The policies of the king were controlled by a system of checks and balances in the councils. Silappadikaram refers
to the two types of councils —Aimperunkulu and Enperayam.
The aimperunkulu or the council of five members was the council of the ministers. Theenperayam or the great
assembly (perayam) consisted of 8 members (government officers).
This worked as an administrative machinery of the state. These two assemblies that of the Five and that of the
Eight functioned as administrative bodies, though their function was generally advisory in character. However,
their advice was rarely rejected by the king.
Their important function was judicial though the aimperunkulu seems to have been solely in chaRige of it as
described by Maduraikkanji.
It is important to note that in spite of all the glory attached to the ancient king, the ethos of Indian administration
has been in the direction of limited or popular monarchy.
This can be observed in South India from very early times even more than in the north and each followed its own
model of administration. Every local unit, however small and in whatever corner it was situated, was
administered by a local assembly.
The avai and the manram are the terms used for this unit in Sangam works. Such assembly is commonly referred
to as arankuravaiyam, which were known for its just decision. These can be taken to be the forerunner of our
modern panchayat.

Q.133 Tolkappiyam refers to four castes namely arasar, anthanar, vanigar and vellalar.
Consider the following about the roles of these castes.
1. The ruling class was called arasar.
2. Anthanars were the untouchables.
3. Vanigars were responsible for protecting the empire.
4. The vellalas were priests.
Select the correct answer using the codes below.
(a) 1 and 4 only
(b) 2, 3 and 4 only
(c) 1 only
(d) 2 and 3 only
Solution: (c)
Justification: This division of caste refers to the Sangam age.
Statement 2: Anthanars played a significant role in the Sangam polity and religion. Statement 3 and 4: Vanigars
carried on trade and commerce. The vellalas were agriculturists.
Learning: Other tribal groups like Parathavar, Panar, Eyinar, Kadambar, Maravar and Pulaiyar were also found in
the Sangam society. Ancient primitive tribes like Thodas, Irulas, Nagas and Vedars lived in this period.

Q.134 With reference to the Sangam age, Panar and Viraliyar were
(a) Temple guards
(b) Bull fighters
(c) Singing bards
(d) Charity donors
Solution: (c)
Learning: Poetry, music and dancing were popular among the people of the Sangam age. Liberal donations were
given to poets by the kings, chieftains and nobles. The royal courts were crowded with singing bards called Panar
and Viraliyar. They were experts in folk songs and folk dances. The arts of music and dancing were highly
developed. A variety of Yazhs and drums are referred to in the Sangam literature.Dancing was performed by
Kanigaiyar. Koothu was the most popular entertainment of the people.

Q.135 Consider the following statements.


1. Manimegalai is a Tamil epic that criticizes Buddhism while investigating the strengths of contemporary Hindu
traditions.
2. Silappatikaram, produced at Sangam, is a poetic rendition with details of Tamil culture.
Which of the above is/are correct?
(a) 1 only
(b) 2 only
(c) Both 1 and 2
(d) None
Solution: (b)
Justification: The corpus of Sangam literature includes Tolkappiyam, Ettutogai, Pattuppattu, Pathinenkilkanakku,
and the two epic – Silappathigaram and Manimegalai.
Statement 1: The aim of the author of Manimegalai, Seethalai Saathanar, was to compare Buddhism favourably
with the other prevailing religions in South India in order to propagate Buddhism.
He criticizes Hinduism, and exposes the weaknesses of the other contemporary Indian religions, while praising the
Buddha's Teaching, the Dharma, as the most perfect religion. So, 1 is wrong.
Statement 2: Regarded as one of the great works of Tamil literature, the Silappatikaram is a poetic rendition with
details of Tamil culture; its varied religions; its town plans and city types; the mingling of different people; and the
arts of dance and music.

Q.136 With reference to Ancient India, who introduced the Pattini cult in Tamilnadu?
(a) Cheran Senguttuvan
(b) Rajaraja II
(c) Pulakesin I
(d) Narasinghvarmana
Solution: (a)
Justification: He belonged to 2nd century A.D
Senguttuvan introduced the Pattini cult or the worship of Kannagi as the ideal wife in Tamil Nadu.
The stone for making the idol of Kannagi was brought by him after his Himalayan expedition.
The consecration ceremony was attended by many princes including Gajabhagu II from Sri Lanka.
Learning: His younger brother was Elango Adigal, the author of Silappathigaram.Among his military
achievements, his expedition to the Himalayas was remarkable. He defeated many north Indian
monarchs.

Q.137 Sangam poems mention the term muvendar. It was used for
(a) Land allotted to temples by Kings
(b) Administrative arrangements made to felicitate scholars of Tamil
(c) Heads of three ruling dynasties of Southern India
(d) Classical commentary on Vedas
Solution: (c)
Learning: This is a Tamil word meaning three chiefs, used for the heads of three ruling families, the Cholas,
Cheras, and Pandyas, who became powerful in south India around 2300 years ago.
Each of the three chiefs had two centres of power: one inland, and one on the coast.
Of these six cities, two were very important: Puhar or Kaveripattinam, the port of the Cholas, and
Madurai, the capital of the Pandyas.

Q.138 Anthanars and thuthars in the Sangam monarchy were


(a) Priests and envoys
(b) Spies and military commanders
(c) Custom officials and seaport traders
(d) Ministers and army officials
Solution: (a)
Learning: The imperial court or avai was attended by a number of chiefs and officials.
The king was assisted by a large body of officials who were divided into five councils.
They were ministers (amaichar), priests (anthanar), military commanders (senapathi), envoys (thuthar) and spies
(orrar).
Each ruler had a regular army and their respective Kodimaram (tutelary tree).
Land revenue was the chief source of state’s income while custom duty was also imposed on foreign trade.
The Pattinappalai refers to the custom officials employed in the seaport of Puhar.
GUPTA PERIOD

Q.139 In the Gupta administration, Sandivigraha were those officials who


(a) Assisted the king in maintaining a close contact with the provincial administration
(b) Looked after the city administration
(c) Were princes and later became ministers of provinces
(d) Administered foreign affairs
Solution: (d)
Learning: A high official called was mentioned in the Gupta inscriptions known as Sandivigraha, most
probably minister for foreign affairs.
The king was assisted in his administration by a council consisting of a chief minister, a Senapati or
commanderin- chief of the army and other important officials.
The king maintained a close contact with the provincial administration through a class of officials called
Kumaramatyas and Ayuktas.
Provinces in the Gupta Empire were known as Bhuktis and provincial governors as Uparikas. They were
mostly chosen from among the princes.

Q.140 The Gupta kings patronized the classical Sanskrit literature. With reference to several scholars
and their composition in this period, consider the following matches:
1. Kalidas: Ritusambara
2. Vishakhdutta: Vikramorvashi
3. Shudraka: Raghuvamsha
Select the correct answer using the codes below.
1. 1 and 2 only
2. 1 only
3. 2 and 3 only
4. 3 only
Solution: (b)
Justification: Statement 1: Poet Kalidas wrote many beautiful poems and plays. His works in Sanskrit are
considered the gems of Literature. He wrote passionate plays and poems.
His wonderful skill is exhibited in his poem Meghaduta, Ritusambara. Kumar Sambhavam and Raghuvamsha.
His plays are Abhijan Shakuntalam, Vikramorvashi and Malvikaganimithram.
Statement 2: Vishakhdutta was another great play writer of this period. He wrote two great historical plays
like- Mudra Rakshas and Dev Chandra Gupta.
Statement 3: Shudraka wrote an exciting play Mrichchha Katikam or the Toy Cart. It is a great source of
socio-cultural conditions of that time.
Learning: Harisena was also among the great poets and play writers of the Gupta period. He wrote poems
praising the valour of Samudra Gupta. It is inscribed on Allahabad pillar.

Q.141 Most of the Gupta kings encouraged


1. Brahmanism
2. Religious education of women
3. Equal treatment of all varnas
Select the correct answer using the codes below.
(a) 1 only
(b) 2 and 3 only
(c) 1 and 3 only
(d) 1 and 2 only
Solution: (a)
Justification & Learning: Statement 1: The Brahmins occupied the top ladder of the society. It had two
branches - Vaishnavism and Saivism.
Most of the Gupta kings were Vaishnavaites. They performed Aswamedha sacrifices. Statement 2 and 3: The
practice of untouchability had slowly begun during this period.
The position of women had also become miserable during the Gupta period. They were prohibited from
studying the religious texts like the Puranas.
The subjection of women to men was thoroughly regularized. But it was insisted that they should be
protected and generously treated by men

Q.142 Who among the following made very significant contributions in finding the exact value of π (pi)?
1. Aryabhatta and Madhava
2. Bhaskaracharya
3. Budhayana
4. Gyandev and Aryabhatta
Solution: (a)
Justification: According to Professor Ramasubramanian from IITB, one of the reasons why Indians would
have wanted to know the value of pi, would have been for the construction of altars for havana, each
household was supposed to have three of them—one square, one circular, and third semi-circular one.
The only condition was that the area of all the three altars be the same. This would have made it necessary
for the society to be aware of the value of pi.

Q.143 Consider the following about the position of women during the Gupta period.
1. State asserted that women had no right to be protected by men.
2. They were prohibited from studying the religious texts like the Puranas.
3. The practice of Swyamvara was given up.
4. Early marriage for girls was suggested by Manusmriti.
Select the correct answer using the codes below.
(a) 2, 3 and 4 only
(b) 1 and 4 only
(c) 1 and 2 only
(d) 2 and 3 only
Solution: (a)
Justification: Statement 1: The subjection of women to men was thoroughly regularized. But it was insisted
that they should be protected and generously treated by men.
Other statements: Position of women had become miserable, alongwith that of the lower castes, during the
Gupta period.
They were prohibited from studying the religious texts. Manusmriti suggested the early marriage for girls.
Learning: In the sphere of religion, Brahmanism reigned supreme during the Gupta period.
Most of the Gupta kings were Vaishnavaites.
During the Gupta period, the caste system became rigid.
The practice of untouchability was slowly entrenched during this period.

Q.144 Bhartṛhari, 5th Century CE, is a Sanskrit writer to whom are normally ascribed two influential
Sanskrit texts, one of which is Vakyapadiya. It deals with
(a) Sanskrit grammar and linguistic philosophy
(b) Social hierarchy in Hinduism
(c) Divine poetry written by laities
(d) Theories of creation and dissolution
Solution: (a)
Learning: Vakyapadiya, on Sanskrit grammar and linguistic philosophy, a foundational text in the Indian
grammatical tradition, explaining numerous theories on the word and on the sentence, including theories which
came to be known under the name of Sphoṭa.
Another work is the Satakatraya. It is a work of Sanskrit poetry, comprising three collections of about 100 stanzas
each; it may or may not be by the same author who composed the two mentioned grammatical works.

Q.145 The prashastis (panegyric) of royal inscriptions, dating to the Gupta empire and its contemporaries,
offer several important details to historians, such as royal genealogies and political events. However,
they are not completely reliable because
1. They generally report political victories rather than defeats.
2. Inscriptions of different dynasties often make conflicting claims about the same event or battle.
Which of the above is/are correct?
(a) 1 only
(b) 2 only
(c) Both 1 and 2
(d) None
Solution: (c)
Justification: They are important because:
The titles and descriptions of kings in inscriptions reflect prevailing power hierarchies and ideals of kingship.
Royal land grant inscriptions represent important socio-economic processes of their time and provide information
on administrative structures and agrarian relations.
Donative inscriptions of private individuals provide information on social history and the sources of patronage of
religious establishments.
Statement 1: There is a popular saying that history is written by the victorious. If this is so, many kingdoms did not
reveal a candid war account to often protect and enhance their image. This does not portray correct history.
Statement 2: This renders a correct historical estimation really difficult.

Q.146 Consider the following officials in the Ancient Indian period and their roles in the Gupta empire.
1. Uparika: Village Headman
2. Mahapratihara: Deputy Minister
3. Mahabaladhikrita: Army commander-in-chief
Select the correct answer using the codes below.
(a) 1 only
(b) 3 only
(c) 2 and 3 only
(d) 1, 2 and 3
Solution: (b)
Justification: Statement 1: The Gupta empire was divided into provinces called bhuktis, administered by
governors called uparikas, directly appointed by the king.
The fact that the uparika had the title maharaja in three of the Damodarpur plates of the Gupta era indicates his
high administrative status and rank. Goptri was another title for the governor.
Saurashtra was an important province of the Gupta empire. The Junagarh inscription of Parnadatta and
Chakrapalita provides details of the repair to the Sudarshana lake that had flooded its banks by Chakrapalita, the
son of Skandagupta’s minister and governor of Saurashtra, Parnadatta.
This inscription highlights the provincial government’s role in initiating the repair of waterworks, as also the
practice of the delegation of official responsibilities from father to son.
Statement 2 and 3: Officials connected with the royal establishment included the mahapratihara (chief of the
palace guards) and the mahabaladhikrita (army commander-in-chief). The standard term senapati does not occur
in Gupta inscriptions but is mentioned in some Vakataka records. The top layer of the administrative structure
also included amatyas and sachivas who were executive officers in charge of various departments.
Learning: The provinces of the Gupta empire were divided into districts called vishayas under officers called
vishayapatis. The Damodarpur copper plates of the reign of Kumaragupta I show that the vishayapati was helped
in his administrative duties by prominent town members.
Administrative units below district level included settlement clusters called pithi, patta, bhumika, pathaka or
petha. Village elders played an important role in village matters as indicated by the Damodarpur copper plate of
Budhagupta’s reign. A Sanchi inscription of Chandragupta II’s time mentions a pancha-mandali, which may have
been a corporate village body.

Q.147 Consider the following statements.


1. Gupta kings issued large numbers of gold coins known as dinaras, with representation of the King on the
obverse and on the reverse side that of a deity.
2. Samudragupta’s coins represent him in poses that indicate his military skills, with occasional obverse displays of
goddesses.
Which of the above is/are correct?
(a) 1 only
(b) 2 only
(c) Both 1 and 2
(d) None
Solution: (c)
Justification: Statement 1: Dinaras bore the names and epithets of kings.
The obverse generally had a representation of the king and the reverse an image of a deity. A notable feature is
the names of the rulers written vertically on the obverse.
Rulers like Chandragupta II, Kumaragupta I, Skandagupta and Budhagupta also issued silver coins. The obverse
had the king’s portrait, often accompanied by a date; the reverse had a motif (like a garuda or peacock)
surrounded by a circular legend. Copper coins of the Guptas are rare.
Statement 2: Samudragupta’s coins represent him in poses that indicate his military skills—for instance, as an
archer wielding a bow and arrow, with a battleaxe and a dwarf looking up at him, or trampling and killing a tiger.
The ‘ashvamedha type’ shows a sacrificial horse standing before a decorated yupa or sacrificial post. The ‘queen’,
in this type, is supposedly holding a ‘needle’, which was used to pierce the horse.
The ‘standard type’, which is the most frequent, shows Samudragupta with a long staff in his left hand and making
offerings to the fire with his right.
A coin showing Chandragupta I and his queen standing face-to-face is attributed either to Chandragupta I or to
Samudragupta.
The obverse of Samudragupta’s coins sometimes depicts the goddess Ardoksho or a goddess sitting on an
elephant-headed fish. In other cases, there is a standing female figure, perhaps a queen, holding a flywhisk.
It should be noted that the ‘standard type’ and the ‘Ardoksho type’ show continuing Kushana influence. Later
issues, though, are more Indianized.
Q.148 The Allahabad inscription is often associated with which of the following rulers?
(a) Allaudin Khilji
(b) Samudragupta
(c) Ranjit Singh
(d) Harshvardhana
Solution: (b)
Learning: It provides very specific details of Samudragupta’s military achievements and conquests from which it is
possible to delineate the extent of his empire. It also presents the Gupta Empire as the centre of a complex
network of political relationships.
Samudragupta’s initial military campaigns were directed towards areas that lay immediately beyond his control.
For e.g. Line 14 of the inscription refers to Samudragupta capturing a king of the Kota family, while Line 21 talks of
him violently destroying several kings of Aryavarta and subordinating all the forest kings.
The territories of the kings mentioned in these two lines seem to have been annexed, leading to an extension of
the Gupta empire over the Ganga-Yamuna valley up to Mathura and Padmavati (Pawaya) in the west.

Q.149 What argument(s) would you advance to justify that the position of women had become miserable
during the Gupta period?
1. Women were prohibited from studying religious texts.
2. The practice of Swyamvara was given up and Manusmriti suggested early marriage for girls.
Which of the above is/are correct?
(a) 1 only
(b) 2 only
(c) Both 1 and 2
(d) None
Solution: (c)
Justification: Statement 1: This ensured the domination of men over women.
Statement 2: Swaymvara was the practice of women choosing their own partners by way of a
competition/selection. When this was given up, women lost an important right. Manusmriti (law book) further
entrenched patriarchal rules to worsen the position of women.
Learning: During the Gupta dynasty period the male members of a family were entitled to inherit property left
behind by their fathers. However, such right of inheritance was not given to women or girls. The power of
inheritance enabled the male members of a family to exercise dominance over the women.
Q.150 With reference to Ancient India, Ashtangasamgraha written by Vagbhata deals with
(a) Astrology
(b) Medicine
(c) Law
(d) Economics
Solution: (b)
Learning: In the field of medicine, Vagbhata lived during this period. He was the last of the great medical trio of
ancient India. The other two scholars Charaka and Susruta lived before the Gupta age. Vagbhata was the author
Ashtangasamgraha (Summary of the eight branches of medicine). The Sushruta and Charaka texts differ in one
major aspect, with Sushruta Samhita providing the foundation of surgery, while Charaka Samhita being primarily a
foundation of medicine.

Q.151 Consider the following:


1. Temple at Deogarh near Jhansi
2. Giant copper statue of Buddha originally found at Sultanganj
3. Delhi Iron pillar
4. Bagh caves paintings
The above belong to periods of which of these dynasties of India?
(a) Peshwas
(b) Guptas
(c) Paramara
(d) Western Ganga
Solution: (b)
Learning: The Delhi Iron pillar of the Gupta period is still free from rust though completely
exposed to sun and rain for so many centuries. The paintings of the Gupta period are seen at Bagh caves near
Gwalior. Moreover, the mural paintings of Ajantha mostly illustrate the life of the Buddha as depicted in the
Jataka stories. There was little influence of Gandhara style on Gupta art. But the beautiful statue of standing
Buddha at Mathura reveals a little Greek style. The Buddha statue unearthed at Saranath was unique piece of
Gupta art.

Q.152 Many Chinese Buddhist pilgrims visited Indian subcontinent to visit places associated with the life of
the Buddha. Who among the following is NOT one of them?
(a) Fa Xian
(b) Xuan Zang
(c) I-Qing
(d) Alan Chen
Solution: (d)
Learning: Three such Chinese Buddhist pilgrims are best-known.
Fa Xian, who came to the subcontinent about 1600 years ago, Xuan Zang (who came around 1400 years ago) and
I-Qing, who came about 50 years after Xuan Zang are the famous one. They have documented previous socio-
economic, political and religious details of Indian life then.

Q.153 Chandragupta II, a ruler of the Gupta Empire, assumed the famous title of ‘Sakari’. What did the title imply?
(a) Destroyer of Sakas
(b) Lord of the world
(c) Well Wisher of masses
(d) Devotee of Shankara
Solution: (a)
Learning: The greatest of the military achievements of Chandragupta II was his war against the Saka satraps of western
India.
Rudrasimha III, the last ruler of the Saka satrap was defeated, dethroned and killed.
His territories in western Malwa and the Kathiawar Peninsula were annexed into the Gupta Empire.
After this victory he performed the horse sacrifice and assumed the title Sakari, meaning, ‘destroyer of Sakas’. He also
called himself Vikramaditya.
Q.154 There are plenty of source materials to reconstruct the history of the Gupta period. Some of the
renowned material, in this context, includes
1. Puranas throw light on the royal genealogy of the Gupta kings.
2. DevichandraguptamandMudhrakshasamprovideinformationregardingtheriseoftheGuptas
3. Meherauli Iron Pillar Inscription mention their conquests.
4. Fahien’swritingsgives anaccountofthesocial,economicandreligious conditionoftheempire.
Select the correct answer using the codes below.
(a) 1 and 4 only
(b) 2, 3 and 4 only
(c) 1 and 3 only
(d) 1, 2, 3 and 4
Solution: (d)
Learning: S1 and S2 mention commonly known sources.
Statement 3: The Meherauli Iron Pillar inscription mentions the extensive conquests of Chandragupta I. He is
considered to be the founder of the Gupta era which starts with his accession in A.D. 320.
The Allahabad Pillar inscription mentions that Samudragupta defeated twelve rulers in his South Indian Expedition.
Statement 4: We will cover a separate question on Fahien’s accounts. It is important to note that his main interest of visiting
India was religious.

Q.155 He worked on the approximation for pi and found out the area of a right-angled triangle. He also worked
on the motions of the solar system and calculated the length of the solar year correctly. A legendary
mathematician of the Gupta Era, he is
(a) Brahmagupta
(b) Aryabhatta
(c) Varahamihira
(d) Yativṛṣabha
Solution: (b)
Learning: He wrote Aryabhattiya at the age of 23 years and later, Arya-Siddhanta. Aryabhatiyam sketches his
mathematical, planetary, and cosmic theories.
Aryabhatta deduced that earth is a rotating sphere: the stars do not move, it is the earth that rotates.
Aryabhatta also deduced that: “The moon eclipses the sun, and the great shadow of the earth eclipses the moon.”

Q.156 The transformation of the ancient brahmanical faith into modern Hinduism can be regarded as
the most distinguishing feature from the Gupta period onwards. Consider the following about it.
1. Buddhism received greater royal patronages than in received earlier.
2. Bhagavata Sect of Hinduism was abandoned and a more scientific creed was evolved by the Guptas.
3. Vishnu emerged was considered as the god of devotion and came to be represented as the preserver of
dharma.
Select the correct answer using the codes below.
(a) 1 and 2 only
(b) 2 and 3 only
(c) 3 only
(d) 1 and 3 only
Solution: (c)
Justification: Statement 1: Buddhism no longer received as much royal patronage as it did earlier because
Brahmanism had come to the forefront.
Statement 2: The Puranas were finally compiled in this age. Vishnu emerged as the god of devotion and came to
be represented as the preserver of dharma.
Numerous legends gathered around him and a whole Purana called Vishnu Purana was compiled in his honour.
Similarly a law book called the Vishnusmriti was also named after him.
Statement 3: The Gupta rulers especially provided filling to the Bhagavata Sect of Hinduism. They called them
selved Bhagavatas, worshipped Lord Vishnu performed Asvamedha yajnas, gave large donations to brahmanas,
and built many temples.

Q.157 Fahien’s account on the Gupta administration provides useful information about the empire.
As per his accounts
1. Imposing a fine was a common punishment.
2. There was no spy system.
3. Roads were not safe for travellers due to frequent thefts.
4. People were not free to move and their movements were heavily controlled by the state.
Select the correct answer using the codes below.
(a) 1 and 2 only
(b) 3 and 4 only
(c) 1 only
(d) 2, 3 and 4 only
Solution: (a)
Justification: Statement 2 and 3: He mentioned that people were generally prosperous and the crimes were
negligible.
There was no state interference in the individual’s life. Punishments were not severe. The administration was so
efficient that the roads were kept safe for travellers, and there was no fear of thieves.
Fahien had also appreciated the efficiency of the Gupta administration as he was able to travel without any fear
throughout the Gangetic valley.
Statement 3 and 4: He characterises the Gupta administration as mild and benevolent. There were no restrictions
on people’s movements and they enjoyed a large degree of personal freedom.

Q.158 Pancha Siddhantika, a work of Varahamihira, deals with


(a) Geographical domains of India
(b) Astronomical systems
(c) Occult and mysticism
(d) Principles of social living
Solution: (b)
Learning: He was also a great authority on astrology. In this context, he composed Pancha Siddhantika, about the
five astronomical systems. Along with Aryabhatta, his contribution is considered seminal in Ancient India.
His work Brihadsamhita is a great work in Sanskrit literature. It deals with a variety of subjects like astronomy,
astrology, geography, architecture, weather, animals, marriage and omens.
His Brihadjataka is considered to be a standard work on astrology.

Q.159 Consider the following about the visit of the famous Chinese traveller, Fahien.
1. He visited India during the reign of Chandragupta II.
2. He stayed for over twenty years in India.
3. He came to India by sea route.
4. His main purpose of visiting India was to study its political and social system.
5. He studied Sanskrit in Patliputra.
Select the correct answer using the codes below.
(a) 2, 3 and 4 only
(b) 1, 2 and 5 only
(c) 1 and 5 only
(d) 1, 2, 4 and 5 only
Solution: (c)
Justification: Statement 1 and 2: Out of his nine years stay in India, he spent six years in the Gupta empire.
Statement 3: He came to India by the land route through Khotan, Kashgar, Gandhara and Punjab.
He visited Peshawar, Mathura, Kanauj, Sravasti, Kapilavastu, Kusinagara, Pataliputra, Kasi and Bodh Gaya among
other places.
He returned by the sea route, visiting on the way Ceylon and Java.
Statement 4: The main purpose of his visit was to see the land of the Buddha and to collect Buddhist manuscripts
from India.
Statement 5: He stayed in Pataliputra for three years studying Sanskrit and copying Buddhist texts.

Q.160 The institution of Samanta finds mention for the first time in epigraphs of northern India
dating to the 6th century. It is most closely associated with the origin and growth of which of
these systems in India?
(a) Feudalism
(b) Bonded Labour
(c) State ownership of land
(d) Standing army system
Solution: (a)
Justification: Options B, C and D existed much prior to 6th century, so they can’t be the answer.
The institution is considered to belong properly to the Gupta Empire, however, some vague evidence is found in
South India as well.
The Samanta in South-India was used to mean a vassal to an emperor. In North-India, the earliest use of the term
in a similar sense was in Bengal where someone was described as Samanta-Chudamanih (best among feudatories)
of the imperial Guptas.
The Samanta vassal provided military support to the Monarch and governed over a portion of a territory.
Learning: The term Indian feudalism is used to describe zamindar, jagirdar, sardar, mankari, deshmukh, chaudhary
and samanta.
In this system, a nobility gave the emperor military services in exchange of land.
There was a separate class of labourers or peasants who were provided military protection in exchange of labour
and share of their produce. They were allowed to live on the lord’s land.

Q.161 Consider the following with reference to the Fa Hien’s account of India.
1. During his visit, stupa veneration was not practiced in India.
2. He frequently mentions the name of Chandragupta II in his works.
3. As per him, Buddhism was established mainly in the Gangetic valley.
4. He was looking for better copies of Buddhist books than were available in China at that time.
Select the correct answer using the codes below.
(a) 2 and 3 only
(b) 4 only
(c) 3 only
(d) 1, 2, 3 and 4
Solution: (b)
Justification: Statement 4: Between 399 and 414 CE, the Chinese monk Fa Hien undertook a trip via Central Asia
to India seeking better copies of Buddhist books than were currently available in China.
Statement 2: As far as the Gupta empire is concerned, although his account is valuable in many respects, he did
not mention the name of Chandragupta II. He was not interested in political affairs. His interest was primarily
religion.
Statement 3: According to him, Buddhism was in a flourishing condition in the north-western India but in the
Gangetic valley it was in a state of neglect. He refers to the Gangetic valley as the ‘land of Brahmanism’.
Statement 1: He indicates clearly the importance of the seven precious substances for Buddhist worship, the
widespread practice of stupa veneration, and his aquaintance with several of the jataka tales about the previous
lives of the Buddha Sakyamuni.

Q.162 In the gupta empire , Paramabhattaraka was a


(a) Minister for foreign affairs
(b) Title adopted by Gupta Kings
(c) Council of Peace and conflict
(d) In charge of granaries
Solution: (b)
Learning: According inscriptions, the Gupta kings assumed titles like Paramabhattaraka, Maharajadhiraja,
Parameswara, Samrat and Chakravartin.
This is a lateral explanation, and we will go into other details of Gupta administration.
Other details: The king was assisted in his administration by a council consisting of a chief minister, a
Senapati or commander in-chief of the army and other important officials.
A high official called Sandivigraha was mentioned in the Gupta inscriptions, most probably minister for foreign
affairs.
The king maintained a close contact with the provincial administration through a class of officials called
Kumaramatyas and Ayuktas. Provinces in the Gupta Empire were known as Bhuktis and provincial governors as
Uparikas.
Q.163 With reference to the views of travellers about caste discrimination in India, consider the
following statements.
1. Chinese Buddhist monk Fa Xian wrote that untouchables had to sound a clapper in the streets so that
people could avoid seeing them.
2. Korean pilgrim, Xuan Zang observed that executioners and scavengers were forced to live outside the city.
Which of the above is/are correct?
(a) 1 only
(b) 2 only
(c) Both 1 and 2
(d) None
Solution: (a)
Justification: Statement 2: Xuan Zang was of Chinese origin, not Korean, so 2 is wrong.
They observed this not only on the social conditions but also the rules laid down in texts like Manusmriti
such as lower castes had to: live outside the village or wear clothes of the dead etc.

Q.164 In Ancient India, shrenis served which of these purposes?


1. Provided training to crafts persons
2. Procured raw material for merchants
3. Served as financial institutions for money deposits
4. Supported religious institutions by donations
Select the correct answer using the codes below.
(a) 1 only
(b) 2, 3 and 4 only
(c) 1 and 3 only
(d) 1, 2, 3 and 4
Solution: (d)
Justification: Many crafts persons and merchants now formed associations known as shrenis.
These shrenis of crafts persons provided training, procured raw material, and distributed the finished
product.
Then shrenis of merchants organised the trade.
Shrenis also served as banks, where rich men and women deposited money.
This was invested, and part of the interest was returned or used to support religious institutions such as
monasteries.

Q.165 Some of Bhaskara's contributions to mathematics include


1. A proof of the Pythagorean theorem
2. Solutions of quadratic, cubic and quartic indeterminate equations
3. Derivation of the Hero’s theorem for calculating a triangle’s area
Select the correct answer using the codes below.
(a) 1 and 2 only
(b) 1 and 3 only
(c) 2 and 3 only
(d) 3 only
Solution: (a)
Justification: Statement 1: In Lilavati, solutions of quadratic, cubic and quartic indeterminate equations are explained.
The rules he gives are (in effect) the same as those given by the Renaissance European mathematicians of the 17th
century
A cyclic Chakravala method for solving indeterminate equations of the form ax2 bx c = y was given by him.
The solution to this equation was traditionally attributed to William Brouncker in 1657, though his method was more
difficult than the chakravala method.
Statement 3: No such work has been produced by him.
POST GUPTAS

Q.166 Which of the following urban centres in India that were thriving in the Pre-Kushana period started declining
from the post-Gupta period?
1. Purushapura
2. Kanauj
3. Ahichchhatra
4. Rajghat
Select the correct answer using the codes below.
(a) 1 and 2 only
(b) 1, 2, 3 and 4
(c) 1 and 4 only
(d) 2 and 3 only
Solution: (c)
Justification: Most historians agree that early historic urban settlements underwent a decline.
Thus Chattopadhyaya, an eminent historian, holds, “the decline was geographically widely distributed and since this
observation is based on a study of archaeological sequences at a number of Early Historic sites, both of the northern and
southern India, the chronology of the decline of this urban phase is not a matter of speculation”.
Some of the centres are shown here
NEW KINGDOMS
HARSHA AND HIS TIMES

Q.167 From the Harshacharitra, one can find that the officer Samanta had five duties. Which of the
following was one of them?
1. Render military aid to the emperor
2. Administer justice to the needy and deserving
3. Manage revenue collection from agricultural fields
4. Coordinate the foreign relations of the Empire
Solution: (a)
Learning: The institution of Samanta finds mention for the first time in epigraphs of northern India dating to
the 6th century century.
The institution is considered to belong properly to the Gupta Empire and is closely associated with the origin
and growth of feudalism in India.
The five duties include (apart from the one mentioned above):
Paying yearly tributes to the emperor.
Paying homage to the emperor in person.
Defeated Samantas had to offer their sons and minor princes to the Emperor so that they are groomed in the
imperial traditions and grow to be loyal to the emperor.
Perform administrative and judicial functions in times of peace.

Q.168 Consider the following statements.


1. Harsha prohibited the use of animal food in his kingdom and punished those who killed any living being.
2. Harsha gave away his enormous wealth as gifts to the members of all religious sects during the Allahabad Conference.
Which of the above is/arecorrect?
(a) 1 only
(b) 2 only
(c) Both 1 and 2
(d) None
Solution: (c)
Justification: Statement 1: In his early life, Harsha was a devout Saiva but later he became an ardent Hinayana Buddhist. Hiuen Tsang
converted him to Mahayana Buddhism. This led him to shed eating meat and enforce the same as well.
Statement 2: Hiuen Tsang mentions in his account about the conference held at Allahabad, known as Prayag.
It was the one among the conferences routinely convened by Harsha once in five years. Harsha gave away his enormous wealth as
gifts to the members of all religious sects.
According to Hiuen Tsang, Harsha was so lavish that he emptied the treasury and even gave away theclothesandjewelshewas
wearing.Hisstatementmightbeoneofadmiringexaggeration.

Q.169 Hiuen Tsang portray the social life in the times of Harsha. Consider the following with reference to it.
1. He mentions that the Sudras were forbidden from agriculture.
2. As per his documentation, the dead could not be subjected to water burial.
3. Dowry system was practiced in the empire.
Select the correct answer using the codes below.
(a) 1 only
(b) 2 and 3 only
(c) 3 only
(d) 1 and 2 only
Solution: (c)
Justification: The fourfold division of the society – Brahmin, Kshatriya, Vysya and Sudra – was prevalent.
Statement 1: The Brahmins were the privileged section of the society and they were given land grants by
the kings. Hiuen Tsang mentions that the Sudras practiced agriculture.
Statement 2: Hiuen Tsang mentions three ways of disposal of the dead – cremation, water burial and
exposure in the woods.
Statement 3: The position of women was not satisfactory. The institution of Swyamvara (the choice of
choosing her husband) had declined.
Remarriage of widows was not permitted, particularly among the higher castes. The system of dowry had
also become common. The practice of sati was also prevalent.
Learning: Trade and commerce had declined during Harsha’s period, which is is evident from the decline of
trade centres, less number of coins, and slow activities of merchant guilds.
The decline of trade in turn affected the handicrafts industry and agriculture.
Since there was no large-scale demand for goods, the farmers began to produce only in a limited way. This
led to the rise of self-sufficient village economy.

Q.170 Naganandam written by Harsha deals with


1. Story of a Bodhisattva
2. Accounts of social justice in the Harsha empire
3. Love story of an emperor
4. The first record of a snake deity
Solution: (a)
Learning: Harsha wrote three plays--the Priyadarshika, the Ratnavali and the Naganandam.
The first two are ‘sister plays’, dealing with harem intrigue, and the third tells the story of Jimutavahana, a bodhisattva.
Jimutavahana (c. 12th century) was an Indian Sanskrit scholar and writer of legal and religious treatises of early medieval
period.
He was the earliest writer on smriti (law) from Bengal whose texts are extant.

NEW DECCAN STATE

Q.171 With reference to Ancient Southern India, Tolkappiyam refers to the five-fold division of lands. Consider the
following matches with their description.
1. Kurinji: Desert
2. Mullai: Pastoral
3. Neydal: Agricultural
4. Marudam: Hilly tracks
5. Palai: Coastal
Select the correct matches using the codes below.
(a) 1, 3 and 4 only
(b) 2 only
(c) 1, 4 and 5 only
(d) 1, 2, 3, 4 and 5
Solution: (b)
Justification: This was the description: Kurinji (hilly tracks), Mullai (pastoral), Marudam (agricultural), Neydal (coastal)
and Palai (desert).
Learning: The people living in these five divisions had their respective chief occupations as well as gods for worship.
Kurinji – chief deity was Murugan – chief occupation, hunting and honey collection.
Mullai – chief deity Mayon (Vishnu) – chief occupation, cattle-rearing and dealing with dairy products.
Marudam – chief deity Indira – chief occupation, agriculture.
Neydal – chief deity Varunan – chief occupation fishing and salt manufacturing.
Palai – chief deity Korravai – chief occupation robbery.

Q.172 In the ancient traditions, traders’ voyages often refer which of these lands to Suvarnabhumi (the land of gold)?
(a) Countries of the East Asia
(b) African nations
(c) Sri Lankan islands
(d) European countries
Solution: (a)
Learning: The east coast of India is studded with numerous ports and Indians undertook frequent voyages to these lands.
Also, Indians began to colonize the East Asia in the Gupta period. It was further encouraged by the Pallavas.
The Indian colonists established great kingdoms (with Indian names) and some of them lasted for more than a thousand years.
Till the arrival of Islam in the fifteenth century, Indian culture dominated this region.

PALLAVAS

Q.173 Mahabalipuram was a 7th Century port city of South Indian dynasty of
(a) Pallavas
(b) Chalukyas
(c) Pandyas
(d) Cheras
Solution: (a)
Learning: By the 7th century Mahabalipuram was a port city of South Indian dynasty of the Pallavas.
It has a group of sanctuaries, which was carved out of rock along the Coromandel coast in the 7th and 8th centuries :
rathas (temples in the form of chariots), mandapas (cave sanctuaries), giant open-air rock reliefs such as the famous
Descent of the Ganges, and the Shore Temple, with thousands of sculptures to the glory of Shiva.
The Group of Monuments at Mahabalipuram has been classified as a UNESCO World Heritage Site

Q.174 He was a follower of Jainism in the early part of his career and converted to Saivism later. He is hailed as
‘Vichitrachitta’ who constructed a temple for Brahma, Vishnu and Siva without the use of bricks, timber, metal
and mortar. Who is the ruler?
(a) Kadambas of Banavasi
(b) Adikavi Pampa
(c) Rajaraja I
(d) Mahendravarman I
Solution: (d)
Learning: Mahendravarman I was converted to Saivism which led him to built a Siva temple at Tiruvadi. He was a great
builder of cave temples. The Mandagappattu inscription hails him as Vichitrachitta who constructed a temple for
Brahma, Vishnu and Siva without the use of
bricks, timber, metal and mortar. His rock-cut temples are found in a number of places like Tiruchirappalli. His title
Chitrakarapuli reveals his talents in painting. He is also regarded as an expert in music. The music inscription at
Kudumianmalai is ascribed to him.

Q.175 In the context of the rule of South Indian dynasties, Devadhana and Brahmadeya referred to?
(a) Pristine land
(b) Divine king rule
(c) Treasure appropriated to theGods
(d) Land grants to the temples
Solution: (d)
Justification:Land taxwas the primary source of the government revenue. The Brahmadeya and Devadhana lands were exempted
from tax.
Learning: This is a lateral explanation and deals with aspects other than those mentioned in the question, because detailing the
topic at hand is not of much use.
The Pallavas had a well organized administrative system. The Pallava state was divided into Kottams. The Kottam was administered
by officers appointed by the king.
It was also the responsibility of the central government to provide irrigation facilities to the lands.
Traders and artisanssuch ascarpenters, goldsmiths, washer-men,oil-pressers andweavers paid taxes to the government.
The Pallava inscriptions throw much light on the village assemblies called sabhas and their committees.Theymaintainedrecordsof
allvillagelands,lookedafterlocalaffairsand managed temples.

Q.176 They introduced the art of excavating temples from the rock, and the Dravidian style of temple architecture
began with their rule. They were?
(a) Cheras
(b) Pallavas
(c) Eastern Chalukyas
(d) Jaffna
Solution: (b)
Learning: Pallava age was a great age of temple building.
It was a gradual evolution starting from the cave temples to monolithic rathas and culminated in structural temples.
The development of temple architecture under the Pallavas can be seen in several stages, which started with Mahendravarman I
introducing rock-cut temples.
This style of Pallava temples are seen at places like Mandagappattu, Mahendravadi, Mamandur, Dalavanur, Tiruchirappalli,
Vallam, Siyamangalam and Tirukalukkunram.

Q.177 In the context of Pallava’s history, what was Dakshinachitra?


(a) A compendium of temples
(b) A spiritual treatise on Pallava practices
(c) An autobiography of a Pallava ruler
(d) An instruction manual for painters
Solution: (d)
Learning: Mahendravarman, one of the greatest of the rulers of the Pallava dynasty, was found of painting and is said to have
commissioned a scholar at his court to pen down an instruction manual for painters, which is a treatise on south Indian paintings
called Dakshinachitra.
Unfortunately, it has been lost in the course of time.
Mahendravarman I gave himself the title of “Vchitrachitta” (the man with new fangled ideas).

Q.178 What is the importance of Kanchi in context of Ancient India?


1. Ghatika at Kanchi was an ancient centre of leaning attracting students from India and abroad.
2. Founder of the Kadamba dynasty, Mayurasarman, studied Vedas at Kanchi.
3. Dharmapala, who later became the Head of the Ancient Nalanda University, belonged to Kanchi.
4. It was known as the religious capital of the Southern India.
5. Kanchi hosts Kailasanatha which is one of the largest and most ornate ancient temples in the whole of
India.
Select the correct answer using the codes below.
(a) 2 and 5 only
(b) 1 and 3 only
(c) 3 and 4 only
(d) 1, 2, 3, 4 and 5
Solution: (d)
Justification: Once a capital of the Pallava dynasty, Kanchipuram was also a noted centre of learning for
Tamil and Sanskrit scholars. Dinganaga, a Buddhist writer came to study at Kanchi.
It hosts many temples dedicated to Shiva and Vishnu.
Kanchi was also the home of the famous 6th century CE poet Bharavi who wrote the Kiratarjuniya and the
famous 11th to 12th century CE Hindu philosopher Ramanuja.
Still today an important religious centre, the site has over hundred temples and is also noted for its
production of fine silk saris.

Q.179 Consider the following statements.


Assertion (A): Pallava rulers issued their charters only in
Prakrit. Reason (R): Pallava rulers were against the dominance of Brahmanism and banned the use of Sanskrit for all
official works.
In the context of the above, which of these is correct?
(a) A is correct, and R is an appropriate explanation of A.
(b) A is correct, but R is not an appropriate explanation of A.
(c) A is correct, but R is incorrect.
(d) Both A and R are incorrect.
Solution: (d)
Justification: The early Pallava rulers from 250 A.D. to 350 A.D. issued their charters in Prakrit. Important among
them were Sivaskandavarman and Vijayaskandavarman.
The second line of Pallava rulers who ruled between 350 A.D. and 550 A.D. issued their charters in Sanskrit. So, R is
wrong.
CHALUKYAS

Q.180 Consider the following:


1. They developed Vesara style of architecture.
2. Their structural temples exist at Aihole, Badami and Pattadakal.
3. Their administration was highly centralized.
The above most accurately refer to?
(a) Cholas
(b) Chalukyas
(c) Cheras
(d) Pandyas
Solution: (b)
Justification: Administration: The Chalukya administration was highly centralized unlike that of
the Pallavas and the Cholas. Village autonomy was absent under the Chalukyas.
Architecture: The Chalukyas were great patrons of art. They developed the vesara style in the building of
structural temples. However, the vesara style reached its culmination only under the Rashtrakutas and
the Hoysalas.
Cave temple architecture was also famous under the Chalukyas. Their cave temples are found in Ajanta,
Ellora and Nasik.
The best specimens of Chalukya paintings can be seen in the Badami.

INDIAN PHILOSOPHY

Q.181 The Aranyaka texts are so-called because


(a) They cannot be read by those other than Siddhas.
(b) They contain metaphysics
(c) They are works to be read in the forests.
(d) They appear after Upanishads
Solution: (c)
Learning: Aranyakas are generally the concluding portions of the several Brahmanas, but on account of their distinct
character, contents and language deserve to be reckoned as a distinct category of literature.
They are partly included in the Brahmanas themselves, but partly they are recognized as independent works. Aranyaka
literature is rather small as compared to the Brahmanas.
The term Aranyaka is derived from the word 'Aranya' meaning 'forest'. The Aranyaka texts are so-called because 'they
were works to be read in the forest' in contradistinction to the regular Brahmanas, which were to be read in the village.
Whereas the Brahmanas deal with the huge bulk of sacrificial paraphernalia which represents Karma-Kanda, the
Aranyakas and Upanishads, on the other hand, chiefly deal with the philosophical and theosophical speculations which
represent Jnana-Kanda.

Q.182 The Samkhya philosophy holds that


1. Reality is constituted of two principles one female and the other male.
2. The creator is unconscious of its creation.
3. Life on earth can be explained by the doctrine of evolution.
Select the correct answer using the codes below.
(a) 1 and 3 only
(b) 2 only
(c) 1 and 2 only
(d) 1, 2 and 3
Solution: (a)
Justification: Statement 1: These are Prakriti and Purusha respectively. Prakriti and Purusha are completely
independent and absolute.
According to this system, Purusha is mere consciousness, hence it cannot be modified or changed.
Prakriti on the other hand is constituted of three attributes, thought, movement and the change or transformation of
these attributes brings about the change in all objects.
Statement 2 and 3: The Samkhya philosophy tries to establish some relationship between Purusha and Prakriti for
explaining the creation of the universe. The propounder of this philosophy was Kapila, who wrote the Samkhya sutra.
Infact Samkhya school explained the phenomena of the doctrine of evolution.

Q.183 Nyaya can most appropriately be considered as technique of


(a) Extra-sensory perception
(b) Out of body experiences
(c) Logical thinking
(d) Intuitive Wisdom
Solution: (c)
Learning: According to Nyaya, valid knowledge is defined as the real knowledge, that is, one knows about the object
as it exists.
For example, it is when one knows a snake as a snake or a cup as a cup. Nyaya system of philosophy considers God who
creates, sustains and destroys the universe.
Gautama is said to be the author of the Nyaya Sutras.

Q.184 Consider the following statements.


1. According to Mimamsa philosophy Vedas are eternal and possess all knowledge.
2. Vedanta philosophy denies the existence of the apparent ego.
Which of the above is/are correct?
(a) 1 only
(b) 2 only
(c) Both 1 and 2
(d) None
Solution: (c)
Justification: Statement 1: Mimamsa philosophy is basically the analysis of interpretation, application and the use of
the text of the Samhita and Brahmana portions of the Veda.
Religion means the fulfilment of duties prescribed by the Vedas.
This philosophy encompasses the Nyaya-Vaisheshika systems and emphasizes the concept of valid knowledge.
It is called the ritualistic school amongst all the six schools of philosophy.
Statement 2: This is the ego as known to us, and in this respect Vedanta has its unique position in the history of
philosophies of the world.
Vedanta implies the philosophy of the Upanishad, the concluding portion of the Vedas.
According to Vedanta philosophy, ‘Brahman is true, the world is false and self and Brahman are not different,
Shankaracharya believes that the Brahman is existent, unchanging, the highest truth and the ultimate knowledge.
He also believes that there is no distinction between Brahman and the self. The knowledge of Brahman is the essence of
all things and the ultimate existence.

Q.185 Which of the following is NOT a Vedanga?


(a) Nirukta – Etymology
(b) Vyakarana – Grammar
(c) Kalpa – Ritual instructions
(d) Virakta – Philosophy
Solution: (d)
Learning: Vedangas literally mean the limbs of the Vedas. They are six in number. Just like the limbs of the body, they
perform various supportive and augmenting functions in the study, preservation and protection of the Vedas and the
vedic traditions.
The six Vedangas are Siksha, Chhanda, Vyakarana, Nirukta, Jyotisha and Kalpa.
These subjects were an integral and essential part of ancient vedic education system, aimed to promote an all-round
development of the students with a better understanding of the Vedas and vedic practices.
Of these six subjects, Siksha deals with the study of sounds and pronunciation associated with each syllable; Chhanda
with the mastery of rhyme and meter; Vyakarana with the study of word and sentence structure; Nirukta with the
meaning of complex words and phrases; Jyotisha with the study of heavenly bodies to find an auspicious time for the
performance of the rituals; and Kalpa with the ethical, moral and procedural precepts associated with the performance
of rituals as a way of life.

Q.186 What is the central feature of all the Upanishads?


1. All of them deny non-dualism or advaita and assert dvaita or dualism.
2. They take the stand that the Universe is without any consciousness and a mere play of the Brahman.
Which of the above is/are correct?
(a) 1 only
(b) 2 only
(c) Both 1 and 2
(d) None
Solution: (d)
Justification: Both these statements can be invalidated by the example of Brihadaranyaka Upanishad.
The metaphysics of Brihadaranyaka Upanishad is non-dualism (Advaita).
"From infinite or fullness, we can get only fullness or infinite". The above verse describes the nature of the Absolute or
Brahman which is infinite or full, i.e., it contains everything. Upanishadic metaphysics is further elucidated in the
Madhu-vidya (honey doctrine), where the essence of every object is described to be same to the essence of every other
object.
The Brihadaranyaka Upanishad looks at reality as being indescribable and its nature to be infinite and consciousness-
bliss.

Q.187 Anahata Nada’ is a term associated in the Indian philosophical tradition to


(a) Purification rites
(b) Divine sounds
(c) Unflinching determination to spiritual discipline
(d) Vibration produced by the mind
Solution: (b)
Learning: Anahata means "unhurt, unstruck, and unbeaten". Anahata Nad refers to the Vedic concept of unstruck sound
(the sound of the celestial realm). Anahata is associated with balance, calmness, and serenity.
The sound is of many types including that of a veena, humming bee, clouds, ocean etc. One with a calm and serene
mind, and one whose energies have penetrated the Anahata chakra is able to hear this sound steadily and clearly. You
can also hear these sounds if you close both your ears and listen to the inner sounds attentively.
Anahata is considered to be the seat of the Jivatman and Parashakti. In the Upanishads, this is described as a tiny flame
inside the heart. Anahata is named as such because sages were believed to hear the sound (Anahata – comes without
the striking of two objects together).

Q.188 With reference to Ancient India, which of the following is NOT one of the system of ashramas?
(a) Vanaprastha
(b) Sanyasa
(c) Purushartha
(d) Brahmacharya
Solution: (c)
Learning: The literal meaning of word Ashrama is “a halting or resting place”.
Ashramas are regarded as different stages in the life of an individual which train him for some period and the individual
exerts himself in the same order to qualify himself for the next stage of life.
The four asramas are: Brahmacharya (student), Grihastha (householder), Vanaprastha (retired) and Sannyasa
(renunciation). The Ashramas system is one facet of the Dharma concept in Hinduism.

Q.189 Which of the following treatises represent the transitional phase between the ritualistic symbolism of the
Brahmanas and the philosophical doctrines of the Upanishads?
(a) Agamas
(b) Aranyakas
(c) Yajur Veda
(d) Shaiva texts
Solution: (b)
Justification: Vedic ritual is preserved in literary texts called the Brahmanas. The main division of the contexts of these
extensive texts is twofold – the ritualistic injuction and discussions on the meaning of Vedic ritual and all that is related
to it.
The Aranyakas or the treatises of the forest present secret explanations of the ritual, have their origin in the
philosophical discussions of the Brahmanas, find their culmination in the Upanishads and represent the transitional
phase between the ritualistic symbolism of the Brahmanas and the philosophical doctrines of the Upanishads.
The Upanishads, written both in prose and poetry, are expressions of philosophical concepts.

Q.190 The Mahapuranas have five subjects. Which of the following is NOT included?
(a) Sarga, the original creation of the universe
(b) Nadabindu, cosmic dance with Anahata Naad (sound)
(c) Surya Vamsha and Chandra Vamsa, the histories of the solar and lunar dynasties of Gods and sages
(d) Vamshanucharita, the genealogies of kings
Solution: (b)
Learning: The main Puranas are 18 encyclopaedic collections of legend and myth. Though the archaic form of the
genre might have existed as early as the fourth or the fifty century B.C., the famous names of the 18 Mahapuranas were
not discovered earlier than the third century A.D.
The phenomenal popularity of these Mahapuranas gave rise to yet another sub-genre known as the Upapuranas or
minor Puranas. They are also 19 in number.
The Mahapuranas have five subjects. These are : (1) Sarga, the original creation of the universe, (2) Pratisarga, the
periodical process of destruction and re-creation, (3) Manvantara, the different eras or cosmic cycles, (4) Surya Vamsha
and Chandra Vamsa, the histories of the solar and lunar dynasties of Gods and sages, (5) Vamshanucharita, the
genealogies of kings.
Around this core skeleton of the five subjects any Purana adds other diverse materials like matters of religious concern,
customs, ceremonies, sacrifices, festivals, the duties of various castes, different types of donations, details of the
construction of temples and images, and descriptions of places of pilgrimage.

Q.191 Madhamayaka and Yogacara are philosophical traditions related to


(a) Tantra
(b) Vedanta
(c) Yoga
(d) Buddhism
Solution: (d)
Learning: The Madhyamaka school of Buddhism, the followers of which are called Mādhyamikas, was one of the two
principal schools of Mahāyāna Buddhism in India, the other school being the Yogācāra.
The name of the school is a reference to the claim made of Buddhism in general that it is a middle path (madhyamā
pratipad) that avoids the two extremes of eternalism—the doctrine that all things exist because of an eternal essence—
and annihilationism—the doctrine that things have essences while they exist but that these essences are annihilated just
when the things themselves go out of existence.
The fundamental doctrine of the Yogacara school is "that all phenomenal existence is fabricated by consciousness."
Consciousness is the basis of all activities from birth to attaining enlightenment; "...all is based upon the coming into
being and the ceasing to be of consciousness, i.e., of distinctions in the mind."

Q.192 With reference to Ancient India, the text Grihya-sutra deals with
1. Science of making garbha-grihas in Hindu temples
2. Domestic Hindu religious rituals
3. Astronomical positions of planets in their own zodiac signs
Select the correct answer using the codes below.
(a) 2 only
(b) 1 and 3 only
(c) 3 only
(d) 1, 2 and 3
Solution: (a)
Justification: Grihya-sutra, in Hinduism, is a manual detailing the domestic (grihya) religious ceremonies
performed by both male and female householders over the fire.
The Grihya-sutras, together with the Shrauta-sutras (which deal with the grand Vedic sacrifices) and the
Dharma-sutras (which deal with rules of conduct), make up the Kalpa-sutras—collections of texts that
emerged within the different Vedic schools.
The Grihya-sutras describe the ceremonies (samskaras) that mark each stage of an individual’s life, from
the moment of conception to the final death rites; the five daily sacrifices (mahayajna); seasonal
ceremonies; and those observed on special occasions, such as house building or cattle
breeding.

Q.193 The Puranas laid down that


1. It was not possible for devotees to receive the grace of God unless they were born in certain castes.
2. One should not be a devout of God idols and instead meditate on the formless reality
Which of the above is/arecorrect?
(a) 1 only
(b) 2 only
(c) Both 1 and 2
(d) None
Solution: (d)
Justification:Statement1:OnecouldattainthegraceofGodirrespectiveofone’scastestatus.
Statement 2: Around the time when the puranas first began to be composed, the belief in particular deities had become established
as one of the principal marks of the Hindu faith, and to some degree the puranas can be described as a form of sectarian
literature.
Some puranas exhibit devotion to Shiva; in others, the devotion to Vishnu predominates.
Learning: The special subject of the puranas is the powers and works of the gods, and one ancient Sanskrit lexicographer,
Amarasinha, writing in the fifth or sixth century A.D., defined a purana as having five characteristic topics, or pancalaksana: "(1)
The creation of the universe; (2) Its destruction and renovation; (3) The genealogy of gods and patriarchs; (4) The reigns of the
Manus, formingthe periodscalled Manwantaras; (5) thehistoryof the SolarandLunarracesof kings."

Q.194 Philosophy of Integral humanism has four pillars. Which of these is NOT one of them?
(a) Dharma
(b) Moksha
(c) Artha
(d) Nikaya
Solution: (d)
Learning: Integral humanism is a doctrine developed by Deendayal Upadhyaya and adopted by the Jana Sangh in 1965
as its official doctrine
Humankind, according to Upadhyaya, had four hierarchically organized attributes of body, mind, intellect and soul which
corresponded to four universal objectives, kama (desire or satisfaction), artha (wealth), dharma (moral duties) and
moksha (total liberation or 'salvation').
While none could be ignored, dharma is the 'basic', and moksha the 'ultimate' objective of humankind and society.
He claimed that the problem with both capitalist and socialist ideologies is that they only consider the needs of body
and mind, and were hence based on the materialist objectives of desire and wealth.

Q.195 Which of these sects advocated renunciation of the world to reach salvation?
1. Yogis
2. Nathpanthis
3. Siddhacharas
Select the correct answer using the codes below.
(a) 1and3only
(b) 1and2only
(c) 2 only
(d) 1, 2 and 3
Solution: (d)
Justification: To them the path to salvation lay in meditation on the formless Ultimate Reality and the realisation of oneness with
it.
To achieve this, they advocated intense training of the mind and body through practices like yogasanas, breathing exercises and
meditation.
These groups became particularly popular among “low” castes.
Their criticism of conventional religion created the ground for devotional religion to become a popular force in northern India.

Q.196 He founded the Vaisheshika school of Indian philosophy which attempted to explain the creation and
existence of the universe by proposing an atomistic theory. He is?
(a) Vashistha
(b) Kanada
(c) Kapila
(d) Agastya
Solution: (b)
Learning: He was an Indian sage and philosopher whose traditional name "Kanada" means "atom eater".
He is known for developing the foundations of an atomistic naturalism Indian philosophy in the Sanskrit text Vaisheshika
Sutra.
Applying logic and realism, it is among one of the earliest known systematic realist ontology in
human history.
Kanada suggested that everything can be subdivided, but this subdivision cannot go on forever, and there must be
smallest entities (parmanu) that cannot be divided, that are eternal,
He used these ideas with the concept of Atman (soul, Self) to develop a non-theistic means to moksha.

Q.197 Which of these is NOT of of the Eight components of Yoga as described by Patanjali?
(a) Yamas
(b) Niyama
(c) Asana
(d) Mulahara
Solution: (d)
Learning: Other components include Praṇayama, Pratyahara, Dharaṇa, Dhyana and Samadhi.
These are a part of the Yoga Sutras of Patanjali which was the most translated ancient Indian text in the medieval era,
having been translated into about forty Indian languages and two non-Indian languages: Old Javanese and Arabic.
The Yoga Sutras are a composite of various traditions. The levels of samādhi taught in the text resemble the Buddhist
jhanas.
According to Feuerstein, the Yoga Sutras are a condensation of two different traditions, namely "eight limb yoga"
(ashtanga yoga) and action yoga (Kriya yoga).

Q.198 Ramananda, Kabir and Nanak


1. Were not linked with any particular religious creed
2. Did not believe in rituals and ceremonies
3. Condemned monotheism and believed in multiple Gods
4. Denounced all forms of idolatry
Select the correct answer using the codes below.
(a) 1, 2 and 3 only
(b) 1, 2 and 4 only
(c) 3 and 4 only
(d) 1 and 3 only
Solution: (b)
Justification: Statement 1: In the fourteenth and fifteenth centuries, Ramananda, Kabir and Nanak remained great
apostles of the Bhakti cult. They drew inspiration from old masters but showed a new path. They helped the common
people to shed age-old superstitions and attain salvation through Bhakti or pure devotion.
Statement 2, 3 and 4: They condemned polytheism and believed in one god. They also denounced all forms of idolatry.
They strongly believed in Bhakti as the only means of salvation. They also emphasised the fundamental unity of all
religions. Unlike the early reformers, they were not linked with any particular religious creed and did not believe in
rituals and ceremonies.
Q.199 The system of four ashrams is advocated in2
(a) Buddhism
(b) Jainism
(c) Lokayatas
(d) Brahmanism
Solution: (d)
Learning: Around the time when Jainism and Buddhism were becoming popular, brahmins developed the system ofashramas.
Ashramas refer to stage of life: brahmacharya, grihastha, vanaprastha and samnyasa.
Brahmin, kshatriya and vaishya men were expected to lead simple lives and study the Vedas during the early years of their life
(brahmacharya).
Then they had to marry and live as householders (grihastha).
Then they had to live in the forest and meditate (vanaprastha). Finally, they had to give up everything and become samnyasins.
The system of ashramas allowed men to spend some part of their lives in meditation.
Generally, women were not allowed to study the Vedas, and they had to follow the ashramas chosen by their husbands.

Q.200 Which of the following propositions have been given by the Vaisheshika Darshana?
1. Dharma is that through which there is the accomplishment of rising to the unsurpassed good.
2. The true being is eternal, having no cause.
3. Dharma does not have the authority of Vedas as it surpasses Vedas.
Select the correct answer using the codes below.
(a) 1 and 2 only
(b) 3 only
(c) 2 and 3 only
(d) 1, 2 and 3
Solution: (a)
Justification: Statement 1 and 3: It says that because the Dharma is an exposition of unsurpassed good, it has the
authority of Vedas.
Statement 2: It says that that there is only one individual (soul) is known from the absence of particularity when it
comes to the emergence of understanding of happiness and suffering, (whereas) a multiplicity of individuals is inferred
from their perseverance in dharma, and from the strength of their teaching.
The true being is eternal, having no cause. Its indicator is its effect. The presence of the effect arises from the presence
of its cause.

Q.201 Consider the following statements.


1. Sankara denounced Hindu revivalist movement citing them as going against rationality and modernity.
2. Ramanuja advocated prabattimarga or the path of self-surrender to God for salvation.
Which of the above is/arecorrect?
(a) 1 only
(b) 2 only
(c) Both 1 and 2
(d) None
Solution: (b)
Learning: Statement 1: In the ninth century Sankara started a Hindu revivalist movement giving a new orientation to Hinduism. So,
1 is wrong.
However, his doctrine of Advaita or Monism was too abstract to appeal to the common man.
Moreover, there was a reaction against the Advaita concept of Nirgunabrahman (God without attributes) with the emergence of
the idea of Sagunabrahman (God with attributes).
Statement 2: In the twelfth century, Ramanuja, preached Visishtadvaita.
According to him God is Sagunabrahman. The creative process and all the objects in creation are real but not illusory as was held
by Sankaracharya.
Therefore, God, soul, matter are real. But God is inner substance and the rest are his attributes.
He invited the downtrodden to Vaishnavism

Q.202 With reference to Ancient texts, consider the following statements.


1. The Brahmanas are the law texts under Manusmriti that helped establish the predominance of
Brahmana community.
2. Upanishads are treatises relating to prayer and sacrificial ceremony.
3. Aranyakas are called forest books and they deal with mysticism.
4. Yajurveda deals with music, chanting and medicine.
Select the correct answer using the codes below.
(a) 1 and 2 only
(b) 3 only
(c) 1, 3 and 4 only
(d) 2 and 4 only
Solution: (b)
Justification: Besides the Vedas, there are other sacred works like the Brahmana, the Upanishads, the
Aranyakas and the epics Ramayana and Mahabharata.
The Brahmanas are the treatises relating to prayer and sacrificial ceremony.
The Upanishads are philosophical texts dealing with topic like the soul, the absolute, the origin of the
world and the mysteries of nature.
The Aranyakas are called forest books and they deal with mysticism, rites, rituals and sacrifices.
Yajurveda is a compilation of ritual offering formulas that were said by a priest while an individual
performed ritual actions such as those before the yajna fire.

Q.203 Rules laid down by the Vishnu Smriti for the grant of land provide for which of the following?
1. Landed property bestowed upon brahmanas by other rulers should not be appropriated by a King.
2. For the information of future rulers, land grants should be recorded containing the names of immediate ancestors.
Which of the above is/are correct?
(a) 1 only
(b) 2 only
(c) Both 1 and 2
(d) None
Solution: (c)
Justification: This is the statement (translated) that appears in Vishnusmriti. “Let him (the king) bestow landed
property upon brahmanas.
To those upon whom he has bestowed (land), he must give a document, destined for the information of a future ruler,
which must be written upon a piece of cloth, or a copper plate, or must contain the names of (three) immediate
ancestors, a declaration of the extent of the land, and a description (of what results) from disrupting the donation and
should be signed with his own seal. Let him not appropriate to himself landed property bestowed (upon brahmanas) by
other rulers.”

Q.204 With reference to Ancient India, according to Manu


1. Father could not give his wealth to daughter as a gift.
2. Mother’s wealth would pass to the daughters.
3. Women could receive wealth in Adhyavahanika.
Select the correct answer using the codes below.
(a) 1 only
(b) 2 and 3 only
(c) 3 only
(d) 1, 2 and 3
Solution: (b)
Justification: Statement 1: Manu did not project that daughters are not the natural heirs of their fathers, but the father
was at liberty to give wealth to daughter as gift.
Manu also refers to the putrika or daughter’s son functioning as an heir for a sonless father. Statement 2: The
mother’s wealth would pass to the daughters, according to the MS.
The different occasions in the MS when gifts can be given to the daughter, over which she has right were:
Adhyagni – before the nuptial fire;
Adhyavahanika – in the bridal procession;
Pritidatta – out of love;
Bhratrimatripitripraptam- received from brother, mother, father
Q.205 Consider the following statements.
1. The Dharmasutras and Dharmashastras contained rules about the ideal occupations of the four varnas.
2. Manusmriti held that no one could engage in agriculture and pastoralism apart from khastriyas.
Which of the above is/are correct?
(a) 1 only
(b) 2 only
(c) Both 1 and 2
(d) None
Solution: (a)
Justification: Statement 1: This division became the basis for caste discrimination later. Brahamanas
were awarded with the highest occuppations whereas the shudras with menial jobs which was socially
and economically exploitative.
Statement 2: Kshatriyas were to engage in warfare, protect people and administer justice, study the
Vedas, get sacrifices performed, and make gifts.
The Vaishyas were expected to engage in agriculture, pastoralism and trade.

TRAVELLERS AND OTHER SOCIAL LIFE

Q.206 Consider the following statements about famous teachers of ancient India.
1. When Acharya Kumarajiva went to China, the king requested him to translate Sanskrit texts into Chinese.
2. Acharya Kamalasheel of Ancient Nalanda University was invited by the king of Tibet.
3. A distinguished scholar, Jnanabhadra, went to Tibet from India to preach Dharma.
Select the correct answer using the codes below.
(a) 1 only
(b) 2 and 3 only
(c) 3 only
(d) 1, 2 and 3
Solution: (d)
Justification: Statement 1: Two Indian teachers went to China on an invitation from the Chinese
Emperor in AD 67.
Their names are Kashyapa Martanga and Dharmarakshita.
They were followed by a number of teachers from universities like Nalanda, Takshila, Vikramashila and Odantapuri.
The scholar Bodhidharma, who specialised in the philosophy of Yoga is still venerated in China and Japan.
Statement 2 and 3: After his death, the Tibetans embalmed his body and kept it in a monastery in Lhasa.
Another distinguished scholar was Jnanabhadra. He went to Tibet with his two sons to preach Dharma.
A monastery was founded in Tibet on the model of Odantapuri University in Bihar.
The head of the Vikramashila University was Acharya Ateesha, also known as Dipankara Shreejnana. He went to Tibet in
the eleventh century and gave a strong foundation to Buddhism in Tibet.

Q.207 ‘Romas’ group of people


1. Were Indians who went abroad as wanderers
2. Are known are Gypsies in Europe
3. Settled as a group mainly in Pakistan and Afghanistan
Select the correct answer using the codes below.
(a) 1 only
(b) 1 and 2 only
(c) 2 and 3 only
(d) 1, 2 and 3
Solution: (b)
Justification: Some groups of Indians went abroad as wanderers. They called themselves Romas and their language
was Romani, but in Europe they are famous as Gypsies.
They went towards the West, crossing the present-day Pakistan and Afghanistan.
From there, their caravans went through Iran and Iraq to Turkey. Travelling through Persia, Taurus mountains and
Constantinople, they spread to many countries of Europe.
Today they live in Greece, Bulgaria, States of former Yugoslavia, Romania, Hungary etc.
The Romas are known for their dance and music as well. It is said that every Roma musician is a splendid artist.

Q.208 In the seventh and eighth centuries, ‘ghatikas’ were


(a) colleges attached to the temples
(b) time keeping devices kept at astronomical houses
(c) guards of religious shrines
(d) royal decrees passed with the consent of the citizens
Solution: (a)
Justification: These ghatikas or colleges attached to the temples emerged as new centres of learning in 7th and 8th
century.
The ‘ghatikas’, provided Brahmanical education. The mediumof instruction was Sanskrit. Entry to these temple colleges
was open only to the upper castes or ‘dvijas’ (twice born).
Learning: Use of Sanskrit as the medium of instruction distanced the common people from education. Education
became the privilege of only the uppermost sections of society.
The ‘mathas’ supported by Brahmins were institutions parallel to Jain and Buddhist monasteries. The ‘mathas’
functioned like ashramas for educational purposes.

Q.209 The Ancient Silk Route traversed through which of the following regions?
1. Europe
2. Africa
3. East Asia
4. South-east Asia
Select the correct answer using the codes below.
(a) 1 and 2 only
(b) 1, 3 and 4 only
(c) 2 and 4 only
(d) 1, 2, 3 and 4
Solution: (d)
Justification: The road connected civilizations across Europe, Asia (including Persia, Arabia) and Africa (only the horn
part).
The disappearance of the Silk Road following the end of the Mongols' reign was one of the main factors that stimulated
the Europeans to reach the prosperous Chinese empire through another route, especially by sea.
The direct ocean route from Europe to the East was finally opened by the expeditions of Bartolomeu Dias (1488), and
Vasco da Gama (1497-1499), by the Atlantic and the Indian oceans.
Q.210 In Ancient India, officers who reported the activities in the country to the government were known as
(a) Agraharas
(b) Dhamma Mahathas
(c) Patidevakas
(d) Gramani
Solution: (c)
Learning: The duty of these officers was to superintend or oversee all that occurred in town or country and to make
private reports to the government.
Arian notes that similar officers were employed by the authorities of the independent nations as well as by the
monarchical governments of India.
King Devanampiya says, “the patidevakas should report to me about the affairs of the people at all times, anywhere,
whether I am eating, in the inner apartment, in the bedroom or in the garden. And I will dispose of the affairs of the
people everywhere.”

Q.211 In the context of Ancient India, consider the following Travellers/Scholars and match them to their patron.
1. Xuan Zang: A. Dara Shikoh
2. Megasthenes: B. Vikramaditya
3. Fa-Hein: C. ChandraGuptaMaurya
4. Francois Bernier: D. Harshavardhana
Select the correct answer using the codes below.
(a) 1B, 2A, 3C, 4D
(b) 1C, 2A, 3D, 4A
(c) 1D, 2C, 3B, 4A
(d) 1A, 2B, 3C, 4D
Solution: (c)
Justification: Statement 1:Hsuan Tsang, a Chinese traveller, has been described as the “Prince of Pilgrims.”
His visit to India was an important event of the reign of Harshavardhana. India is much indebted to this Chinese traveller
for the valuable accounts he left behind with many details of political, religious, economic, social conditions of those
days.
Statement 2: Megasthenes became an ambassador of Seleucus I Nicator of the Seleucid dynasty to Chandragupta
Maurya in Pataliputra.
He was an ancient Greek historian, diplomat and Indian ethnographer and explorer in the
Hellenistic period. He described India in his book Indika, which is now lost, but has been partially reconstructed from the
writings of the later authors.
Statement 3: Fa-Hien is the famous Chinese pilgrim who visited India during the rule of Chandra Gupta II.
He covered, inter alia, Taxila, Mathura, Kanauj, Kosambi, Kashi, Kusinara, Saraswati, Kapilvastu, Vaishali, Patliputra and
Nalanda.
Statement 4: Francois Bernier was a French physician and traveller. He was briefly personal physician to Mughal prince
Dara Shikoh, the eldest son of the Mughal emperor Shah Jahan, and after Dara Shikoh's demise, was attached to the
court of the Mughal emperor Aurangzeb (1618
–1707), for around 12 years during his stay in India.

MEDIEVAL INDIAN HISTORY

Q.212 Which of the following dynasties do NOT belong to Medieval India?


(a) Western Ganga dynasty
(b) Kakatiya Kingdom
(c) Rashtrakuta dynasty
(d) Nanda dynasty
Solution: (d)
Justification: Option A: Western Ganga dynasty was an important ruling dynasty of ancient Karnataka, often under the
over lordship of larger states, from about 350 to 1000 AD. The large monolithic Bahubali of Shravanabelagola was built
during their rule.
Option B: Kakatiya Kingdom was a Telugu dynasty that ruled most of current day Andhra Pradesh, India from 1083 CE to
1323 CE.
Option C: Rashtrakuta dynasty was a Kannada Royal Dynasty Ruling large parts Of the Indian subcontinent between the
6th and the 10th centuries and one who built World Heritage centre Ellora, Maharashtra.
Option D: Mahapadma Nanda (from 345 BCE), son of Mahanandin, founded the Nanda Empire after inheriting
Mahanandin's empire

BHAKTI MOVEMENT

Q.213 With reference to Chaitanya, consider the following statements.


1. He popularised the Shakti cult in Medieval India.
2. He was against renunciation and asceticism as a path of spiritual liberation.
3. He believed that through song and dance done with love and devotion, one can feel the presence of God.
Select the correct answer using the codes below.
(a) 1 and 2 only
(b) 3 only
(c) 1 only
(d) 2 and 3 only
Solution: (b)
Justification: He was a well-known saint and reformer of Bengal who popularised the Krishna cult. He renounced the
material world, became an ascetic and wandered all over the country preaching his ideas.
He proclaimed the universal brotherhood of man and condemned all distinction based on religion and caste. He
emphasised love and peace and showed great sympathy to the sufferings of other people, especially that of the poor
and the weak.
He believed that through love and devotion, song and dance, a devotee can feel the presence of God. He accepted
disciples from all classes and castes and his teachings are widely followed in Bengal even today.

Q.214 Located on an island in Cauvery River, this temple is mentioned in the Thiviya Pirabandham, the early
medieval Tamil literature canon of the Alvar saints from the 6th to 9th centuries AD. It is
(a) Dattadeva temple
(b) Sangameshwara
(c) Kashivishweshwar temple
(d) Ranganathaswamy temple
Solution: (d)
Learning: The Sri Ranganathaswamy temple at Srirangam is one of the most illustrious Vaishnava temples in South
India rich in legend and history. Its location, on an island in Cauvery river.
It is counted among the 108 Divya Desams dedicated to Vishnu. The temple follows Thenkalai tradition of worship.
It has bagged an award of merit from UNESCO for protecting and conserving cultural heritage, thus becoming the first
temple from Tamil Nadu to grab the prestigious honour from the UN body.
Launched in 2000, UNESCO Asia-Pacific awards for cultural heritage conservation programme is aimed at acknowledging
the efforts taken to restore and conserve historical structures without affecting their heritage value in the region
comprising 48 countries.

Q.215 Consider the following statements. Shaiva Movement in the South India
1. Flourished through the activities of Alvars
2. Was popularized through singing emotional songs in Tamil
3. Opposed the views of the Virashaivas and Agamantas
Select the correct answer using the codes below.
(a) 1 only
(b) 2 and 3 only
(c) 2 only
(d) 1, 2 and 3
Solution: (c)
Justification: Statement 1: The Shaiva movement in the South flourished at the beginning through the activities of many
of the 63 saints known in Tamil as Nayanars (Siva-bhakts).
Statement 2: Their appealing emotional songs in Tamil were called Tevaram Stotras, also known as Dravida Veda and
ceremonially sung in the local Shiva temples.
Statement 3: The Nayanars hailed from all castes. This was supplemented on the doctrinal side by a large number of Shaiva
intellectuals whose names were associated with several forms of Shaiva movements like Agamanta, Shudha and Vira-
shaivism.

Q.216 With reference to Bhakti saints, consider the following statements.


1. Ekanatha opposed caste distinctions and was sympathetic towards the lower castes.
2. Gnanadeva was the founder of the Bhakti Movement in Maharashtra in fifth Century.
3. Namadeva preached the gospel of love through idol worship.
Select the correct answer using the codes below.
(a) 1 and 3 only
(b) 1 only
(c) 1 and 2 only
(d) 2 and 3 only
Solution: (b)
Justification: Statement 1: In the sixteenth century, Ekanatha opposed caste distinctions and sympathetic towards the
lower castes. He composed many lyrics and his bhajans and kirtans were famous.
Statement 2: It can’t be fifth century because Bhakti movement is a Medieval India movement. It was in the thirteenth
century. It was called Maharashtra dharma.
He wrote a commentary of Bhagavat Gita called Gnaneswari.
Statement 3: Namadeva preached the gospel of love. He opposed idol worship and priestly domination. He also
opposed the caste system.
Learning: Another Bhakti saint of Maharashtra was Tukaram, a contemporary of Sivaji. He was responsible for creating
a background for Maratha nationalism. He opposed all social distinctions.

Q.217 Consider the following statements.


1. As per Ramanuja, all the objects in creation are real but not illusory.
2. As per Madhava, dualism of Jivatma and Paramatma does not exist and they dwell in an eternal cosmic entity.
Which of the above is/are correct?
(a) 1 only
(b) 2 only
(c) Both 1 and 2
(d) None
Solution: (a)
Justification: Statement 1: In the twelfth century, Ramanuja preached Visishtadvaita.
According to him God is Sagunabrahman. The creative process and all the objects in creation are real but not illusory as
was held by Sankaracharya. Therefore, God, soul, matter are real. But God is inner substance and the rest are his
attributes.
He also advocated prabattimarga or path of self-surrender to God. He invited the downtrodden to Vaishnavism.
Statement 2: In the thirteenth century, Madhava from Kannada region propagated Dvaita or dualism of Jivatma and
Paramatma.
According to his philosophy, the world is not an illusion but a reality. God, soul, matter are unique in nature.
Learning: Nimbarka and Vallabhacharya were also other preachers of Vaishnavite Bhakti in the Telungana region.
Surdas was the disciple of Vallabhacharya and he popularized Krishna cult in north India.

Q.218 Anuvyakhyana is a major work of


(a) Adi Shankaracharya
(b) Madhvacharya
(c) Ramanuja
(d) Vasudevbandhu
Solution: (b)
Learning: He was a Hindu philosopher and the chief proponent of the Dvaita (dualism) school of Vedanta. He was born
in Karnataka in 13th-century India.
As a teenager, he became a Sanyasi (monk) joining Brahma-sampradaya guru Achyutapreksha, of the Ekadandi order.
Madhva studied the classics of Hindu philosophy, particularly the Principal Upanishads, the Bhagavad Gita and the
Brahma Sutras (Prasthanatrayi). He commented on these, and is credited with thirty seven works in Sanskrit.
His writing style was of extreme brevity and condensed expression. His greatest work is considered to be the
Anuvyakhyana, a philosophical supplement to his bhasya on the Brahma Sutras composed with a poetic structure.
In some of his works, he proclaimed himself to be an avatar of Vayu, the son of god Vishnu.
He was a critic of Adi Shankara's Advaita Vedanta and Ramanuja's Vishishtadvaita Vedanta teachings.

Q.219 The Nalayira Divyaprabandham is a major compositions of the


(a) Alvars
(b) Nayanars
(c) Lingayatas
(d) Lokayatas
Solution: (a)
Learning: The Nalayira Divya is a collection of 4,000 Tamil verses (Naalayiram in Tamil means 'four thousand')
composed by the 12 Alvars, and was compiled in its present form by Nathamuni during the 9th – 10th centuries.
The work, an important liturgical compilation of the Tamil Alvar Bhaktas, marks the beginning of the canonization of 12
Vaishnava poet saints, and these hymns are still sung extensively today. The works were lost before they were collected
and organized in the form of an anthology by Nathamuni.
It was frequently described as the Tamil Veda, thus claiming that the text was as significant as the four Vedas in Sanskrit
that were cherished by the Brahmanas.
Q.220 With reference to Kabir, a fifteenth century poet and mystic, consider the following statements.
1. Salvation cannot be attained by search for the knowledge, rather results from complete surrender of oneself.
2. Kabir was critical of social discrimination but maintained that caste system was useful for the division of labour in
the society.
Which of the above is/are correct?
(a) 1 only
(b) 2 only
(c) Both 1 and 2
(d) None
Solution: (d)
Justification: He was a poet and weaver who also belonged to the Bhakti tradition. Kabir’s poetry spoke about his love
for the Supreme Being free of ritual and priests.
Statement 1: In his view every person had the ability to reach the highest level of spiritual salvation and deep knowledge
within themselves through their own experience.
Statement 2: It also expresses his sharp and pointed criticism of those he saw as powerful.
Kabir attacked those who attempted to define individuals on the basis of their religious and caste identities.
His poetry brings out the powerful idea of the equality of all human beings and their labour.
He writes about valuing the work of the ordinary potter, the weaver and the woman carrying the water pot – labour that
in his poetry becomes the basis of understanding the entire universe.

Q.221 The Devaram composed by Nayanmars and the Nalayradivyaprabandam composed by Alwars
represent the religious literature of the
(a) Pallava period
(b) Shunga period
(c) Kushana period
(d) Gupta period
Solution: (a)
Learning: The Nayanmars and Alwars composed religious hymns in Tamil.
The Devaram denotes the first seven volumes of the Tirumurai, the twelve-volume collection of Śaiva devotional poetry.
The Nalayira Divya Prabandham compiled in its present form by Nathamuni during the 9th – 10th centuries.
Perundevanar was patronized by Nandivarman II and he translated the Mahabharata as Bharathavenba in Tamil.
Nandikkalambagam was another important work but the name of the author of this work is not known.

Q.222 Consider the following statements.


1. Gnanadeva wrote a commentary on Ramayana called Gnaneswari.
2. Namadeva opposed idol worship and priestly domination.
3. Tukaram was against parochial concept of Maratha nationalism.
Select the correct answer using the codes below.
(a) 2 only
(b) 3 only
(c) 2 and 3 only
(d) 1 and 2 only
Solution: (a)
Justification: Statement 1: Gnanadeva was the founder of the Bhakti Movement in Maharashtra in the thirteenth
century. It was called Maharashtra dharma. He wrote a commentary of Bhagavat Gita called Gnaneswari.
Statement 2: Namadeva preached the gospel of love. He opposed idol worship and priestly domination. He also
opposed the caste system.
In the sixteenth century, Ekanatha opposed caste distinctions and sympathetic towards the lower castes. He composed
many lyrics and his bhajans and kirtans were famous.
Statement 3: Another Bhakti saint of Maharashtra was Tukaram, a contemporary of Sivaji. He was responsible for
creating a background for Maratha nationalism. He opposed all social distinctions.

Q.223 Kabir used the word “alakh” to describe


(a) Bhakti movements in India
(b) Religious reforms in favour of lower castes
(c) The unseen reality
(d) Artistic inspiration from nature
Solution: (c)
Learning: Kabir used terms drawn from Vedantic traditions, alakh (the unseen), nirakar (formless), Brahman, Atman, etc.
Other terms with mystical connotations such as shabda (sound) or shunya (emptiness) were drawn from yogic
traditions.
Some of Kabir’s poems are known as ulatbansi (upside-down sayings), are written in a form in which everyday meanings
are inverted.
These hint at the difficulties of capturing the nature of the Ultimate Reality in words: expressions such as “the lotus
which blooms without flower” or the “fire raging in the ocean” convey a sense of Kabir’s mystical experiences.

Q.224 What were the reason(s) for Lingayats not practising funerary rites?
1. They believed that the body was too sacred to be purified by any rites or rituals.
2. They observed that such rites pollute the environment.
3. They held the belief that after death a devotee will not return to this world.
4. They believed funeral rites were not sanctioned by the Vedas.
Select the correct answer using the codes below.
(a) 1, 2 and 3 only
(b) 2 and 4 only
(c) 3 only
(d) 4 only
Solution: (c)
Justification: The Lingayats challenged the theory of rebirth and thus disapproved of this practice.
Lingayats believe that on death the devotee will be united with Shiva and will not return to this world.
Therefore, they do not practise funerary rites such as cremation, prescribed in the Dharmashastras. Instead, they
ceremonially bury their dead.

Q.225 Sufi silsilas were


1. An unbroken spiritual genealogy to the Prophet
2. Chanting practices taught by the master
3. Rest houses meant for Sufi travellers
4. Holy locations signifying the final demise of Sufi masters
Solution: (a)
Learning: Institutionally, the sufis began to organise communities around the hospice or khanqah (Persian) controlled by
a teaching master known as shaikh (in Arabic), pir or murshid (in Persian).
Sufi silsilas began to crystallise in different parts of the Islamic world around the twelfth century. The word silsila literally
means a chain, signifying a continuous link between master and disciple, stretching as an unbroken spiritual genealogy
to the Prophet.
Chisti silsila, tracing its origin to Khwaja Chisti was one such popular silsila.

Q.226 Consider the following about Virashaiva movement.


1. It was initiated by Basavanna.
2. It began in Karnataka in the mid-eighteenth century.
3. It was against Brahmanical ideas about caste and the treatment of women.
4. It did not support rituals but was inclined towards idol worship as a symbol of the eternal truth.
Select the correct answer using the codes below.
(a) 1 and 3 only
(b) 1, 2 and 4 only
(c) 2, 3 and 4 only
(d) 1, 2, 3 and 4
Solution: (a)
Justification: Statement 1: It was initiated by Basavanna and his companions like Allama Prabhu and Akkamahadevi.
Statement 2: Being associated with the Bhakti movement, it could not be initiated in 18th century. You could have easily
eliminated this option and got the right answer, which can only be (a). It was started in the mid-twelfth century.
Statement 3 and 4: The Virashaivas argued strongly for the equality of all human beings and against Brahmanical ideas
about caste and the treatment of women. They were also against all forms of ritual and idol worship.
Q.227 What characterizes the teachings of Kabir?
1. He did not move place to place to give his teachings and stationed near a hermitage.
2. He believed in institutionalism in religion which leads to a creation of a non-hierarchical social structure.
Which of the above is/are correct?
(a) 1 only
(b) 2 only
(c) Both 1 and 2
(d) None
Solution: (d)
Justification: Statement 1: Kabir was one of the chief exponents of the Bhakti movement in the medieval period.
Similar to all Bhakti saints, he moved from place to place preaching.
Kabir spent much of his time in the company of Hindu ascetics, saints and Muslim sufis. So, he imbibed the tenets of
both the religions and realized the best of both.
Statement 2: As per him, God was to be found neither in temples nor in mosques, neither in Benares nor in Mecca but
only in the heart of a true devotee. This is why he did not support the establishment of a religion.

Q.228 Consider the following about the Nayanars and Alvars.


1. They were sharply critical of the Buddhists and Jainas and preached ardent love of Shiva or Vishnu as the path to salvation.
2. They drew upon the ideals of love and heroism as found in the Sangam literature with the values of bhakti.
3. They went from place to place composing exquisite poems in praise of the deities enshrined in the villages they visited.
Select the correct answer using the codes below.
(a) 2 only
(b) 1 only
(c) 1 and 3 only
(d) 1, 2 and 3
Solution: (d)
Justification: The seventh to ninth centuries saw the emergence of new religious movements, led by the Nayanars (saints devoted to
Shiva) and Alvars (saints devoted to Vishnu) who came from all castes including those considered “untouchable” like the Pulaiyar
and the Panars.
They drew upon ideals from the earliest Tamil literature, Sangam, and preached the language of devotion, love and non-
discrimination.
Other details about them are covered separately in this test.

Q.229 Virashaiva movement initiated by Basavanna and his companions like Allama Prabhu and Akkamahadevi
argued for
1. Equality of all human beings
2. Revival of a new caste system based on divine origin
3. Promoting idol worship
4. Rituals to be conducted by all segments of society
Select the correct answer using the codes below.
(a) 1 only
(b) 2, 3 and 4 only
(c) 1 and 4 only
(d) None of the above
Solution: (a)
Justification: This movement began in Karnataka in the mid-twelfth century.
The Virashaivas argued strongly for the equality of all human beings and against Brahmanical ideas about caste and the treatment
of women.
They were also against all forms of ritual and idol worship.

Q.230 Consider these major bhakti saints and the regions associated with them.
1. Ramdas
2. Nammalvar
3. Surdas
4. Vallabhacharya
What is the correct North-South order of the regions associated with them?
(a) 3412
(b) 1234
(c) 1324
(d) 3124
Solution: (a)
Justification: Statement 1: A 17th-century brahmin saint and spiritual poet of Maharashtra, he is most remembered for his
Advaita Vedantist text, the Dasbodh.
Statement 2: Nammalvar is one of the twelve alwar saints of Tamil Nadu, India, who are known for their affiliation to the
Vaishnava tradition of Hinduism.
Statement 3: He was a 16th-century blind Hindu devotional poet and singer, who was known for his lyrics written in praise of
Krishna. They are usually written in Braj Bhasa, one of the two literary dialects of Hindi, the other being Awadhi.
Statement 4: Also known as Vallabha, was a devotional philosopher, who founded the Krishna- centered Pushti sect of Vaishnavism
in the Braj region of India, and the philosophy of Shuddha advaita.

Q.231 Who, among the following, is a saint NOT associated with the Bhakti movement in India?
(a) Matsyendranath
(b) Vallabhacharya
(c) Ramananda
(d) Namdev
Solution: (a)
Justification: Matsyendranath was associated with the founding of Hatha Yoga in India, and occurs a bit earlier than
the Medieval Bhakti period.
The Bhakti movement regionally developed around different gods and goddesses, such as Vaishnavism (Vishnu),
Shaivism (Shiva), Shaktism (Shakti goddesses), and Smartism.
It originated in the eighth-century Tamil south India (now parts of Tamil Nadu and Kerala), and spread northward

Q.232 Who among the following are Religious reformers of the Medieval India?

1. Kabir
2. Guru Nanak
3. Mira Bai
4. Raskhan
5. Baba Amte
Select the correct answer using the codes below.
(a) 1, 2 and 3 only
(b) 2, 4 and 5 only
(c) 1, 2, 3 and 4 only
(d) 1, 3 and 5
Solution: (c)
Justification: Others were Abdur Rahim Khan e Khannah, Surdas, Kalidas, Chaitanya, Namdev Sufis and Bhaki saints.
They looked upon religion not as a worship but as a loving bond based upon love between the worshipped and the
worshipper.
They made no distinction between caste, creed or religion before God.
They stressed on equality before God, the caste system and attacked institutional religion.
They opposed Sati and Female infanticide.
They aimed to bridge between Hindus and Muslims.

Q.233 Consider the following statements. Tasawwuf


1. stressed the elements of love and devotion as effective means of the realisation of God
2. considered rituals more important than self-discipline in reaching the highest goal
Which of the above is/are correct?
(a) 1 only
(b) 2 only
(c) Both 1 and 2
(d) None
Solution: (a)
Justification: Statement 1 and 2: Tasawwuf is another name for Sufism, as it is known in the Muslim world.
In Sufism, self-discipline was considered an essential condition to gain knowledge of God by sense of perception.
While orthodox Muslims emphasise external conduct and rituals, the Sufis lay stress on inner purity.
According to them one must have the guidance of a pir or guru, without which spiritual development is impossible.

Q.234 With reference to Medieval India, the popular saintly figures like Appar, Sambandar, Sundarar and Manikkavasagar
belonged to which of these sects?
(a) Ajivikas
(b) Nayanars
(c) Alvars
(d) Ligayatas
Solution: (b)
Learning: There were sixty three Nayanars, who belonged to different caste backgrounds such as potters, “untouchable” workers,
peasants, hunters, soldiers, Brahmanas and chiefs.
The best known among them were Appar, Sambandar, Sundarar and Manikkavasagar. There are two sets of compilations of their
songs – Tevaram and Tiruvacakam.
There were 12 Alvars, who came from equally divergent backgrounds, the best known being Periyalvar, his daughter Andal,
Tondaradippodi Alvar and Nammalvar. Their songs were compiled in the DivyaPrabandham.

NORTHERN KINGDOMS
Q.235 The Tripartite Struggle for control of northern India took place in the ninth century. The struggle was between the
(a) Pratihara Empire, the Pala Empire and the Rashtrakuta Empire
(b) Mauryas, Guptas and Harshavardhana
(c) Nandas, Sungas and Kanvas
(d) Satvahanas, Chalukyas and PalaEmpire
Solution: (a)
Learning: The Rastrakutas gained a lot of power by forming a matrimonial relationship with the Gangas and defeated the
kingdom of Vengi.
By the end of the 9th Century the power of the Rastrakutas started to decline along with the Palas.
This was seen as an ideal opportunity by a feudal king Taila II who defeated the Rastrakuta ruler and declared his kingdom there. This
came to be known the Later Chalukya dynasty.
Their kingdom included the states of Karnataka, Konkan and northern Godavari.
By the end of the tripartite struggle, the Pratiharas emerged victorious and established themselves as the rulers of central India.

PALAS
PRATIHARAS
RASHTRAKUTAS

Q.236 Considering the period of rule of the Rashtrakutas, among the following, their earliest known
inscription will most likely be a
(a) 7th CE copper plate grant
(b) 12th CE rock-cut cave
(c) 2nd CE pillar inscription
(d) 4th BC metal carvings
Solution: (a)
Justification: Rashtrakuta was a royal dynasty ruling large parts of the Indian subcontinent between the sixth and 10th
centuries. So, all other options apart from option A are clearly ruled out.
The earliest known Rashtrakuta inscription is a 7th-century copper plate grant detailing their rule from Manapura, a city
in Central or West India.
Other ruling Rashtrakuta clans from the same period mentioned in inscriptions were the kings of Achalapur (modern
Elichpur in Maharashtra) and the rulers of Kannauj

Q.237 Which scholar described the Emperor of the Rashtrakuta dynasty as one of the four great Kings of the World
in the 9th century?
(a) Traveller Marco Polo
(b) Francois Bernier who visited Mughal India
(c) Arab scholar Sulaiman
(d) Persian scholar Al-Biruni
Solution: (c)
Learning: Attracted by the grandiose of the Rashtrakutas, many scholars have attempted to write on them.
Sulaiman was impressed by the state of arts, architecture, economy and military conquests of the Rashtrakutas and
hence described them so.
Al-Masudi has also written on limitary conquests of the Rashtrakutas as the 9th Century was a melting pot of many
military expeditions between Pratiharas, Palas, Rashtrakutas etc.

Q.238 In the Rashtrakuta rule, ‘hiranya-garbha’ sacrifice was done in order to


(a) Donate land to Brahmins
(b) Start excavation for establishing a new empire
(c) To convert the caste of an individual to Kshatriya
(d) Begin a war
Solution: (c)
Justification: These rituals were popular with subordinate samantas who later overpowered their emperors and became rulers.
One such instance was that of the Rashtrakutas in the Deccan. Initially they were subordinate to the Chalukyas of Karnataka.
In the mid-eighth century, Dantidurga, a Rashtrakuta chief, overthrew his Chalukya overlord and performed a ritual called hiranya-
garbha (literally, the golden womb).
When this ritual was performed with the help of Brahmanas, it was thought to lead to the “rebirth” of the sacrificer as a Kshatriya,
even if he was not one by birth.

Q.239 Under Rashtrakuta Empire, Vishayapatis and bhogapatis were


1. Administrators of districts and its sub-divisions respectively
2. Accountants of the royal treasury
3. Religious priests who performed sacrificial ceremonies
4. Land guards who protected the extent of the empire
Solution: (a)
Learning: The Rashtrakuta Empire was divided into several provinces called rashtras under the control of
rashtrapatis.
They were further divided into vishayas or districts governed by vishayapatis.
The next subdivision was bhukti consisting of 50 to 70 villages under the control of bhogapatis.
These officers were directly appointed by the central government. The village administration was carried
on by the village headmen.
However, the village assemblies played a significant role in the village administration.

Q.240 In the context of Ancient India, Amogavarsha’s work Kavirajamarga is significant because?
(a) It was the first poetic work in Kannada language.
(b) The largest repository of royal assets was commissioned in this work.
(c) All artists of the empire contributed to this work.
(d) It was a prose written in the middle of a battle ground.
Solution: (a)
Learning: The Kannada literature saw its beginning during the period of the Rashtrakutas. Amogavarsha’s Kavirajamarga was the
first poetic work in Kannada language.
The name literally means "Royal Path for Poets" and was written as a guide book for poets and scholars (Kavishiksha).
Q.241 Consider the following about the Kakatiyas.
(a) Kakatiyas were worshippers of Shiva.
(b) They were the historic overlords and progenitors of the Rashtrakutas.
(c) The Kakatiya polity was based a monarchical system that preferred women over men in rulership.
(d) Their empire was conquered by the Delhi Sultanate.
Select the correct answer using the codes below.
(a) 1 and 4 only
(b) 2, 3 and 4 only
(c) 1 and 3 only
(d) 1, 2, 3 and 4
Solution: (a)
Justification: Statement 1: They are known as worshippers of Svayambhudeva, i.e., Siva.
Statement 2: During the 9th and 10th century, they were the subordinates of the Rashtrakutas. After the overthrow of
the Rashtrakuta power by the Western Chalukyas of Kalyani, the Kakatiyas became their feudatories or
Mahamandalesvaras under the Chalukyas of Kalyani.
Statement 3: The Kakatiya ruler was the pivot of the entire administrative structure, yet he was not an absolute
autocrat. Generally, in the process of succession, they followed the law of primogeniture and a woman coming to the
throne was a remarkable exception.
Statement 4: The Kakatiya dynasty was a South Indian dynasty but was eventually conquered by the Delhi Sultanate.

IMPERIAL CHOLAS
Q.242 Rathakaras, mentioned in a twelfth-century inscription from Uyyakondan Udaiyar, were engaged in the profession of
(a) temple architecture
(b) preparing metal equipment used to perform sacrifices
(c) making chariots
(d) manual scavenging
Solution: (c)
Learning: This inscription found in Tiruchirapalli taluka (in present-day Tamil Nadu), describes the deliberations in a sabha of
Brahmanas.
They deliberated on the status of a group known as rathakaras (literally, chariot makers).
They laid down their occupations, which were to include architecture, building coaches and chariots, erecting gateways for
temples with images in them, preparing wooden equipment used to perform sacrifices, building mandapas, making jewels for
the king etc.

www.insightsias.com
© InsightsActiveLearning| Allrightsreserved.Youmay notreproduce, distributeorexploitthecontentsin anyformwithoutwritten
Q.243 The inscriptions of the Cholas mention various kinds of taxes. Which of these is the most frequently mentioned tax in
theseinscriptions?
(a) Ladamol
(b) Vetti
(c) Surnam
(d) Rumolna
Solution: (b)
Learning: Their inscriptions refer to more than 400 terms for different kinds of taxes.
The most frequently mentioned tax is vetti, taken not in cash but in the form of forced labour, and kadamai, or land revenue.
There were also taxes on thatching the house, the use of a ladder to climb palm trees, a cess on succession to family
property,etc.

Q.244 The system of village autonomy with sabhas and their committees developed through the ages and reached
its culmination during the Chola rule. A village was divided in several wards, what was/were the qualifications
to become a ward member?
1. A woman could not become a ward member.
2. Only those who have completed seventy years of age
3. Have knowledge of Vedas
4. Owns a residence
Select the correct answer using the codes below.
(a) 3 and 4 only
(b) 1, 2 and 3 only
(c) 1 and 4 only
(d) 1, 2, 3 and 4
Solution: (a)
Learning: That village was divided into thirty wards and each was to nominate its members to the village council.
The qualifications to become a ward member were:
Ownership of at least one fourth veli of land.
Own residence.
Above thirty years and below seventy years of age.
Knowledge of Vedas.
However, certainnormsof disqualificationwere alsomentioned inthe inscriptions. Theywere:
Those who had been members of the committees for the past three years.
Those who had failed to submit accounts as committee members.
Those who had committed sins.
Those who had stolen the property of others
Two inscriptions belonging to the period of Parantaka I found at Uttiramerur provide details of the formation and functions
of village councils.

Q.245 With reference to the Chola empire, consider the following statements.
1. Settlements of peasants were known as ur.
2. Nadu was a group of villages that often administered collection of taxes.
3. Muvendavelanwas atitle given to uppercastes thatworked for thewelfareof backward castes.
Select the correct answer using the codes below.
(a) 1and2only
(b) 2and3only
(c) 1 only
(d) 3 only
Solution: (a)
Justification:Statement1:Theybecameprosperouswiththespreadofirrigationagriculture.
Statement 2: Groups of such villages formed larger units called nadu. The village council and the nadu had several
administrative functions including dispensing justice and collecting taxes.
Rich peasants of the Vellala caste exercised considerable control over the affairs of the nadu under the supervision of the
central Chola government.
Statement 3: The Chola kings gave some rich landowners titles like muvendavelan (a velan or peasant serving three kings),
araiyar (chief), etc. as markers of respect, and entrusted them with important offices of the state at the centre.

Q.246 Tamil culture reached its peak under Cholas. Consider the following matches of major text works and their
description.
1. Periya Puranam: Lives of the Nayanars
2. Kalingattupparani: Kalinga war fought by Kulotunga
3. Moovarula: Life of Chola kings
4. Kalladam: Work on Tamil grammar
Select the correct answer using the codes below.
(a) 2 and 3 only
(b) 1, 2 and 3 only
(c) 3 and 4 only
(d) D. 1, 2, 3 and 4
Solution: (d)
Justification: Statement 1: Sometimes called Tiruttontarpuranam, the Purana of the Holy Devotees, is a Tamil
poetic account depicting the legendary lives of the sixty-three Nayanars, the canonical poets of Tamil Shaivism.
Statement 2: Jayankondar’s Kalingattupparani describes the Kalinga war fought by Kulotunga I.
Statement 3: Ottakoothar (another name) was a Tamil court poet to three legendary early Chola kings, namely
Vikrama Chola, Kulotunga II and Rajaraja II.
Statement 4: The works on Tamil grammar like Kalladam by Kalladanar, Yapperungalam by Amirthasagarar, a
Jain, Nannul by Pavanandhi and Virasoliyam by Buddhamitra were the products of the Chola age.

LIFE IN INDIA
DELHI SULTANATE

Q.247 Unani system of medicinewas introduced to India


(a) In the later Vedic period
(b) With the establishment of Delhi Sultanate
(c) During the reign of Mauryas
(d) During the rule of the Kushanas and Indo Greeks
Solution: (b)
Learning: Unani system of medicine is the term for Perso-Arabic traditional medicine as practiced in Mughal India
and in Muslim culture in South Asia and modern day Central Asia.
It had originated in Greece. Hippocrates is known as the father of this system of medicine.
The term Unani (Greek) means Perso-Arabic system of medicine. It was introduced to India in 13th century with
the establishment of Delhi Sultanate and it took its own course of development during Mughal Empire after it was
influenced by Indian medical teachings of Sushruta and Charaka.

Q.248 Under the Delhi sultanate muqtis or walis


(a) Maintained law and order and collected the land revenue
(b) Managed financial accounts of central treasury
(c) Lead mercenary military units in special warfares
(d) Administered village assemblies
Solution: (a)
Learning: The provinces under the Delhi Sultanate were called iqtas. They were initially under the control of the
nobles.
But the governors of the provinces were called the muqtis or walis. They were to maintain law and order and
collect the land revenue.
The provinces were divided into shiqs and the next division was pargana. The shiq was under the control of
shiqdar.
The pargana comprising a number of villages was headed by amil.
The village remained the basic unit of the administration. The village headman was known as muqaddam or
chaudhri. The village accountant was called patwari.

Q.249 Which of the following dynasties do NOT form a part of the Delhi Sultanate?
(a) Khalji
(b) Lodi
(c) Mamluk
(d) Tomara
Solution: (d)
Learning: This is a lateral explanation.
During and in the Delhi Sultanate, there was a synthesis of Indian civilization with that of Islamic civilization, and
the further integration of the Indian subcontinent with a growing world system and wider international networks
spanning large parts of Afro-Eurasia, which had a significant impacton Indian culture and society, as well as the
wider world.
The time of their rule included the earliest forms of Indo-Islamic architecture, increased growth rates in India's
population and economy, and the emergence of the Hindi-Urdu language.
The Delhi Sultanate was also responsible for repelling the Mongol Empire's potentially devastating invasions of
India in the 13th and 14th centuries.

SLAVES
KHALJIS

Q.250 Consider the following statements.


1. Alauddin Khalji was the first Sultan of Delhi who ordered for the measurement of land.
2. Firoz Tughlaq banned the iqta system of land due to its hereditary transmission from one heir to another.
Which of the above is/are correct?
(a) 1 only
(b) 2 only
(c) Both 1 and 2
(d) None
Solution: (a)
Justification: Statement 1: Apart from market reforms, Alauddin Khalji took important steps in the land revenue
administration, not sparing even big landlords from paying land tax.
Land revenue was collected in cash in order to enable the Sultan to pay the soldiers in cash. His land revenue
reforms provided a basis for the future reforms of Sher Shah and Akbar.Statement 2: The reign of Firoz Tughlaq
was more notable for his administration. He strictly followed the advice of the ulemas in running the
administration.
He pleased the nobles and assured hereditary succession to their properties. Thus the iqta system was not only
revived but also it was made hereditary.
Q.251 It was a system whereby nobles were granted the rights to hold a jagir, which meant revenue
assignments for services rendered by them but the authority bestowed upon them was under the direct
control of the king. This system of Medieval India is known as
(a) Muzaffarid
(b) Mansabdari
(c) Dagh and Hulia system
(d) Iqtadari
Solution: (b)
Justification: Option C: Sultans like Ala-ud-Din Khilji introduced the practice of ‘Dag’ (branding of horses) and
‘huliya’ (descriptive rolls of soldiers). This was not a military system in itself.
Learning: The word mansab means rank or position. The system, hence, determined the rank of a government
official.
Every civil and military officer was given a ‘mansab’ and different numbers which could be
divided by ten were used for ranking officers. It was also meant for fixing the salaries and allowances of officers.
A mansabdar could be asked to perform any civil or military service

Q.252 During the reign of Alauddin Khalji


1. Different markets such as those of commodities, slaves and horses were integrated into a single market
2. Land reforms were introduced where taxes were decided on the measurement of land
Which of the above is/are correct?
(a) 1 only
(b) 2 only
(c) Both 1 and 2
(d) None
Solution: (b)
Learning: Market reforms:
Alauddin Khalji established four separate markets in Delhi, one for grain; another for cloth, sugar, dried fruits,
butter and oil; a third for horses, slaves and cattle; and a fourth for miscellaneous commodities.
Each market was under the control of a high officer called Shahna-i-Mandi.
The supply of grain was ensured by holding stocks in government store-houses. Regulations were issued to fix the
price of all commodities.
Land reforms:
Apart from market reforms, Alauddin Khalji took important steps in the land revenue administration.
He was the first Sultan of Delhi who ordered for the measurement of land. Even the big landlords could not
escape from paying land tax.
Land revenue was collected in cash in order to enable the Sultan to pay the soldiers in cash. His land revenue
reforms provided a basis for the future reforms of Sher Shah and Akbar.

Q.253 Which of these systems were popularly introduced by Alauddin Khalji?


1. Branding of horses
2. Maintaining a descriptive list of soldiers
3. Paying army in kind rather than cash
4. Market based pricing for all products
Select the correct answer using the codes below.
(a) 1 and 2 only
(b) 3 and 4 only
(c) 2 and 4 only
(d) 1 and 3 only
Solution: (a)
Justification:
Statement 1, 2 and 3: Alauddin Khalji maintained a large permanent standing army and paid them in cash from
the royal treasury. He introduced the system of dagh (branding of horses) and prepared huliya (descriptive list of
soldiers). In order to ensure maximum efficiency, a strict review of army from time to time was carried out.
Statement 4: The introduction of paying salaries in cash to the soldiers led to price regulations popularly called as
Market Reforms. Markets were under the control of a high officer called Shahna-i- Mandi. The supply of grain was
ensured by holding stocks in government store-houses. Regulations were issued to fix the price of all
commodities.
Learning: Every merchant was registered under the Market department. There were secret agents called
munhiyans who sent reports to the Sultan regarding the functioning of these markets. Violation of regulations
was severely punished. Harsh punishment was given if any shopkeeper charged a higher price, or tried to cheat by
using false weights and measures. Even during the famine the same price was maintained.

Q.254 In Delhi Sultanate, the Sultan was assisted by a number of departments and officials in his
administration. Consider the following with reference to these departments:
1. Diwani Insha dealt with the correspondence between the ruler and the officials.
2. Diwani Ariz gave grants for the construction and maintenance of mosques, tombs and madrasas.
Which of the above is/arecorrect?
(a) 1 only
(b) 2 only
(c) Both 1 and 2
(d) None
Solution: (a)
Justification:Statement 1: It is correct.
Statement 2: The military department was called Diwani Ariz. It was headed by Ariz-i-mumalik. He was responsible for
recruiting the soldiers and administering the military department.
Learning: Under the Delhi sultanate, the post of Naib was the most powerful one. The Naib practicallyenjoyedallthepowers
oftheSultanandexercisedgeneralcontroloverallthe departments.
NexttohimwastheWazirwho washeadingthefinancedepartment calledDiwaniWizarat.
Diwani Rasalat was the department of religious affairs. It was headed by chief Sadr. Grants were made by this department
for religious construction.

TUGHLAQS

Q.255 His rule is significant for the introduction of token currency. He took keen interest to circulate
gold and silver coins and introduced the gold coin as Dinar in India. He was
(a) Tughlaq
(b) Mir Kasim
(c) Aurangjeb
(d) Khiliji
Solution: (a)
Learning: He understood the importance of currency as a medium of commercial exchange and that is why he
took keen interest to circulate gold and silver coins.
The gold coin was introduced as Dinar. Tughlaq’s silver coin was named Adl. However, it was difficult to maintain
the supply of gold and silver coins on a large scale.
So, Tughlaq replaced those coins and started the circulation of copper and brass coins as the token currency
which had the same value of gold or silver coins in 1330-32 CE.
He was well aware that the state had to act as a responsible guarantor for the token money by ensuring high
degree of security which will prevent others from making fake currencies.

Q.256 The first person to translate Sanskrit works into Persian was
(a) Amir Khusrau
(b) Zia Nakshabi
(c) Tuti Namah
(d) Rahman Biddur
Solution: (b)
Learning: The first translation of Sanskrit works into Persian can be traced back to the 6th century.
Zia Nakshabi’s book Tuti Namah (book of the parrot), written in the time of Muhammad Tughluq, was a
Persian translation of Sanskrit stories which were related by a parrot to a woman whose husband had gone
on a journey.
Later, in the time of Firoz, Sanskrit books on medicine and music were translated into Persian

Q.257 The Fatwa-i-Jahandari is a work of Ziauddin Barani dealing with


(a) Economic principles of Mughal Empire
(b) Political ideals to be pursued by a Muslim ruler
(c) Crime and punishment in Islam
(d) Accounts of travellers to South Asia
Solution: (b)
Learning: Barani was a Muslim political thinker of the Delhi Sultanate located in present-day North India during
Muhammad bin Tughlaq and Firuz Shah's reign.
The Fatwa-i-Jahandari is a work containing the political ideals to be pursued by a Muslim ruler in order to earn religious
merit and the gratitude of his subjects.
HewasbestknownforcomposingtheTarikh-i-FiruzShahi,aworkonmedievalIndia,which coverstheperiodfromthereign
ofGhiyasuddinBalbantothefirstsixyearsofreignofFiruz Shah Tughluq and the Fatwa-i-Jahandari which details the caste
system among Muslims in South Asia.

Q.258 In 1329-30 Muhammad bin Tughlaq introduced a token copper currency in place of silver, but banned it later
and promised to exchange silver coins for these copper coins. This was done because
1. Copper was scarce in the empire, but had huge demand in the world.
2. Token currency was trampled and disrespected by people.
3. Goldsmiths began to forge the token coins on a large scale
4. Treasury became empty with issue of token coins.
Solution: (c)
Learning: There was a shortage of silver through out the world in the fourteenth century.
Kublai Khan issued paper money in China. In the same manner, Muhammad bin Tughlaq issued copper coins at par with the
value of the silver tanka coins.
But he was not able to prevent forging the new coins. The goldsmiths beganto forgethe token coins on a large scale. Soon
the new coins were not accepted in the markets due to its widescale duplication and thuscredibility.
Finally, Muhammad bin Tughlaq stopped the circulation of token currency and promised to exchange silver coins for the
copper coins.
Many people exchanged the new coins but the treasury became empty.
AccordingtheBarani,theheapofcoppercoinsremainedlyingonroadsideinTughlaqabad.

Q.259 In medieval India, takkavi loans were given in order to


1. Improve agriculture
2. Build religious shrines
3. Construct flood saving embankments in cities
4. Serve sovereign debt
Solution: (a)
Learning: Takkavi loans were a measure of reprieve taken after few suppressing moves of Tughlaq.
In order to overcome financial difficulties in his empire, Muhammad bin Tughlaq increased the land revenue onthe farmers
of Doab (land between Ganges and Yamuna rivers).
It was an excessive and arbitrary step on the farmers. A severe famine was also ravaging that regionatthattime.Ithad
resultedinaseriouspeasantrevolts.Therevoltswerecrushed.
However, the Sultan realized later that adequate relief measures and the promotion of agriculture were the real solution to
the problem.
He launched a scheme by which takkavi loans (loans for cultivation) were given to the farmers to buy seed and to extend
agriculture.
Q.260 Firoz Tughlaq’s welfare state included which of these administrative measures?
1. Treating Hindus and Muslims alike in terms of political and economic rights
2. Abolition of the practice ofslavery
3. Establishment of government department to take care of orphans and widows
Select the correct answer using the codes below.
(a) 1and3only
(b) 2and3only
(c) 3 only
(d) 1 and 2 only
Solution: (c)
Justification: Statement 1: As Firoz was guided by the ulemas, he was intolerant towards Shia Muslims and Sufis. He treated
Hindus as second grade citizens and imposed Jiziya. In this respect he was the precursor of Sikandar Lodi and
Aurangazeb.
Statement 2: Also he increased the number of slaves by capturing the defeated soldiers and young persons. So, 2 is wrong.
Statement 3: A new department called Diwan-i-Khairat was created to take care of orphans and widows. Free hospitals and
marriage bureaus for poor Muslims were also established.

SAYYID
LODI
SOCIAL LIFE AND DEVELOPMENT

Q.261 Sarais, in Medieval India, were meant to provide


(a) Temporary accommodation to travellers and pilgrims
(b) Discussion places for ascetics
(c) Space for addressing local grievances
(d) Tributes to royal householders
Solution: (a)
Learning: A very interesting feature of medieval India was the sarais which ringed cities and dotted the vast space
of the Indian subcontinent.
Sarais were largely built on a simple square or rectangular plan and were meant to provide temporary
accommodation to Indian and foreign travellers, pilgrims, merchants, traders, etc.
In effect, sarais were public domains which thronged with people of varied cultural backgrounds.
This led to cross-cultural interaction, influences and syncretic tendencies in the cultural mores of the times and at
the level of the people.

Q.262 In the context of Medieval India, “Kunjali Marakkar” was


(a) Caliphate campaigns in India when they tried to move eastward
(b) The title given to the Muslim naval chief of the Zamorin
(c) An earlier name for the independent Maratha zone carved out by Shivaji Bhosale
(d) A trading strategy of the Ahom Kingdom
Solution: (b)
Learning: It was titled to the Hindu king of Calicut, in present-day state of Kerala, during the 16th century.
The Marakkars are credited with organizing the first naval defence of the Indian coast, to be later succeeded in
the 18th century by the Maratha Sarkhel Kanhoji Angre.
There were four major Kunhalis who played a part in the Zamorin's naval wars with the Portuguese from 1502 to
1600.
Of the four Marakkars, Kunjali Marakkar II is the most famous.
Q.263 One Thousand and One Nights is a collection of Middle Eastern folk tales compiled in Arabic during
(a) The Islamic Golden Age ranging from 8th to 13th century
(b) 13th to 16th century CE by scholars from Western Indian subcontinent
(c) First Century CE by translators from Mesopotamia
(d) None of the above
Solution: (a)
Learning: It is often known in English as the Arabian Nights, from the first English-language edition (1706), which
rendered the title as The Arabian Nights' Entertainment.
The Islamic Golden Age is the era in the history of Islam, traditionally dated from the 8th century to
the 13th century, during which much of the historically Islamic world was ruled by various caliphates, and science,
economic development and cultural works flourished.
The work was collected over many centuries by various authors, translators, and scholars across West, Central,
and South Asia and North Africa.

Q.264 Famous in India by the name of "City of Weavers" and "Textile City", it is also known as the "cast-off
capital" due to being "the global centre for recycling textiles". The place has witnessed three important
historical battles in Medieval India. The best guess for this place, from among the following, would be?
(a) Hyderabad
(b) Surat
(c) Tarain
(d) Panipat
Solution: (d)
Learning: Panipat is called the city of weavers, as it produces textiles and carpets. It is the biggest centre for
quality blankets and carpets in India and has a hand loom weaving industry.
Panipat city is the biggest centre of "shoddy yarn" in the World. Blankets prepared through hand looms and
power looms are sent to soldiers.
The Battle of Panipat Memorial society, set up by the Government of Haryana, highlights the major events that
took place for over two hundred years which made Panipat a place of great historical importance.

Q.265 In late 1334, the famous traveller, Ibn Battuta went to Delhi to seek official employment from the Delhi
sultanate. He was employed as a
(a) Judge
(b) Translator
(c) Military commander
(d) Doctor
Solution: (a)
Learning: On the strength of his years of study in Mecca, Ibn Battuta was appointed a qadi, or judge, by the
sultan. However, he found it difficult to enforce Islamic law beyond the sultan's court in Delhi, due to lack of
Islamic appeal in India.
Option B: Because he didn’t speak Persian well, he was given two assistants. So, he could not have been a
translator, which means Option B is incorrect.
He also joined the Sultan and high officials on elaborate hunting expeditions.

Q.266 In Medieval India, Muqaddam or mandal was


(a) Royal adviser to the Emperor
(b) Panchayat head
(c) Revenue officer
(d) Police chief
Solution: (b)
Learning: Some sources suggest that the headman was chosen through the consensus of the village elders, and
that this choice had to be ratified by the zamindar.
Headmen held office as long as they enjoyed the confidence of the village elders, failing which they could be
dismissed by them.
The chief function of the headman was to supervise the preparation of village accounts, assisted by the
accountant or patwari of the panchayat.

Q.267 Who among the following translated Patanjali’s work on Sanskrit grammar into Arabic?
(a) Al-Biruni
(b) Amir Khusrau
(c) Ibn Abi Ishaq
(d) Al-Idrisi
Solution: (a)
Learning: Al-Biruni spent years in the company of Brahmana priests and scholars, learning Sanskrit, and studying
religious and philosophical texts.
Al-Biruni’s expertise in several languages allowed him to compare languages and translate texts.
He translated several Sanskrit works, including Patanjali’s work on grammar, into Arabic. For his Brahmana
friends, he translated the works of Euclid (a Greek mathematician) into Sanskrit.

Q.268 With reference to Medieval India village panchayats, consider the following statements.
1. These used to be an assembly of elders.
2. The decisions made by these panchayats were binding on the members.
3. Village Headmen held office until his death post which a new headman was nominated by the state.
4. The panchayat derived its funds from contributions made by individuals to a common financial pool.
Select the correct answer using the codes below.
(a) 1, 2 and 4 only
(b) 2 and 3 only
(c) 3 and 4 only
(d) 1 and 2 only
Solution: (a)
Justification: Statement 1 and 2: The village panchayat was an assembly of elders, usually important people of
the village with hereditary rights over their property
An oligarchy, the panchayat represented various castes and communities in the village, though the village menial-
cum-agricultural worker was unlikely to be represented there. The decisions made by these panchayats were
binding on the members.
Statement 3 and 4: The panchayat was headed by a headman known as muqaddam or mandal. Some sources
suggest that the headman was chosen through the consensus of the village elders, and that this choice had to be
ratified by the zamindar.
Headmen held office as long as they enjoyed the confidence of the village elders, failing which they could be
dismissed by them.
The chief function of the headman was to supervise the preparation of village accounts, assisted by the
accountant or patwari of the panchayat.

Q.269 With reference to Delhi Sultanate, who were the bandagan?


(a) Royal spies posted to other kingdoms
(b) Translators and scribes hired by the royalty
(c) Cavalrymen deputed to hilly regions
(d) Special slaves purchased for military service
Solution: (d)
Justification: The consolidation of a kingdom as vast as the Delhi Sultanate needed reliable governors and administrators.
Rather than appointing aristocrats and landed chieftains as governors, the early Delhi Sultans, especially Iltutmish, favoured
their special slaves purchased for military service, called bandagan in Persian.
They were carefully trained to man some of the most important political offices in the kingdom.
Since they were totally dependent upon their master, the Sultan could trust and rely upon them.
Q.270 Consider the following statements about the authors of tawrikhs.
1. They lived mainly in cities.
2. These authors prescribed elitism rather than merit to preserve purity in the royal lineage.
3. They were sharply critical of the Sultans and the bureaucratic establishment.
Select the correct answer using the codes below.
(a) 1 only
(b) 2 and 3 only
(c) 2 only
(d) 3 only
Solution: (a)
Justification: Although inscriptions, coins and architecture provide a lot of information, especially valuableare“histories”,tarikh
(singular)/tawarikh(plural),writteninPersian,thelanguageof administration under the DelhiSultans.
Statement 1: The authors of tawarikh were learned men: secretaries, administrators, poets and courtiers,whobothrecountedevents
andadvisedrulersongovernance,emphasisingthe importance of just rule.
The authors of tawarikh lived in cities (mainly Delhi) and hardly ever in villages.
Statement 2: These authors advised rulers on the need to preserve an “ideal” social order based on birthright and gender
distinctions.
Statement 3: They often wrote their histories for Sultans in the hope of rich rewards.

Q.271 Consider the following about Amir Khusrau, who was an iconic figure in the cultural history of the Indian
subcontinent.
1. He wrote poetry only inPersian.
2. He is regarded as the "father of qawwali".
3. He rejected Sufism as an unnecessary appendage of Islam.
Select the correct answer using the codes below.
(a) 1 and 3 only
(b) 2 only
(c) 3 only
(d) 1, 2 and 3 only
Solution: (b)
Justification: Statement 1: He was an Indian Sufi musician, poet and scholar.
He was a mystic and a spiritual disciple of Nizamuddin Auliya of Delhi. He wrote poetry primarily in Persian, but also in
Hindavi.
Statement 2: Khusrow is credited with fusing the Persian, Arabic, Turkish, and Indian musical traditions in the late 13th
century to create qawwali, a form of Sufi devotional music.
He also introduced the ghazal style of song into India, both of which still exist widely in India and Pakistan.
He also wrote a war ballad in Punjabi. In addition, he spoke Arabic and Sanskrit.
His poetry is still sung today at Sufi shrines throughout Pakistan and India.

Q.272 Consider the following about the travel accounts of the Geographer Al-Idrisi.
1. He mentions sailing time across the world as two days.
2. He labelled silk found in Quanzhou as the best.
3. As per him, Indian subcontinent was circular in shape.
Select the correct answer using the codes below.
(a) 2 only
(b) 1and3only
(c) 2and3only
(d) 1and2only
Solution: (a)
Justification: Statement 1: Al-Idrisi in his famous Tabula Rogeriana mentioned Irlandah-al-Kabirah (Great Ireland).
According to him, "from the extremity of Iceland to that of Great Ireland," the sailing time was "one day."
Historians, however, note that both al-Idrisi and the Norse tend to understate distances.
Statement 2: Al-Idrisi mentioned that Chinese junks carried leather, swords, iron and silk. He mentions the glassware of the
city of Hangzhou and labels Quanzhou's silk as the best.
Statement 3: Al-Idrisi produced a compendium of geographical information with the title Kitab nuzhat al-mushtaq
fi'khtiraq al-'afaq.
The title has been translated as ‘The book of pleasant journeys into faraway lands’ or ‘The pleasure of him who longs to
cross the horizons’.

Q.273 Mallahzadas, in Medieval India, were


1. Agricultural labourers in state-owned collective farms
2. Considered to be belong to the highest occupational caste
3. From a different race than all other castes in India
Select the correct answer using the codes below.
(a) 1 and 2 only
(b) 2 and 3 only
(c) 1 and 3 only
(d) None of the above
Solution: (d)
Justification & Learning: In Medieval India, deep inequities on the basis of caste and other caste like
distinctions meant that the cultivators were a highly heterogeneous group.
Despite the abundance of cultivable land, certain caste groups were assigned menial tasks and thus relegated to
poverty.
Among those who tilled the land, there was a sizeable number who worked as menials or agricultural labourers
(majur).
In Muslim Communities menials like the halalkhoran (scavengers) were housed outside the boundaries of the
village; similarly, the mallahzadas (literally, sons of boatmen) in Bihar were comparable to slaves.
There was a direct correlation between caste, poverty and social status at the lower strata of society.

Q.274 With reference to Al-Biruni, consider the following:


1. He travelled to the South Asia and authored a study of Indian culture.
2. Mahmud Ghazni entrusted the writing of Kitab-al Hind to Al-Biruni.
3. He was given the title "founder of Indology".
Select the correct answer using the codes below.
(a) 1 only
(b) 2and3only
(c) 1and3only
(d) 1, 2 and 3
Solution: (d)
Justification: Statement 1: In 1017 he travelled to the South Asia and authored a study of Indian culture (Tahqiq ma li-l-hind)
after exploring the Hinduism practised in India.
Statement 2: Much of the wealth Mahmud carried away was used to create a splendid capital city at Ghazni.
He was interested in finding out more about the people he conquered, and entrusted a scholar named al-Biruni to write an
account of the subcontinent.
This Arabic work, known as the Kitab-al Hind, remains an important source for historians. He consulted Sanskrit scholars to
prepare this account.
Statement 3: Al-Biruni is regarded as one of the greatest scholars of the medieval Islamic era and was well versed in physics,
mathematics, astronomy, and natural sciences, and also distinguished himself as a historian, chronologist and linguist.
Al-Biruni contributed to the introduction of the experimental scientific method to mechanics, unified statics anddynamics
into the science ofmechanics, andcombined the fields of hydrostatics with dynamics to create hydrodynamics.
He came up with different methods for exploring densities, weight, and even gravity.
Q.275 After consolidating their position in India, the Delhi Sultans introduced reforms in the land revenue
administration. In this context, inam lands were assigned to
1. officials against the payment for their services
2. religious leaders or religious institutions
3. courtyards for mosques
4. expand the market in the hinterland of the empire
Solution: (b)
Learning: The lands were classified into three categories:
iqta land – lands assigned to officials as iqtas instead of payment for their services.
khalisa land – land under the direct control of the Sultan and the revenues collected were spent for the
maintenance of royal court and royal household.
inam land – land assigned or granted to religious leaders or religious institutions.
However, despite the land reforms, the peasantry paid one third of their produce as land revenue, and sometimes
even one half of the produce. They also paid other taxes and always led a hand-to- mouth living.

Q.276 In the context of Medieval India, kudirai chettis were


1. horse merchants
2. rebel nomads
3. port and harbour controllers
4. owner of private military stations
Solution: (a)
Learning: As warfare during these times depended upon effective cavalry, the import of horses from Arabia and
Central Asia was very important for rival kingdoms.
This trade was initially controlled by Arab traders. Local communities of merchants known as kudirai chettis or
horse merchants also participated in these exchanges.
From 1498 other actors appeared on the scene. These were the Portuguese, who arrived on the west coast of the
subcontinent and attempted to establish trading and military stations.
Their superior military technology, especially the use of muskets, enabled them to become important players in
the tangled politics of the period.

VIJAYANAGARA EMPIRE

Q.277 The amaram lands in the Vijayanagar Empire were


(a) State forest Reserves
(b) Land granted to army officers
(c) Donations made to the temple sites
(d) Lands dedicated for industries
Solution: (b)
Learning: The Vijayanagar army was well-organized and efficient. It consisted of the cavalry, infantry, artillery and
elephants.
The top-grade officers of the army were known as Nayaks or Poligars. They were granted land in lieu of their services.
These lands were called amaram. Soldiers were usually paid in cash.
That army was given considerable focus is clear from the fact that High-breed horses were procured from foreign
traders.

Q.278 Amukthamalyada, Gangadevi’s Maduravijayam and Allasani Peddanna’s Manucharitam are


some of the indigenous literature of the period of
(a) Cholas
(b) Vijayanagar Empire
(c) Pallavas
(d) Sangam age
Solution: (b)
Learning: Krishna Deva Raya, though a Vaishnavaite, he respected all religions. He was a great patron of
literature and art and he was known as Andhra Bhoja.
Eight eminent scholars known as Ashtadiggajas were at his royal court. Allasani Peddanna was the greatest and he
was called Andhrakavita Pitamaga. His important works include Manucharitam and Harikathasaram.
Pingali Suranna and Tenali Ramakrishna were other important scholars. Krishna Deva Raya himself authored a
Telugu work, Amukthamalyadha and Sanskrit works, Jambavati Kalyanam and Ushaparinayam.

Q.279 Krishnadeva Raya’s rule was characterised by


1. Absence of any military expeditions
2. Acquisition of the land between the Tungabhadra and Krishna rivers
3. Building of temples and adding gopurams to many important south Indian temples
Select the correct answer using the codes below.
(a) 1 and 2 only
(b) 3 only
(c) 2 and 3 only
(d) 1 only
Solution: (c)
Answer Justification :
Justification: Krishnadeva Raya’s rule was characterised by expansion and consolidation.
This was the time when the land between the Tungabhadra and Krishna rivers (the Raichur doab) was acquired
(1512), the rulers of Orissa were subdued (1514) and severe defeats were inflicted on the Sultan of Bijapur (1520).
Although the kingdom remained in a constant state of military preparedness, it flourished under conditions of
unparalleled peace and prosperity.
Krishnadeva Raya is credited with building some fine temples and adding impressive gopurams to many important
south Indian temples.
He also founded a suburban township near Vijayanagara called Nagalapuram after his mother. Some of the most
detailed descriptions of Vijayanagara come from his time or just after.

Q.280 Which of these travellers wrote of the Emperor of the Vijayanagara Empire as the most powerful ruler of India
in the 15th century?
(a) The Italian traveller Niccolo de Conti
(b) Venetian merchant traveller, Marco Polo
(c) Moroccan scholar, Ibn Battuta
(d) The Arab historian, Al-Mas‘udi
Solution: (a)
Learning: He was one of the human sources used to create the 1450 Fra Mauro map, which indicated that there
was a sea route from Europe around Africa to India.
He travelled in India to "Pacamuria", "Helly" and Vijayanagar, capital of the Deccan before 1555.
It was in India that he coined the phrase 'Italian of the East' to refer to the Telugu language, which he found had
words ending with vowels, similar to Italian.

Q.281 The amara-nayaka system was a major political innovation of the


(a) Vijayanagara Empire
(b) Cholas and Cheras
(c) Pandya Empire
(d) Hoysala Empire
Solution: (a)
Learning: The amara-nayakas were military commanders who were given territories to govern.
Amara-nayakas collected taxes and other dues from peasants, craftspersons and traders in the area.
They retained part of the revenue for personal use and for maintaining a stipulated contingent of horses and
elephants.
These contingents provided the Vijayanagara kings with an effective fighting force with which they brought the
entire southern peninsula under their control.
The amara-nayakas sent tribute to the king annually and personally appeared in the royal court with gifts to
express their loyalty.

Q.282 Consider the following with reference to the traveller, Marco Polo.
1. He authored the ‘Book of the Marvels of the World’.
2. He was the first European to reach China.
3. He was the first European cartographer.
Select the correct answer using the codes below.
(a) 1 only
(b) 2 and 3 only
(c) 2 only
(d) None of the above
Solution: (a)
Justification: His travels are recorded in Book of the Marvels of the World, also known as The Travels of Marco
Polo, c. 1300), a book that described to Europeans the wealth and great size of China.
Statement 2: Marco Polo was not the first European to reach China (see Europeans in Medieval China), but he
was the first to leave a detailed chronicle of his experience.
Statement 3: This book inspired Christopher Columbus and many other travellers. There is a substantial literature
based on Polo's writings; he also influenced European cartography, leading to the introduction of the Fra Mauro
map.

Q.283 Which of the following can be related to the origins of the Vijayanagar Empire?
1. Sangama dynasty
2. Tungabhadra River
3. Krishnadeva Raya
Select the correct answer using the codes below.
(a) 2 only
(b) 1 and 2 only
(c) 2 and 3 only
(d) 1 and 3 only
Solution: (b)
Justification: It was founded in 1336 by Harihara and Bukka of the Sangama dynasty. They were originally served
under the Kakatiya rulers of Warangal.
Then they went to Kampili where they were imprisoned and converted to Islam. Later, they returned to the Hindu
fold at the initiative of the saint Vidyaranya.
They also proclaimed their independence and founded a new city on the south bank of the Tungabhadra River.
There were 4 dynasties that ruled Vijayanagara – Sangam (founding dynasty), Saluva, Tuluva (of which
Krishnadeva Raya was one), and Aravidu.

Q.284 The highest court of appeal in the Vijayanagar Empire was


(a) Gram Sabha
(b) Mantri Parishad
(c) Qazi’s Court
(d) The King
Solution: (d)
Learning: The administration under the Vijayanagar Empire was organized in the way that the king enjoyed absolute authority
in executive, judicial and legislative matters. He was the highest court of appeal.
The king was assisted by a council of ministers in his day to day administration. But, word of the king was final. Moreover,
the succession to the throne was on the principle of hereditary

Q.285 What is described by contemporaries as “Karnataka samrajyamu” was defined by historians as


(a) Vijayanagara Empire
(b) Kadambas and Western Ganges Dynasty
(c) Bahmani Sultanate
(d) Badami Chalukyas
Solution: (a)
Justification: Karnataka samrajyamu was the name used by the contemporaries to describe the Vijayanagara
Empire and it is quite true with finding of the evidences of Empire in Karnataka at Hampi. Historians, however,
use the term Vijayanagara Empire.
Learning: Markets in the Vijayanagara Empire were known for its spices, textiles and other precious stones.
Trade was a status symbol for cities in the empire and boasted of a wealthy population that was in need of high-
value exotic goods such as precious stones and jewellery.
The revenue that came from the trade was used for the development of the State.

Q.286 The ruins at Hampi were brought to light in 1800 by an engineer and antiquarian who was also
the first Surveyor-general of India. He was?
(a) Colin Mackenzie
(b) Alexander Cunningham
(c) Gregory Schopen
(d) Julia Shaw
Solution: (a)
Learning: Colin Mackenzie became famous as an engineer, surveyor and cartographer.
In 1815 he was appointed the first Surveyor General of India, a post he held till his death in 1821.
He embarked on collecting local histories and surveying historic sites in order to better understand India’s past
and make governance of the colony easier.
He says that “it struggled long under the miseries of bad management … before the South came under the benign
influence of the British government”.
By studying Vijayanagara, Mackenzie believed that the East India Company could gain “much useful information
on many of these institutions, laws and customs whose influence still prevails among the various Tribes of Natives
forming the general mass of the population to this day”.

Q.287 Which of the following statements about the administration under the Vijayanagar Empire is
INCORRECT?
Q.288 The king enjoyed absolute authority in executive, judicial and legislative matters.
Q.289 The King was the highest court of appeal.
Q.290 The succession to the throne was on the principle of merit.
Q.291 The king was assisted by a council of ministers in his day to day administration.
Q.292 Solution: (c)

Q.293 Answer Justification :


Q.294 Justification: The succession to the throne was hereditary. So, (c) is the answer. However, sometimes
usurpation to the throne took place as Saluva Narasimha came to power by ending the Sangama dynasty.
Q.295 Learning: The Empire was divided into different administrative units called Mandalams, Nadus, sthalas and
finally into gramas. The governor of Mandalam was called Mandaleswara or Nayak. ijayanagar rulers gave full
powers to the local authorities in the administration.
Q.296 In the matter of justice, harsh punishments such as mutilation and throwing to elephants were followed.
Q.297 Besides land revenue, tributes and gifts from vassals and feudal chiefs, customs collected at the ports, taxes on
various professions were other sources of income to the government.

Q.298 Which of the following is correct with regard to the reign of Krishnadeva Raya?
1. He composed a work on statecraft in Telugu known as the Amuktamalyada.
2. He strictly prohibited entry of foreign ships and sailors from the port harbour of the kingdom, other than for
official purposes.
3. To encourage the export of horses, elephants and precious gems from the empire, he severely curtailed their
imports.
Select the correct answer using the codes below.
(a) 1 only
(b) 2 and 3 only
(c) 3 only
(d) None of the above
Solution: (a)
Justification: Statement 1: He (ruled 1509-29) was the most famous ruler of Vijayanagara, composed a work on
statecraft known as the Amuktamalyada.
Statement 3: About traders he wrote: A king should improve the harbours of his country and so encourage its
commerce that horses, elephants, precious gems, sandalwood, pearls and other articles are freely imported.
Statement 2: He further wrote that he should arrange that the foreign sailors who land in his country on account
of storms, illness and exhaustion are looked after in a suitable manner.

Q.299 In the context of Medieval India, who called themselves the rayas?
(a) Hoysalas of Karnataka
(b) The rulers of Vijayanagara
(c) Deccan Sultans who were also termed as ashvapati
(d) Bahmani sultans who carved a social niche for the royal empire
Solution: (b)
Justification: The rulers of Vijayanagara, who called themselves rayas, built on temple construction traditions of
Cholas and Hoysalas to new heights.
Ruling elites in these areas had extended patronage to elaborate temples such as the Brihadishvara temple at
Thanjavur and the Chennakeshava temple at Belur.
Learning: Gajapati literally means lord of elephants. This was the name of a ruling lineage that was very
powerful in Orissa in the fifteenth century.
In the popular traditions of Vijayanagara the Deccan Sultans are termed as ashvapati or lord of horses and the
rayas are called narapati or lord of men.

Q.300 The Architectural Splendour of Hampi is located in which of these river basins?
(a) Godavari-Vaigai basin
(b) Cauvery-Penna basin
(c) Krishna-Tungabhadra basin
(d) Mahanadi-Bhima basin
Solution: (c)
Learning: Hampi is located in the Krishna-Tungabhadra basin, which formed the nucleus of the Vijayanagara Empire,
founded in1336.
The magnificent ruins at Hampi reveal a well-fortified city. No mortar or cementing agent was used in the construction of
these walls andthe technique followed was to wedge them together by interlocking.Hampi fell into ruin following the
defeat of Vijayanagara in 1565 by the Deccani Sultans – the rulers of Golconda, Bijapur, Ahmadnagar, Berar and Bidar.

Q.301 What are the defining features of the chariot of the Vitthala temple in Virupaksha temple?
1. It is a monolithic structure carved out of a single rock.
2. Remains of painting can be found on the carvings of the chariot.
3. In front of the chariot two elephants are positioned as if they are pulling the chariot.
Select the correct answer using the codes below.
(a) 1 and 3 only
(b) 2 and 3 only
(c) 1 only
(d) 2 only
Solution: (b)
Justification: Statement 1: It may appear as a monolithic structure. In reality this stone shrine was built with
many giant granite blocks. The joints are smartly hidden in the carvings and other decorative features that adorn
the Stone Chariot.
Statement 2: One can still see the remains of the painting on the carvings of the chariot. Probably because it was
relatively protected from the natural weather elements, the undercarriage of the chariot spots one of the best
preserved specimens of this kind of paintings.
It is believed the whole of the Vittala Temple’s sculptures were once beautifully painted in similar fashion using
the minerals as medium
Statement 3: But, these stone elephants were brought from elsewhere and positioned here at a later stage.
Originally two horses were carved in that position.

STRUGLE FOR EMPIRE IN NORTH INDIA

Q.302 Buranjis in earliest Assamese literature were essentially


(a) Court chronicles
(b) War memorials
(c) Genealogies of Daibu kings
(d) Compilation of agricultural rituals
Solution: (a)
Learning: Buranjis are a class of historical chronicles, written initially in the Ahom and afterwards in Assamese
language. The first such Buranji was written on the instructions of the first Ahom king Sukaphaa who established
the Ahom kingdom in 1228.
There were two kinds of Buranjis: one maintained by the state (official) and the other maintained by families.
Many such manuscripts were written by scribes under the office of the Likhakar Barua, which were based on state
papers, diplomatic correspondences, judicial proceedings, etc.
Others were written by nobles or by people under their supervision, sometimes anonymously.
These documents reveal chronology of events, language, culture, society and the inner workings of the state
machinery of the kingdom.

Q.303 They occupied the political space after the decline of the Pala power in Bengal. Great patrons of literature,
major growth in Bengali language were witnessed during their reign. They were
(a) Yadavas
(b) Nayakas
(c) Empire of Harsha
(d) Senas
Solution: (d)
Learning: The Senas were the supporters of orthodox Hinduism. The dynasty traces its origin to theSouth, to the
Western Chalukya Empire of southern India
The Sena rulers were also great patrons of literature. During the Pala dynasty and the Sena dynasty, major growth
in Bengali was witnessed.
Some Bengali authors believe that Jayadeva, the famous Sanskrit poet and author of Gita Govinda, was one of the
Pancharatnas (five gems) in the court of Lakshmana Sena.
The Sena dynasty is famous for building Hindu temples and monasteries, which include the renowned
Dhakeshwari Temple in what is now Dhaka, Bangladesh.
They also consolidated the caste system in Bengal. Although Bengal borrowed from the caste system of Mithila,
caste was not so strong in Bengal as in Mithila.

Q.304 What are Moidams?


(a) Most significant urban hub in all of Nagaland after Dimapur and Kohima and the home of Ao Naga
(b) Battleground of the Manikya Kings in the past
(c) The Burial System of the Ahom Dynasty in Assam
(d) Traditional attire wore in the Tirap festival in Arunachal Pradesh
Solution: (c)
Learning: The Tai-Ahom clan upon their migration from China established their capital in different parts of the
Brahmaputra River Valleybetween 12th to 18th CE.
Usurping the Barahi tribe, Chau-lung Siu-ka-pha established the first capital of the Ahoms at the foothill of Patkai
hills and named it Che-rai-doi or Che-tam-doi, meaning “a dazzling city above the mountain” in their language and
consecrated site with a ritual.
While the clan moved from city to city, the landscape of Che-Rai-Doi or Choraideo continued to retain its position
as most sacred where the departed soul of the Royals could transcend into the after-life.
Their unique system of vaulted mounds continued for 600 years, till many Tai-Ahoms converted to Buddhism
while others adopted the Hindu system of cremation.

Q.305 In Assam, the Ahom kings had their paiks. These were people who
1. Were obliged to render military service in exchange for land
2. Were foreign mercenaries and spies who were kept under captivation
Which of the above is/are correct?
(a) 1 only
(b) 2 only
(c) Both 1 and 2
(d) None
Solution: (a)
Justification: The Paik system was a type of unpaid and bounded labour system on which the Ahom kingdom of
medieval Assam was based.
Every male in the Ahom kingdom between the ages of fifteen and fifty who was not a noble, a priest, a high caste
or a slave was a paik.
The duty of a paik was to render service to the Ahom state in exchange for which he was granted 2 puras of
cultivable land (gaa mati), which was neither hereditary nor transferable.
The paiks in a khel (groups) were organized under a gradation of officials who commanded a set number of them.

Q.306 Consider the following with reference to Ahom Kingdom.


1. They made land grants to religious institutions and figures.
2. Ahoms were persecutors of Hinduism.
3. Artisan population was predominant in the Kingdom.
Select the correct answer using the codes below.
(a) 1, 2 and 3
(b) 2 only
(c) 1 only
(d) 3 only
Solution: (c)
Justification: Statement 1: Ahom society was very sophisticated. Poets, temples, brahmins and scholars were given land
grants. Theatre was encouraged. Important works of Sanskrit were translated into the local language.
The peasant was given land by his village community. Even the king could not take it away without the community’s
consent.
Statement 2: Originally, the Ahoms worshipped their own tribal gods. During the first half of the seventeenth century,
however, the influence of Brahmanas increased.
In the reign of Sib Singh (1714-1744), Hinduism became the predominant religion. But the Ahom kings did not completely
give up their traditional beliefs after adopting Hinduism.
Statement 3: Ahom society was divided into clans or khels. There were very few castes of artisans, so artisans in the Ahom
areas came from the adjoining kingdoms. A khel often controlled several villages.

Q.307 Consider the following about the state of Garha Katanga, a Gond kingdom.
1. It earned much wealth by trapping and exporting wild elephants to other kingdoms.
2. It banned the system of coin minting due to widespread counterfeiting.
3. It conquered Bundelas and Marathas for much of Medieval Indian time period.
Select the correct answer using the codes below.
(a) 1 and 2 only
(b) 3 only
(c) 1 only
(d) 1, 2 and 3 only
Solution: (c)
Justification: It was a rich state of Central India.
When the Mughals defeated the Gonds, they captured a huge booty of precious coins and elephants. So, 2 is wrong.
They annexed part of the kingdom and granted the rest toChandra Shah, an uncle of BirNarain.
Despite the fall of Garha Katanga, the Gond kingdoms survived for some time. However, they became much weaker and later
struggled unsuccessfully against the stronger Bundelas and Marathas.

Q.308 With reference to Medieval India, buranjis were


(a) Historical chronicles, written initially in the Ahom and afterwards in Assamese language
(b) An encyclopedia of diverse topics written in a variety of local languages
(c) Works discussing the four Vedas and the six Vedangas
(d) Stories of Lingam, entailing description of disputes between Vishnu and Brahma
Solution: (a)
Learning: The first such Buranji was written on the instructions of the first Ahom king Sukaphaa who established the
Ahom kingdom in 1228.
There were two kinds of Buranjis: one maintained by the state (official) and the other maintained by families.
Many such manuscripts were written by scribes under the office of the Likhakar Barua, which were based on state papers,
diplomatic correspondences, judicial proceedings, etc.

EARLY MUGHALS
BABUR

Q.309 With reference to Medieval India, consider the following statements about Babur.
1. He was the founder of the Mughal Empire in India.
2. He wrote his memoirs, Tuzuk-i-Baburi in Turki language.
3. His son, Humanyun, was the first Mughal ruler to use cavalry and artillery in a war.
` Select the correct answer using the codes below.
(a) 1 and 2 only
(b) 2 and 3 only
(c) 1 and 3 only
(d) 1, 2 and 3
Solution: (a)
Justification: Statement 1: He was related to Timur from his father’s side and to Chengiz Khan through his
mother.
Babur succeeded his father Umar Shaikh Mirza as the ruler of Farghana. But he was soon defeated by his distant
relative and as a result lost his kingdom. He became a wanderer for sometime till he captured Kabul from one of
his uncles.
Then, Babur took interest in conquering India and launched four expeditions between 1519 and 1523.
Statement 2: Babur was a great statesman and a man of solid achievements. He was also a great scholar in Arabic
and Persian languages. Turki was his mother tongue. He wrote his memoirs, Tuzuk-i-Baburi in Turki language. It
provides a vivid account of India.
Statement 3: By the end of 1525, Babur started from Kabul to conquer India. He occupied Lahore easily by
defeating its governor, Daulat Khan Lodi.
Then he proceeded against Delhi where Ibrahim Lodi was the Sultan. The first Battle of Panipat, 1526, took place
between Babur and Ibrahim Lodi, who was killed in the battle.
Babur’s success was due his cavalry and artillery. Babur occupied Delhi and sent his son Humayun to seize Agra.
Babur proclaimed himself as “Emperor of Hindustan”.
Q.310 With reference to Araghatta or Persian wheels, consider the following statements.
1. It was a water lifting device used to irrigate small fields.
2. Babur Nama mentions the existence and use of Araghatta.
3. There is evidence of the use of Araghatta in Kolar, Karnataka.
Select the correct answer using the codes below.
(a) 1 only
(b) 2 and 3 only
(c) 3 only
(d) 1, 2 and 3
Solution: (d)
Justification: Statement 1: It's a simple water lifting device, where a number of small pots are attached to a
long chain. It is believed that the technology originated in Egypt and as world shrunk through extensive
trading, it spread to India and China.
Statement 2: One of the earliest mentions of the Persian Wheel occurs in the Babur's memoirs, the Babur
Nama (1526-30).
Statement 3: However, despite that evidence, several factors have also contributed to the disappearance of
the Persian Wheel from Kolar town.
This is because the biggest drawback of Persian wheels was its inability to draw water when the level is low.
HUMAYUN
SHER SHAH AND SUR DYNASTY
CONSOLIDATION OF MUGHALS

Q.311 The zamindars held extensive personal lands – milkiyat – in Mughal India. These
lands could be
1. Cultivated for the private use of zamindars
2. Sold or mortgaged at will by Zamindars
Which of the above is/are correct?
(a) 1 only
(b) 2 only
(c) Both 1 and 2
(d) None
Solution: (c)
Justification & Learning: Milkiyat lands were cultivated for the private use of
zamindars, often with the help of hired or servile labour. The zamindars could sell, bequeath
or mortgage these lands at will.
Zamindars also derived their power from the fact that they could often collect revenue on
behalf of the state, a service for which they were compensated financially

Q.312 Consider the following about the administrative arrangements in Mughal India.
1. Sarkars were further subdivided as paraganas.
2. In each paragana a Shiqdar and a Fotehdar were appointed.
3. Muqaddam acted as intermediaries between the government and the people.
Select the correct answer using the codes below.
(a) 1 only
(b) 2 only
(c) 2 and 3 only
(d) 1, 2 and 3
Solution: (d)
Justification: Statement 2: Apart from them, a Munsif, a Hindi writer and a Persian writer
were appointed.
Statement 3: Besides the Patwari, Chaudari and the Muqaddam who acted as the
intermediaries between the government and the people, the duty of the Shiqdar was to
maintain law and order, to collect the revenue and also decide the legal matters.

Q.313 The duty of Fotehdar, an important Mughal official in India, was to


1. Try civil and revenue cases
2. Keep the custody of royal treasury
3. Maintain accounts of the income and expenditure of the paraganas
Select the correct answer using the codes below.
(a) 2 only
(b) 1 and 2 only
(c) 3 only
(d) 1, 2 and 3
Solution: (a)
Justification: Statement 1: The Amin was in charge of collection of the land revenue and he
tried civil and revenue cases as well.
Statement 2: The treasurer was Fotehdar and the entire amount collected was kept in his
custody.
Statement 3: Clerks or Karkuns were appointed to maintain duplicate accounts in Hindi and
Persian.
Q.314 In Mughal India, the official Amin was responsible for

1. Collecting dues from laities who didn’t follow any particular religion
2. Ensuring that imperial regulations were carried out in the provinces
3. Looking after the security and diplomatic relations of the empire
4. Implementing market reforms in mandis
Solution: (b)
Learning: The duty of the Shiqdar was to maintain law and order, to collect the revenue and also
decide the legal matters.
The Amin was in charge of collection of the land revenue and he too tried civil and revenue
cases. The Amin and the Shiqdar were of the same rank.

Q.315 Consider the following about land revenue collection in Mughal India.
1. While fixing revenue, the state first assessed the land.
2. Akbar mandated that payment be accepted both in kind and cash.
Which of the above is/are correct?
(a) 1 only
(b) 2 only
(c) Both 1 and 2
(d) None
Solution: (c)
Justification: Statement 1: The Mughal state tried to first acquire specific information about
the extent of the agricultural lands in the empire and what these lands produced before
fixing the burden of taxes on people.
The land revenue arrangements consisted of two stages – first, assessment and then actual
collection. The jama was the amount assessed, as opposed to hasil, the amount collected.
Statement 2: Akbar decreed that while he should strive to make cultivators pay in cash, the
option of payment in kind was also to be kept open.
While fixing revenue, the attempt of the state was to maximise its claims.

AKBAR

Q.316 Ain-i Akbari written by Abu’l Fazl that deals with records of administrative
arrangements in Medieval India was part of which of these texts?
(a) Akbarnama
(b) Rizla
(c) Bhaktamal
(d) Tuzk-e-Taimuri
Solution: (a)
Learning: The Ain-i-Akbari is the third volume of the Akbarnama containing information
regarding Akbar's reign in the form of, what would be called in modern times, administration
reports, statistical compilations, or gazetteers.
The Ain-i-Akbari is itself divided into five books.
The first book deals with the imperial household, and the second with the servants of the
emperor, the military and civil services.
The third book deals with the imperial administration, containing the regulations for the
judicial and executive departments.
The fourth book contains information about Hindu philosophy, science, social customs and
literature.
The fifth book contains sayings of Akbar.

Q.317 Consider the following statements.


1. Mughal chronicle Akbar Nama was written in Persian.
2. Babur’s memoirs were translated from the Turkish into the Persian Babur Nama.
3. Mughal emperors commissioned translations of Sanskrit texts into Persian.
Select the correct answer using the codes below.
(a) 1 only
(b) 2 and 3 only
(c) 1 and 3 only
(d) 1, 2 and 3
Solution: (d)
Justification: Statement 1: It is the official chronicle of the reign of Akbar, the third Mughal
Emperor, commissioned by Akbar himself by his court historian and biographer, Abul Fazl
who was one of the nine jewels in Akbar's court.
Statement 2: Babur was the founder of the Mughal Empire and a great-great-great-grandson
of Timur.
During Emperor Akbar's reign, the work was completely translated to Persian by a Mughal
courtier, Abdul Rahim.
Statement 3: Translations of Sanskrit texts such as the Mahabharata and the Ramayana into
Persian were commissioned by the Mughal emperors. The Mahabharata was translated as
the Razmnama (Book of Wars).

Q.318 The following is a listing of criteria of classification excerpted from the Ain-I-Akbari
for land.
1. Polaj was land which has never been cultivated and can be used by the royalty.
2. Parauti is land left out of cultivation for a time that it may recover its strength.
3. Chachar is land that is annually cultivated for each crop in succession and is never
allowed to lie fallow.
Select the correct matches using the codes below.
(a) 1 and 2 only
(b) 2 only
(c) 1 and 3 only
(d) 2 and 3 only
Solution: (b)
Justification: The Emperor Akbar in his sagacity classified the lands and fixed a different
revenue to be paid by each.
Polaj is land which is annually cultivated for each crop in succession and is never allowed to
lie fallow.
Parauti is land left out of cultivation for a time that it may recover its strength.
Chachar is land that has lain fallow for three or four years.
Banjar is land uncultivated for five years and more.
Of the first two kinds of land, there are three classes, good, middling, and bad. They add
together the produce of each sort, and the third of this represents the medium produce,
one-third part of which is exacted as the Royal dues.

Q.319 Consider the following with reference to sulh-i kul.


1. In sulh-i kul all religions and schools of thought had freedom of expression.
2. The ideal of sulh-i kul was implemented through state policies.
3. Abu’l Fazl describes the ideal of sulh-i kul as the cornerstone of enlightened rule.
4. Practiotioners of Sulh-i-kul were free to challenge and undermine the authority of the state
as long as it was done non-violently.
Select the correct answer using the codes below.
(a) 1, 2 and 3 only
(b) 2 and 4 only
(c) 1 and 3 only
(d) 1, 2, 3 and 4
Solution: (a)
Justification: Akbar ruled with a social and religious toleration which was based on his concept
of sulh-i-kul (for the general good of all people) built on his liberal views of religion.
Statement 1 and 4: There was freedom of expression but on condition that they did not
undermine the authority of the state or fight among themselves.
Statement 2: The ideal of sulh-i kul was implemented through state policies – the nobility under
the Mughals was a composite one comprising Iranis, Turanis, Afghans, Rajputs, Deccanis – all of
whom were given positions and awards purely on the basis of their service
Statement 3: Akbar took the Sufi mystic notion of sulh-i-kul and transformed it to become a
principle denoting amity- within a culturally pluralistic India.
Akbar established separation of state and religion and opened government positions to
members of all religions.
Sulh- i-kul was to become his method of judging what was legally right or wrong within his
empire and was created because Akbar understood that he was trying to build political
institutions for predominately non-Muslim society.

Q.320 During Akbars reign, Ibadat khana was used as a


(a) House of sacrifices
(b) Place for religious discussions
(c) Rest house
(d) Place for initiation of monks
Solution: (b)
Justification:WhileAkbarwasatFatehpurSikriduringthe1570shestarteddiscussionson religion with
the ulama, Brahmanas, Jesuit priests who were Roman Catholics, and Zoroastrians.
These discussions took place in the ibadat khana. He was interested in the religion and social customs of
different people.
Itmadehimrealisethat religious scholarswhoemphasised ritualanddogmawereoften bigots.

Q.321 Consider the following about Sulh-i kul, started by Akbar.


1. It was an idea of “universal peace”.
2. Abul Fazl was against this system of beliefs.
3. This system of governance was abandoned by rulers who followed Akbar
Select the correct answer using the codes below.
(a) 2 and 3 only
(b) 3 only
(c) 1 only
(d) 1 and 2 only
Solution: (c)
Justification: It was an idea of tolerance that did not discriminate between people of different
religions. It was a universal religion of peace and harmony.
Instead it focused on a system of ethics – honesty, justice, peace – that was universally applicable. Abul
Fazl helped Akbar in framing a vision of governance around this idea of sulh-i kul.
This principle of governance was followed by Jahangir and Shah Jahan as well.

Q.322 Akbarorderedoneofhisclosefriendsandcourtiers,AbulFazltowriteAkkbarNama,aworkof
three volumes. The volumes deal with
1. Akbar’s ancestors
2. Evolution of administrative setup in Indian subcontinent
3. Geography of India
4. Events of Akbar’s reign
Select the correct answer using the codes below.
(a) 1 and 4 only
(b) 2 and 3 only
(c) 1, 2 and 3 only
(d) 1, 2, 3 and 4
Solution: (a)
Justification: The first volume dealt with Akbar’s ancestors and the second volume recorded the
events of Akbar’s reign. The third volume is the Ain-I Akbari. It deals with Akbar’s administration,
household, army, the revenues and geography of his empire.
It also provides rich details about the traditions and culture of the people living in India. The most
interestingaspectabouttheAin-iAkbariisitsrichstatisticaldetailsaboutthingsasdiverseas crops,
yields, prices, wages and revenues.

JAHANGIR

Q.323 Tuzuk-i-Jahangiri is a work on

(a) Memoirs of Mughals in the Indian sub-continent


(b) Poetry on Social oppression
(c) Jahangir’s autobiography
(d) Compendium of Martial techniques used in Mughal warfare
Solution: (c)
Learning: It is in Persian and is a unique piece of literature.
Similarly, we have a work named Tabqat-i-Alamgïri, shedding light on Aurangzeb.
Abul Fazl’s Akbarnamah and Ain-e-Akbari is another such example. From there we get a good
deal of information about Akbar and his times.
A name in prose and history writing is that of Chandra Bhan, a writer of Shahjahan’s days.
Badauni was another writer who belonged during Akbar’s time. In the twentieth century, Iqbal
wrotegood Persian poetry.

Q.324 Which of these rulers issued a ‘firman’ (decree) permitting the East India Company
to establish its first trading post at Surat?
(a) Peshwa Nana Saheb
(b) Jahangir
(c) Murshid Khan
(d) Mir Zafar
Solution: (b)
Learning: The English East India Company was established in 1600 as per the Royal Charter
issued by the Queen of England, Elizabeth I.
The Company had sent Captain Hawkins to the court of the Mughal Emperor, Jahangir in
1608 to secure permission to establish a “factory” (store house of goods) at Surat. It was
turned down initially.
However, in 1613, Jahangir issued the firman permitting the East India Company to establish
its first trading post at Surat. Subsequently,
Sir Thomas Roe obtained more trading rights and privileges for the East India Company.
Accordingly, the English set up business centres at Agra, Ahmedabad and Broach.

SHAH JAHAN
AURANGZEB
LIFE AND DEVELOPMENT

Q.325 Which of these languages was popularly known as Dakshini in Medieval India?
(a) Telugu
(b) Urdu
(c) Kannada
(d) Classical Sanskrit
Solution: (b)
Learning: After the conquest of Delhi (1192), Urdu was born out of the interaction of the
Turkish settlers and soldiers in the barracks with the common people.
Originally it was a dialect but slowly it acquired all the features of a formal language when
the authors started using Persian script. It was further given an impetus by its use in
Bahamani states of Ahmadnagar, Golkunda, Bijapur and Berar.
Here it was even called dakshini or daccani (southern). As time passed, it became popular
with the masses of Delhi

Q.326 The kanungos were the


(a) hereditary revenue officers since the time of Mughals.
(b) law givers as the assistant of the Ulemmas
(c) judicial magistrates in the village panchayats
(d) bonded labourers released by the British
Solution: (a)
Learning: The kanungo comes into prominence in the reign of Akbar, who employed him,
as the name implies, to keep the records of the pargana, a revenue sub-division.
He was in fact a registrar of a district appointed to see that the crown received its dues and
that the ryot was not oppressed; his duties were responsible and onerous; he had to register
the usages of a district, the rate and of its and all regulation relating thereto.
The kanungos' duties also included the keeping of a record of all events, such as the
appointments, deaths or removals of zamindars; to preserve the records of the Tumar and
Taksim Jama etc.

Q.327 In the 16th-17th century India, Muqaddam or mandal was the


(a) Local police head
(b) Adviser to the King
(c) Headman of the panchayat
(d) Tax collection officer
Solution: (c)
Learning: Some sources suggest that the headman was chosen through the consensus of
the village elders, and that this choice had to be ratified by the zamindar.
Headmen held office as long as they enjoyed the confidence of the village elders, failing
which they could be dismissed by them.
The chief function of the headman was to supervise the preparation of village accounts,
assisted by the accountant or patwari of the panchayat.

Q.328 As per Ibn Battuta’s account of India


Indian rhinoceros lived on the banks of the Indus.
Hindu Kush mountains were used for slave trading in India.
Female slaves were employed by the Sultan to keep a watch on his nobles.
Select the correct answer using the codes below.
2 only
1 and 2 only
3 only
1, 2 and 3
Solution: (d)
Answer Justification :
Justification: Statement 2: He journeyed south to Afghanistan, then crossed into India via the
mountain passes of the Hindu Kush. In the Rihla, he mentions these mountains and the history of the
range in slave trading.
Statement 1: Upon his arrival in Sindh, Ibn Battuta mentions the Indian rhinoceros that lived on the
banks of the Indus.
Statement 3: It appears from Ibn Battuta’s account that there was considerable differentiation
among slaves.
Some female slaves in the service of the Sultan were experts in music and dance, and Ibn Battuta
enjoyed their performance at the wedding of the Sultan’s sister, and some to keep a watch on his
nobles.

Q.329 In Medieval India, according to Francois Bernier, what was the reason for
averseness to any long- term investment in the sustenance and expansion of crop
production on land?
1. Crown ownership of land
2. Low crop tax that deferred state investments on land
Which of the above is/are correct?
(a) 1 only
(b) 2 only
(c) Both 1 and 2
(d) None
Solution: (a)
Justification: Statement 1: Owing to crown ownership of land, argued Bernier, landholders
could not pass on their land to their children. So, they did not intent upon any long-term
investments on land, such as increasing its productive capacity.
Statement 2: The crop tax was very high in Mughal India, and formed the basis for higher
taxes by other ruling dynasties.

Q.330 consider the following with reference to Francois Bernier, a traveller to Medieval India.
1. He came to the Mughal Empire as the ambassador of Mesopotamia.
2. He despised the presence of army in the Mughal Empire.
3. He described the culture and scientific developments in India as far superior to that in
Europe.
Select the correct answer using the codes below.
(a) 1 and 2 only
(b) 3 only
(c) 2 and 3 only
(d) None of the above
Solution: (d)
Justification: Statement 1: François Bernier was a Frenchman, a doctor, political philosopher
and historian.
Like many other travellers, he came to the Mughal Empire in search of opportunities.
Statement 2: Bernier often travelled with the army and found its presence important for the
Mughal empire.
Statement 3: In virtually every instance Bernier described what he saw in India as a bleak
situation in comparison to developments in Europe.

Q.331 Which of the following are associated with the contributions of Mughals in the
development of Arms and Ammunition in India?
1. Technique of production of gunpowder
2. Fire works and explosion techniques
3. Casting of cannons
Select the correct answer using the codes below.
(a) 1 only
(b) 2 and 3 only
(c) 3 only
(d) 1, 2 and 3
Solution: (d)
Justification: Mughal artillery included a variety of cannons, rockets, and mines employed
by the Mughal Empire. This gunpowder technology played an important role in the
formation and expansion of the empire.
Following the decisive Ottoman victory over the Safavid Empire at the 1514 Battle of
Chaldiran, Babur incorporated artillery and Ottoman artillery tactics into his military.
In 1526, the First Battle of Panipat saw the introduction of massed artillery tactics to Indian
warfare.
Later emperors paid less attention to the technical aspects of artillery, allowing the Mughal
Empire to gradually fall behind in weapon technology, although the degree to which this
decline affected military operations is debated

Q.332 In the Mughal state, the amil-guzar was


(a) A Hereditary craftsman
(b) Keeper of Law and Order
(c) The chief cultivator of state-owned lands
(d) A revenue collector
Solution: (d)
Learning: The Mughal state tried to first acquire specific information about the extent of
the agricultural lands in the empire and what these lands produced before fixing the burden
of taxes on people.
The land revenue arrangements consisted of two stages – first, assessment and then actual
collection.
The jama was the amount assessed, as opposed to hasil, the amount collected. In his list of
duties of the amil-guzar or revenue collector, Akbar decreed that while he should strive to
make cultivators pay in cash, the option of payment in kind was also to be kept open.

Q.333 Muzarian, khud-kashta and pahi-kashta were terms used by Indo-Persian


sources of the Mughal period most frequently to denote
(a) Non-resident Scholars
(b) Peasants
(c) Foreign spies
(d) Samantas
Solution: (b)
Learning: Terms used to denote a peasant was raiyat (plural, riaya) or muzarian. In
addition, we also encounter the terms kisan or asami.
Sources of the seventeenth century refer to two kinds of peasants – khud-kashta and pahi-
kashta.
The former were residents of the village in which they held their lands.
The latter were non-resident cultivators who belonged to some other village, but cultivated
lands elsewhere on a contractual basis.
People became pahi-kashta either out of choice – for example, when terms of revenue in a
distant village were more favourable – or out of compulsion

Q.334 The period between the sixteenth and eighteenth centuries was marked by a
remarkable stability in the availability of metal currency, particularly the silver
rupya in India. This was mainly due to
1. The large natural reserves of silver in India that were successfully exploited and mined
by the Mughal state
2. An expanding trade brought in huge amounts of silver bullion into Asia to pay for goods
procured from India
Which of the above is/are correct?
(a) 1 only
(b) 2 only
(c) Both 1 and 2
(d) None
Solution: (b)
Justification: The political stability achieved by Mughal empires in India helped create
vibrant networks of overland trade from China to the Mediterranean Sea.
Voyages of discovery and the opening up of the New World resulted in a massive expansion
of Asia’s (particularly India’s) trade with Europe.
This resulted in a greater geographical diversity of India’s overseas trade as well as
an expansion in the commodity composition of this trade.
This was good for India as it did not have natural resources of silver.
This facilitated an unprecedented expansion of minting of coins and the circulation of money
in the economy as well as the ability of the Mughal state to extract taxes and revenue in
cash.

Q.335 Jajmani system, as popular in parts of Medieval India in regions like Bengal, was
1. A system of hereditary holding
2. A terms of payment of daily allowances
3. A military service rendered by the local populace
Select the correct answer using the codes below.
(a) 1 only
(b) 2 only
(c) 2 and 3 only
(d) 1, 2 and 3
Solution: (b)
Justification: Statement 1: In Maharashtra some lands became the artisans’ miras or watan
– their hereditary holding. This wasn’t the jajmani system.
Statement 2: But, another variant of this was a system where artisans and individual peasant
households entered into a mutually negotiated system of remuneration, most of the time
goods for services.
For example, eighteenth-century records tell us of zamindars in Bengal who remunerated
blacksmiths, carpenters, even goldsmiths for their work by paying them “a small daily
allowance and diet money”. This later came to be described as the jajmani system.

THE DECCAN AND SOUTH INDIA

Q.336 With reference to the Bahmani Kingdom, consider the following statements.
1. It was the one of those few Persian Kingdoms in India that had only a single ruling
Sultan.
2. The kingdom was confined to Southern Peninsular India.
3. The Kingdom was at friendly terms with the Vijayanagara Empire.
Select the correct answer using the codes below.
(a) 1 only
(b) 2 and 3 only
(c) 1 and 3 only
(d) None of the above
Solution: (d)
Justification: Statement 1: There were a total of fourteen Sultans ruling over this kingdom.
The founder of the Bahmani kingdom was Alauddin Bahman Shah also known as Hasan
Gangu in 1347. Its capital was Gulbarga.
Among them, Alauddin Bahman Shah, Muhammad Shah I and Firoz Shah were important.
Statement 2: It extended from the Arabian sea to the Bay of Bengal. On the west it
extended from Goat to Bombay. On the east, it extended from Kakinada to the mouth
of the river Krishna.
Statement 3: The Bahmani kingdom reached its peak under the guidance of Mahmud
Gawan. He waged successful wars against Vijayanagar, Orissa and the sea pirates on
the Arabian sea.

Q.337 Mahmud Gawan, serving under Bahmani Sultans, introduced which


of the following reform measures?
1. He reduced the powers of the governors of the Provinces and strengthened that of the
Central government.
2. He introduced lands measurement and fixation of land taxes on that basis.
Which of the above is/are correct?
(a) 1 only
(b) 2 only
(c) Both 1 and 2
(d) None
Solution: (a)
Justification: Mahmud Gawan served as the Prime Minister of three Bahamni Sultans from
1458 to 1481 A.D.
Statement 1: The existing four provinces were divided into two each and created eight
‘Tarafs’.
In each of these eight ‘Tarafs’ some areas were kept reserve for the control of the Central
Government and for their administrative control, officials were appointed by the Centre.
Orders were issued by him for placing one fort in each province under the charge of the
governor and others under the control of military commanders appointed by the Sultan.
He put a check on the powers of the nobles. Nobles were paid salaries and were asked to
maintain contingents of horses
Statement 2: No such reform was introduced by him.
Learning: He used gunpowder in the war against the Vijayanagar kings in Belgaum.
He is considered the architect of medieval Deccan who invited Persian chemists to teach his
soldiers the preparation and use of gunpowder.

Q.338 The text ‘Lilatilakam’ deals with


(a) Grammar and poetics
(b) Commentary on the Upanishads
(c) Love story of a CholaKing
(d) War rituals in Rashtrakuta empire
Solution: (a)
Learning: The first literary works in Malayalam, dated to about the twelfth century, are directly
indebted to Sanskrit.
A fourteenth-century text, the Lilatilakam, dealing with grammar and poetics, was composed in
Manipravalam.
Manipravalam, literally means "diamonds and corals, refers to the two language Sanskrit and the
regional language of Kerala.

Q.339 Uttaramerur Inscription is a famous Tamil inscription datingto theTenth Century A.D. It deals
with
(a) Recording the Mode of Election to Village Assemblies
(b) Sacrificial ceremonies to be performed in the North
(c) Rights and inheritances of a women
(d) The segregation of the law-making bodies and executive bodies
Solution: (a)
Learning: The inscription gives several qualifications for the candidate:
It says, “In these thirty wards, those that live in each ward shall assemble and shall choose for “pot-
tickets” (Kudav Olai) anyone possessing the following qualifications:
He must own more than a quarter veli of tax-paying land;
He must live in a house built on his own site;
His age must be below 70 and above 35;
He must know the Mantrabrahmana, i.e., he must know it by teaching others;
Among those possessing the foregoing qualifications:
Only such as are well conversant with business and are virtuous shall be taken and,
One who possesses honest earnings, whose mind is pure and who has not been on any of the
committees for the last three years shall also be chosen.

MODERN INDIAN HISTORY


ADVENT OF THE EUROPEANS
INDIA ON THE EVE OF BRITISH CONQUEST
DECLINE OF MUGHALS
RISE OF REGIONAL STATES
BRITISH EXPANSION AND CONSOLIDATION
Q.340 Which one of the following statements about the Treaty of Allahabad is correct?
(a) It was concerned with the Battle of Plassey.
(b) It was imposed by force on the incumbent Mughal Emperor and deprived the Emperor
of his property and land in lieu of protection.
(c) The rights given by this treaty allowed the Company to collect revenue directly from the
people certain provinces.
(d) None of the above is correct.
Solution: (c)
Justification and Learning: It was signed in 1765, between the Mughal Emperor Shah
Alam II, and Robert, Lord Clive as a result of the Battle of Buxar of 1764.
The Treaty marks the political and constitutional involvement and the beginning of British
rule in India.
Based on the terms of the agreement, Alam granted the East India Company Diwani rights,
or the right to collect taxes on behalf of the Emperor from the eastern province of Bengal-
Bihar- Orissa.
Thus East India Company got appointed as the imperial tax collector for the Eastern province
(Bengal-Bihar-Orissa).
These rights allowed the Company to collect revenue directly from the people of Bengal,
Bihar and Orissa. In return, the Company paid an annual tribute of twenty-six lakhs of rupees
(equal to 260,000 pounds sterling) while securing for Shah Alam II the districts of Kora
and Allahabad.
The tribute money paid to the emperor was for the maintenance of the Emperor's court in
Allahabad.

Q.341 Which of the following could be the possible implications of the Battle of
Plassey?
1. It resulted in the victory of East India Company (EIC) leading greater powers to extract
revenue and its consolidation of its presence in Bengal.
2. It resulted in the EIC gaining greater military might which allowed them to push other
European colonial powers away from Bengal.
Which of the above is/are correct?
(a) 1 only
(b) 2 only
(c) Both 1 and 2
(d) None
Solution: (c)
Justification: The Battle of Plassey was essentially due the conflicts between the Bengal
Nawab and the East India Company.
This is judged to be one of the pivotal battles in the control of Indian subcontinent by the
colonial powers.
The British now wielded enormous influence over the Nawab and consequently acquired
significant concessions for previous losses and revenue from trade.
The British further used this revenue to increase their military might and push the other
European colonial powers such as the Dutch and the French out of South Asia, thus
expanding the British Empire.
Q.342 Which of these can be said to be deciding battles after which the East India
Company became a political power in India?
(a) Battle of Plassey in 1757 and Battle of Buxar in 1764
(b) Battle of Bandiwash in 1762 and Battle of Panipat in 1785
(c) Battle of Surat in 1754 and Battle of Tuicorin in 1772
(d) Battle of Turicorin in 1772 and Battle of Plassey in 1757
Solution: (a)
Justification & Learning: The Battle of Buxar was fought between British East India Company
and the combined forces of Nawabs and the Mughal Emperor leading to the victory of the
British.
It led to the signing of the Allahabad Treaty in 1765. With the defeat of Mir Kasim, the rule of
Nawabs came to an end.
Diwani rights or fiscal rights were secured which meant that the British would administer and
manage revenues of large areas which included the present-day West Bengal, Jharkhand, Bihar,
and Uttar Pradesh, as well as of Bangladesh.
Battle of Plassey is judged to be one of the pivotal battles in the control of Indian subcontinent
by the colonial powers.
The British, after the war, wielded enormous influence over the Nawab and consequently
acquired significant concessions for previous losses and revenue from trade.
The British further used this revenue to increase their military might and push the other
European colonial powers such as the Dutch and the French out of South Asia, thus expanding
the British Empire.

Q.343 Consider the following statements. The Treaty of Allahabad granted rights to the British
to
1. Collect taxes from the eastern province of Bengal-Bihar-Orissa
2. Keep an armed continent on behalf of the Bengal nawab
Which of the above is/are correct?
(a) 1 only
(b) 2 only
(c) Both 1 and 2
(d) None
Solution: (a)
Justification: It was signed in 1765, between the Mughal Emperor Shah Alam II and Robert,
Lord Clive, as a result of the Battle of Buxar.
The Treaty marks the political and constitutional involvement and the beginning of British
rule in India.
Based on the terms of the agreement, Alam granted the East India Company Diwani rights,
or the right to collect taxes on behalf of the Emperor from the eastern province of Bengal-
Bihar- Orissa.
Thus, East India Company got appointed as the imperial tax collector for the Eastern
province (Bengal-Bihar-Orissa).
These rights allowed the Company to collect revenue directly from the people of Bengal,
Bihar and Orissa.

MYSORE

Q.344 Consider the following statements. Treaty of Srirangapattinam


1. Abolished the princely status of the state of Mysore
2. Made Tipu Sultan pay war indemnity to the British
3. Resulted in British ceding the territory of Malabar Coast
Select the correct answer using the codes below.
(a) 2 only
(b) 1 and 3 only
(c) 3 only
(d) 1 only
Solution: (a)
Background: A War broke out in May 1790 between the English and Tipu. It was fought in
three phases.
The third phase of the war began when timely aid from the Marathas with plenty of
provisions helped him to resume his campaign and marched against Srirangapattinam again.
Tipu Sultan concluded the Treaty of Srirangapattinam with the British.
Justification: Statement 1 and 2: The terms of the treaty were as follows:
Tipu had to give up half his dominions; (ii) He had to pay a war indemnity of three crore
rupees and surrender two of his sons as hostages to the English; and (iii) Both sides agreed
to release the prisoners of war.
Statement 3: The Treaty of Srirangapattinam is a significant event in the history of South
India. The British secured a large territory on the Malabar Coast. In addition, they obtained
the Baramahal district and Dindugal.
After this war, although the strength of Mysore had been reduced, it was not extinguished.
Tipu had been defeated but not destroyed.

MARATHAS
SINDH
PUNJAB
OTHERS
Q.345 The Indus Navigation Treaty that was signed between the British and the ruler of
Kashmir, Ranjit Singh, provided for
1. Opening Sutlej river for navigation
2. Sharing of Indus waters with North-western provinces that were not under British
control
3. Sole right of Kashmiris to store, cultivate and develop Indus water
4. None of the above
Solution: (a)
Background & Learning: Lord William Bentinck was the first Governor-General to visualise a
Russian threat to India. Hence, he was eager to negotiate friendly relations both with the
ruler of Punjab, Maharajah Ranjit Singh and also with the Amirs of Sind.
His earnest desire was that Afghanistan should be made a buffer state between India and
any possible invader.
As an initial measure, an exchange of gifts took place between Lahore, the capital of Punjab
and Calcutta, the seat of Governor-General.
It was then followed by the meeting of Bentinck and Ranjit Singh in 1831 at Rupar on the
bank of the river Sutlej.
The Governor-General was successful in winning the friendship of Ranjit Singh and the Indus
Navigation Treaty was concluded between them.
This treaty opened up the Sutlej for navigation. In addition, a commercial treaty was
negotiated with Ranjit Singh. A similar treaty was also concluded with the Amirs of Sind.

Q.346 With reference to colonial India, the Treaty of Sagauli is concerned with which of the
following South Asian neighbours of India with which present-day India shares a porous
border?
(a) Nepal
(b) Bangladesh
(c) Bhutan
(d) Myanmar
Solution: (a)
Learning: In 1816, the Treaty of Sagauli was concluded. The Gurkhas gave up their claim
over the Tarai region and ceded the areas of Kumaon and Garhwal to the British.
The British now secured the area around Simla and their north-western borders touched the
Himalayas.
The Gurkhas had to withdraw from Sikkim and they also agreed to keep a British Resident at
Katmandu.
It was also agreed that the kingdom of Nepal would not employ any other foreigner in its
services other than the English.
The British had also obtained the sites of hill stations like Simla, Mussoori, Nainital, Ranikhet
and developed them as tourist and health resorts.
After this victory in the Gurkha War Hastings was honoured with English peerage and he
became Marquis of Hastings.

Q.347 The 1890 UK- China treaty deals with


(a) Recognition of the frontier between Sikkim and Tibet
(b) Accession of Shortughai as an industrial centre
(c) Political status of Taiwan
(d) Release of local colonies like Manchuria
Solution: (a)
Learning: As per Article (1) of Convention of 1890, it was agreed that the boundary of
Sikkim and Tibet shall be the crest of the mountain range separating the waters flowing into
the Sikkim Teesta and its affluents, from the waters flowing into the Tibetan Mochu and
northwards into other rivers of Tibet.
The line commences at Mount Gipmochi, on the Bhutan frontier, and follows the above-
mentioned water-parting to the point where it meets Nepal territory.
However, Tibet refused to recognise the validity of Convention of 1890 and further refused
to carry into effect the provisions of the said Convention. In 1904, a treaty known as a
Convention between Great Britain and Tibet was signed at Lhasa.
In 1906, a treaty was signed between Great Britain and China at Peking, which confirmed the
Convention of 1904 between Great Britain and Tibet.

THE REVOLT OF 1857

Q.348 In the context of British India, which among the following mainly contributed to the
Vellore Mutiny?
1. New import and export policy of spices near Malabar Coast
2. Annexation of princely states
3. Imposition of taxes on locals
4. Introduction of new army regulations
Select the correct answer using the codes below.
(a) 1 and 2 only
(b) 2, 3 and 4 only
(c) 4 only
(d) 1, 2 and 3 only
Solution: (c)
Justification: The Vellore Mutiny was a sepoy mutiny and the causes were rooted in the
conditions of the Indian army contingents.
Several causes are attributed to the Vellore Mutiny. Indian sepoys had to experience
numerous difficulties when they went to serve in the Company’s army.
The sepoys were forced to serve under the Company since their earlier patrons (the native
chieftains) were all disappearing from the scene.
The strict discipline, practice, new weapons, new methods and uniforms were all new to the
sepoys.
The sepoys were asked to shave the chin and to trim the moustache. The sepoys felt that
these were designed to insult them and their religious and social traditions. There was also a
popular belief that this was the beginning of a process by which all of them would be
converted to Christianity.
The English treated the Indian sepoys as their inferior. There was the racial prejudice. This
was the psychological base for the sepoy mutinies in India during the Company’s rule.
The Vellore uprising was preceded by a series of protests by the Indian troops.

Q.349 What was the position of paiks, with reference to Medieval kingdoms of North-eastern India?
(a) They were forced to work for the state.
(b) They were mercenaries recruited from other kingdoms.
(c) They held large amount of land and gold and donated it for public welfare.
(d) They were the patrons of literature and arts.
Solution: (a)
Justification: The Ahoms migrated to the Brahmaputra valley from present-day Myanmar in the
thirteenthcentury.Theycreatedanewstatebysuppressingtheolderpoliticalsystemofthe
bhuiyans (landlords).
The Ahom state depended upon forced labour. Those forced to work for the state were called paiks. A
census of the population was taken. Each village had to send a number of paiks by rotation.
People from heavily populated areas were shifted to less populated ones.

Q.350 Many researchers and historians have opined that the Paika Rebellion of 1817
was India’s first organized armed rebellion against British Raj. Who were the
Paiks?
(a) Traditional landed militia of Odisha
(b) Sepoys recruited by the Uttar Pradesh regiment of the British army
(c) Absentee landlords of Malabar
(d) Landless tribal peasants of Central India
Solution: (a)
Learning: Paika rebellion predates the 1857 Revolt i.e. first war of independence.
Paiks were the traditional landed militia of Odisha, used to perform policing functions. They
owned rent-free land that was given to them for their military service to Kingdom of
Khurda.
However, by the policies of British East India Company, it was forcefully usurped rendering
them landless.
They were also subjected to repressive land revenue policies and humiliation by the British.
At that critical juncture, Bakshi Jagabandhu Bidyadhar, the military chief of the King of
Khurda, led army of Paikas forcing the East India Company to retreat.

CAUSES

Q.351 Consider the following statements.


Assertion (A): 1857 revolt was largely limited to Northern and Central India. Reason (R): The
territories beyond Aurangabad in Southern India were not under British control.
In the context of the above, which of these is correct?
(a) A is correct, and R is an appropriate explanation of A.
(b) A is correct, but R is not an appropriate explanation of A.
(c) A is correct, but R is incorrect.
(d) A is incorrect, but R is correct.
Solution: (c)
Justification: The map below shows that it was not so. British control extended almost all
across India.
CONCEQUENCES
Q.352 As per Queen Victoria’s proclamation of 1858
1. British Crown assumed the sovereignty and control over all princely states and their
subjects.
2. The crown would gradually introduce responsible government in India where subjects
would be the source of power.
Which of the above is/are correct?
(a) 1 only
(b) 2 only
(c) Both 1 and 2
(d) None
Solution: (d)
Justification: As per the proclamation, all treaties and agreements made with the Indian
native princes under the authority of the East India company did not cease to exist but were
there to stay and accepted by the crown.
The declaration expressed faith and the rights, dignity and status of the native princes. The
Indian public was given an assurance that there would be no extension of the current
territorial possessions.
The most important result was the now onwards the Indian Sepoys were enlisted in the
regular service in the British Army and participated in the world wars in the next century.

RELIGIOUS AND SOCIAL REFORM MOVEMENTS

Q.353 With reference to the Poona Sarvajanik Sabha, consider the following
statements.
1. It was established by Mahadev Govind Ranade in opposition to the expansionist
propaganda of the Hindu Mahasabha.
2. It was dedicated to the cause of social and political reforms in India advocating the
principle of Swadeshi and Swaraj.
Which of the above is/are correct?
(a) 1 only
(b) 2 only
(c) Both 1 and 2
(d) None
Solution: (d)
Justification: Statement 1: The Akhil Bharatiya Hindu Mahasabha was formed in 1906
much after the formation of the Pune mahasabha.
Poona Sarvajanik Sabha was established by MG Ranade in 1870 along with Ganesh Vasudeo
and others.
The Hindu Mahasabha was formed to protect the rights of the Hindu community in British
India, after the formation of the All India Muslim League in 1906 and the British India
government's creation of separate Muslim electorate under the Morley-Minto reforms of
1909
Statement 2: It did not advocate radical reforms as swadeshi or swaraj. It was a socio-
political organisation in British India which started with the aim of working as a mediating
body between the government and people of India.
The organisation was a precursor to the Indian National Congress which started with its first
session from Maharashtra itself.
The Pune Sarvajanik Sabha provided many of the prominent leaders of national stature to
the Indian freedom struggle including Bal Gangadhar Tilak. It was formed in 1870 by S. H.
Chiplunkar, Ganesh Vasudeo Joshi, Mahadev Govind Ranade, et al.

Q.354 Consider the following about the Satnami movement.


1. Satnami community was established first in East Bengal.
2. Their principles treated all people as equal.
3. They believed in the polytheism and multiplicity of Gods.
4. Satnamis often organized mass rebellions against the British citing them as ‘Dikus’ or
outsiders.
Select the correct answer using the codes below.
(a) 1 and 3 only
(b) 2, 3 and 4 only
(c) 2 only
(d) 1, 3 and 4 only
Solution: (c)
Justification: Statement 1: Ghasidas established Satnami community in Chhattisgarh,
India based on "Satnam", meaning Truth, and equality.
Ghasi Das instigated a socio-religious order that discarded and helped in demolishing the
hierarchical caste system from the society.
Statement 2 and 3: Satnami principles state that 'there is only one God', which is described
as formless and infinite.
Ghasi Das was of the view that the measures to eradicate social injustice and disparity would
stay insufficient and unfinished without the proper transformation of the individuals.
Statement 4: They were focussed on reforms rather than on large scale movements.

Q.355 The main objective of the Prarthana Samaj was to


1. Spiritually purify the Hindu community from western pollution
2. Securing social justice for the oppressed
3. Revive the Gurukul education system in India
4. Establish a swatantra (free) India
Solution: (b)
Learning: Prarthana Samaj was founded by Aatma Ram Pandurang in 1867 with an aim to
make people believe in one God and worship only one God. The main reformers were the
intellectuals who advocate reforms of the social system of the Hindus.
It sought to remove caste restrictions, abolish child marriage, encourage the education of
women, and end the ban on widow remarriage.
The religious meetings of the Samaj drew upon Hindu, Buddhist and Christian texts.

Q.356 Consider the following about Shuddhi movement, in British India.


1. It was a socio-political movement aimed at reducing the conversions of Hindus to Islam
and Christianity.
2. It aimed at abolishing the practise of untouchability by converting outcasts from other
religions to Hinduism.
Select the correct answer using the codes below.
(a) 1 only
(b) 2 only
(c) Both 1 and 2
(d) None
Solution: (c)
Justification: Derived from ancient rite of shuddhikaran, or purification, it was started by
the Arya Samaj, and its founder Swami Dayanand Saraswati and his followers like Swami
Shraddhanand, who also worked on the Sangathan consolidation aspect of Hinduism, in
North India, especially Punjab in early 1900s, though it gradually spread across India.
Shuddhi had a social reform agenda behind its belligerent rationale and was aimed at
abolishing the practise of untouchability by converting outcasts from other religions to
Hinduism and integrating them into the mainstream community by elevating their position,
and instilling self-confidence and self-determination in them.
The movement strove to reduce the conversions of Hindus to Islam and Christianity, which
were underway at the time.

Q.357 With reference to Raja Rammohan Roy, consider the following statements.
1. Atmiya Sabha was founded by Debendranath Tagore which was later re-established and
renamed as Brahmo Samaj by Rammohan Roy.
2. Brahmo Samaj preached polytheism.
3. He helped William Bentinck to declare the practice of Sati a punishable offence.
4. He did not favour inter-caste marriages.
Select the correct answer using the codes below.
(a) 1 and 3 only
(b) 3 only
(c) 2 and 4 only
(d) 1, 2, 3 and 4
Solution: (b)
Justification: Statement 1 and 2: 1815, he established the Atmiya Sabha. The work of the
Atmiya Sabha was carried on by Maharishi Debendranath Tagore (father of Rabindranath
Tagore), who renamed it as Brahmo Samaj.
Statement 3: Through this organisation, he preached that there is only one God. He
combined the teachings of the Upanishads, the Bible and the Koran in developing unity
among the people of different religions.
Raj Rammohan Roy is most remembered for helping Lord William Bentinck to declare the
practice of Sati a punishable offence in 1829. He also protested against the child marriage
and female infanticide. He favored the remarriage of widows, female education and
women’s right to property.
Rammohan Roy started the first Bengali weekly Samvad Kaumudi and edited a Persian
weekly Mirat-ul-akhbar. He stood for the freedom of the press.
Statement 4: He believed in the equality of mankind. He did not believe in the supremacy of
the Brahmin priests. He favoured inter-caste marriages.

Q.358 With reference to the Theosophical Society, consider the following statements:
1. It was founded in Calcutta by Bhikaji Cama.
2. Main aim was to promote the study of ancient religions and philosophies.
3. Central Hindu School was chosen as the parent organization that acted as a catalyst for
the growth of this society.
Select the correct answer using the codes below.
(a) 2 only
(b) 1 and 3 only
(c) 2 and 3 only
(d) None of the above
Solution: (a)
Justification: Statement 1: The Theosophical Society was founded in New York (USA) in
1875 by Madam H.P. Blavatsky, a Russian lady, and Henry Steel Olcott, an American colonel.
Statement 2: Their main objectives were to form a universal brotherhood of man without
any distinction of race, colour or creed and to promote the study of ancient religions and
philosophies. They arrived in India and established their headquarters at Adyar in Madras in
1882.
Statement 3: Later in 1893, Mrs. Annie Besant arrived in India and took over the leadership
of the Society after the death of Olcott. Mrs. Annie Besant founded the Central Hindu School
along with Madan Mohan Malaviya at Benaras which later developed into the Banaras Hindu
University.

Q.359 The Aligarh Movement was started for


(a) For the social and educational advancement of the Muslims in India
(b) Opposing the establishment of the Indian National Congress
(c) Creating the All-India Muslim League
(d) To reform the University system as proposed by the Wood’s Despatch
Solution: (a)
Learning: It was started by Sir Syed Ahmad Khan (1817-98) for the social and educational
advancement of the Muslims in India.
He fought against the medieval backwardness and advocated a rational approach towards
religion.
In 1866, he started the Mohammadan Educational Conference as a general forum for
spreading liberal ideas among the Muslims.
In 1875, he founded a modern school at Aligarh to promote English education among the
Muslims. This had later grown into the Mohammadan Anglo Oriental College and then into
the Aligarh Muslim University.

Q.360 The Young Bengal Movement


(a) attacked old traditions and decadent customs
(b) advocated women’s rights and their education
(c) organized debates against idol worship
(d) All of the above
Solution: (d)
Learning: Henry Vivian Derozio was the founder of the Young Bengal Movement. He
taught in the
Hindu College, Calcutta.
His followers were known as the Derozians and their movement the Young Bengal
Movement.
They attacked old traditions and decadent customs. They also advocated women’s rights
and their education.
They founded associations and organized debates against idol worship, casteism and
superstitions.

Q.361 The objective(s) of the Deoband Movement was/were to


1. To propagate among the Muslims Western and rational thought
2. To promote women centred social reforms
Which of the above is/are correct?
(a) 1 only
(b) 2 only
(c) Both 1 and 2
(d) None
Solution: (d)
Justification: The orthodox section among the Muslim ulema organised the Deoband
Moovement.
It was a revivalist movement whose twin objectives were : (i) to propagate among the
Muslims the pure teachings of the Koran and the Hadis and (ii) to keep alive the spirit of
jihad aganist the foreign rulers.
The new Deoband leader Mahmud-ul-Hasan (1851-1920) sought to impart a political and
intellectualcontent to the religious ideas of the school. The liberal interpretation of Islam
created a political awakening among its followers.

Q.362 Which of the following is/are correct with reference to the views of Swami Vivekananda?
(a) He championed the supremacy of Vedantic philosophy
(b) He was against the idea and concept of Indian nationalism.
(c) He resisted the idea of idol worship and deitifying religious figures.
(d) None of the above.
Solution: (a)
Justification: Option A: He was a key figure in the introduction of the Indian
philosophies of Vedanta and Yoga to the Western world and is credited with raising
interfaith awareness, bringing Hinduism to the status of a major world religion during the
late 19th century.
A disciple of Ramakrishna Paramahamsa, he championed the supremacy of Vedantic
philosophy. His talk at the Chicago (USA) Conference of World Religions in 1893 made the
westerners realize the importance of Hinduism.
Option B: He was a major force in the revival of Hinduism in India, and contributed to the
concept of nationalism in colonial India.
Option C: He was a devotee of Hindu Gods and Goddesses and himself used to meditate in
the presence of their idols.
Q.363 The Young Bengal Movement by Louis Vivian Derozio
1. Worked towards the religious and spiritual liberation of the youth
2. Pushed its followers to question all authority
3. Demanded education for women
Select the correct answer using the codes below.
(a) 1 and 2 only
(b) 2 and 3 only
(c) 1 and 3 only
(d) 1, 2 and 3
Solution: (b)
Justification: The Young Bengal movement was a group of radical Bengali free thinkers
emerging from Hindu College, Calcutta.
The Young Bengals were inspired and excited by the spirit of free thought and revolt against
the existing social and religious structure of Hindu society.
Derozio joined Hindu College in 1828 and within a short period attracted students. The
Academic Association, established in 1828 under the guidance of Derozio, arranged
discussions on subjects such as free will, free ordination, fate, faith, the sacredness of truth.

Q.364 The Veda Samaj worked to


1. Promote the belief of One God
2. Rationalize superstitious Hindu practices based on Vedas
3. Open Educational Institutions to promote the study of Eastern mysticism
Select the correct answer using the codes below.
(a) 1 only
(b) 1 and 2 only
(c) 2 and 3 only
(d) None of the above
Solution: (a)
Justification & Learning: It was established in Madras (Chennai) in 1864.
It was inspired by the Brahmo Samaj and worked to abolish caste distinctions and promote
widow remarriage and women’s education.
Its members believed in one God. They condemned the superstitions and rituals of orthodox

Q.365 Which of the following statements about Ishwarchandra Vidyasagar is correct?


1. He was a key figure of the Bengal Renaissance.
2. He took the initiative in proposing and pushing the Widow Remarriage Act 1856.
Which of the above is/are correct?
(a) 1 only
(b) 2 only
(c) Both 1 and 2
(d) None
Solution: (c)
Learning: He also demonstrated that the system of polygamy without restriction was not
sanctioned by the ancient Hindu Shastras.
He reconstructed the Bengali alphabet and reformed Bengali typography into an alphabet of
twelve vowels and forty consonants. He contributed significantly to Bengali and Sanskrit
literature.Vidyasagar's "Barna Porichoy" is still considered a classic.

Q.366 Consider the following about Tattwabodini Sabha.


1. It was a splinter group of the Brahmo Samaj.
2. It was founded by Debendranath Tagore.
3. Its main objective was to promote a rational and humanist form of Hinduism based on
the Vedanta.
Select the correct answer using the codes below.
(a) 1 and 2 only
(b) 3 only
(c) 2 and 3 only
(d) 1, 2 and 3
Solution: (d)
Learning: As per some commentators, “Their view, at least in the early years, was that the
world is created by God, and all things within it are pathways to knowledge of Brahman, the
Ultimate Self, and the ultimate goal. Similarly, they saw that material wealth, if made and
possessed with the correct intention - that of helping society and others – was in fact not
only ethically sound, but an utter necessity for harmonious society.”
In 1859, the Sabha was dissolved back into the Brahmo Samaj by Debendranath Tagore.

Q.367 With reference to Sister Nivedita, consider the following statements.


1. She became the first Western woman to be received into an Indian monastic order.
2. She served as the President of Ramakrishna Mission.
3. She termed the teachings of Buddhists as Anti-Vedic and regressive.
Select the correct answer using the codes below.
(a) 1 only
(b) 2 and 3 only
(c) 1 and 3 only
(d) 1 and 2 only
Solution: (a)
Justification: Sister Nivedita met Swami Vivekananda in 1895 in London and traveled to
Calcutta, India in 1898. Her 150th birth anniversary was celebrated recently.
Statement 1: Swami Vivekananda gave her the name Nivedita (meaning "Dedicated to God")
when he initiated her into the vow of Brahmacharya in 1898.
Statement 2: Nivedita had close associations with the newly established Ramakrishna
Mission. However, because of her active contribution in the field of Indian Nationalism, she
had to publicly dissociate herself from the activities of the Ramakrishna Mission under the
then president Swami Brahmananda.
Statement 3: She started taking interest in the teachings of Gautama Buddha, Swami
Vivekananda as alternate source of peace and benediction.

Q.368 Consider the following about Tattwabodini Sabha.


1. It was a splinter group of the Brahmo Samaj.
2. It was founded by Debendranath Tagore.
3. It was opposed to the Vedantic views of Hinduism.
Select the correct answer using the codes below.
(a) 3 only
(b) 1 and 2 only
(c) 2 and 3 only
(d) 1, 2 and 3
Solution: (b)
Learning: As per some commentators, “Their view, at least in the early years, was that the
world is created by God, and all things within it are pathways to knowledge of Brahman, the
Ultimate Self, and the ultimate goal.
Similarly, they saw that material wealth, if made and possessed with the correct intention -
that of helping society and others – was in fact not only ethically sound, but an utter
necessity for harmonious society.”
In 1859, the Sabha was dissolved back into the Brahmo Samaj by Debendranath Tagore.

Q.369 Sayyid Ahmad Khan had a notable contribution in spreading education among
Muslims in India. He would NOT agree with which of the following in the initiative
towards social reforms?
(a) Belief in Western Education
(b) Education of women
(c) Interpretation of the Quran in the light of traditional sharia customs rather than
rationalism and Science
(d) He would agree with all of the above
Solution: (c)
Learning: He was a reformer of the nineteenth century. He led liberal, social and cultural
movements within the Muslim society.
Option A: While pursuing studies of different subjects including European jurisprudence,
Syed began to realise the advantages of Western-style education, which was being offered
at newly established colleges across India.
Despite being a devout Muslim, Syed criticised the influence of traditional dogma and
religious orthodoxy, which had made most Indian Muslims suspicious of British influences
Option C: Syed studied the Quran and sciences within the court and later studied law from
the University of Edinburgh. Being a modernist, he supported the rational studies of religious
scriptures.

Q.370 With reference to the Servants of India Society, consider the following
statements.
1. It was the first nationalist organization to be established and operated from
outside India.
2. The society was only concerned with promotion of swadeshi and achievement
of political swaraj.
3. It published Hitavada in all the major regional languages of India.
Select the correct answer using the codes below.
(a) 1 and 2 only
(b) 2 and 3 only
(c) 1 only
(d) None of the above
Solution: (d)
Justification: Statement 1: The Servants of India Society was formed in Pune,
Maharashtra, in 1905 by Gopal Krishna Gokhale.
Statement 2: The Society organized many campaigns to promote education,
sanitation, health care and fight the social evils of untouchability and
discrimination, alcoholism, poverty, oppression of women and domestic abuse.
Statement 3: The publication of The Hitavada, the organ of the Society in English
from Nagpur commenced in 1911.

THE STRUGGLE BEGINS


PRE CONGRESS

Q.371 The Ilbert Bill Controversy is said to be a high watermark in the history of Indian National
Movement. This is because it invoked issues of
1. Racial discrimination between Indian and Europeans
2. Security lapse on the Indian borders, especially the North-Western frontier
3. Suppression of Indian media houses and their nationalization by the Government
Select the correct answer using the codes below.
(a) 1 only
(b) 1 and 2 only
(c) 2 and 3 only
(d) 1 and 3 only
Solution: (a)
Background & Justification: Lord Ripon wanted to remove two kinds of law that had
been prevalent in India.
According to the system of law, a European could be tried only by a European Judge or a
European Magistrate.
The disqualification was unjust and it was sought to cast a needless discredit and dishonour
upon the Indian-born members of the judiciary.
C.P. Ilbert, Law Member, introduced a bill in 1883 to abolish this discrimination in judiciary.
But Europeans opposed this Bill strongly.
They even raised a fund of one lakh fifty thousand rupees and established an organisation
called the Defence Association.
They also suggested that it was better to end the English rule in India than to allow the
English to be subjected to the Indian Judges and Magistrates.
The press in England joined the issue. Hence, Ripon amended the bill to satisfy the English in
India and England.
Learning: The Ilbert Bill controversy helped the cause of Indian nationalism.
Ripon was totally disillusioned and heartbroken and he tendered his resignation and left for
England. The immediate result of this awakening of India was the birth of the Indian National
Congress in 1885, the very next year of Ripon’s departure.

Q.372 Amongst the following, the earliest to form was


(a) The British Indian Association
(b) The Bombay Association
(c) The Madras Mahajana Sabha
(d) Poona Sarvojanik Sabha
Solution: (a)
Justification: These were some of the early Political Associations that paved the way for
organized struggle in India:
The British Indian Association – 1851 Bengal
The Bombay Association — 1852 Dadabhai Naoroji
East India Association 1856 London
Madras Native Association 1852
Poona Sarvojanik Sabha—1870
The Madras Mahajana Sabha—1884
We have covered some of them in previous tests, and some will be covered later.
Learning: Bombay Association was started to vent public grievances to the British.
The first organisation in the Madras Presidency to agitate for the rights of Indians was the
Madras Native Association which was established by publicist Gazulu Lakshminarasu Chetty
in 1849. This organisation did not survive for long and was eventually disbanded.
The members of the Sabha felt the necessity of creating an organization at All India level to
relieve and free the nation from the clutches of British rule and solve the problems of
Indians leading to the Birth of the Madras Mahajana Sabha.
Along with the Poona Sarvajanik Sabha, Bombay Presidency Association and the Indian
Association, Madras Mahajana Sabha is considered to be a predecessor of the Indian
National Congress.

Q.373 With reference to the East India Association (London), consider the following statements.
1. It was founded by Surendranath Banerjee.
2. It superseded the London Indian Society.
3. It was an extremist organization that had its main goal of organizing an armed struggle
against the British in India.
Select the correct answer using the codes below.
(a) 1 only
(b) 2 and 3 only
(c) 2 only
(d) 1, 2 and 3
Solution: (c)
Justification: It was founded by Dadabhai Naoroji in 1866, in collaboration with Indians
and retired British officials in London.
It superseded the London Indian Society and was a platform for discussing matters and ideas
about India, and to provide representation for Indians to the Government.
In 1868, the East India Association had nearly 600 members. This had increased to 1,000 in
1878. Female members were admitted from 1912.
The Association produced a journal (Journal of the East India Association) from its inception
which included the papers that were delivered before their meetings.
Papers and proceedings of these meetings were then produced in the Asiatic Quarterly
Review, which eventually superseded the Journal of the East India Association.

Q.374 The aim of the Indian Association found by Suredranath Banerjee was to
1. Represent the views of the educated middle class and take united community action
2. Give shape to and establish the Indian National Congress (INC)
3. Advocate constitutional reforms through legislative action
4. Organize seditious movements to dethrone the British Empire
Solution: (a)
Learning: The foundations of the Indian National Movement were laid by Suredranath
Banerjee with the formation of Indian Association at Calcutta in 1876.
The aim of the Association was to represent the views of the educated middle class, inspire
the Indian community to take the value of united action.
Option B: The Indian Association was, in a way, the forerunner of the Indian National
Congress, which was founded, with the help of A.O. Hume, a retired British official. However,
the aim of IA was not to establish INC.
The birth of Indian National Congress (INC) in 1885 marked the entry of new educated
middle-class into politics and transformed the Indian political horizon.
The first session of the Indian National Congress was held in Bombay in December 1885
under the president ship of Womesh Chandra Banerjee and was attended among others by
and Badr-uddin- Tyabji.

INDIAN NATIONAL CONGRESS AIMS AND OBJECTIVES

Q.375 Consider the following statements with reference to Indian National Congress (INC)
towards princely states in British India.
1. The INC, for the first time, at its Nagpur Session in 1920, enunciated its policy towards
the peoples’ movement in the Princely States.
2. Post withdrawal of non-cooperation movement, INC adopted the policy of total non-
interference in the political status of the princely states.
Which of the above is/are correct?
(a) 1 only
(b) 2 only
(c) Both 1 and 2
(d) None
Solution: (a)
Justification: The general policy of the Indian National Congress (INC) towards the Indian
princely states was first enunciated in 1920 at Nagpur and later carried forward till
independence. It was first of exhortation for a responsible government in the states and
later active political intervention.
It called upon the Princes to grant full responsible government in their States. However, no
direct political activity was to be resorted for.
This position continued till 1935.
Two developments in the mid-30s brought a radical change in the relations between the
Princely States and British India. The Government of India Act of 1935 projected a scheme of
federation in which the States were brought into a direct constitutional relationship with
British India.
The second major impact on the States was created by the acceptance of office by the
Congress in majority of the British Indian provinces in 1937.
The installation of the Congress ministries in the neighbouring British Indian provinces
encouraged the Praja Mandal leaders to step up their political activities for demanding
responsible government in the Princely States

Q.376 Who among the following are associated with the founding of Indian National
Congress (INC)?
1. Badruddin Taiyabji
2. A.O. Hume
3. Dinshaw Edulji Wacha
4. Surendranath Banerjee
Select the correct answer using the codes below.
(a) 2 only
(b) 1, 2 and 3 only
(c) 2 and 3 only
(d) 1, 3 and 4 only
Solution: (c)
Justification: Founded in 1885, it was the first modern nationalist movement to emerge in
the British Empire in Asia and Africa.
The Indian National Congress conducted its first session in Bombay in 1885 at the initiative
of retired Civil service officer Allan Octavian Hume.
Other founders include Dadabhai Naoroji and Dinshaw Edulji Wacha. Edulji Wacha worked in
close association with Dadabhai Naoroji and Pherozeshah Mehta in the Congress and along
with his political activities was active in both social reform and education.
Womesh Chandra Bonnerjee was the first president of Congress.

MODERATES

Q.377 The Moderates were called so because

1. They had rejected the idea of participating in legislative councils and instead believed in
peaceful protests.
2. They relied on organized struggle against the British, something that was opposite to the
extremist camp.
Which of the above is/are correct?
(a) 1 only
(b) 2 only
(c) Both 1 and 2
(d) None
Solution: (d)
Justification: The Moderates had faith in the British justice and goodwill. They were
called moderates because they adopted peaceful and constitutional means to achieve their
demands. They organized in parties and associations, even extremists did the same, but they
the latter were more radical in their approach.
Main Demands of Moderates were:
Expansion and reform of legislative councils.
Greater opportunities for Indians in higher posts by holding the ICS examination
simultaneously in England and in India.
Separation of the judiciary from the executive.
More powers for the local bodies.
Reduction of land revenue and protection of peasants from unjust landlords.
Abolition of salt tax and sugar duty.
Reduction of spending on army.
Freedom of speech and expression and freedom to form Associations
NATIONAL MOVEMENT 1905-1918
MITITANT NATIONALISM

Q.378 A manifesto titled “The Revolutionary”, that was produced as evidence in the Kakori
conspiracy case of 1925, was written by
(a) Sachindra Nath Sanyal
(b) W.C. Banerjee
(c) Feroze Shah Mehta
(d) Badruddin Tyabji
Solution: (a)
Justification: Use elimination. It is clear that the manifesto was an extremist project, so
all options other than A can be rejected because they were all moderates. It was a manifesto
of HSRA.
Learning: Hindustan Socialist Republican Association (HSRA) was a revolutionary
organisation established in 1928 at Feroz Shah Kotla in New Delhi by Chandrasekhar Azad,
Bhagat Singh, Sukhdev Thapar and others.
Sanyal wrote a manifesto for the HRA entitled Revolutionary. This was distributed around
large cities of North India in 1924.
It proposed the overthrow of British colonial rule and its replacement with what it termed a
"Federal Republic of the United States of India".
In addition, it sought universal suffrage and the socialist-oriented aim of abolition of "all
systems which make any kind of exploitation of man by man possible"

Q.379 Which of the following revolutionary organizations, of the colonial period, were
based out of India?
1. Swadesh Sevak Home
2. Jashn-e-Hind
3. United India House
Select the correct answer using the codes below.
(a) 1 and 2 only
(b) 2 and 3 only
(c) 1 and 3 only
(d) 1, 2 and 3
Solution: (c)
Justification: Several nationalists were residing in countries abroad, but the discriminatory
policies of the host countries soon resulted in a flurry of political activity among Indian
nationalists.
As early as 1907, Ramnath Purl, a political exile on the West Coast, issued a Circular-e-Azadi
(Circular of Liberty) in which he also pledged support to the Swadeshi Movement; Tarak
Nath Das in Vancouver started the Free Hindustan and adopted a very militant nationalist
tone.
G.D. Kumar set up a Swadesh Sevak Home in Vancouver on the lines of the India House in
London and also began to bring out a Gurmukhi paper called Swadesh Sevak which
advocated social reform and also asked Indian troops to rise in revolt against the British.
In 1910, Tarak Nath Das and G.D. Kumar, by now forced out of Vancouver, set up the
United India House in Seattle in the US, where every Saturday they lectured to a group of
twenty- five Indian labourers.
Close links also developed between the United India House group, consisting mainly of
radical nationalist students, and the Khalsa Diwan Society, and in 1913 they decided to send
a deputation to meet the Colonial Secretary in London and the Viceroy and other officials in
India.

Q.380 The Government of India had decided to convert the famous headquarters of
the Ghadr party into a library and a museum. This famous place is known as
(a) Gandhar Ashram
(b) Mani Bhawan
(c) Shaantigram Ashra
(d) Yugantar Ashram
Solution: (d)
Learning: The Hindustan Association of the Pacific Coast, known as the Gadar Party was
founded in 1913 to free India from British slavery.
The headquarters of the association was established initially at 436 Hill Street, San Francisco
and named as “Yugantar Ashram.”
The GoI decided in 2013 to convert this memorial into a library and Museum.
The association began publishing a magazine, Gadar (revolt), for free distribution.
The Gadar publication exposed the British imperialism and called upon the Indian people to
unite and rise up against British rule and throw the British out of India. The publication
Gadar, over a period of time, became well known among Indians and the Hindustan
Association of the Pacific Coast itself became known as the Gadar party.

Q.381 With reference to pre-independent history of India, what was Komagata Maru?
(a) It was an Indian steamship that sailed to Japan carrying Muslim passengers.
(b) It was a Japanese steamship that sailed to Canada carrying immigrants from Punjab.
(c) It was a Canadian steamship that sailed to Japan carrying Indian labourers.
(d) It was an Indian steamship that sailed to Canada carrying people from Punjab.
Solution: (b)
Learning: The Komagata Maru incident provoked revolutionary nationalistic response in
India.
It was a Japanese steamship that sailed from Hong Kong to Vancouver, Canada via Japan in
1914.
It was carrying 376 passengers who were immigrants from Punjab, India. Of these, only 24
were granted admittance in Canada when the ship docked in Vancouver.
At that time, Canada had laws restricting entry of migrants of Asian origin. Following a two
month stalemate, the ship and its 352 passengers were escorted out of the dock by the
Canadian military and forced to sail back to India.
At every port that the ship touched, it triggered off a wave of resentment and anger among
the Indian community and became the occasion for anti-British mobilization.

Q.382 The Ghadar party was


1. A revolutionary organization
2. Aiming at securing India's independence from British rule
3. Associated with Rashbehari Bose
4. Was founded in Punjab
Select the correct answer using the codes below.
(a) 1, 2 and 3 only
(b) 1 and 4 only
(c) 2, 3 and 4 only
(d) 1, 2 and 4 only
Solution: (a)
Learning: The Ghadar Party was an organization founded by Punjabi Indians in the United
States and Canada with the aim of securing India's independence from British rule.
Key members included Lala Har Dayal, Sohan Singh Bhakna, Abdul Hafiz Mohamed
Barakatullah, Kartar Singh Sarabha, and Rashbehari Bose.
After the outbreak of World War I, Ghadar party members returned to Punjab to agitate for
rebellion alongside the Babbar Akali Movement.
In 1915 they conducted revolutionary activities in central Punjab and organised uprisings.
Their presence shook the hold of the British empire and police surveillance in Punjabi
villages increased in an attempt to crush the rebellion.

Q.383 The Anushilan Samiti formed in 1906 propounded


(a) Constitutional methods to achieve administrative reforms
(b) British support for bringing social reforms in India
(c) Revolutionary violence as means for ending British rule in India
(d) The doctrine of ‘Council entry’ to weaken the British raj from within
Solution: (c)
Learning: It was a Bengali Indian organisation and arose from conglomerations of local youth
groups and gyms in Bengal.
It had two prominent if somewhat independent arms in East and West Bengal identified as
Dhaka Anushilan Samiti and the Jugantar group respectively.
Between its foundations to its gradual dissolution through 1930s, the Samiti collaborated
with other revolutionary organisations in India and abroad.
Led by notable revolutionaries of the likes of Aurobindo Ghosh, Rash Behari Bose and
Jatindranath Mukherjee, the Samiti was involved in a number of noted incidences of
revolutionary terrorism
against British interests and administration in India. These included the early attempts to
assassinate Raj officials, the 1912 attempt on the life of Viceroy of India, as well as the
Sedetious conspiracy during World War I.

Q.384 Consider the following statements about Madam Bhikaji Cama.


1. She co-founded the Paris Indian Society.
2. She has once served as the president of the British Committee of the Indian National
Congress (INC).
3. She unfurled the National Flag at the International Socialist Conference in 1907.
4. She has served as the Secretary General of the Indian Home Rule Society.
Select the correct answer using the codes below.
(a) 1 and 3 only
(b) 2, 3 and 4 only
(c) 3 only
(d) 1, 2 and 4 only
Solution: (a)
Justification: Statement 1: She co-founded the Paris Indian Society together with Singh
Rewabhai Rana and Munchershah Burjorji Godre.
Statement 2 and 4: She served as the private secretary of Dadabhai Naoroji, the president of
the British Committee of the Indian National Congress. Together with Naoroji and Singh
Rewabhai Rana, Cama supported the founding of Varma's Indian Home Rule Society in 1905.
Statement 3: In her appeal for human rights, equality and for autonomy from Great Britain,
she unfurled what she called the "Flag of Indian Independence"

Q.385 In British India, what was the contention against the passage of the Public Safety Bill
and the Trade Disputes Bill, amongst the revolutionaries?
1. The act gave more power to the British Government and the police against Indian
revolutionaries and freedom fighters.
2. It affected the civil liberties of citizens and workers.
Which of the above is/are correct?
(a) 1 only
(b) 2 only
(c) Both 1 and 2
(d) None
Solution: (c)
Justification: The Act declared unlawful secondary action and any strike whose purpose was
to coerce the government of the day directly or indirectly.
In addition, incitement to participate in an unlawful strike was made a criminal offence,
punishable by imprisonment for up to two years; and the attorney general was empowered
to sequester the assets and funds of unions involved in such strikes.
It declared unlawful mass picketing which gave rise to the intimidation of a worker.
Q.386 In Modern India, Yugantar Ashram was
1. The headquarters of the Gadar party
2. An Ashram adopted by the Indian National Congress in the memory of Mahatma Gandhi
3. A seditious publication of the India House, London
4. A war memorial established by the British government for Indian freedom fighters in
1948
Solution: (a)
Learning: The Hindustan Association of the Pacific Coast, known as the Gadar Party was
founded in 1913 to free India from British slavery.
The headquarters of the association was established initially at 436 Hill Street, San Francisco
and named as “Yugantar Ashram.”
The GoI decided in 2013 to convert this memorial into a library and Museum.

SWADESHI AND BOYCOTT

Q.387 With reference to the Partition of Bengal, 1905, consider the following statements.
1. It was done when Lord Dalhousie was the Governor General of British India.
2. The incident gave birth to the Swadeshi and Rithala movement.
3. New provinces of East Bengal and Assam were created after the partition of Bengal.
Select the correct answer using the codes below.
(a) 1 and 2 only
(b) 3 only
(c) 2 and 3 only
(d) 1 only
Solution: (b)
Justification: The Partition of Bengal into two provinces was effected in July 1905.
The new province of Eastern Bengal and Assam included the whole of Assam and the Dacca,
Rajshahi and Chittagong divisions of Bengal with headquarters at Dacca.
Though Curzon justified his action on administrative lines, partition divided the Hindus and
Muslims in Bengal. This led to the anti-partition agitation all over the country.
This had also intensified the National Movement leading to the Swadeshi movement.
Statement 2: Rithala is a place in Delhi, the movement or the riotsin Rithala that happened
between 1911-1932 cannot be related to the Swadeshi movement.
Learning: In 1874, Bengal had become too large unit to be administered as a single unit. The
government wanted to solve the problem permanently.
It was thought that Assam should be extended and it should include some eastern districts of
Bengal.
Congress instead suggested that the Government should separate the Non-bengali speaking
areas (Bihar or Orissa) rather than splitting the Bengali speaking area.
This was because the new Western Bengal made the Bengali people minority in their own
country because now the population of Hindi and Oriya speaking people was 37 Million while
the population of Bengali Speaking people was only 17 Million.

Q.388 The Swadeshi movement that spanned over several decades in India
1. Followed ‘moderate’ methods as well boycotts of institutions
2. Was also called vandemataram movement
3. Was based on Gandhian ideals
Select the correct answer using the codes below.
(a) 1 and 2 only
(b) 1, 2 and 3
(c) 1 only
(d) 2 and 3 only
Solution: (b)
Justification: L. M. Bhole identifies five phases of the Swadeshi movement.
1850 to 1904: developed by leaders like Dadabhai Naoroji, Gokhale, Ranade, Tilak, G.V. Joshi
and Bhaswat.K.Nigoni. This was also known as First Swadeshi Movement.
1905 to 1917: Began with and because of the partition of Bengal in 1905 by Lord Curzon.
1918 to 1947: Swadeshi thought shaped by Gandhi, accompanied by the rise of Indian
industrialists.
1948 to 1991: Widespread curbs on international and inter-state trade. India became a
bastion of obsolete technology during the licence-permit raj.
1991 onwards: liberalization and globalization. Foreign capital, foreign technology, and many
foreign goods are not excluded and doctrine of export-led growth resulted in modern
industrialism.
The second Swadeshi movement started with the partition of Bengal by the Viceroy of India,
Lord Curzon in 1905 and continued up to 1911. It was the most successful of the pre-
Gandhian movement.

EXTREMIST LEADERSHIP

Q.389 Consider the following statements. The Indian National Congress (INC) was
divided into two groups
1. Between extremists and moderates
2. In the year 1915 at the Lucknow session
3. Based on a resolution passed in the leadership of Pandit Nehru
Select the correct answer using the codes below.
(a) 1 and 2 only
(b) 1 only
(c) 2 and 3 only
(d) 1 and 3 only
Solution: (b)
Justification: This was in the year 1907 between extremists and moderates at the Surat
Session of the Congress.
The period 1885-1905 was known as the period of the moderates as moderates dominated
the INC.
These Moderates used petition, prayers, and protest to influence the (British) colonial
government. Moderates were not able to achieve their goals.
This created dissatisfaction among the people.
In 1907 the INC meeting was to be held in Pune and the extremists wanted Lala Lajpat Rai or
Bal Gangadhar Tilak as president. But moderates wanted Rash Behari Ghosh to be president.
This conflict, based on the differences in the larger anti-British ideology, lead to the split.

Q.390 He was one of the first and strongest advocates of Swaraj, was called as the “The father of
the Indian unrest" and published newspapers such as Mahratta and Kesari. He was
(a) Bipin Chandra Pal
(b) Bal Gangadhar Tilak
(c) Lala Lajpat Rai
(d) Gopal Ganesh Agarkar
Solution: (b)
Learning: These newspapers championed nationalistic causes.
The British colonial authorities called him "The father of the Indian unrest." He was also
conferre with the title of "Lokmanya", which means "accepted by the people (as their
leader)"
He organised the Deccan Education Society in 1884 with a few of his college friends,
including Gopal Ganesh Agarkar, Mahadev BallalNamjoshi and VishnushastriChiplunkar.
Their goal was to improve the quality of education for India's youth.
Tilak joined the Indian National Congress in 1890. He opposed its moderate attitude,
especially towards the fight for self-government. He was one of the most-eminent radicals at
the time

Q.391 What is the importance of Tashkent in the Indian freedom struggle?


(a) The Communist Party of India was founded in Tashkent.
(b) The Indian National Army was supervised from Tashkent.
(c) Third round table conference was held in Tashkent.
(d) Tashkent agreement was signed between Pandit Nehru and Lord Irwin here.
Solution: (a)
Learning: Attracted by the Soviet Union and its revolutionary commitment, a large number
of Indian revolutionaries and exiles abroad made their way there.

The most well-known and the tallest of them was M.N. Roy, who along with Lenin, helped
evolve the Communist International’s policy towards the colonies.
Seven such Indians, headed by Roy, met at Tashkent in 1920 and set up a Communist Party
of India.
Independently of this effort a number of left-wing and communist groups and organizations
had begun to come into existence in India after 1920

Q.392 The movement that precedes the first split in the Indian National Congress
between ‘moderates’ and ‘extremists’ is
(a) Swadeshi Movement
(b) Quit India Movement
(c) Non-Cooperation Movement
(d) Civil Disobedient Movement
Solution: (a)
Learning: Moderates believed in the policy of settlement of minor issues with the
government by deliberations. But the extremists believed in agitation, strikes and boycotts
to force their demands.
Some nationalists led by Lokmanya Tilak agitated against the moderate behaviour of
congress against British rule. The split between these two sections became outward at the
end of Banaras Session of congress (1905).
In Surat Session (1907), extremists wanted Lala Lajpat Rai or Tilak as a President candidate of
congress and Moderates supported Dr. Rashbihari Ghosh to be the President.
But Lala Lajpat Rai stepped down and Dr. Rashbihari Ghosh became the President.

WORLD WAR 1 AND INDIAN RESPONSE

Q.393 Consequences of the first world war for India and Indian National movement included:
1. Huge increase in defence expenditure which was financed by war loans and increasing
taxes
2. Customs duties were lowered to zero to import cheap British products in India
3. The Rowlatt Act was repealed
4. The Indian industries were rejuvenated as demand for industrial goods during the WW-I
increased.
Select the correct answer using the codes below.
(a) 1 and 4 only
(b) 2, 3 and 4 only
(c) 1 and 3 only
(d) 1, 2 and 4 only
Solution: (a)
Justification: The war created a new economic and political situation.
It led to a huge increase in defence expenditure which was financed by war loans and
increasing taxes: customs duties were raised and income tax introduced.
Through the war years prices increased – doubling between 1913 and 1918 – leading to
extreme hardship for the common people.
Villages were called upon to supply soldiers, and the forced recruitment in rural areas
caused widespread anger.
Then in 1918-19 and 1920-21, crops failed in many parts of India, resulting in acute
shortages of food. This was accompanied by an influenza epidemic.
According to the census of 1921, 12 to 13 million people perished as a result of famines and
the epidemic.
However, the urban India faced a different situation. The Indian industries were rejuvenated
as demand for industrial goods during the WW-I increased. The same industrialists lent
support the nationalist by funding and indirect participation.
Learning: The FWW also gave a push to Indian National Movement.
End of the world war ended the myth of invincibility of British Empire in India.
The soldiers that returned after war understood the way British were exploiting the Indian.
To fight the British they decided to raise the morale of masses.
This led to the rise of national consciousness and soon Non Cooperation movement was
launched.
Formation of USSR also led to the spread of the ideology of socialism in India with the
formation of CPI and imparted a socialist tinge to freedom struggle

Q.394 The Jallianwala Bagh assembly of people 1919 intended to


1. Participate in Baisakhi celebrations
2. Launch Rowlatt Satyagraha with Amritsar as the first base
3. Condemn the arrest and deportation of Satya Pal and Dr Saifuddin Kitchlew
Select the correct answer using the codes below.
(a) 1 only
(b) 1 and 3 only
(c) 1 and 2 only
(d) 3 only
Solution: (b)
Justification: Civilians, in the majority Sikhs, had assembled to participate in the annual
Baisakhi celebrations, a religious and cultural festival for Punjabi people and also to condemn
the arrest and deportation of two national leaders, Satya Pal and Dr Saifuddin Kitchlew.
Coming from outside the city, many may have been unaware of the imposition of martial law.
The Jallianwala Bagh massacre took place on 13 April 1919 when troops of the British Indian
Army under the command of Colonel Reginald Dyer fired machine guns into the crowd.
Both the Defence of India Act and Rowlatt Act were in force then, but the Rowlatt Satyagraha
was launched by Gandhiji (not by the assembly at Jallianwala).

Q.395 Consider the following about the Indian Home Rule movement.
1. It wanted complete independence from the British Empire and establish a constitutional
democracy in India.
2. Gandhiji boycotted the All India Home Rule League.
3. It worked as a joint movement of the Indian National Congress and the All India Muslim
League.
Select the correct answer using the codes below.
(a) 1 and 2 only
(b) 2 and 3 only
(c) 1 only
(d) None of the above
Solution: (d)
Justification: Statement 1: Between 1916 and 1918, when the World War-I was gradually
approaching an end, prominent Indians like Muhammad Ali Jinnah, Bal Gangadhar Tilak, Sir
S. Subramania Iyer, Annie Besant decided to organise a national alliance of leagues across
India.
It was specifically to demand Home Rule, or self-government within the British Empire for all
of India.
Statement 2: In 1920, the All India Home Rule League elected Mahatma Gandhi as its
President. In a year, the body would merge into the Indian National Congress to form a
united Indian political front.
Statement 3: It was run separately from both parties. Some members of both INC and AIML
joined the Home Rule movement.

Q.396 Consider the following about the Indian Home Rule movement.
1. It wanted to obtain the status of a Dominion for India within the British Empire.
2. It worked as a joint movement of the Indian National Congress and the All India Muslim
League.
3. Gandhiji was once elected as a President of the All India Home Rule League.
Select the correct answer using the codes below.
(a) 1 and 2 only
(b) 2 and 3 only
(c) 1 and 3 only
(d) 1, 2 and 3
Solution: (c)
Justification: Statement 1: Between 1916 and 1918, when the World War-I was gradually
approaching an end, prominent Indians like Muhammad Ali Jinnah, Bal Gangadhar Tilak, Sir
S. Subramania Iyer, Annie Besant decided to organise a national alliance of leagues across
India.
It was specifically to demand Home Rule, or self-government within the British Empire for all
of India.
Statement 2: It was run separately from both parties. Some members of both INC and AIML
joined the Home Rule movement.
Statement 3: In 1920, the All India Home Rule League elected Mahatma Gandhi as its
President. Ina year, the body would merge into the Indian National Congress to form a
united Indian political front.

Q.397 Indian Home Rule movement demanded


(a) Status of a Dominion for India within the British Empire
(b) Complete independence from the British Empire
(c) Entry for Indians in the legislative and executive councils
(d) Uprooting of the British Empire from India
Solution: (a)
Justification: Between 1916 and 1918, when the World War-I was gradually approaching an
end, prominent Indians like Muhammad Ali Jinnah, Bal Gangadhar Tilak, Sir S. Subramania
Iyer, Annie Besant decided to organise a national alliance of leagues across India.
It was specifically to demand Home Rule, or self-government within the British Empire for all
of India.
It was run separately from both parties. Some members of both INC and AIML joined the
Home Rule movement.
In 1920, the All India Home Rule League elected Mahatma Gandhi as its President. In a year,
the body would merge into the Indian National Congress to form a united Indian political
front.

BRITISH RESPONSE

Q.398 Why was there an outrage against the enactment of the Defence of India Act
1915?
1. It was an emergency criminal law enacted with the intention of curtailing the nationalist and
revolutionary activities.
2. It applied only to Indian subjects and white Europeans were exempt under the provisions of
the law.
Which of the above is/are correct?
(a) 1 only
(b) 2 only
(c) Both 1 and 2
(d) None
Solution: (a)
Justification: Statement 1 and 2: It was similar to the British Defence of the Realm Acts, and
granted the Executive very wide powers of preventive detention, internment without trial,
restriction of writing, speech, and of movement.
However, unlike the English law which was limited to persons of hostile associations or origin,
the Defence of India act could be applied to any subject of the King, and was used to an
overwhelming extent against Indians.
The passage of the act was supported unanimously by the non-official Indian members in the
Viceroy's legislative council, and was seen as necessary to protect against British India from
subversive nationalist violence.
The act was first applied during the First Lahore Conspiracy trial in the aftermath of the failed
Ghadar Conspiracy of 1915, and was instrumental in crushing the Ghadr movement in Punjab
and the Anushilan Samiti in Bengal.

NATIONAL MOVEMENT 1919-1939


EMERGENCE OF GANDHI

Q.399 What was the Kheda peasant movement, that Gandhiji organized, mainly
concerned with?
(a) Tenancy reforms in the British province of the present-day Gujarat
(b) Remission of taxes for the farmers following the failure of harvest
(c) Better working conditions for the landless labourers
(d) Implementation of ceiling laws in the Kutch region
Solution: (b)
Learning: The Kheda peasant struggle is also known as no-tax peasant struggle. It was a
satyagraha launched in 1919 under the leadership of Gandhiji, Sardar Vallabhbhai Patel,
Indulal Yajnik, N.M. Joshi, Shankerlal Pareekh and several others.
It was the second Satyagraha movement after Champaran Satyagraha. Gandhi ji organise
this movement to support peasants of kheda district.The peasants of kheda could not be
able to pay high taxes of british due to crop failure and plague epidemic.
Patel was a close ally of Gandhi in this satyagraha. Patel and his colleagues organised a major
tax revolt, and all the different ethnic and caste communities of Kheda rallied around it. The
peasants of Kheda signed a petition calling for the tax for this year to be scrapped in wake of
the famine. The government in Bombay rejected the charter.
But, the Government finally sought to foster an honourable agreement for both parties. The
tax for the year in question, and the next would be suspended, and the increase in rate
reduced, while all confiscated property would be returned.

Q.400 By the time Gandhiji arrived in India


1. He had already forged a technique of non-violent Satyagraha in South Africa
2. Indian National Congress (INC) was already established in India
3. The first Swadeshi movement had already been waged
Select the correct answer using the codes below.
(a) 1 and 2 only
(b) 2 and 3 only
(c) 1 and 3 only
(d) 1, 2 and 3
Solution: (d)
Justification: As the historian Chandran Devanesan has remarked, South Africa was “the
making of the Mahatma”. It was in South Africa that Mahatma Gandhi first forged the
distinctive techniques of non-violent protest known as satyagraha, first promoted harmony
between religions, and first alerted upper-caste Indians to their discriminatory treatment of
low castes and women.
The India that Mahatma Gandhi came back to in 1915 was rather different from the one that
he had left in 1893. Although still a colony of the British, it was far more active in a political
sense.
The Indian National Congress now had branches in most major cities and towns. Through the
Swadeshi movement of 1905-07 it had greatly broadened its appeal among the middle
classes. That movement had thrown up some towering leaders – among them Bal
Gangadhar Tilak of Maharashtra, Bipin Chandra Pal of Bengal, and Lala Lajpat Rai of Punjab.

Q.401 In his first public appearance in India in a speech at the Banaras Hindu
University (BHU), Gandhiji aired which of these major concerns?
1. Lack of the spirit of self-government in India
2. Lack of concern for the farmers and the labouring poor in India
3. Exploitation of the University education system by the British
4. Inappropriateness of the political organizations in India
Select the correct answer using the codes below.
(a) 1 and 2 only
(b) 2, 3 and 4 only
(c) 1 and 4 only
(d) 1, 3 and 4 only
Solution: (a)
Justification: His first major public appearance was at the opening of the Banaras Hindu
University (BHU) in February 1916. Compared to the dignitaries at the opening speech,
Gandhiji was relatively unknown.
He had been invited on account of his work in South Africa, rather than his status within
India.
When his turn came to speak, Gandhiji charged the Indian elite with a lack of concern for the
labouring poor.
He worried about the contrast between the “richly bedecked noblemen” present and
“millions of the poor” Indians who were absent. Gandhiji told the privileged invitees that
“there is no salvation for India unless you strip yourself of this jewellery and hold it in trust
for your countrymen in India”.
“There can be no spirit of self-government about us,” he went on, “if we take away or allow
others
to take away from the peasants almost the whole of the results of their labour. Our salvation
can only come through the farmer. Neither the lawyers, nor the doctors, nor the rich
landlords are going to secure it.”

Q.402 The unpopular legislation Rowlatt Act, against which Gandhi launched a
satyagraha, did NOT provide for
(a) Arrests without warrant
(b) Indefinite detention for a suspect of terrorism
(c) Scrapping the process of Judicial trial in India
(d) Stricter control of the press
Solution: (c)
Learning: The act was known as the black Act on account of its unpopularity.
It provided for stricter control of the press, arrests without warrant, indefinite detention
without trial, and juryless in camera trials for proscribed political acts. The accused were
denied the right to know the accusers and the evidence used in the trial.
The act provided speedy trial of the offenses by a special cell that consisted of 3 High Court
Judges. There was no court of appeal above that panel.
Those convicted were required to deposit securities upon release, and were prohibited from
taking part in any political, educational, or religious activities.
It also gave enormous powers to the police to search a place and arrest any person they
disapproved of without warrant.
This act gave a new direction to the movement. Gandhi organized a mass protest at all India
level.
In April, 1919, an all India strike was organized. There was mob violence in Bombay,
Ahmadabad and all other major towns. The Satyagraha lost momentum with the Jallianwala
Bagh tragedy on April 13, 1919.

Q.403 Hind Swaraj or Indian Home Rule is a book written by Mohandas K. Gandhi in
1909. The Gujarati edition was banned by the British on its publication in India but the
English edition was not banned because
(a) The book was never published in English in India
(b) British believed that English edition would not affect the majority of Indian population
that was native Hindi
(c) Gandhi revised the English edition cutting down the radical and seditious content
(d) The then Viceroy favoured the book
Solution: (b)
Justification & Learning: Mohandas Gandhi wrote this book in his native language,
Gujarati, while traveling from London to South Africa onboard SS Kildonan Castle in 1909.
In it he expresses his views on Swaraj, modern civilization, mechanisation etc
The Gujarati edition was banned by the British on its publication in India. Gandhi then
translated it into English.
The English edition was not banned by the British, who concluded that the book would have
little impact on the English-speaking Indians' subservience to the British and British ideas.

Q.404 In the Words of Gandhiji


(a) Satyagraha is physical force
(b) Satyagraha is passive resistance
(c) Satyagraha is pure soul-force
(d) In the use of satyagraha, there is no ill-will whatsoever.
Select the correct answer using the codes below.
(a) 3 and 4 only
(b) 2, 3 and 4 only
(c) 1 and 2 only
(d) D. 1, 2, 3 and 4
Solution: (a)
Justification: These are the words of Gandhiji:
‘It is said of “passive resistance” that it is the weapon of the weak, but the power which is
the subject of this article can be used only by the strong.
This power is not passive resistance; indeed it calls for intense activity. The movement in
South Africa was not passive but active …
‘ Satyagraha is not physical force. A satyagrahi does not inflict pain on the adversary; he does
not seek his destruction … In the use of satyagraha, there is no ill-will whatever.
‘ Satyagraha is pure soul-force. Truth is the very substance of the soul. That is why this force
is called satyagraha. The soul is informed with knowledge. In it burns the flame of love. …
Nonviolence is the supreme dharma …
‘It is certain that India cannot rival Britain or Europe in force of arms. The British worship the
war-god and they can all of them become, as they are becoming, bearers of arms.
The hundreds of millions in India can never carry arms. They have made the religion of non-
violence their own ...’

Q.405 With reference to Champaran Satyagraha of 1917, consider the following statements.
1. The word Satyagraha was used for the first time in this agitation.
2. The tinkathia system was associated with this movement
3. Rajendra Prasad and J. B. Kripalani participated in the movement.
Select the correct answer using the codes below.
(a) 2 and 3 only
(b) 1 and 2 only
(c) 3 only
(d) 1, 2 and 3
Solution: (a)
Justification: Statement 1: The word Satyagraha was used for the first time in Anti
Rowlatt Act agitation.
Statement 2: Indigo plantations extended into Bihar, where too European planters used the
zamindari system to force their peasant tenants to bow to their will.
Where they could not buy zamindaris they obtained leases from local zamindars, and in the
form of ‘thekadars’ exercised the same rights over peasants as they would have had as
zamindars.
In Champaran district of Bihar, most European planters obtained thekas or leases for whole
villages from the large Bettiah zamindari. Here, as the demand for indigo grew with
expanding textile imports, the planters imposed what came to be known as the tinkathia
system, the peasants being forced to raise indigo on the best parts of their rented lands.
Statement 3: To address the situation, Gandhi arrived in Champaran in 1917 with a team of
eminent lawyers: Brajkishore Prasad, Rajendra Prasad, Anugrah Narayan Sinha Ramnavmi
Prasad, and others including J. B. Kripalani.

Q.406 The first Ashram established by Gandhiji in India was


(a) Kochrab Ashram, near Ahmedabad
(b) Sabarmati Asharam
(c) Khadi Pratishthan and Ashram at Sodepur, Kolkata
(d) Bardoli Taluka Ashram
Solution: (a)
Learning: Kochrab was the first Ashram established by Gandhiji in India in 1915 CE, it was a
pioneering centre for students of Gandhian ideas to practise Satyagraha, self-sufficiency,
Swadeshi, work for the upliftment of the poor, women and untouchables, and public
education and sanitation.
Sabarmati Ashram was the residence of Mahatma Gandhi for over twelve years (1917-1930
CE) and laboratory for his social experiments, e.g. agriculture and literacy promotion.
It was also from here that in 1930, Mahatma Gandhi started his Dandi March.
This was not his first Ashram though. You can read about a lot of important Gandhian sites
here.

Q.407 Consider the following statements about the Kheda Satyagraha of 1918.
1. It was the first Satyagraha movement launched in India by Gandhi
2. The peasants of Kheda were unable to pay high taxes of the British due to crop failure
and plague epidemic which inspired the Satyagraha.
3. During this time, Sardar Patel organised a major tax revolt within the Kheda community.
Select the correct answer using the codes below.
(a) 1 and 2 only
(b) 2 and 3 only
(c) 1 and 3 only
(d) 2 only
Solution: (b)
Justification: S1: It was the third Satyagraha movement after Champaran Satyagraha and
Ahmedabad mill strike. Gandhi ji organise this movement to support peasants of kheda
district.
The peasants of Kheda could not be able to pay high taxes of British due to crop failure and
plague epidemic.
In Gujarat, Gandhi was chiefly the spiritual head of the struggle. His chief lieutenant, Sardar
Vallabhbhai Patel and a close coterie of devoted Gandhians, toured the countryside,
organised the villagers and gave them political leadership and direction.
The tax withheld, the government's collectors and inspectors sent in thugs to seize property
and cattle, while the police forfeited the lands and all agrarian property. The farmers did not
resist arrest, nor retaliate to the force employed with violence.
The Government finally sought to foster an honourable agreement for both parties. The tax
for the year in question, and the next would be suspended, and the increase in rate reduced,
while all confiscated property would be returned.

Q.408 Why did Gandhiji resign from Congress party membership in 1934?
(a) Difference of opinion on suitable candidate for Presidency of the Indian National
Congress (INC)
(b) He wanted to quit active political life.
(c) To pursue his own vision of an independent India by doing constructive work
(d) Excessive interference of the British in the administration of INC
Solution: (c)
Learning: Gandhiji was convinced that he was out of tune with powerful trends in the
Congress.
He felt that a large section of the intelligentsia favoured parliamentary politics with which he
was in fundamental disagreement.
Another section of the intelligentsia felt estranged from the Congress because of his
emphasis on the spinning wheel as ‘the second lung of the nation,’ on Harijan work based on
a moral and religious approach, and on other items of the constructive programme.
Similarly, the socialist group, whose leader was Jawaharlal, was growing in influence and
importance but he had fundamental differences with it. Yet the Socialists felt constrained by
the weight of his personality.
He said, “for me to dominate the Congress in spite of these fundamental differences is
almost a species of violence which I must refrain from.”
Hence, in October 1934, he announced his resignation from the Congress.

Q.409 Gandhiji believed that


1. Swaraj is not just freedom but liberation in redeeming one’s self-respect and capacities
for self- realisation from institutions of dehumanisation.
2. Understanding the real ‘Self ’, and its relation to communities and society, is critical to
the project of attaining Swaraj.
Which of the above is/are correct?
(a) 1 only
(b) 2 only
(c) Both 1 and 2
(d) None
Solution: (c)
Justification: An analogous concept to Freedom in Indian political thought is ‘Swaraj’.
The term Swaraj incorporates within it two words — Swa (Self) and Raj (Rule). It can be
understood to mean both the rule of the self and rule over self.
Swaraj, in the context of the freedom struggle in India referred to freedom as a
constitutional and political demand, and as a value at the social-collective level.
That is why Swaraj was such an important rallying cry in the freedom movement inspiring
Tilak’s famous statement — “Swaraj is my birth right and I shall have it.”
It is the understanding of Swaraj as Rule over the Self that was highlighted by Mahatma
Gandhi in his work Hind Swaraj where he states, “It is swaraj when we learn to rule
ourselves”.
Swaraj is not just freedom but liberation in redeeming one’s self-respect, self-responsibility,
and capacities for self-realisation from institutions of dehumanisation.
Understanding the real ‘Self ’, and its relation to communities and society, is critical to the
project of attaining Swaraj.
Gandhiji believed the development that follows would liberate both individual and collective
potentialities guided by the principle of justice.
Needless to say, such an understanding is as relevant to the Twenty first century as it was
when Gandhiji wrote the Hind Swaraj in 1909.

Q.410 Fasting became an important way of Satyagraha in the Indian National Movement.
Gandhiji took his first fast unto death in
(a) Champaran Satyagraha
(b) Kheda Satyagraha
(c) Ahmadabad Mill Strike
(d) Jallianwala Bagh tragedy
Solution: (c)
Learning: The Ahmadabad cotton mill strike was spearheaded by Gandhiji in 1918.
Gandhiji did fast unto death to get 35% raise for the mill workers.
In 1932, Mahatma Gandhi began a “fast unto death” to protest the Communal award that
gave separate electorates to the depressed classes

Q.411 Ahmadabad Mill Strike 1918 led by Gandhiji was based on the issue of
(a) Plague Bonus of the previous year to workers
(b) Inhuman treatment of mill workers by the management
(c) Objection of management on participation of workers in the freedom struggle
(d) Large-scale layoffs of mill workers
Solution: (a)
Learning: In 1918, there was a situation of conflict between the Gujarat Mill owners and
workers on the question of Plague Bonus of 1917.
The Mill Owners wanted to withdraw the bonus whole the workers demanded a 50% wage
hike. The Mill Owners were willing to give only 20% wage hike.
In March later, under the leadership of Gandhi, there was a strike in the cotton mills. In this
strike Gandhi used the weapon of Hunger strike.
It was carried out in pure non-violent disciplined way. The result was that the strike was
successful and the workers got a 35% wage increase.

Q.412 The following places were associated with the imprisonment of Mahatma Gandhi.
Arrange them chronologically in order of their appearance/association with the life
of Gandhi.
1. Yervada
2. Sabarmati
3. Johannesburg
Select the correct answer using the codes below.
(a) 123
(b) 312
(c) 321
(d) 231
Solution: (c)
Justification: Statement 3: His first few imprisonments were in:
10-1-1908 to 31-1-1908 Johannesburg
1-11-1908 to 30-11-1908 Johannesburg and Volksrust
Statement 1 and 2: In India, the first three imprisonments were in:
9-4-1919 to 11-4-1919 Railway Carriage
10-3-1922 to 20-3-1922 Sabarmati
21-3-1922 to 11-1-1924 Yeravda
Then he was imprisoned in Yeravda and Sabarmati several times, with the last imprisonment
in Agakhan Palace, Delhi

Q.413 The 'tinkathia system’ associated with the Champaran movement was actually
(a) A solution devised by Gandhi to end the exploitation of peasants by European planters
(b) A system that made peasants cultivate indigo on part of their land at unremunerative
prices
(c) A division of land that was held by the European planters in the name of peasants
(d) A three-pronged separation of peasants on the basis of caste, gender and religion
Solution: (b)
Learning: There had been sporadic resistance in Champaran since the 1860s to the 'tinkathia
system by which European planters holding thikadari leases from the big zamindars made
peasants cultivate indigo on part of their land at unremunerative prices.
As indigo declined from about 1900 in face of competition from synthetic dyes, the planters
tried to pass the burden on the peasants by charging sharahbeshi (rent-enhancement) or
tawan (lump-sum compensation) in return for releasing them from the obligation to grow
indigo.
Widespread resistance developed in the Motihari-Bettiah region between 1905-08, and even
after it continued.
It was as a part of this on-going confrontation that Raj Kumar Shukla, a prosperous peasant
cum-petty moneylender, contacted Gandhi at the Lucknow Congress of 1916.
This became the historic Champaran movement.

Q.414 What is the significance of Sabarmati Ashram in the modern history of India?
1. It served as the last residence of Mahatma Gandhi.
2. It has been declared as a national monument by the Indian government.
3. Salt Satyagraha was started from this Ashram in 1930.
Select the correct answer using the codes below.
(a) 1 only
(b) 2 and 3 only
(c) 3 only
(d) 1 and 3 only
Solution: (b)
Justification: The Sabarmati Ashram (also known as Harijan or Satyagraha Ashram) was
home to Mohandas Gandhi from 1917 until 1930 and served as one of the main centres of
the Indian freedom struggle.
Statement 1: This was Sevagram cottage (not Sabarmati Ashram). Gandhiji left to Delhi from
Sevargam Cottage in 1946 and then went to Noakhali. From there he did not return to
Sewagram while engaged in his task of communal unity and peace making. He was martyred
in Delhi in 1948.
Statement 2 and 3: It is common knowledge.
Learning: On his return from South Africa, Gandhi’s first Ashram in India was established in
the Kochrab area of Ahmedabad in 1915. The Ashram was then shifted in 1917 to a piece of
open barren land on the banks of the river Sabarmati. Reasons for this shift included: he
wanted to do some experiments in living eg farming, animal husbandry, cow breeding, Khadi
and related constructive activities, for which he was in search of this kind of barren land.
The Prime Minister has recently released the coin and postal stamp in the honour of
Shrimad Rajchandraji on his 150th birth anniversary at Abhay Ghat. Shrimad Rajchandraji is
best known for his teachings on Jainism and his spiritual guidance to Mahatma Gandhi.
Q.415 Writing in the journal Young India in 1931, who said, "I cannot possibly bear the idea
that a man who has got wealth should get the vote but a man who has got character but no
wealth or literacy should have no vote…. for the crime of being a poor man…"
(a) Jawahar Lal Nehru
(b) M.K. Gandhi
(c) Rajendra Prasad
(d) Maulana Abul Kalam Azad
Solution: (b)
Learning: Young India, Harijan and Hind Swaraj were some of the most famous works of Gandhi.
Young India was a weekshed - a weekly paper or journal.
He used Young India to spread his unique ideology and thoughts regarding the use of nonviolence in
organising movements and to urge readers to consider, organise, and plan for India's eventual
independence from Britain.
The journal was reprinted in USA by the India Home Rule League of America

NON COOPERATION MOVEMENT

Q.416 Why was the Non-cooperation movement called on account of the


Chauri-Chaura incident that happened in the United Provinces?
1. The incident was against the ideals of non-violence of Gandhi.
2. The incident was initiated by extremist leaders of India, which Gandhi did not want to
include in the Non-cooperation movement
3. The non-cooperation movement was not launched in the United Provinces and Gandhi
feared that its inclusion might destabilize the movement.
Select the correct answer using the codes below.
(a) 1 only
(b) 1 and 3 only
(c) 1 and 2 only
(d) 2 and 3 only
Solution: (a)
Justification: The Chauri Chaura incident occurred at Chauri Chaura in the Gorakhpur district
of the United Province, (modern Uttar Pradesh) in British India in February 1922, when a
large group of protesters, participating in the Non-cooperation movement, clashed with
police, who opened fire.
In retaliation the demonstrators attacked and set fire to a police station, killing all of its
occupants. The incident led to the deaths of three civilians and several policemen.
Mahatma Gandhi who was strictly against violence halted the Non-cooperation Movement
on the national level in February 1922 itself, as a direct result of this incident.

Q.417 Consider the following statements about the Khilafat Movement.


1. It was led by Muhammad Ali Jinnah.
2. It was concerned with the Jaziat-Ul-Arab region that encompasses Iran, Arabia, Syria and
Palestine.
3. It was not supported by the Indian National Congress.
Select the correct answer using the codes below.
(a) 1 and 2 only
(b) 2 and 3 only
(c) 2 only
(d) 1 and 3 only
Solution: (c)
Justification: Launched in 1919-1920, it was a movement of Indian Muslims, led by
Muhammad Ali and Shaukat Ali, that demanded the following:
The Turkish Sultan or Khalifa must retain control over the Muslim sacred places in the
erstwhile Ottoman empire;
The jazirat-ul-Arab (Arabia, Syria, Iraq, Palestine) must remain under Muslim sovereignty;
And the Khalifa must be left with sufficient territory to enable him to defend the Islamic
faith.
The Congress supported the movement and Mahatma Gandhi sought to conjoin it to the
Non- cooperation Movement.

Q.418 Who among the following leaders were associated with the Khilafat agitation?
1. Mohammad Ali
2. Maulana Abul Kalam Azad
3. Chittaranjan Das
Select the correct answer using the codes below.
(a) 1 only
(b) 2 and 3 only
(c) 1 and 3 only
(d) 1, 2 and 3
Solution: (d)
Background: In 1920 the British imposed a harsh treaty on the Turkish Sultan or Khalifa.
People were furious about this as they had been about the Jallianwala massacre. Also, Indian
Muslims were keen that the Khalifa be allowed to retain control over Muslim sacred places
in the erstwhile Ottoman Empire.
Justification: The leaders of the Khilafat agitation, Mohammad Ali and Shaukat Ali, wished
to initiate a full-fledged Non-Cooperation Movement.
Gandhiji supported their call and urged the Congress to campaign against “Punjab wrongs”
(Jallianwala massacre), the Khilafat wrong and demand swaraj.
Statement 1, 2 and 3: Mohammad Ali and his brother Maulana Shaukat Ali joined with other
Muslim leaders.
These were Pir Ghulam Mujaddid Sarhandi Sheikh Shaukat Ali Siddiqui, Dr. Mukhtar Ahmed
Ansari, Raees-Ul-Muhajireen Barrister Jan Muhammad Junejo, Hasrat Mohani, Syed Ata
Ullah Shah Bukhari, Maulana Abul Kalam Azad and Dr. Hakim Ajmal Khan to form the All
India Khilafat Committee.
The organisation was based in Lucknow.
The Khilafat Committee in Bengal included Mohmmad Akram Khan, Manruzzaman
Islamabadi, Mujibur Rahman Khan and Chittaranjan Das.

Q.419 A Khilafat Committee was formed in Bombay in 1919 with the main objective to
1. defend the Khalifa’s temporal powers
2. call off the Khilafat movement
3. decide on whether Muslims should participate in the Non-cooperation movement
4. All of the above
Solution: (a)
Justification: The First World War had ended with the defeat of Ottoman Turkey. And
there were rumours that a harsh peace treaty was going to be imposed on the Ottoman
emperor – the spiritual head of the Islamic world (the Khalifa).
To defend the Khalifa’s temporal powers, a Khilafat Committee was formed in Bombay in
March 1919.
A young generation of Muslim leaders like the brothers Muhammad Ali and Shaukat Ali,
began discussing with Mahatma Gandhi about the possibility of a united mass action on the
issue.
Gandhiji saw this as an opportunity to bring Muslims under the umbrella of a unified
national movement.
At the Calcutta session of the Congress in September 1920, he convinced other leaders of
the need to start a non-cooperation movement in support of Khilafat as well as for swaraj.

Q.420 Mahatma Gandhi called a halt to the Non-Cooperation Movement due to


1. Motilal Nehru and Chittaranjan Das forming the Swaraj Party amidst the movement
2. Chauri Chaura incident that led to violent clashes
3. Gandhi’s arrest by the authorities
Select the correct answer using the codes below.
(a) 2 and 3 only
(b) 2 only
(c) 1 and 3 only
(d) 1, 2 and 3
Solution: (b)
Justification: Statement 2 and 3: The non-cooperation movement was withdrawn
because of the Chauri Chaura incident. Although he had stopped the national revolt single-
handedly, on March 10, 1922, Gandhi was arrested. On March 18, 1922, he was imprisoned
for six years for publishing seditious materials. This led to suppression of the movement and
was followed by the arrest of other leaders.
Statement 1: Although most Congress leaders remained firmly behind Gandhi, the
determined broke away.
The Ali brothers would soon become fierce critics. Motilal Nehru and Chittaranjan Das
formed the Swaraj Party (after the movement, not in between), rejecting Gandhi's
leadership.
Many nationalists had felt that the non-cooperation movement should not have been
stopped due to isolated incidents of violence, and most nationalists, while retaining
confidence in Gandhi, were discouraged.

Q.421 The Non-cooperation movement urged


1. The use of khadi and Indian material as alternatives to those shipped from Britain
2. Boycott of British educational institutions and law courts
Which of the above is/are correct?
(a) 1 only
(b) 2 only
(c) Both 1 and 2
(d) None
Solution: (c)
Justification: From 1920 to 1922, Gandhi started the Non-Cooperation Movement.
As per the call, all offices and factories would be closed. Indians would be encouraged to
withdraw from Raj-sponsored schools, police services, the military, and the civil service, and
lawyers were asked to leave the Raj's courts. Public transportation and English-
manufactured goods, especially clothing, was boycotted.
Although most Congress leaders remained firmly behind Gandhi, the determined broke
away. The Ali brothers would soon become fierce critics. Motilal Nehru and Chittaranjan Das
formed the Swaraj Party, rejecting Gandhi's leadership.

SWARAJIST AND NO CHANGERS

Q.422 The Swaraj Party was formed by members of the Indian National Congress who
(a) Were unable to get elected to the provincial legislative councils
(b) Opposed Gandhi's suspension of the Non-cooperation movement in response to the Chauri
Chaura tragedy
(c) Did not approve of the rural reform agenda of INC
(d) Were against Indian forces supporting the British in the First World War
Solution: (b)
Learning: It was formed in 1923 by members of the INC who had opposed the suspension of all
civil resistance in 1922 in response to the Chauri Chaura tragedy.
They suggested that the nationalists should end the boycott of the legislative councils, enter
them, expose them as ‘sham parliaments’ and as ‘a mask which the bureaucracy has put on,’ and
obstruct ‘every work of the council.’
This, they argued, would not be giving up non-cooperation but continuing it in a more effective
form by extending it to the councils themselves.
C.R. Das and Motilal Nehru thus formed the Swaraj party.

Q.423 Which of the following is correct with reference to Gandhiji's view towards
Swarajist leaders?
1. He was opposed to the council-entry program of Swarajists.
2. He never considered them patriots and avoided any personal relations with Swarajists.
Select the correct answer using the codes below.
(a) 1 only
(b) 2 only
(c) Both 1 and 2
(d) None
Solution: (a)
Justification: The following was a statements made by Gandhiji, “Nor is the difference
between them and myself one of mere detail. There is fundamental difference.
I retain the opinion that Council-entry is inconsistent with Non-co-operation, as I conceive it.
Nor is this difference a mere matter of interpretation of the word “Non-co-operation,” but
relates to the essential mental attitude resulting in different treatment of vital points.”
He further says, “It is with reference to such mental attitude that the success or the failure
of the Triple Boycott is to be judged, and not merely by a reference to the actual results
attained. It is from advantages to the country than to be in them. I have, however, failed to
convince my Swarajist friends but I recognise that so long as they are otherwise, their place
is undoubtedly in the Councils. It is the best for us all.”
This clearly tells us that he was opposed to the council entry programme of Swarajists
(statement 1 will be correct), and that Swrajists and Gandhi were not in enemy camps, so 2
will be incorrect

ANTI SIMON MOVEMENT

Q.424 The first ‘White Paper on Constitutional Reforms’ for India was prepared
and submitted for the consideration of the Joint Select Committee of the
British Parliament on the recommendations of the
(a) Hunter Commission
(b) Radcliffe Commission
(c) Butler Commissions
(d) Simon Commission
Solution: (d)
Learning: The commission submitted its report in 1930.
To consider the proposals of the commission, the British Government convened three round
table conferences of the representatives of the British Government, British India and Indian
princely states.
The recommendations of this committee were incorporated (with certain changes) in the
next Government of India Act of 1935.
These were:
There should be a constitutional reconstruction.
It would be a federal constitution.
The provinces should be given full autonomy including law.
The governor should have discretionary power to relate to internal security and
administrative powers to protect the different communities.
The number of members of provincial legislative council should be increased.
Governor general should have complete power to appoint the members of the cabinet.
The government of India should have complete control over the high court.

Q.425 Following the rejection of the recommendations of the Simon Commission by Indians, an
all- party conference was held at Mumbai in May 1928. The conference appointed a drafting
committee under Motilal Nehru to
(a) Draw up a constitution for India
(b) Prepare the roadmap to the next non-cooperation movement
(c) Dwell on the future leadership of the Indian National Congress (INC)
(d) None of the above
Solution: (a)
Learning: The draft constitution was prepared which was called “Nehru Committee
Report“. This report was submitted on August 28, 1928 at the Lucknow conference of all the
parties.
The main points of the Nehru report were as follows:
India would be given Dominion status. This means independence within the British
Commonwealth. India will be a federation which shall have a bicameral legislature at the
centre and Ministry would be responsible to the legislature.
Governor General of India would be the constitutional head of India and will have the same
powers as that of British Crown. There will be no separate electorate. The draft report also
defined the citizenship and fundamental rights.

Q.426 Which of the following was/were the recommendations of the Simon Commission
setup in 1927?
1. India’s constitution should be unitary in nature.
2. The provincial governments should devolve financial powers to the local bodies.
3. Separate electorates should be abolished.
4. Elections to the legislative assemblies will be based on Universal adult franchise.
Select the correct answer using the codes below.
(a) 1 and 3 only
(b) 4 only
(c) 1 and 2 only
(d) None of the above
Solution: (d)
Justification: As per the commission, there should be a constitutional reconstruction in the
form of a federal constitution. The provinces should be given full autonomy including law.
Other major recommendations include:
The number of members of provincial legislative council should be increased. Governor-
general should have complete power to appoint the members of the cabinet.
The governor should have discretionary power to relate to internal security and
administrative powers to protect the different communities.
The government of India should have complete control over the high court.
There were no Indian members in the commission. No universal franchise was proposed and
the position of governor-general remained unaffected.
There was no provision to abolish separate electorate but it was rather extended to other
communities as well. No financial devolution was proposed.

NEHRU REPORT

Q.427 Consider the following statements about the events that transpired in the late 1920s in
British India.
1. Gandhiji supported and applauded Pandit Nehru for his successful passing of the
Independence resolution at the Madras Congress, 1927.
2. Pandit Nehru opposed Gandhiji’s ‘trusteeship’ solution to zamindar- peasant conflicts.
Which of the above is/are correct?
(a) 1 only
(b) 2 only
(c) Both 1 and 2
(d) None
Solution: (b)
Justification: Throughout 1928 and 1929, the period before the launch of the Civil
Disobedience Movement, Gandhi acted as a brake on mounting pressure for another round
of all-India mass struggle.
Excerpts from Sumit Sarkar clarify all question statements:
He (Gandhiji) had strongly disapproved of Jawaharlal's snap Independence resolution passed
in his absence at the Madras Congress (1927).
At Calcutta next year he was able to push through a compromise formula which accepted
the Nehru Report's dominion status objective provided the British granted it by the end of
1929, failing which the Congress would be free to go in for Civil Disobedience and Purna
Swaraj.
Gandhi tried to confine Congress activities during 1929 to constructive work in villages,
prohibition, and boycott of British goods, plus redress, along Bardoli lines, of 'specific
grievances'.
He encouraged public bonfires of foreign cloth (for which he was arrested in Calcutta in
March, and awarded a token fine), and toured the country collecting funds for khadi, but
repeatedly rejected pressures for any all-out struggle.
Jawaharlal graced the Lahore Congress (December 1929) with the first of his stirring
Presidential addresses, boldly sketching out a new internationalist and socially-radical
perspective for the freedom movement—a perspective so far confined to small Leftist sects.
He said - 'I must frankly confess that I am a socialist and a republican, and am no believer in
kings and princes, or in the order which produces the modern kings of industry”…..He
attacked Gandhi's pet 'trusteeship' solution for zamindar-peasant and capital-labour
conflicts:
“Many Englishmen honestly consider themselves the trustees for India, and yet to what a
condition they have reduced our country!” was the view Nehru held which made him
reject the trusteeship thesis of Gandhiji.

CIVIL DISOBEDIENCE MOVEMENT

Q.428 The issue of protest chosen by the Dandi March was the
(a) State monopoly in the manufacture and sale of salt
(b) High taxes on salt leading to high purchase price
(c) Both (a) and (b)
(d) None of the above
Solution: (c)
Learning: The 1882 Salt Act gave the British a monopoly on the collection and manufacture
of salt, limiting its handling to government salt depots and levying a salt tax.
Violation of the Salt Act was a criminal offence. Even though salt was freely available to
those living on the coast (by evaporation of sea water), Indians were forced to purchase it
from the colonial government.
The salt tax was a deeply symbolic choice, since salt was used by nearly everyone in India, to
replace the salt lost by sweating in India's tropical climate.
The salt tax represented 8.2% of the British Raj tax revenue, and hurt the poorest Indians the
most significantly.
In contrast to the other leaders who were not pleased with the object of protest, future
Governor- General of India, C. Rajagopalachari, supported this choice.
Q.429 Introduction of uniform salt tax throughout British India, that later inspired
movements like Civil Disobedience, was associated with?
(a) Lord Lytton
(b) Lord Mayo
(c) Lord Cunningham
(d) Lord Dufferin
Solution: (a)
Learning: He also abolished many import duties and supported the Free Trade Policy. This
had seriously affected the Indian economic interest.
The system of decentralisation of finance that had begun in the time of
Lord Mayo was continued during the time of Lord Lytton.
The provincial governments were empowered with some control over the expenditure of all
provincial matters like land-revenue, excise, stamps, law and justice.
Lytton wanted to encourage the provinces in collecting the revenue and thereby strengthen
the financial power and position of the provinces.
In 1878, the Statutory Civil Service was established exclusively for Indians but this was
abolished later.

Q.430 Why the Karachi session (1931) of the Indian National Congress (INC) holds
importance in our freedom struggle?
1. The Gandhi-Irwin pact was endorsed in this session.
2. The ‘Karachi Resolution’ on Fundamental Rights and the National Economic Programme
was passed in this session.
Which of the above is/are correct?
(a) 1 only
(b) 2 only
(c) Both 1 and 2
(d) None
Solution: (c)
Learning: Statement 1: In the Karachi session, congress passed a resolution to dissociate
itself from and disapprove the political violence in any shape. INC also reiterated the goal of
Poorna Swaraj here.
Statement 2: Some important aspects of these resolutions were: Basic civil rights of freedom
of speech, Freedom of Press, Freedom of assembly, Freedom of association, Equality before
law Elections on the basis of Universal Adult Franchise Free and compulsory primary
education, Substantial reduction in rent and taxes etc.

Q.431 The Civil Disobedience movement was withdrawn


(a) Right before the 1937 provincial elections
(b) After the Poona Pact
(c) Before the beginning of the Second World War
(d) By Gandhiji in 1934
Solution: (d)
Learning: Gandhiji assured Nehru and the leftists that the withdrawal of the civil
disobedience was dictated by the reality of the political situation. But this did not mean
following a policy of drift or bowing down before political opportunists or compromising
with imperialism.
Only civil disobedience had been discontinued, the war continued. The new policy, he said,
‘is founded upon one central idea — that of consolidating the power of the people with a
view to peaceful action.’

Q.432 The Indian National Congress (INC) adopted the Karachi resolution on
Fundamental Rights and Economic Policy. Which of the following were its
constituents?
1. Elections on the basis of Universal Adult Franchise
2. No interference by government in economic activities
3. Free and compulsory primary education.
Select the correct answer using the codes below.
(a) 1 only
(b) 1 and 3 only
(c) 3 only
(d) 2 and 3 only
Solution: (b)
Justification: Some important aspects of these resolutions were:
Basic civil rights of freedom of speech, Freedom of Press, Freedom of assembly, Freedom of
association, Equality before law
Elections on the basis of Universal Adult Franchise
Free and compulsory primary education
Substantial reduction in rent and taxes
Better conditions for workers including a living wage, limited hours of work
Protection of women and peasants Government ownership or control of key industries,
mines, and transport
Protection of Minorities.

Q.433 Which of the following is NOT correct with reference to the Civil Disobedience
movement?
(a) Women and teenagers participated in large numbers.
(b) Muslim participation remained low throughout the Civil Disobedience years.
(c) Unlike Non-Cooperation, Civil Disobedience did not coincide with any major labour upsurge.
(d) Gandhiji at the Lahore Congress gave a call for massive boycott of schools, courts and
workplaces to embolden the movement.
Solution: (d)
Justification: Civil Disobedience marked in fact a major step forward in the emancipation of
Indian women due to their large scale participation.
Muslim participation remained low throughout the Civil Disobedience years. In U.P., for
instance, where the Congress-Khilafat alliance had been so formidable in 1921-22, very few Civil
Disobedience prisoners in Allahabad between 1930 and 1933 were Muslims.
Apart from option (c), yet another contrast lay in the evident decline in the older, more purely
intelligentsia forms of protest like lawyers giving up their practice and students leaving official
institutions to start national schools and colleges.
Gandhi at the Lahore Congress rejected a call for boycott of schools and courts as unpractical—'I
do not see today the atmosphere about us necessary for such boycott.'
A Bihar Congress report of July 1930 admitted that there had been 'practically no response from
lawyers and students. So, clearly (d) is wrong.
The lag in respect of labour and the urban intelligentsia was counter-balanced, however, by the
massive response obtained from business groups and large sections of the peasantry

ROUND TABLE CONFERENCE

Q.434 At the Second Round Table Conference, the claim of the Indian National
Congress representing all of India was challenged by
1. The Muslim League saying that Congress doesn’t represent Muslims.
2. Dr. Ambedkar citing that Congress is not a representative of lower castes.
3. Motilal Nehru quoting that Congress is the opposite of Socialism.
4. The Princes saying that Congress had no stake in the territories of the princely states.
Select the correct answer using the codes below.
(a) 1, 2 and 4 only
(b) 2, 3 and 4 only
(c) 1 and 3 only
(d) 2 and 4 only
Solution: (a)
Justification: Statement 3: Motilal Nehru was an important leader of the Indian National
Congress, who also served as the Congress President twice, 1919–1920 and 1928–1929. So,
S3 is naturally incorrect.
Due to the opposition faced by Gandhi, representing the Congress, by these leaders and the
reluctance of the British to ceded to Congress’s demands, the Conference in London was
inconclusive.
In 1935, however, a new Government of India Act promised some form of representative
government.
Two years later, in an election held on the basis of a restricted franchise, the Congress won a
comprehensive victory. Gandhiji returned to India and resumed civil disobedience.

Q.435 Consider the following statements. Poona Pact led to


1. Gandhi ending his fast in the jail as a protest against separate electorates for the
depressed classes
2. Reservation of seats for the depressed classes out of general electorate in the provincial
Legislatures
Which of the above is/are correct?
(a) 1 only
(b) 2 only
(c) Both 1 and 2
(d) None
Solution: (c)
Justification: Statement 1: The Poona Pact refers to an agreement between Ambedkar and
Gandhi on the reservation of electoral seats for the depressed classes in the legislature of
then British India.
It was made in 1932 at Yerwada Central Jail in Poona and was signed by Madan Mohan
Malviya, Ambedkar and some other leaders as a means to end the fast that Gandhi was
undertaking in jail.
Statement 2: As per the pact:
Seat reservation for the Scheduled Castes (SC) and Scheduled Tribes (ST) in provincial
legislature
The STs and SCs would form an electoral college which would elect four candidates for the
general electorate
The representation of these classes was based on the standards of joint electorates and
reserved seats
About 19 per cent of seats were to be reserved for these classes in legislature
The system of election to the panel of candidates in both, Central and Provincial Legislature
should come to end in 10 years, unless it ends on mutual terms

Q.436 As per the Gandhi-Irwin Pact


1. The Congress would participate in the Round Table Conference and discontinue the Civil
Disobedience Movement.
2. The Government would withdraw all ordinances issued to curb the Congress.
3. The Government would revoke the Defence of India Act.
4. The Congress would stop picketing of liquor and foreign cloth shops by any means
Select the correct answer using the codes below.
(a) 1 and 2 only
(b) 2, 3 and 4 only
(c) 1, 2 and 3 only
(d) 1, 3 and 4 only
Solution: (a)
Justification: Under the Pact, the British Government agreed to:-
Withdraw all ordinances against the Congress and end prosecutions
Release all political prisoners, except those guilty of violence
Permit peaceful picketing of liquor and foreign cloth shops
Restore confiscated properties of the satyagrahis
Permit free collection or manufacture of salt by persons near the sea-coast
Lift the ban over the congress.
Much before the pact, the viceroy Irwin announced in October 1929, a vague offer of
'dominion status' for India in an unspecified future and a Round Table Conference to discuss
a future constitution. Later, the second round table conference which was held in 1931.

Q.437 The Round Table Conference was organized to


1. Discuss a future constitution for India
2. Discuss the implementation of a dominion status for India
Which of the above is/are correct?
(a) 1 only
(b) 2 only
(c) Both 1 and 2
(d) None
Solution: (a)
Justification: The three Round Table Conferences of 1930–32 were a series of
conferences organized by the British Government to discuss constitutional reforms in India.
They were conducted as per the recommendation of Jinnah to Lord Irwin viceroy and his
friend Ramsay MacDonald then Prime Minister of Great Britain, and by the report submitted
by the Simon Commission in May 1930.
Demands for swaraj, or self-rule, in India had been growing increasingly strong. By the
1930s, many British politicians believed that India needed to move towards dominion status.
However, there were significant disagreements between the Indian and the British political
parties that the Conferences would not resolve.
Statement 2: Dominion status was officially promised as a Right only by the Cripps Mission
which was much after the Round Table Conference.

Q.438 Consider the following about the Third Round Table Conference.
1. Indian National Congress (INC) did not attend it.
2. White paper published as a result of the conference became the basis of the
Government of India Act 1935.
3. British proposed the idea of a ‘commonwealth’ of colonial nations in this conference.
Select the correct answer using the codes below.
(a) 1 and 2 only
(b) 3 only
(c) 2 only
(d) 1, 2 and 3
Solution: (a)
Justification: Statement 1: The Labour Party from Britain and the Indian National Congress
refused to attend. So, 1 is correct.
Statement 2: The recommendations of conference were published in a White Paper in
March 1933 and debated in Parliament afterwards. A Joint Select Committee was formed to
analyse the recommendations and formulate a new GoI Act 1935 for India. So, 2 is correct.
Statement 3: The Commonwealth dates back to the mid-20th century with the
decolonisation of the British Empire through increased self-governance of its territories. It
was formally constituted by the London Declaration in 1949. So, 3 is wrong.

Q.439 Why was the Communal Award of 1932 severely opposed by the leaders of the
national freedom struggle?
The Award allotted to each minority a number of seats in the legislatures to be elected on the basis
of a separate electorate.
The Award declared the Depressed Classes to be a minority community.
The Award removed reservations for persons with disability in government jobs.
Both (a) and (b)
Solution: (d)
Answer Justification :
Learning: The British policy of ‘Divide and Rule’ found another expression in the announcement of
the Communal Award in 1932.
The Award allotted to each minority a number of seats in the legislatures to be elected on the basis
of a separate electorate that is Muslims would be elected only by Muslims and Sikhs only by Sikhs,
and so on.
Muslims, Sikhs and Christians had already been treated as minorities. The Award declared the
Depressed Classes (Scheduled Castes of today) also to be a minority community entitled to separate
electorate and thus separated them from the rest of the Hindus.
The Congress was opposed to a separate electorate for Muslims, Sikhs and ‘Christians as it
encouraged the communal notion that they formed separate groups or communities having
interests different from the general body of Indians.

Q.440 Under the Gandhi-Irwin Pact 1931, what was agreed upon?
1. Setting up of a responsible government for an independent dominion of India
2. Discontinuation of the civil disobedience movement by the Indian National Congress
Removal of the tax on salt
3. Participation by the Indian National Congress in the Second Round Table Conference
Select the correct answer using the codes below.
(a) 3 and 4 only
(b) 2, 3 and 4 only
(c) 1 and 2 only
(d) 1, 2, 3 and 4
Justification & Learning: Salient features of this act were as following:
The Congress would participate in the Round Table Conference.
The Congress would discontinue the Civil Disobedience Movement.
The Government would withdraw all ordinances issued to curb the Congress.
The Government would withdraw all prosecutions relating to offenses other than violent
one.
The Government would release all persons undergoing sentences of imprisonment for their
activities in the civil disobedience movement.
Manufacture of salt by Indians will be allowed.

Q.441 The Act of 1935 was unanimously rejected by the Congress. Why?
1. It did not provide for convening of a Constituent Assembly for India.
2. There was no provision for universal adult franchise.
Which of the above is/are correct?
(a) 1 only
(b) 2 only
(c) Both 1 and 2
(d) None
Solution: (c)
Justification: The Act provided for the establishment of an All-India Federation to be based
on theunion of the British Indian provinces and Princely States.
The representatives of the States to the federal legislature were to be appointed directly by
the Princes who were to be used to check and counter the nationalists.
The franchise was limited to about one-sixth of the adults.
The Governors, appointed by the British Government, retained special powers. They could
veto legislative and administrative measures
Q.442 Consider the following about the Third Round Table Conference.
1. Indian National Congress (INC) did not attend it.
2. White paper published as a result of the conference became the basis of the
Government of India Act 1935.
3. British proposed the idea of a ‘commonwealth’ of colonial nations in this conference.
Select the correct answer using the codes below.
(a) 1 and 2 only
(b) 3 only
(c) 2 only
(d) 1, 2 and 3
Solution: (a)
Justification: Statement 1: The Labour Party from Britain and the Indian National Congress
refused to attend.
Statement 2: The recommendations of conference were published in a White Paper in
March 1933 and debated in Parliament afterwards. A Joint Select Committee was formed to
analyse the recommendations and formulate a new GoI Act 1935 for India.
Statement 3: The Commonwealth dates back to the mid-20th century with the
decolonisation of the British Empire through increased self-governance of its territories. It
was formally constituted by the London Declaration in 1949.

Q.443 Why was the Communal Award of 1932 severely opposed by the leaders of the
national freedom struggle?
1. The Award allotted to each minority a number of seats in the legislatures to be elected
on the basis of a separate electorate.
2. The Award declared the Depressed Classes to be a minority community.
Which of the above is/are correct?
(a) 1 only
(b) 2 only
(c) Both 1 and 2
(d) None
Solution: (c)
Justification: The British policy of ‘Divide and Rule’ found another expression in the
announcement of the Communal Award in 1932.
The Award allotted to each minority a number of seats in the legislatures to be elected on
the basis of a separate electorate that is Muslims would be elected only by Muslims and
Sikhs only by Sikhs, and so on.
Muslims, Sikhs and Christians had already been treated as minorities. The Award declared
the Depressed Classes (Scheduled Castes of today) also to be a minority community entitled
to separate electorate and thus separated them from the rest of the Hindus.
The Congress was opposed to a separate electorate for Muslims, Sikhs and ‘Christians as it
encouraged the communal notion that they formed separate groups or communities having
interests different from the general body of Indians.

Q.444 The Round Table Conference (1930–32) was conducted based on a


provision under which of these acts?
1. Councils Act, 1892
2. Government of India Act, 1919
3. Councils Act, 1861
4. Defence of India Act, 1905
Solution: (b)
Learning: These were a series of meetings in three sessions called by the British
government to consider the future constitution of India.
The conference resulted from a review of the Government of India Act of 1919, undertaken
in 1927 by the Simon Commission, whose report was published in 1930.
The first session had 73 representatives, from all Indian states and all parties except the
Indian National Congress, which was waging a civil disobedience campaign against the
government.
Its principal achievement was an insistence on parliamentarianism—an acceptance by all,
including the princes, of the federal principle—and on dominion status as the goal
of constitutional development.
The second session was attended by Mahatma Gandhi as the Congress
representative; it failed to reach agreement, either constitutionally or on communal
representation.
The third session was shorter and less important, with neither the Congress nor the British
Labour Party attending.
The result of these deliberations was the Government of India Act, 1935, establishing
provincial autonomy and also a federal system that was never implemented.

Q.445 The Poona Pact 1932 was signed to


(a) Grant separate electorates to depressed classes
(b) Implement the recommendations of the Second Round Table Conference
(c) Take back the Civil Disobedience movement
(d) None of the above
Solution: (d)
Learning: The Pact provided for reservations for depressed classes, not separate
electorates. So,
is wrong.
According to the Communal Award, the depressed classes were considered as a separate
community and as such provisions were made for separate electorates for them.
Mahatma Gandhi protested against the Communal Award and went on a fast unto death in
the Yeravada jail on in 1932.
Finally, an agreement was reached between Dr Ambedkar and Gandhi. This agreement came
to be called as the Poona Pact. The British Government also approved of it. Accordingly, 148
seats in different Provincial Legislatures were reserved for the Depressed Classes in place of
71 as provided in the Communal Award

OTHER EVENTS AND MISC

Q.446 With reference to the All India Forward Bloc (AIFB), consider the following
statements.
1. It emerged as a faction within the Indian National Congress (INC).
2. The party was dissolved post-independence of India.
3. The party leaned ideologically towards Socialism and Marxism.
Select the correct answer using the codes below.
(a) 1 only
(b) 2 and 3 only
(c) 1 and 3 only
(d) 1 and 2 only
Solution: (c)
Justification: Statement 1: A left-wing nationalist political party, the Forward Bloc of the INC
was formed in 1939 by Netaji Subhas Chandra Bose, who had resigned from the presidency
of the Indian National Congress after being outmaneuvered by Mohandas K. Gandhi.
Statement 2: Following Independence and Partition, the party national council met in
Varanasi February 1948. The national council meeting was also preceded by a decision of the
Indian National Congress in the beginning of the year to expel all dissenting tendencies
within the Congress, including the Forward Bloc.
Thus the party decided to renounce any links with the Congress once and for all, and
reconstruct itself as an independent opposition party
Statement 3: The party subscribes to Left-wing nationalism; Socialism; Anti-Imperialism; and
Marxism.
Inside India, local activists of the Forward Bloc continued the anti-British activities without
central
co-ordination.
The party is still active in India. AIFB has branches throughout the country, but the main
strength of the party is concentrated in West Bengal. It is part of the Left Front government
in there, and Forward Bloc has various ministers in the state government.

Q.447 Consider the following chronology and rearrange it suitably.


1. Lahore Congress adopts the demand for Purna Swaraj
2. Second Round Table Conference
3. Ambedkar establishes Depressed Classes Association
4. Civil Disobedience movement relaunched
Select the correct answer using the codes below.
(a) 1324
(b) 1234
(c) 2413
(d) 3142
Solution: (a)
Justification: These are some of the most important events in Indian freedom struggle post-
1918:

Q.448 The formation of All India Forward Bloc coincides with which of the following events?
(a) First World War
(b) Civil Disobedience Movement
(c) Second World War
(d) Quit India Movement
Solution: (c)
Learning: The entry of India into the war was strongly opposed by Subhas Chandra Bose,
who had been elected President of the Congress in 1938 and 1939, but later resigned due to
differences in opinion with Gandhi.
After resignation he formed his own wing separated from the mainstream congress
leadership known as Forward bloc which was the centre of ex-congressmen with socialist
views; however he remained emotionally attached with him for the remainder of his life.
Bose then founded the All India Forward Bloc. In 1940, a year after war broke out, the British
had put Bose under house arrest in Calcutta. However, he escaped and made his way
through Afghanistan to Nazi Germany to seek Hitler and Mussolini's help for raising an army
to fight the British.

Q.449 Which of the following statements about the National Planning Committee
(NPC) of 1938 is CORRECT?
(a) The Congress led Governments in provinces never joined the NPC.
(b) It was appointed by Subash Chandra Bose.
(c) It rejected the path of path of indigenous industrial expansion.
(d) Report prepared by the committee was deliberated by the British government right after
the Cripps Mission.
Solution: (b)
Learning: 1938 was the year that witnessed the first attempt to develop a national plan for
India when National Planning Committee was set up.
This committee was set up by the then Congress President Subhash Chandra Bose and
chaired by Jawaharlal Nehru.
However the reports of the committee could not be prepared and only for the first time in
1948 -49 some papers came out
The Congress Governments removed impediments in the path of indigenous industrial
expansion and, in fact, actively attempted to promote several modern industrial ventures
such as automobile manufacture. So, (a) and (c) cannot be correct.

Q.450 Tripuri Crisis of the Indian National Congress (INC) relates to


(a) Decision of Muslim League not to support Congress ministries formed in provinces
(b) British banning regional organizations of Congress from conducting any business
(c) Contentions related to leadership of the INC
(d) Divide between moderates and extremists on the way ahead post- Civil Disobedience
movement
Solution: (c)
Learning: In 1939, S.C. Bose contended for the presidentship of Congress and he defeated
Pattabhi Sitaramaiya, candidate of Gandhi.
Gandhi declared the defeat of Pattabhi as his own defeat.
Ultimately in April 1939, S.C. Bose resigned and Dr. Rajendra Prasad was made new
President. In May 1939, S.C. Bose founded a new party 'Forward Block' at Makur, Unnao.
In August 1939, S.C. Bose was removed from the post of President of All India Congress
committee and Bengal Congress Committee and was declared unfit for any post in Congress
for the next 3 years

Q.451 Meerut Thesis, 1936 of the Congress Socialist Party (CSP) is related to
(a) Involving princely states in the freedom movement
(b) Joining the Imperial Legislative Councils to convince the British government
(c) Developing leadership of revolutionary socialism within the Congress
(d) A peasant disobedience movement planned from Meerut.
Solution: (c)
Learning: Initially, the task of CSP was interpreted as the displacement of the existing
leadership in Congress, which was declared to be incapable of developing the struggle of the
masses to a higher level.
The CSP was to develop as the nucleus of the alternative socialist leadership of the Congress.
As the Meerut Thesis of the CSP put it in 1935, the task was to ‘wean the anti-imperialist
elements in the Congress away from its present bourgeois leadership and to bring them
under the leadership of revolutionary socialism.
This perspective was, however, soon found to be unrealistic and was abandoned in favour of
a ‘composite’ leadership in wh ich socialists would be taken into the leadership at all levels.

NATIONAL MOVEMENT 1939-INDEPENDENCE


WORLD WAR 2

Q.452 Congress Ministries resigned from the provinces in 1939 because


1. The Government did not allot them seats in the Executive Council.
2. Muslim league was given a disproportionate share of seats in the Provincial Legislature.
Which of the above is/are correct?
(a) 1 only
(b) 2 only
(c) Both 1 and 2
(d) None
Solution: (d)
Justification: In 1937, elections were held on the basis of a restricted franchise as provided
under the GoI Act of 1935.
The Congress won a comprehensive victory. Now eight out of 11 provinces had a Congress
“Prime Minister”, working under the supervision of a British Governor.
In September 1939, two years after the Congress ministries assumed office, the Second
World War broke out. Mahatma Gandhi and Jawaharlal Nehru had both been strongly
critical of Hitler and the Nazis.
Accordingly, they promised Congress support to the war effort if the British, in return,
promised to grant India independence once hostilities ended.
The offer was refused. In protest, the Congress ministries resigned in October 1939.
Through 1940 and 1941, the Congress organised a series of individual satyagrahas to
pressure the rulers to promise freedom once the war had ended.

Q.453 When did the Muslim League pass a resolution committing itself to the
creation of a separate nation called “Pakistan”?
(a) First Non-cooperation movement, 1919
(b) Civil Disobedience Movement, 1930
(c) In 1940, few months after the Congress resigned from the provinces
(d) Direct Action Day, 1946
Solution: (c)
Learning: The Lahore Resolution written by Muhammad Zafarullah Khan and others and
presented by A. K. Fazlul Huq, the Prime Minister of Bengal was a formal political statement
adopted by the All-India Muslim League on the occasion of its three-day general session in
Lahore on 22–24 March 1940.
The resolution called for independent states as seen by the statement:
“That geographically contiguous units are demarcated into regions which should be
constituted, with such territorial readjustments as may be necessary that the areas in which
the Muslims are numerically in a majority as in the North Western and Eastern Zones of
(British) India should be grouped to constitute ‘independent states’ in which the constituent
units should be autonomous and sovereign.”
Although the name "Pakistan" had been proposed by Choudhary Rahmat Ali in his Pakistan
Declaration, it was not until after the resolution that it began to be widely used.

Q.454 Which of the following could NOT be a causative factor behind the Bengal
Famine, 1943?
(a) Spread of communicable and vector-borne diseases such as cholera, malaria and Kala-
Azar
(b) Poor agricultural production and rice crop diseases
(c) Forced Indigo plantations on most of the net sown area of Bengal
(d) Debt bondage and land grabbing of peasants
Solution: (c)
Justification: It was one of the biggest famines to ever take place in British India.
Bengal's economy was predominantly agrarian. For at least a decade before the crisis,
between half
and three quarters of those dependent on agriculture were already at near subsistence
level.
Underlying causes of the famine included inefficient agricultural practices, dense population,
and de-peasantisation through debt bondage and land grabbing.
An estimated 2.1 million people died in the famine, the deaths occurring first from
starvation and then from diseases, which included cholera, malaria, smallpox, dysentery,
and kala-azar.
Other factors, such as malnutrition, population displacement, unsanitary conditions, and
lack of health care, further increased disease fatalities.
Proximate causes comprise localised natural disasters (a cyclone, storm surges and flooding,
and rice crop disease) and at least five consequences of war: initial, general war-time
inflation of both demand-pull and monetary origin; loss of rice imports due to the Japanese
occupation of Burma (modern Myanmar); near-total disruption of Bengal's market supplies
and transport systems by the preemptive etc.

Q.455 Which of the following events were involved in and around the Tripuri
Crisis of the Indian National Congress (INC)?

1. All-India Muslim League forming a separate unit in the Indian National Congress (INC).
2. British banning the Indian National Congress (INC)
3. Contentions related to leadership of the INC
Select the correct answer using the codes below.
(a) 1 and 2 only
(b) 1 and 3 only
(c) 3 only
(d) 2 and 3 only
Solution: (c)
Justification: In 1939, S.C. Bose contended for the presidentship of Congress and he
defeated Pattabhi Sitaramaiya, candidate of Gandhi.
Gandhi declared the defeat of Pattabhi as his own defeat.
Ultimately in April 1939, S.C. Bose resigned and Dr. Rajendra Prasad was made new
President. In May 1939, S.C. Bose founded a new party 'Forward Block' at Makur, Unnao.
In August 1939, S.C. Bose was removed from the post of President of All India Congress
committee and Bengal Congress Committee and was declared unfit for any post in Congress
for the next 3 years.
This was the Tripuri crisis of the INC.

Q.456 Consider the following with reference to events that transpired by the end of 1930s.
1. The congress ministries resigned in several provinces in 1939.
2. The British government dragged India in the Second World War without the consent of
Indians.
3. There was a major split in the Indian National Congress in 1938 for following the policy
of mendicancy.
Select the correct answer using the codes below.
(a) 1 and 3 only
(b) 1 and 2 only
(c) 1, 2 and 3
(d) 2 only
Solution: (b)
Justification: Provincial elections were held in British India in the winter of 1936-37 as
mandated by the Government of India Act 1935. Elections were held in eleven provinces.
The Indian National Congress emerged in power in eight of the provinces. The All-India
Muslim League failed to form the government in any province.
The Congress ministries resigned in October and November 1939, in protest against Viceroy
Lord Linlithgow's action of declaring India to be a belligerent in the Second World War
without consulting the Indian people.

AUGUST OFFER

Q.457 With reference to Individual Satyagraha, consider the following:


1. It was launched with an aim to seek independence and self-governance.
2. Women did not participate in individual satyagraha.
3. Gandhi himself chose the first few individual satyagrahis.
Select the correct answer using the codes below.
(a) 2 only
(b) 1 and 3 only
(c) 3 only
(d) 2 and 3 only
Solution: (c)
Background: The Congress, at its Ramgarh Session in March 1940, resolved to launch some
kind of a Civil Disobedience Movement under the leadership of Gandhiji.
The congress leader authorised him to decide the time and strategy of such a movement.
Gandhiji, who was initially reluctant to start a Civil Disobedience Movement. Finally, he
agreed to launch it.
But still being absorbed with the ideas of difficulties of Britain, he did not want to start a
mass movement. Instead, he opted for limited Satyagraha.
Justification: Statement 1: The issue underlying the Satyagraha was confined to the demand
for carrying on anti-war propaganda and preaching non co-operation with the Government
in their war efforts. It was to affirm the right of free speech and not to seek independence.
Statement 3: Only persons having faith in non-violence and constructive programmes were
eligible for offering Satyagraha and they were selected by Gandhi himself.
Vinoba Bhave was the first to offer Satyagraha by making anti-war speeches in the village
Panaur and he was arrested subsequently. Jawaharlal Nehru was to follow Vinoba Bhave in
November 1940, after giving due notice to the authorities. But he was arrested a week in
advance and was sentenced to four year imprisonment for his speeches delivered early in
October.
Then the third person chosen to offer Satyagraha was Brahmo Dutt, an inmate of Gandhi's
ashram.
One by one all the congress leaders, who offered themselves for the Individual Satyagraha
against the Government, were imprisoned.
Statement 2: There was a huge rush of people conducting Satyagraha in the different parts
of the country.
The women Satyagrahis also responded remarkably for Individual Satyagraha and courted
arrest. One of the first to do so was Sucheta Kirpalani who had been incharge of the
women's department of the All India Congress Committee since 1939.
Bhag Devi (M.L.A), Priyamvada Devi and Mahadevi Kajariwal took the initiative of offering
"Individual Satyagraha" in Punjab, Orissa and Bihar respectively.

Q.458 Consider the following statements about the August Offer of 1940 of the British
government.
1. It abolished the system of communal electorates in India.
2. It suggested the setting up provisional governments in the Princely states.
3. The Congress did not approve the August Offer.
Select the correct answer using the codes below.
(a) 1 and 2 only
(b) 3 only
(c) 1 and 3 only
(d) 2 only
Solution: (b)
Justification: After the Congress ministries (in the Provinces) resigned in 1939, the British
asked again for the support of the Congress in WW-II.
Later in 1940, INC passed a resolution offering the British Government support in war, if a
provisional National Government is setup at Centre.
This was responded by Lord Linlithgow in the sort of a proposal which is called August Offer.
The august Offer turned down the demand of the Congress to set up a national Government
at the centre but proposed the following:
A representative “Constitution Making Body” shall be appointed immediately after the war.
The number of the Indians in the Viceroy’s Executive council will be increased.
A war advisory Council would be set up.
The Congress did not approve the August Offer.

CRIPPS MISSION

Q.459 The Cripps mission was an attempt by the British government to


(a) Extend communal representation in the Imperial legislature
(b) Convince the Congress to join the Central legislature
(c) Secure full Indian cooperation and support for their efforts in World War II
(d) Conclude the Round Table Conferences successfully
Solution: (c)
Learning: Cripps worked to keep India loyal to the British war effort in exchange for a
promise of full self-government after the war. Cripps promised to give dominion status after
the war as well as elections to be held after the war.
Cripps discussed the proposals with the Indian leaders and published them. Both the major
parties, the Congress and the League rejected his proposals and the mission proved a failure.
Cripps had designed the proposals himself, but they were too radical for both Churchill and
the Indians; no middle way was found.
Congress moved towards the Quit India movement whereby it refused to cooperate in the
war effort, while the British imprisoned practically the entire Congress leadership for the
duration of the war.

Q.460 The proposal of the Cripps mission was that


1. India would be a dominion associated with the United kingdom.
2. An elected body charged with the task of making the constitution would be setup
immediately after the WW-II is stopped.
Which of the above is/are correct?
(a) 1 only
(b) 2 only
(c) Both 1 and 2
(d) None
Solution: (c)
Learning: The main proposals of the mission were as follows:
An Indian Union with a dominion status; would be set up; it would be free to decide its
relations with the Commonwealth and free to participate in the United Nations and other
international bodies.
After the end of the war, a constituent assembly would be convened to frame a new
constitution. Members of this assembly would be partly elected by the provincial assemblies
through proportional representation and partly nominated by the princes.
The British Government would accept the new constitution subject to two conditions.
any province not willing to join the Union could have a separate constitution and form a
separate Union, and (ii) the new constitution- making body and the British Government
would negotiate a treaty to effect the transfer of power and to safeguard racial and religious
minorities.
In the meantime, defence of India would remain in British hands and the governor-general’s
powers would remain intact.

Q.461 The proposal of the Cripps mission was that


1. An Indian Union with a dominion status would be set up and it would be free to decide
its relations with the Commonwealth.
2. After the end of the First World War, a constituent assembly would be convened to
frame a new constitution.
3. Any province not willing to join the Indian Union could have a separate constitution and
form a separate Union.
Which of the above is/are correct?
(a) 1 only
(b) 1 and 3 only
(c) 2 and 3 only
(d) 1, 2 and 3
Solution: (b)
Learning: The main proposals of the mission were as follows:
An Indian Union with a dominion status; would be set up; it would be free to decide its
relations with the Commonwealth and free to participate in the United Nations and other
international bodies.
After the end of the Second World war (not first), a constituent assembly would be
convened to frame a new constitution. Members of this assembly would be partly elected by
the provincial assemblies through proportional representation and partly nominated by the
princes.
The British Government would accept the new constitution subject to two conditions.
any province not willing to join the Union could have a separate constitution and form a
separate Union, and (ii) the new constitution- making body and the British Government
would negotiate a treaty to effect the transfer of power and to safeguard racial and religious
minorities.
In the meantime, defence of India would remain in British hands and the governor-
general’s powers would remain intact.

Q.462 Why Indian National Congress (INC) leaders were not satisfied with the
Cripps Mission (1942) proposals?
(a) Cripps Mission did not address the issue of separate electorates.
(b) Cripps Mission did not deliberate any matter with the Indian leaders and proposed
unilateral measures.
(c) Leaders wanted full independence rather than a dominion status.
(d) Cripps Mission agreed for post-war real transfer of power but maintained that the
Viceroy and Secretary of State shall be appointed by the crown.
Solution: (c)
Justification: Cripps was sent to negotiate an agreement with the nationalist leaders,
speaking for the majority Indians, and Muhammad Ali Jinnah, speaking for the minority
Muslim population.
Cripps discussed the proposals with the Indian leaders and published them. So, (b) is wrong.
Cripps worked to keep India loyal to the British war effort in exchange for a promise of full
self- government after the war.
Cripps promised to give dominion status after the war as well as elections to be held after
the war. So, (c) is correct.
Both the major parties, the Congress and the League rejected his proposals and the mission
proved a failure.

Q.463 Gandhiji decided to launch “Individual Satyagraha” due to dissatisfaction with


(a) Government of India Act, 1935 which eroded autonomy of local bodies
(b) Religious intolerance in the countryside
(c) August offer made by the British during the course of the Second World War
(d) State of untouchables in the country
Solution: (c)
Learning: During the course of the Second World War in order to secure the cooperation of
the Indians, the British Government made an announcement on August 1940, which came to
be known as the ‘August Offer’.
The August Offer envisaged that after the War a representative body of Indians would be
set up to frame the new Constitution. Gandhi was not satisfied with is offer and decided to
launch Individual Satyagraha.
Individual Satyagraha was limited, symbolic and non-violent in nature and it was left to
Mahatma Gandhi to choose the Satyagrahis.
Acharya Vinoba Bhave was the first to offer Satyagraha and he was sentenced to three
months imprisonment.
Jawaharlal Nehru was the second Satyagrahi and imprisoned for four months. The individual
Satyagraha continued for nearly 15 months.

QUIT INDIA MOVEMENT

Q.464 Which of the following characterize Dandi march, consider the following statements.
1. March covering 1000 miles
2. Over 1000 volunteers marching together
3. Spanned over 500 hours
Select the correct answer using the codes below.
(a) 1 and 2 only
(b) 3 only
(c) 1 and 3 only
(d) None of the above
Solution: (b)
Justification: The march was over 240 miles, from Gandhiji’s ashram in Sabarmati to the
Gujarati coastal town of Dandi. The volunteers walked for 24 days (576 hours), about 10
miles a day.
Thousands came to hear Mahatma Gandhi wherever he stopped, and he told them what he
meant by swaraj and urged them to peacefully defy the British.
On 6 April he reached Dandi, and ceremonially violated the law, manufacturing salt by
boiling sea water.
Mahatma Gandhi started this march because his demands made to Irwin in a letter were not
fulfilled.

Q.465 Which of the following statements about the Song ‘Amar Sonar Bangla’ is
INCORRECT?
(a) It was written during the Quit India Movement.
(b) It was adopted as the National Anthem of Bangladesh.
(c) It inspired the Liberation Struggle of Bangladesh.
(d) Rabindranath Tagore wrote the song.
Solution: (a)
Learning: As an Ode to Mother Bengal, it was written by Bengali polymath Rabindranath
Tagore in 1905.
The song was written during the Swadeshi movement, when the ruling British Empire had an
undivided India's province of Bengal split into two parts.
The word sonar literally means 'made of gold' or 'beloved', but in the song sonar Bangla may
be interpreted to either express the preciousness of Bengal or refer to the colour of paddy
fields before harvest.
Considering the importance of the song in the liberation struggle of Bangladesh and the
contribution of Rabindranath Tagore, it was made the National Anthem of Bangladesh.

Q.466 The decision to launch the Quit India movement


(a) Was taken by Gandhiji alone
(b) Was ratified by the Indian National Congress (INC)
(c) Wasn’t taken officially as the movement was spontaneous and sporadic in nature
(d) Taken by the forward bloc
Solution: (b)
Learning: In August 1942 at the All-India Congress Committee session in Bombay, Gandhiji
launched the 'Quit India' movement.
The next day, Gandhi, Nehru and many other leaders of the Indian National Congress were
arrested by the British Government.
Disorderly and non-violent demonstrations took place throughout the country in the
following days
By the middle of 1942, Japanese troops were approaching the borders of India. Pressure was
mounting from China, the United States and Britain to solve the issue of the future status of
India before the end of the war

Q.467 “Sankalp Se Sidhi” program is being organized across the country


to commemorate the 75th Anniversary of
(a) Swadeshi movement
(b) Komagata Maru incident
(c) Quit India Movement
(d) Founding of Communist Party of India
Solution: (c)
Learning: On this occasion, the Agriculture & Farmers Welfare Ministry has
planned to organize the Sankalp Se Sidhi program at several Krishi Vigyan Kendras
(KVKs) and ICAR institutes.
The programs comprise of Kisan Film having message of PM to double farmers’
income by year 2022.
The farmers will also be shown various activities in the KVKs and institutes to
acquaint them with the latest technologies.

WAVELL PLAN

Q.468 The Wavell Plan arrived at the Simla Conference 1945 provided for which of the
following?
1. Indianization of the Viceroy’s Executive Council
2. Removing any caste and religion based quota in the Executive Council
3. Partition of India
Select the correct answer using the codes below.
(a) 1 and 2 only
(b) 2 and 3 only
(c) 1 only
(d) 1, 2 and 3
Solution: (c)
Justification: Statement 1: As per the Plan, all the members of the Council, except the
Viceroy and the Commander-in-Chief, would be Indians.
Statement 2: It said, in the Council there would be equal representation of high-caste Hindus
and Muslims. Other minorities including low-caste Hindus, Shudders and Sikhs would be
given representation in the Council.
Statement 3: It proposed for a future constitution of India, not its partition.

Q.469 The Wavell Plan arrived at the Simla Conference 1945 provided for which of the
following?
1. Partition of India
2. Removing any caste and religion based quota in the Executive Council
3. Indianization of the Viceroy’s Executive Council
Select the correct answer using the codes below.
(a) 3 only
(b) 2 and 3 only
(c) 1 and 2 only
(d) 1, 2 and 3
Solution: (c)
Justification: It proposed for a future constitution of India, not its partition.
As per the Plan, all the members of the Council, except the Viceroy and the Commander-in-
Chief, would be Indians.
It said, in the Council there would be equal representation of high-caste Hindus and
Muslims. Other minorities including low-caste Hindus, Shudders and Sikhs would be given
representation in the Council.

OTHER EVENTS

Q.470 The Royal Indian Navy (RIN) mutiny was triggered by the issue of
(a) General Service conditions in the Navy
(b) British occupation of India
(c) Proposals of the Cabinet Mission
(d) Forced participation of India in WW-II
Solution: (a)
Learning: The immediate issues of the revolt were living conditions and food. Issue of INA
trials further fuelled the mutiny.
The mutineers in the armed forces got no support from the national leaders and were
largely leaderless.
The revolt was called off following a meeting between the President of the Naval Central
Strike Committee (NCSC), M. S. Khan, and Vallab Bhai Patel of the Congress, who had been
sent to Bombay to settle the crisis.

Q.471 The Indian National Army (INA) agitation 1945 was against the
1. Killing of large number of Indian soldiers employed by the British in the WW-II
2. Repatriation of Indian currency for funding British army abroad
3. Indian National Army (INA) trials who were captured by the British
Select the correct answer using the codes below.
(a) 1 only
(b) 2 and 3 only
(c) 3 only
(d) 1 and 2 only
Solution: (c)
Justification: After the WW-II, the fate of the members of Netaji Subhash Chandra Bose's
Indian National Army (INA) became a very popular issue.
They were captured by the British in the eastern theatre of the war. An announcement by
the Government, limiting trials of the INA personals to those guilty of brutality or active
complicity, was due to be made by the end of August 1945.
However, before this statement could be issued. Nehru raised the demand for leniency.
The defence of the INA prisoners was taken up by the Congress and Bhulabhai Desai, Tej
Bahadur Sapru, K.N. Katju, Nehru and Asaf Ali appeared in court at the historic Red Fort
trials.
The Congress organized an INA relief and enquiry committee, which provided small sums of
money and food to the men on their release, and attempted to secure employment for
them

CABINET MISSION

Q.472 The recommendations of the Cabinet Mission 1946 included


1. Partition of India
2. Constitution of a Unitary government with centralized powers
3. A nominated Constituent Assembly
4. All the members of the Interim cabinet would be Indians
Select the correct answer using the codes below.
(a) 2 and 4 only
(b) 4 only
(c) 1 and 3 only
(d) 1, 2 and 3 only
Solution: (b)
Justification: The Cabinet Mission 1946 recommended an undivided India and turned
down the Muslim league’s demand for a separate Pakistan.
Important recommendations are listed below.
Recognition of Indian Right to cede from the Commonwealth
There shall be a Union of India which was to be empowered to deal with the defense,
foreign affairs and communications.
Union Government and its legislature were to have limited powers, with greater autonomy
to states
Constituent Assembly to be formed of the representatives of the Provincial Assemblies and
the Princely states.
It provided that all the members of the Interim cabinet would be Indians and there would be
minimum interference by the Viceroy.
All subjects other than the Union Subjects and all the residuary powers would be vested in
the provinces.
The Princely states would retain all subjects and all residuary powers
The representation of the Provincial legislatures was to be break up into 3 sections. Section
A: Madras, UP, Central provinces, Bombay, Bihar & Orissa Section B: Punjab, Sindh, NWFP,
Baluchistan Section C: Assam and Bengal.

TOWARDS INDEPENDENCE AND PARTITION

Q.473 The Radcliffe Line was


(a) the boundary demarcation line between India and Pakistan
(b) the agreed line on the partition of Bengal
(c) the frontier between East Pakistan and West-Pakistan
(d) the line between Indo-China border as historically demarcated by the British
Solution: (a)
Learning: The line was published on 17 August 1947 upon the Partition of India.
It was named after its architect, Sir Cyril Radcliffe, who, as chairman of the Border
Commissions, was charged with equitably dividing 175,000 square miles of territory with 88
million people.
Today its western side still serves as the Indo-Pakistani border and the eastern side serves as
the India-Bangladesh border.

Q.474 The Mountbatten Plan logically culminated in


(a) The August offer extended by Clement Atlee
(b) Indian Independence Act, 1947
(c) Wavell Plan of partition of India
(d) Royal Indian Naval mutiny
Solution: (b)
Learning: Atlee’s Announcement: The Prime Minister of the United Kingdom announced in
February 1947 that:
The British Government would grant full self-government to British India by June 1948 at the
latest,
The future of the Princely States would be decided after the date of final transfer is decided.
The Indian Independence Act was formulated by the government of Attlee and the
Governor General of India Lord Mountbatten, after representatives of the Indian National
Congress, the Muslim League, and the Sikh community came to an agreement with Lord
Mountbatten on what has come to be known as the 3 June Plan or Mountbatten Plan.
This plan was the last plan for independence.

Q.475 Consider the following about the events 1946 onwards in Modern India.
1. A Cabinet Mission was sent that proposed convening of a Constituent Assembly
comprising members elected by the provincial legislatures and nominees of the Indian
states.
2. An interim Government was formed headed by Jawaharlal Nehru.
3. The Muslim League refused to participate in the deliberations of the Constituent
Assembly and pressed for the separate state for Pakistan.
Select the correct answer using the codes below.
(a) 1 only
(b) 2 and 3 only
(c) 1 and 2 only
(d) 1, 2 and 3
Solution: (d)
Justification: At the conclusion of the Second World War, the Labour Party, under Prime
Minister Clement Richard Attlee, came to power in Britain. The Labour Party was
largely sympathetic towards Indian people for freedom.
A Cabinet Mission was sent to India in March 1946, which after
a careful study of the Indian political scenario, proposed the formation of an interim
Government and convening of a Constituent Assembly comprising members elected by the
provincial legislatures and nominees of the Indian states.
An interim Government was formed headed by Jawaharlal Nehru. However, the Muslim
League refused to participate in the deliberations of the Constituent Assembly and pressed
for the separate state for Pakistan.
Lord Mountbatten, the Viceroy of India, presented a plan for the division of India into India
and Pakistan, and the Indian leaders had no choice but to accept the division, as the Muslim
League was adamant.
Q.476 When did the Indian National Congress Working Committee adopt a resolution
for the abolition of landlordism?
(a) 1885 with the constitution of INC
(b) In 1945, based on a resolution by the Congress Working Committee
(c) Karachi Session, 1931
(d) The advent of Non-cooperation movement
Solution: (b)
Learning: In 1945, the Congress Working Committee adopted a resolution for the abolition
of landlordism and the granting of land to the tiller.
This was also the time for the consolidation of the Indian capitalist class within the national
movement.
This was not adopted earlier as INC felt it may generate backlash with the landlord
community and may not augur well for the Indian national movement.

Q.477 The main point(s) outlined in the British Prime Minister Clement Attlee’s statement, 1947
was/were
1. The British would relinquish power only to a Central government and rejected the idea
of autonomous provincial governments.
2. Even after India’s independence, the princely states would continue to remain a part of
British Empire.
3. The British would not transfer power to Indians if the modalities of the framing of
constitution are not arrived at by consensus.
Select the correct answer using the codes below.
(a) 1 only
(b) 2 and 3 only
(c) 3 only
(d) None of the above
Solution: (d)
Justification: Statement 1: He said the British would relinquish power either to some form
of central government or in some areas to the existing provincial governments if the
constituent assembly formed was not fully representative, i.e. if the Muslim majority
provinces did not join.
Statement 2: British powers and obligations with respect to them would lapse but they
would not be transferred to any successor government.
Statement 3: Clement Attlee made the historic announcement regarding the British
Government’s “definite intention to take necessary steps to effect the transfer of power to
responsible Indian hands by a date not later than June 1948.” It was unconditional transfer
of power.

Q.478 What was/were the salient features of the Mountbatten Plan?


1. It provided for the partition of India.
2. It planned that separate electorates provision would be approved in the constituent
assembly.
Select the correct answer using the codes below.
(a) 1 only
(b) 2 only
(c) Both 1 and 2
(d) None
Solution: (a)
Justification: Upon his arrival, Lord Mountbatten immediately began the procedure to
transfer the power.
He put forth the partition plan, known as the Mountbatten Plan. The plan was accepted by
the Congress and Muslim League.
The British Government accorded formal approval to the Mountbatten Plan by enacting the
Indian Independence Act in 1947.
The salient features of the Act were:
The partition of the country into India and Pakistan would come into effect from 15 August
1947.
A Boundary Commission would demarcate the boundaries of the provinces of the Punjab
and Bengal.
The Act provided for the transfer of power to the Constituent Assemblies of the two
Dominions, which will have full authority to frame their respective Constitutions.
No provision was made for separate electorates.

ADMINISTRATIVE CHANGES

Q.479 Consider the following statements.


1. The Arms Act 1878
2. Prevented any Indian subject to keep arms
3. Provided that Europeans and the Anglo-Indians can keep arms only with appropriate
license
4. Mandated that any public gathering of four or more people with arms would be illegal
Which of the above is/are correct?
(a) 1 and 2 only
(b) 2 only
(c) 1 and 3 only
(d) None of the above
Solution: (d)
Justification: Prior to the Indian First War Of Independence in 1857, there were few gun
control laws in India.
But, in the same year as the passing of the Vernacular Press Act, the Arms Act was passed.
This Act prevented the Indians to keep arms without appropriate license. Its violation would
be a criminal offence.
The Europeans and the Anglo- Indians were exempted from the operation of these
legislations.
The Arms Act, 1959 (amended n 21 was enacted to consolidate and amend the law relating
to arms and ammunition in order to curb illegal weapons and violence stemming from them.

Q.480 In the context of Modern India, what is often referred to as the Fifth Report, was actually
a
(a) Manifesto of the Indian National Congress
(b) A report on the East India Company’s administration of India
(c) Compilation of demands of the moderate leaders
(d) A review of charter acts produced by the British Parliament
Solution: (b)
Background & Learning: From the time the Company established its rule in Bengal in
the mid-1760s, its activities were closely watched and debated in England.
There were many groups in Britain who were opposed to the monopoly that the East India
Company had over trade with India and China. These groups wanted a revocation of the
Royal Charter that gave the Company this monopoly.
Company officials were widely publicised in the press. The British Parliament passed a series
of Acts in the late eighteenth century to regulate and control Company rule in India.
It forced the Company to produce regular reports on the administration of India and
appointed committees to enquire into the affairs of the Company.
The Fifth Report was one such report produced by a Select Committee.
It became the basis of intense parliamentary debates on the nature of the East India
Company’s rule in India.

Q.481 Gomasthas’ in pre-independent India referred to


(a) A guild of merchants
(b) Local zamindars
(c) Entrepreneurs that received protection by the British
(d) An Indian agent of the British East India Company
Solution: (d)
Learning: In the 18th century, the East India Company had established itself in India.
Indian cotton and silk fabrics were in great demand worldwide and hence were of special
interest to them
It proceeded to develop a system of management and control that would eliminate
competition, control costs, and ensure regular supplies of cotton and silk goods. Given the
small number of Englishmen, and their unfamiliarity with the local language and society, the
Company turned to local intermediaries, and gave them legal authority to enforce contracts
The Company tried to eliminate the existing traders and brokers connected with the cloth
trade, and establish a more direct control over the weaver. For this purpose they appointed
paid servants called the gomasthas were employed who would obtain goods and from local
weavers and fix their prices
The prices fixed were 15 per cent lower than market price and in extreme cases, even 40 per
cent lower than the market price
They would also supervise weavers, collect supplies, and examine the quality of cloth. They
also prevented Company weavers from dealing with other buyers

Q.482 According to the terms of the ‘Subsidiary Alliance’, Indian rulers were
1. Allowed to have their independent armed forces but had to pay for these “subsidiary
forces”
2. To be protected under the Company’s Suzerainty for which the Company charged a
nominal annual payment from the states
Which of the above is/are correct?
(a) 1 only
(b) 2 only
(c) Both 1 and 2
(d) None
Solution: (d)
Justification: According to the terms of this alliance, Indian rulers were not allowed to
have their independent armed forces. They were to be protected by the Company, but had
to pay for the “subsidiary forces” that the Company was supposed to maintain for the
purpose of this protection. If the Indian rulers failed to make the payment, then part of their
territory was taken away as penalty.
For example, when Richard Wellesley was Governor-General (1798-1805), the Nawab of
Awadh was forced to give over half of his territory to the Company in 1801, as he failed to
pay for the “subsidiary forces”. Hyderabad was also forced to cede territories on similar
grounds.

Q.483 Consider the following with reference to Modern Indian History.


1. The Doctrine of Lapse
2. The Subsidiary Alliance
3. The Treaty of Lahore
4. The Pitt's India Act
The correct sequence of the following events of Indian history (starting with the earliest) is?
(a) 3214
(b) 1234
(c) 2143
(d) 4231
Solution: (d)
Justification: Elimination can easily yield the right answer. There is only option where
Doctrine of lapse (Dalhousie 1848-1856) is shown to have happened after enactment of
Pitt’s India Act 1784.The clear answer is (d).
The doctrine of subsidiary alliance was introduced by Lord Wellesley, British Governor-
General in India from 1798 to 1805.
After the defeat of the Sikhs at the Battle of Sobraon the British were able to march into
Lahore without any further battles. The Treaty of Lahore was signed in 1846 after the First
Sikh War.

CONSTITUTIONAL CHANGES

Q.484 What do you understand by ‘responsible government’ that the British tried
to incorporate into colonial India by several enactments?
1. That the government is now responsible for the social security and welfare of people at
large
2. That the government would follow the Rule of Law
3. That the Bureaucracy will be nominated by the representatives of the people
4. That the Governor was required to act with the advice of ministers responsible to the
provincial legislature.
Select the correct answer using the codes below.
(a) 1, 2 and 4 only
(b) 1 and 4 only
(c) 4 only
(d) 2 and 3 only
Solution: (c)
Justification: Responsible government is a system which embodies the principle of
parliamentary accountability, such as in India.
It is also the foundation of the Westminster system of parliamentary democracy.
Government is responsible to the parliament rather than to the monarch, or, in a colonial
context, to the imperial government.
If the parliament is bicameral, then the government is responsible first to the parliament's
lower house, which is more numerous, directly elected and thus more representative than
the upper house.
Several enactments gradually increased the responsibility of the Ministers and officials to
the legislature and indirectly to the people, for e.g. GoI Act, 1935 created a dual system
where some subjects were to be administered by the Governor based on Ministerial advice.

Q.485 Which of the following acts abolished the office of the secretary of state for India as well
that of Viceroy?
(a) Government of India Act 1935
(b) Independence Act, 1947
(c) Act of 1919
(d) Councils Act of 1892
Solution: (b)
Learning: The Act abolished the office of viceroy and provided, for each dominion, a
governor-
general, who was to be appointed by the British King on the advice of the dominion cabinet.
His Majesty’s Government in Britain was to have no responsibility with respect to the
Government of India or Pakistan.
It also abolished the office of the secretary of state for India and transferred his functions to
the secretary of state for Commonwealth Affairs.

Q.486 East India Company’s monopoly of India trade was abolished by the
(a) Regulating Act of 1773
(b) Charter Act of 1813
(c) Charter Act of 1853
(d) Pitt’s India Act
Solution: (b)
Learning: It was an Act of the Parliament of the United Kingdom which renewed the
charter issued to the British East India Company, and continued the Company's rule in India.
However, the Company's commercial monopoly was ended, except for the tea trade and the
trade with China. Reflecting the growth of British power in India,
The Act expressly asserted the Crown's sovereignty over British India.
It allotted a financial grant to promote education in Indian masses.
This act permitted Christian missionaries to propagate English

Q.487 The origin of the second Chamber or House of the Parliament can be traced to
the
(a) Montague-Chelmsford Report of 1918
(b) Queens Proclamation, 1858
(c) Indian Councils Act, 1861
(d) Government of India Act, 1947
Solution: (a)
Learning: The Government of India Act, 1919 provided for the creation of a ‘Council of
State’ as a second chamber of the then legislature with a restricted franchise which actually
came into existence in 1921.
The Governor-General was the ex-officio President of the then Council of State. The
Government of India Act, 1935, hardly made any changes in its composition.

Q.488 The Charter Act of 1833 was a significant constitutional instrument defining the scope
and authority of the East India Company. With regard to this act,
1. The East India Company ceased to be a commercial agency in India.
2. The act debarred those Indians from entering the public services who had not been
educated in England.
Which of the above is/are correct?
(a) 1 only
(b) 2 only
(c) Both 1 and 2
(d) None
Solution: (a)
Justification: Statement 1: So, it would function as the political agent for the Crown only.
Statement 2: The Act categorically stated ‘that no native of India, nor any natural born
subject of His Majesty, should be disabled from holding any place, office, or employment, by
reason of his religion, place of birth, descent or colour”. It was this enactment which laid the
foundation for the Indianisation of public services.
Learning: The Governor-General of Fort William was hereafter called ‘the Governor- General
of India’. Thus, Bentinck was the first Governor-General of India’.
A Law Member was appointed to the Governor-General’s Council. T. B. Macaulay was the
first Law Member of the Governor-General-in-Council.

Q.489 The Governor-General of India was given exclusive legislative powers for the
entire British India by this Act:
(a) Government of India Act 1935
(b) Act of 1919
(c) Act of 1909
(d) Charter Act of 1833
Solution: (d)
Justification & Learning: This Act was the final step towards centralisation in British India.
It made the Governor-General of Bengal as the Governor-General of India and vested in him
all civil and military powers. In effect, the act created for the first time, a Government of
India having authority over the entire territorial area possessed by the British in India.
It deprived the governor of Bombay and Madras of their legislative powers.
It ended the activities of the East India Company as a commercial body, which became a
purely administrative body. It provided that the company’s territories in India were held by it
‘in trust for His Majesty, His heirs and successors’.

Q.490 The Indian states committee (1927) was appointed to

1. Report on the status and usefulness of ‘dyarchy’ introduced by the Morley Minto Act
(1919)
2. Rejuvenate the Congress organization in various provinces of India
3. Investigate and clarify the relationship between the paramount power and the Princes
4. Delineate the balance of power between the Centre and Provinces
Solution: (c)
Learning: Relations between the Indian Princes and the crown were not well defined
because the extent of sovereignty of the Paramount power was not settled properly.
The Indian states committee in 1927 was appointed to investigate and clarify the
relationship between the paramount power and the Princes.
Sir Harcourt Butler was its chairman and this committee was popularly known as the Butler
Committee.
It gave the following recommendations:
Paramountey must remain supreme.
States should not be handed over to an Indian government in British India, responsible to an
Indian Legislature.
But it could be done with the consent of states.
It must fulfill its obligation, adopting and defining itself according to the shifting necessities
of time and progressive development of states.
The viceroy was made the Crown’s agent in dealing with states.

Q.491 With regards to “Dual Administration” system introduced in Bengal by


Robert Clives, consider the following about it.
1. The Nawab of Bengal was the incharge of civil and police administration.
2. The English got the right of collecting the revenue but they had no responsibility for the
internal administration.
Which of the above is/are correct?
(a) 1 only
(b) 2 only
(c) Both 1 and 2
(d) None
Solution: (c)
Justification: The principal aim of Clive was to make the Nawab a figurehead in the matter
of administration by taking away from him all the powers.
For that purposes he was determined to give a clear meaning with regard to the relation
between the Nawab of Bengal and East India Company and the company with the Emperor
of Delhi.
He successfully persuaded the Nawab of Bengal to practically grant the company the
Nizamat or military power and criminal justice in 1765.
Wtihin six months, the Emperor Shah Alam granted to the company the Diwani or right to
collect revenue from Bengal, Bihar and Orissa. Thus, the company remained in charge of
army and revenue which added to its power and position.
The Nawab of Bengal remained charge of civil and police administration by receiving
annually a sum of rupees 53 lakhs from the company.
Thus, the administration of Bengal was sharply divided between the company and the
Nawab and was, therefore, called the Dual Government.

Q.492 Before the adoption of the portfolio system in the Government of India, all
governmental business was disposed of by the
(a) Governor-General-in Council
(b) Committee of the Lords, Upper House
(c) Court of Directors, East India Company
(d) Council of Ministers
Solution: (a)
Learning: It was the Council functioning as a joint consultative board. As the amount and
complexity of business of the Government increased, the work of the various departments
wasdistributed amongst the members of the Council only the more important cases being
dealt with by the Governor-General or the Council collectively.
This procedure was legalised by the Councils Act of 1861 during the time of Lord Canning,
leading to the introduction of the portfolio system and the inception of the Executive
Council of the Governor-General.
The Secretariat of the Executive Council was headed by the Private Secretary to the Viceroy,
but he did not attend the Council meetings. Lord Willingdon first started the practice of
having his Private Secretary by his side at these meetings.
Later, this practice continued and in November, 1935, the Viceroy's Private Secretary was
given the additional designation of Secretary to the Executive Council.
The constitution of the Interim Government in September 1946 brought a change in the
name, though little in functions, of this Office. The Executive Councils Secretariat was then
designated as Cabinet Secretariat.

Q.493 As per the Indian Independence Act of 1947, the Governor-general of India was to be
(a) Elected by the first Constituent Assembly or Central Legislative Assembly of India
(b) Appointed by the Viceroy on the advice of the executive council
(c) Appointed by the British Monarch on the advice of the dominion cabinet
(d) Nominated by the dominion cabinet based on the recommendation of the Viceroy
Solution: (c)
Learning: The Act abolished the office of viceroy and provided, for each dominion, a
governor- general, who was to be appointed by the British King on the advice of the
dominion cabinet.
His Majesty’s Government in Britain was to have no responsibility with respect to the
Government of India or Pakistan.
It also abolished the office of the secretary of state for India and transferred his functions to
the secretary of state for Commonwealth Affairs

GOVERNOR GENERALS AND VICEROYS

Q.494 He was the longest reigning Viceroy of India and many important events
occurred during his tenure such as
1. Government of India Act 1935 came into force
2. Foundation of Indian National Army
3. August Offer
4. Launch of Quit India Movement
He was?
(a) Lord Linlithgow
(b) Lord Mayo
(c) Lord Elgin
(d) Lord Lytton
Solution: (a)
Learning: He was the Viceroy of India from 1936 to 1944 and this eight years period was
longest reign as Viceroy of India.
During this period, parts of Government of India Act 1935 came into force in 1937.

Q.495 Appointed as the first Governor-General of Bengal, and thus the first
Governor-General of British India, he became the British Resident in the
Bengali capital of Murshidabad. He was
(a) Colin McKenzie
(b) Warren Hastings
(c) Cornwallis
(d) William Bentick
Solution: (b)
Learning: He was the first Governor of the Presidency of Fort William (Bengal), the head of
the Supreme Council of Bengal, and thereby the first de facto Governor-General of India
from 1773 to 1785.
Hastings shared Clive's view that the three major British Presidencies (settlements),
Madras,Bombay and Calcutta, should all be brought under a single rule rather than being
governed separately as they currently were.
The first thing Warren Hastings did was to end the Dual System put forth by his predecessor
Clive. When he abolished the system, he cut down the Nawab of Bengal’s Pension to one-
half.

Q.496 Lord Canning served as Governor General of India from 1856 to 1862, but also
served as the First Viceroy of India during that period. How can this be explained?
(a) He was a former Secretary of State who was allowed to hold dual positions of power in
British India.
(b) The Government of India Act, 1858, was passed in his tenure.
(c) He passed legislations with the help of the Central council to assign the position of
Viceroy to the Governor General of India.
(d) Canning was given the title of Viceroy for the exemplary administrative work done under
his tenure in India.
Solution: (b)
Justification: Lord Canning served as Governor General of India from 1856 to 1862.
During his tenure, the Government of India Act, 1858 was passed which created the office of
Viceroy to be held by the same person who was Governor General of India. Thus, Lord
Canning also served as first Viceroy of India.
After the 1857 Revolt, the responsibility of ruling India was directly assumed by the British
Crown.
The Government of India Act of 1858 and the Queen’s Proclamation in the same year signify
this change in the Indian administration.
The Queen’s Proclamation remained the basis of the British policy in India for more than 60
years.

Q.497 Lord Ripon introduced the (First) Factory Act of 1881 to


1. Improve the service condition of the factory workers in India
2. Categorize organized and unorganized industries in India
3. Push duty free mercantile exports from Small manufacturing units
Select the correct answer using the codes below.
(a) 1 only
(b) 2 and 3 only
(c) 1 and 3 only
(d) 1, 2 and 3
Solution: (a)
Justification: The Act banned the appointment of children below the age of seven in
factories. It reduced the working hours for children.
It made compulsory for all dangerous machines in the factories to be properly fenced to
ensure security to the workers.
Following this act, a Factory Commission was appointed in 1885.
There was another Factories Act in 1891, and a Royal Commission on Labor was appointed in
1892.
The result of these enactments was the limitation on the factory working hours. This was an
answer of the Government to the pathetic conditions of the workers in the factory, wherein,
only when a laborer exhausted, new laborer was to take his / her place.

Q.498 What is notable about the legacy of Lord Ripon?


1. He helped the growth of local bodies like the Municipal Committees in towns and the
local boards in taluks and villages.
2. He was responsible for the rendition of Mysore to its Hindu ruler.
3. He introduced a uniform primary education policy throughout British India.
Select the correct answer using the codes below.
(a) 1 and 2 only
(b) 1 only
(c) 2 and 3 only
(d) 1 and 3 only
Solution: (a)
Justification: He devoted himself to task of liberalising the Indian administration.
He repealed the Vernacular Press Act and earned much popularity among Indians.
Ripon believed that self-government is the highest and noblest principles of politics.
Therefore, he powers of municipalities were increased.
Their chairmen were to be non-officials. They were entrusted the care of local amenities,
sanitation, drainage and water-supply and also primary education. District and taluk boards
were created.
It was insisted that the majority of the members of these boards should be elected non-
officials. The local bodies were given executive powers with financial resources of their own.

Q.499 The Cornwallis Code is a body of legislation enacted in 1793 by the East India
Company to improve the governance of its territories in India. It provided which of
the following?
1. The local administration was placed in the hands of the revenue collectors of districts.
2. The land revenue assessment was fixed permanently with zamindars.
3. East India Company’s service personnel be divided into three branches: revenue, judicial,
and commercial.
4. Private trade was forbidden to any members of the East India Company.
Select the correct answer using the codes below.
(a) 2 and 4 only
(b) 1, 2 and 3 only
(c) 1, 3 and 4 only
(d) 2, 3 and 4 only
Solution: (b)
Justification: Statement 2: The code contained significant provisions - governing, policing
and judicial and civil administration and Its best known provision was the Permanent
Settlement (or the zamindari system enacted in 1793), which established a revenue
collection scheme which lasted until the 20th century.
On these the government of British India virtually rested until the Charter Act of 1833.
This “permanent settlement” provided the British with an Indian landed class interested in
supporting British authority. The local administration was placed in the hands of the revenue
collectors of districts.
Statement 1 and 3: The judiciary was reorganized; there were district judges with magisterial
powers responsible to provincial courts in civil cases and to courts of circuit in criminal cases.
The law administered was Hindu and Muslim personal law and a modified Muslim criminal
code. The higher ranks of the services were restricted to the British, thus depriving the
Indians of any responsible office.
Statement 4: Private trade was forbidden to the members of the first two branches, and
they were instead compensated by a new and generous scale of pay. The land revenue
assessment (the major source of revenue) was fixed permanently with zamindars, or
hereditary revenue collectors.
These native Indians, provided they paid their land taxes punctually, were treated as
landowners, but they were deprived of magisterial and police functions, which were
discharged by a newly organized government police.

Q.500 Wellesley’s forward policy in India was in direct


ideological opposition to
(a) The policy of neutrality adopted by John Shore
(b) The policy of subsidiary alliance adopted by Dalhousie
(c) The policy of annexation adopted by Hastings and others
(d) All of the above
Solution: (a)
Background & Learning: Wellesley came to India with a determination to launch a forward
policy in order to make ‘the British Empire in India’ into ‘the British Empire of India’.
The system that he adopted to achieve his object is known as the ‘Subsidiary Alliance’.
Moreover, the policy of neutrality adopted by Sir John Shore, the successor of Cornwallis,
created a kind of political unrest in India and greatly affected the prestige of the English.
His non-intervention policy contributed much to the growth of anti-British feelings. Further,
Napoleon’s move for an Eastern invasion created a fear among English statesmen.
It was in this light that Wellesley moulded his policy. Preservation of British prestige and
removal of French danger from India were Wellesley’s twin aims.

Q.501 Ripon’s resolution of 1882 was a landmark official statement in British India related to
(a) Women Empowerment
(b) Vernacular education
(c) Local self-government
(d) Judicial reforms
Solution: (c)
Learning: The development of local self government in India owes their progress to the
sincere efforts of Lord Ripon in this direction. His most significant contribution was in the
field of local self- Government.
He passed a resolution in 1881 which clearly stated that the time had come when further
steps could be taken to develop the system of Local self-Government.
The said resolution directed the provincial Government to transfer considerable revenue to
local bodies that were to deal with matters of local importance. The next step in this
direction was taken by him when he passed the famous Resolution of 1882.
It was made quite clear in this resolution that the expansion of the system of local self-
Government was not to improve the efficiency of the administration. It is chiefly desirable as
an instrument of political and popular education.

Q.502 The Dual System introduced by Robert Clive stood abolished when
1. The East India Company decided to act as Diwan and undertake the collection of
revenue by its own
2. agents in Bengal
3. The British decided to legislate both in central and provincial subjects based on
deliberations in the Governor-General’s council
Which of the above is/are correct?
(a) 1 only
(b) 2 only
(c) Both 1 and 2
(d) None
Solution: (a)
Background: The Dual System of Government in Bengal was the brainchild of Lord Clive.
The Diwani (Fiscal) was carried out by the company so Company was Diwan.
The Nizamat (territorial) jurisdiction was carried out by these decrepit Indians so they were
Nizam.
So, this system of separate Diwan and Nizam is called Dual Administration. However, the
real authority was East India Company in the Nizamat also.
The biggest fall out of this system was that the Indian Merchants were reduced to beggars.
On the one side, British kept enjoying the duty free trade; the Indian merchants were to pay
around 40% of the revenue.
The peasants were now under the British revenue collection.
Learning: After the abolition of the Dual System, the responsibility of collecting the revenue
fell on the shoulders of the Company.
For that purpose, a Board of Revenue was established at Calcutta to supervise the collection
of revenue.
English Collectors were appointed in each district. This is the foundation stone for the
present day collectorates.
The treasury was removed from Murshidabad to Calcutta and an Accountant General was
appointed. Calcutta thus became the capital of Bengal in 1772 and shortly after of British
India.

Q.503 What were the main features of the system of Subsidiary Alliance?
1. The ruler entering the alliance must disband his own army.
2. The crown had all the right to interfere in the internal affairs of the protected state.
3. A ruler entering into subsidiary alliance with the British had to maintain a contingent of
British troops in his territory.
4. The protected state under the alliance should cut off its connection with all other
European powers other than the English.
The ruler of the protected state should keep a British Resident at his court.
Select the correct answer using the codes below.
(a) 1 and 2 only
(b) 3, 4 and 5 only
(c) 1, 3, 4 and 5 only
(d) 1, 2, 3, 4 and 5
Solution: (c)
Justification: Statement 1, 3 and 5: Under the alliance, the Indian state was called ‘the
protected state’ and the British were referred to as ‘the paramount power’.
It was the duty of the British to safeguard that state from external aggression and to help its
ruler maintain internal peace.
The protected state should give some money or give part of its territory to the British to
support the subsidiary force.
Statement 2: The paramount power should not interfere in the internal affairs of the
protected state.
But, he should not employ Europeans in his service without the sanction of the paramount
power.
Statement 4: The state was also forbidden to have any political contact even with other
Indian powers without the permission of the British.
Hyderabad was the first state which was brought under Wellesley’s Subsidiary System in
1798.

Q.504 Consider the following about the reforms introduced by Cornwallis


in British India.
1. He initiated the policy of making appointments mainly on the basis of merit thereby
laying the foundation of the Indian Civil Service.
2. He amalgamated the three branches of the Civil service, namely commercial, judicial and
revenue for improved coordination.
3. He introduced a Uniform Civil Code for all British subjects in certain provinces of India.
Select the correct answer using the codes below.
(a) 1 only
(b) 2 and 3 only
(c) 1 and 3 only
(d) 1 and 2 only
Solution: (a)
Justification: Statement 1: It is for this reason that he is called the father of Indian civil
services. Cornwallis realised that in order to consolidate the British rule in India, it was very
important to organise the civil services.
Statement 2: Another major reform that Cornwallis introduced was the separation of the
three branches of service, namely commercial, judicial and revenue.
The collectors, the king-pins of the administrative system were deprived of their judicial
powers and their work became merely the collection of revenue.
Statement 3: In criminal cases, Muslim law was improved and followed.
In civil cases, Hindu and Muslim laws were followed according to the religion of the litigants.
In suits between Hindus and Muslims, the judge was the deciding authority.

Q.505 Which of the following do NOT coincide with Hasting’s policy and view of education in
British India?
(a) He was against spread of vernacular education in India.
(b) He wanted to spread the teaching of English and western science via colleges and
universities in India.
(c) He heavily regulated the freedom of the Press and introduced its censorship to curb
nationalism.
(d) Both (a) and (c)
Solution: (d)
Justification: Option A: Hastings encouraged the foundation of vernacular schools by
missionaries and others.
Option B: In 1817, the Hindu College was established at Calcutta by the public for the
teaching of English and western science. Hastings was the Patron of this college.
Option C: He encouraged the freedom of the Press and abolished the censorship introduced
in 1799. The Bengali Weekly, Samachar Darpan was started in 1818 by Marshman, a
Serampore missionary.
Learning: The Governor-Generalship of Lord Hastings witnessed not only territorial
expansion but also the progress of administration.
He approved the Ryotwari system of land revenue introduced in the Madras Presidency by
Sir Thomas Munroe.
In the sphere of judiciary, the Cornwallis Code was improved. The Police system of Bengal
was extended to other regions.

Q.506 Warren Hastings abolished the system of dastaks. Dastak was


(a) Pre-paid postage system widely used in rural areas
(b) Superlative system of legislation over provincial subjects
(c) Royal decree issued by the Governor-General to the local merchants
(d) A system of free passes for internal trade
Solution: (d)
Learning: Eliminating these free passes, he intended to regulate the internal trade.
He reduced the number of custom houses and enforced a uniform tariff for Indian and non-
Indian goods.
Private trade by the Company’s servants continued but within enforceable limits.
He also introduced a uniform system of pre-paid postage system. A bank was started in
Calcutta.

Q.507 Residents appointed by the East India Company in Indian states were
(a) Representative natives of the Princely states who were also a part of the British
Legislative council
(b) Heads of Contingent army units placed under Subsidiary alliance in the states.
(c) Political and commercial agents of the company in the states.
(d) Ex-Governors of British Presidencies who controlled the states
Solution: (c)
Explanation & Learning: They were political or commercial agents and their job was to serve
and further the interests of the Company.
Through the Residents, the Company officials began interfering in the internal affairs of
Indian states.
They tried to decide who was to be the successor to the throne, and who was to be
appointed in administrative posts.
Sometimes the Company forced the states into a “subsidiary alliance”.

Q.508 Which of the following is associated with Lord Curzon?


1. Official Secrets Act
2. Educational reforms
3. The Sedition Act
Select the correct answer using the codes below.
(a) 1 and 2 only
(b) 2 and 3 only
(c) 1, 2 and 3
(d) 1 and 3 only
Solution: (c)
Justification: The immediate cause for the rise of extremism in India is said to be the
reactionary rule of Lord Curzon.
The Sedition Act and the Official Secrets Act reduced the freedoms of all people.
Curzon took a serious view of the fall in the standard of education and discipline in the
educational institutions.
In his view the universities had degenerated into factories for producing political
revolutionaries. To set the educational system in order, he instituted in 1902, a Universities
Commission to go into the entire question of university education in the country.
On the basis of the findings and recommendations of the Commission, Curzon brought in the
Indian Universities Act of 1904, which brought all the universities in India under the
control of the government.
He passed the Calcutta Corporation Act, (1899) reducing the Indian control of this local
body.

Q.509 Which of the following administrative reforms is/are associated with


Cornwallis, a Governor-General of British India?
1. Making appointments mainly on the basis of merit thereby laying the foundation of the
Indian Civil Service
2. Separation of the commercial, judicial and revenue branch of services
Which of the above is/are correct?
(a) 1 only
(b) 2 only
(c) Both 1 and 2
(d) None
Solution: (c)
Justification: Statement 1: The statements implies the principle of meritocracy vis-a-vis a
"spoils system" that was prevalent earlier.
A strong ICS was to be strictly based on the principles of merit and competence tested via an
examination. Later, Indians were associated too, but the services were called ICS even when
Indians were not allowed to take part in it.
The greatest work of Cornwallis was the purification of the civil service by the employment
of capable and honest public servants.
Cornwallis, who aimed at cleansing the administration, abolished the vicious system of
paying small salaries and allowing enormous perquisites.
He persuaded the Directors of the Company to pay handsome salaries to the Company
servants in order that they might free themselves from commercial and corrupting activities.
Further, Cornwallis inaugurated the policy of making appointments mainly on the basis of
merit thereby laying the foundation of the Indian Civil Service.
Statement 2: A major reform that Cornwallis introduced was the separation of the three
branches of service, namely commercial, judicial and revenue. The collectors, the king-pins
of the administrative system were deprived of their judicial powers and their work became
merely the collection of revenue.
Also, Civil and criminal courts were completely reorganized.

ECONOMIC IMPACT

Q.510 With reference to the Mahalwari system, consider the following statements.
1. Holt McKenzie was associated with the conception of this system.
2. Under the system, ryots paid a variable sum to the Zamindars who then paid to the
British.
3. A field was a basic unit of revenue assessment.
Select the correct answer using the codes below.
(a) 1 only
(b) 2 and 3 only
(c) 1 and 3 only
(d) 1 and 2 only
Solution: (a)
Justification: Holt Mackenzie felt that the village was an important social institution in
north
Indian society and needed to be preserved.
Under his directions, collectors went from village to village, inspecting the land, measuring
the fields, and recording the customs and rights of different groups.
The estimated revenue of each plot within a village was added up to calculate the revenue
that each village (mahal) had to pay.
This demand was to be revised periodically, not permanently fixed. The charge of collecting
the revenue and paying it to the Company was given to the village headman, rather than the
zamindar.
This system came to be known as the mahalwari settlement.

Q.511 Figures associated with Permanent settlement 1793 were


1. Cornwallis
2. Kenneth Mckenzie
3. Thomas Munro
Select the correct answer using the codes below.
(a) 1 only
(b) 2 and 3 only
(c) 1 and 3 only
(d) 1 and 2 only
Solution: (a)
Justification: The Permanent Settlement, also known as the Permanent Settlement of
Bengal was an agreement between the East India Company and Bengali landlords to fix
revenues to be raised from land.
It had far-reaching consequences for both agricultural methods and productivity in the
entire British Empire and the political realities of the Indian countryside.
Permanent Settlement introduced first in Bengal and Bihar; and then to South district of
Madras and district of Varanasi.
The system eventually spread all over northern India by a series of regulations in 1793.
These regulations remained in place until the Charter Act of 1833.

Q.512 Which of the following land arrangements were prevalent after the
application of permanent settlement in Rural Bengal?
1. Zamindars were responsible for paying revenue to the company and distributing the
revenue demand over villages.
2. Each village ryot paid rent to the zamindar.
3. Jotedars gave out loans to ryots and sold their produce.
Select the correct answer using the codes below.
(a) 1 and 2 only
(b) 2 and 3 only
(c) 3 only
(d) 1, 2 and 3
Solution: (d)
Background and Justification: Earlier zamindars in Bengal, Bihar and Odisha had been
functionaries who held the right to collect revenue on behalf of the Mughal emperor and his
representative, the diwan, in Bengal.
The diwan supervised the zamindars to ensure they were neither lax nor overly stringent.
When the East India Company was awarded the diwani or overlordship of Bengal they
changed the system by giving zamindars more power in certain areas and assuming the
ownership of land by themselves.
Statement 3: Jotedars were powerful people who were often rich ryots. Next in hierarchy
were Ryots who cultivated some land and gave out the rest to under-ryots. Under-ryots paid
rent to the ryots.
The image shows the distribution of Power in rural Bengal of that time:
Q.513 The famous book ‘The Economic History of India’ was written by
(a) Romesh Chandra Dutt
(b) Dadabhai Nairoji
(c) Prof. Prakash Malik
(d) James Dowell
Solution: (a)
Learning: Romesh Chandra Dutt was a retired ICS officer and a major economic historian
of India of the nineteenth century. In this work, he has examined in minute detail the entire
economic record of colonial rule since 1757.
His thesis on deindustrialization of India remains forceful argument in Indian historiography.
He also directed attention to the deepening internal differentiation of Indian society
appearing in the abrupt articulation of local economies with the world market, accelerated
urban-rural polarisation, the division between intellectual and manual labour, and the toll of
recurrent devastating famines

Q.514 Which of the following argument(s) were made by Dadabhai Naoroji about the
economic aspects of the British rule in India?
1. Foreign capital should not be preferred for India’s growth.
2. The drain of wealth from India was the basic cause of India’s poverty.
Which of the above is/are correct?
(a) 1 only
(b) 2 only
(c) Both 1 and 2
(d) None
Solution: (c)
Learning: He argued that it was the economic laws and policies of the British that were
draining India and causing crisis in every sphere especially in loss of livelihoods and loss of
precious capital that could have been used for India’s development.
He and his contemporaries argued that using foreign capital for a nation’s growth was
undesirable and should be stopped. Building domestic capacity and funding it by domestic
sources was the most desirable since no drain would be involved.

Q.515 Consider the following about Famine codes and their evolution in India.
1. Khiljis were the first to formulate a ‘famine-code’ to provide relief to famine-affected
people in India.
2. The British codes, developed 1880s onwards, classified food scarcity on a scale of
intensity.
3. The lack of a famine response protocol by the government was noticeably missing from
the British India famines codes.
Select the correct answer using the codes below.
(a) 1 and 2 only
(b) 2 and 3 only
(c) 2 only
(d) 1 and 3 only
Solution: (c)
Justification: Statement 1: Muhammad Bin Tughlaq, of Delhi Sultanate, had for the first
time formulated ‘famine-code’ to provide relief to famine-affected people.
He was the first sultan to advance loans known as sondhar to peasants for digging wells to
extend cultivation.
Statement 2 and 3: The famine of 1876-78 had resulted from the failure of two monsoons. It
covered an area of two lakh fifty thousand square miles and affected fifty eight million
people.
The government’s relief measures seemed to be inadequate. The first Famine Commission
(1878-80) under Sir Richard Strachey was appointed and it made many commendable
recommendations. They include provision of funds for famine relief and construction work in
the annual budget. The Famine Code came into existence in 1883.
Subsequent famine codes were also issued by the British government. They clearly
mentioned steps that governments were required to take to mitigate the risk of famine.

Q.516 Other than Dadabhai Naoroji and R. C. Dutt, who were staunch critics of
Economic colonialism of India?
1. Mahadev Govind Ranade
2. Gopal Krishna Gokhale
3. G. Subramania Iyer
Select the correct answer using the codes below.
(a) 1 and 2 only
(b) 2 and 3 only
(c) 1 and 3 only
(d) 1, 2 and 3
Solution: (d)
Justification: Naoroji, Dutt, Ranade, Gokhale and G. Subramania Iyer were among those
who grounded Indian nationalism firmly on the foundation of anti-imperialism by fashioning
the world’s first economic critique of colonialism, before Hobson and Lenin.
The drain theory was established by Dadabhai Naoroji.
Increased poverty and lower wages were among the indirect products of colonial rule
according to Dutt.
All of them exposed the so called benevolent face of British Raj in India that relied on
the cooperation of Indians to rule India

Q.517 With reference to 18th CE British India, records tell us that ijaradars had little
incentive in making improvements in the land because
(a) The land was auctioned periodically.
(b) Ijaradars had no right to collect land revenue.
(c) Land was assigned to owners based on patronage.
(d) Only barren or remote lands were allotted under the system.
Solution: (a)
Justification: The ijaradari system was introduced by Warren Hastings according to this
the right to collect revenue was given to the highest bidder called contractor for a period of
five years. Once they got the right to collect the taxes, the bankers had to promise to pay a
fixed some of amount to the state.
This system was a failure because bidding was often not related to the actual productivity of
the land.
The ijaradars were not interested in making improvements in land since the land was
auctioned periodically.
The peasants were exploited by the contractors to extract more and more revenue.

Q.518 The Welby Commission was setup by the British Government to


(a) Enquire into the unnecessary expenditures by the British Indian government.
(b) Report on the atrocities under the 1857 Mutiny.
(c) Investigate the charges made against Indian soldiers serving abroad
(d) Bring educational reforms in Indian Universities
Solution: (a)
Learning: Dadabhai Naoroji in his famous book Poverty and UnBritish Rule in India wrote
his Drain Theory.
He showed how India’s wealth was going away to England in the form of: (a)
salaries,(b)savings, (c) pensions, (d) payments to British troops in India and (e) profits of the
British companies.
The British Government was forced to appoint the Welby Commission, with Dadabhai as the
first Indian as its member, to enquire into the Matter.
The Welby Commission's report, published in 1900, showed a number of cases where
excessive or unjust payments had been made by the Indian government.

Q.519 Whitley Commission established in British India was mandated to report on the
(a) Existing conditions of labour and to make recommendations
(b) Police and military reforms needed in the wake of the 1857 mutiny
(c) Financial reforms needed in British administration
(d) Trade and commerce within the princely states
Solution: (a)
Justification: During World War I, in 1917, John Henry Whitley chaired a committee which
produced a report on the ‘Relations of Employers and Employees’ identifying good industrial
relations as a vital need for industrial relations.
Whitely proposed a system of regular formal consultative meetings between workers and
employers to address industrial unrests, which evolved as wage negotiating bodies over
time. Councils were established from 1919.
Another important commission is Hunter Commission which is associated with Jallianwala
Bagh inquiry. Another Hunter commission was an educational reforms commission setup
under Ripon.
The Hunter (Ripon) Commission recommended for the expansion and improvement of the
elementary education of the masses.

Q.520 As per the revenue settlement systems introduced by the British


(a) The Zamindars were made the owners of farm lands and were assigned the
responsibility of the development of agriculture
(b) All land from peasants and zamindars were confiscated by the official treasury
(c) Peasants were made the real land owners of their farm properties and the British their
legal protectorates
(d) British made payment of agricultural rent mandatory
Solution: (d)
Justification: Under the Permanent settlement system, it was thought that if British fix the
land rent, and if productivity increases in future, zamindars will have an incentive to invest
more in agriculture to earn more later. But, this was the philosophy behind only the fixed
revenue settlement system, and not the most appropriate out of the 4 options here. So, (a)
is inappropriate.
Land was only confiscated when payment of rent failed. So, (b) is incorrect.
British were the real land owners, and any other party like Zamindars or Ryots were given
nominal rights over the land. Land could be confiscated by the state as its property. So, (c) is
also wrong

DEVELOPMENT OF PRESS

Q.521 Gandhiji was the editor of three English weeklies, these included
1. Indian Opinion
2. Harijan
3. Heritage India
4. Navjivan
Select the correct answer using the codes below.
(a) 1 and 2 only
(b) 2, 3 and 4 only
(c) 1 and 4 only
(d) 1, 3 and 4 only
Solution: (a)
Justification: These were Indian Opinion (in South Africa during 1903-1915), Young India
(1919- 1931), and Harijan (1933-1942 and 1946-January 1948).
Indian Opinion was bi-lingual (English and Gujarati). For some time it had also Hindi and
Tamil sections.
Young India had a Gujarati edition - Navajivan. Harijan had both Gujarati and Hindi editions.
All these journals which he described as "newspapers" were organs of political and social
movements and discussed with intensity and concentration, problems that demanded
immediate action.

Q.522 The Vernacular Press Act, 1878, empowered a Magistrate to


1. Secure an undertaking from the editor of a vernacular newspaper that nothing would be
published against the English Government
2. Shut down English presses established by Indians
3. Stop government funding to a media house if it does not adhere to the government
policy of expanding the reach of newspapers to rural areas
Select the correct answer using the codes below.
(a) 1 and 2 only
(b) 1 only
(c) 2 and 3 only
(d) 1 and 3 only
Solution: (b)
Justification: This Act empowered a Magistrate to secure an undertaking from the editor,
publisher and printer of a vernacular newspaper that nothing would be published against
the English Government.
The equipment of the press could be seized if the offence was committed. This Act crushed
the freedom of the Indian press.
This was in response to the opposition that had grown with the outset of the Second Anglo-
Afghan War (1878–80).
The act excluded English-language publications.
This created adverse public opinion against the British Government.

Q.523 Powerful newspapers emerged during the years before the Swadeshi
movement. Match the following newspapers and their editors.
1. Swadesamitran: G. Subramaniya Iyer
2. Amrita Bazar Patrika: Motilal Ghosh
3. Sudharak: Surendranath Banerjee
4. Voice of India: N. N. Sen
Select the correct answer using the codes below.
(a) 1 and 2 only
(b) 2, 3 and 4 only
(c) 1 and 4 only
(d) 1, 2, 3 and 4
Solution: (a)
Justification: These were the Hindu and Swadesamitran under the editorship of G.
Subramaniya Iyer, Kesari and Mahratta under B.G. Tilak, Bengalee under Surendranath
Banerjea, Amrita Bazar Patrika under Sisir Kumar Ghosh and Motilal Ghosh, Sudharak under
G.K. Gokhale, Indian Mirror under N.N. Sen, Voice of India under Dadabhai Naoroji.
Others were: Hindustani and Advocate under G.P. Varma and Tribune and Akhbar-i-Am in
Punjab, Indu Prakash, Dnyan Prakash, Kal and Gujarati in Bombay, and Som Prakash,
Banganivasi, and Sadharani in Bengal.
In fact, there hardly existed a major political leader in India who did not possess a
newspaper or was not writing for one in some capacity or the other.

Q.524 Consider the following statements.


1. He started the first Tamil newspaper Swadeshmitram in 1882.
2. He founded ‘The Hindu’ newspaper and was also its editor till 1898.
The above refer to?
(a) G. Subrahmaniya Iyer
(b) Vaidyanatha Iyer
(c) Alluri Sita Rama Raju
(d) Chinnaswami Subramanya Bharathiyar
Solution: (a)
Learning: He was proprietor, editor and managing director of The Hindu Subramania Iyer
actively participated in the Indian Independence movement.
Subramania Iyer campaigned vehemently for reforms in Hindu society. He supported widow
remarriage and desired to abolish untouchability and child marriages.
In 1898, Subramania Iyer relinquished his claims over 'The Hindu' and concentrated his
energies on Swadesamitran, the Tamil language newspaper which he had started in 1882.

Q.525 The Vernacular Press Act in British India was concerned with
(a) Generating local employment with print news
(b) Regulation of material published by indigenous people for its growth and development
(c) Banning all vernacular language publications in India
(d) Controlling the printing and circulation of seditious material in vernacular languages
Solution: (d)
Learning: The Vernacular Press Act was passed in 1878 under the Governor Generalship
and Viceroyalty of Lord Lytton, for better control of Indian language newspapers.
The purpose of the Act was to control the printing and circulation of seditious material,
specifically that which could produce disaffection against the British Government in India in
the minds of the masses. It was passed on the model of irish press laws.
It provided the government extensive rights to censor report and editorial in vernacular
press.
At the time the Vernacular Press Act was passed, there were thirty five vernacular papers in
Bengal, including the Amrita Bazar Patrika

Q.526 The oldest Indian-owned English daily, started during British India, was
(a) The Hindu
(b) Amrit Bazaar Patrika
(c) Swadesh Mitra
(d) Young India
Solution: (b)
Learning: Amrita Bazaar Patrika played a major role in the evolution and growth of Indian
journalism and and nurturing the Indian freedom struggle.
It was launched in Bengali and later continue in English.
It used to be a nationalist newspaper during the British rule, discontinued its publication
from 1986.
It was the first Indian-owned English daily to go into investigative journalism.
The Patrika had many brushes with Lord Curzon, the Viceroy of India at the time of the
Partition of Bengal (1905).
It referred to him as 'Young and a little foppish, and without previous training but invested
with unlimited powers.'
Because of such editorials, the Press Act of 1910 was passed and Motilal Ghosh was also
charged with sedition.
DEVELOPMENT OF EDUCATION

Q.527 Consider the following about the English


Education Act, 1835.
1. It supported the promotion of oriental studies alongside the introduction of English
language instruction mode in higher institutions of learning.
2. It provided that the British Government ought to promote European literature and
science among the natives of India.
Which of the above is/are correct?
(a) 1 only
(b) 2 only
(c) Both 1 and 2
(d) None
Solution: (b)
Justification: It was a legislative Act of the Council of India in 1835 giving effect to a decision
in 1835 by Lord William Bentinck, the then Governor- General of British India, to reallocate
funds the East India Company was required by the British Parliament to spend on education
and literature in India.
It was based on the belief that the British Government ought to be promoting European
literature and science among the natives of India; and that all the funds appropriated for the
purpose of education would be best employed on English education alone.
Formerly, they had supported traditional Muslim and Hindu education and the publication of
literature in the native learned tongues (Sanskrit and Persian); henceforward they were to
support establishments teaching a Western curriculum with English as the language of
instruction.
Together with other measures promoting English as the language of administration and of
the higher law courts (replacing Persian), this led eventually to English becoming one of the
languages of India, rather than simply the native tongue of its foreign rulers.

Q.528 Hunter Commission was appointed in 1882 to


(a) Recommend modifications in the Indian Councils Act 1861 for more responsible
governance of India
(b) Review the working of the educational system in India
(c) Consolidate sunset and sunrise laws related to the municif and panchayat in India
(d) Assess the secular credentials of the British Government
Solution: (b)
Background & Learning: Like Lord William Bentinck, Lord Ripon was a champion of
education of the Indians. Ripon wanted to review the working of the educational system on
the basis of the recommendations of the Wood’s Despatch.
For further improvement of the system Ripon appointed a Commission in 1882 under the
chairmanship of Sir William Hunter. The Commission came to be known as the Hunter
Commission.
The Commission recommended for the expansion and improvement of the elementary
education of the masses.
The Commission suggested two channels for the secondary education-one was literary
education leading up to the Entrance Examination of the university and the other preparing
the students for a vocational career.
The Commission noted the poor status of women education. It encouraged the local bodies
in the villages and towns to manage the elementary education.
This had resulted in the extraordinary rise in the number of educational institutions in India.

Q.529 Curzon brought in the Indian Universities Act which


(a) Removed social sciences from the curriculum of private universities to curb nationalism
(b) Did away with the territorial jurisdiction of the Universities
(c) Brought all the universities in India under the control of the government
(d) Nationalized all higher educational institutions in India
Solution: (c)
Background & Justification: Curzon took a serious view of the fall in the standard of
education and discipline in the educational institutions.
In his view the universities had degenerated into factories for producing political
revolutionaries.
To set the educational system in order, he instituted in 1902, a Universities Commission to
go into the entire question of university education in the country.
On the basis of the findings and recommendations of the Commission, Curzon brought in the
Indian Universities Act of 1904, which brought all the universities in India under the
control of the government.
Option A: This, however, did not mean that social sciences stood removed, but that now the
government kept a close vigil of these educational institutions.
Learning: This act allowed the Government to appoint a majority of the fellows in a
university.
The Governor General was now empowered to decide a University’s territorial limits and
also affiliation between the universities and colleges.
Universities were empowered to appoint their own staff including the teaching staff
However, for education and research a grant per year for 5 years was also accepted.
This was the starting of university grants in India that later became a permanent feature in
the structure of India education.

Q.530 The famous committee headed by Lord Macaulay to make recommendations for
the promotion of education in India emphasized
1. The promotion of oriental knowledge over occidental
2. Pushing English as the official and literary language of India
Which of the above is/are correct?
(a) 1 only
(b) 2 only
(c) Both 1 and 2
(d) None
Solution: (b)
Justification: Statement 1: Oriental means belonging to the east and occidental means
belonging to the west. The committee recommended the opposite.
The introduction of English Education was a significant event of Lord William Bentinck’s
administration.
He appointed a committee headed by Lord Macaulay to make recommendations for the
promotion of education.
In his report, Macaulay emphasized the promotion of European literature and science
through English medium to the people of India.
This recommendation was wholeheartedly accepted by William Bentinck. The Government
Resolution in 1835 made English the official and literary language of India.
In the same year, William Bentinck laid foundation of the Calcutta Medical College.

Q.531 According to Mahatma Gandhi, highest development of the mind and the soul
is possible under which system of education?
(a) Where individuals are left exposed to nature after achieving basic literacy
(b) Technical and scientific education system
(c) Religious education that imparts an understanding of our cultural heritage
(d) Basic education where vocational training or work experience is given importance
Solution: (d)
Justification: For Gandhiji, literacy was neither the beginning nor the end of education.
For him, education was of the mind and soul. It ought not be restricted to knowing how to
read and write, and receiving a degree.
He further says, “I would therefore begin the child’s education by teaching it a useful
handicraft and enabling it to produce from the moment it begins its training … I hold that the
highest development of the mind and the soul is possible under such a system of education.”
“Only every handicraft has to be taught not merely mechanically as is done today but
scientifically, i.e. the child should know the why and the wherefore of every process”.

Q.532 What were the contributions of Christian Missionaries in the development of


literature in India?
1. Publishing dictionaries in local language
2. Publishing Sanskrit grammar in English language for dissemination to the elites
3. Starting the trend of lithographic printing press
Select the correct answer using the codes below.
(a) 1 and 3 only
(b) 2 only
(c) 2 and 3 only
(d) 1, 2 and 3
Solution: (a)
Justification: Statement 1: The British Government had three roles in India, first that of a
trader, second that of ruler and then that of a Christian propagandist. British rulers held and
professed Christianity.
Consequently, British rule was equated with Christian domination.
Statement 2: They instead published English grammar in local language so that people could
understand it.
Statement 3: They also published helping books for clergymen and helped in the
establishment of schools and colleges

ADMINISTRATIVE POLICY

Q.533 The British Residents in the courts of various princely states were
1. Political and commercial agents of the East India Company
2. Commanders of the troops of the forces instituted by the Subsidiary alliance
3. Members of the British Cabinet who visited princely courts
Select the correct answer using the codes below.
(a) 3 only
(b) 1 only
(c) 2 and 3 only
(d) 1 and 2 only
Solution: (b)
Justification: Their job was to serve and further the interests of the Company.
Through the Residents, the Company officials began interfering in the internal affairs of
Indian states.
They tried to decide who was to be the successor to the throne, and who was to be
appointed in administrative posts.
Sometimes the Company forced the states into a “subsidiary alliance”.
The policy of placing residents and this interference was in contravention with the policy of
non- intervention of the British crown as it was agreed post-1858 Act.

Q.534 The policy of “paramountcy” primarily aimed at


(a) Stopping all the wars that the East India Company was engaged in with the native
princely states
(b) Suspending all subsidiary alliances with the princely courts
(c) Subordinating all European companies trading in India
(d) Making all Indian states accept the suzerainty of the British
Solution: (d)
Learning: Policy of paramountcy was designed to make all Indian states accept the
suzerainty of the British.
The Company claimed that its authority was paramount or supreme; hence its power was
greater than that of Indian states.
In order to protect its interests it was justified in annexing or threatening to annex any
Indian kingdom.
This policy was followed by policies of subsidiary alliances; keeping British residents in courts
etc.

THE INDIAN STATES


CIVIL REBELLION

Q.535 The Punnapra-Vayalar uprising was


1. A communist uprising
2. Organized in Travancore
3. Against the rich landlords
Select the correct answer using the codes below.
(a) 1 only
(b) 1 and 3 only
(c) 1 and 2 only
(d) 2 and 3 only
Solution: (c)
Justification: It developed as a reaction to the constitutional scheme proposed by the
Dewan, C.P.Ramaswamy Iyer, early in January, 1946.
The scheme provided for adult franchise, but retained the dewanship as an irremovable
excertive. The State Congress rejected the scheme.
The Communists decided to launch a violent struggle to bring an end to the oppressive rule
of the Dewan. It was an uprising against the Prime Minister and the state.

Caste movements

Q.536 Consider the following statements.


1. Nirankari Movement insisted on the worship of the formless God.
2. Namdhari Movement followers started wearing black clothes and performed tantric
practices.
3. Akalis started a movement to remove the corrupt Mahants from the Sikh gurudwaras,
something that turned into a political association later.
Which of the above is/are correct?
(a) 1 only
(b) 2 and 3 only
(c) 1 and 3 only
(d) 1, 2 and 3
Solution: (c)
Justification: Punjab came under the spell of many social reforms.
Baba Dayal Das founded the Nirankari Movement. He insisted the worship of God as
nirankar (formless).
The Namdhari Movement was founded by Baba Ram Singh. His followers wore white clothes
and gave up meat eating.
The Singh Sabhas started in Lahore and Amritsar in 1870 were aimed at reforming the Sikh
society. They helped to set up the Khalsa College at Amritsar in 1892.They also encouraged
Gurmukhi and Punjabi literature.
In 1920, the Akalis started a movement to remove the corrupt Mahants (priests) from the
Sikh gurudwaras. The British government was forced to make laws on this matter. Later, the
Akalis organised themselves into a political party.

Q.537 With reference to the Mahar movement, which was an important backward
class movement in India, consider the following ideological strands.

1. It followed the dictum of no religion, no God and no caste.


2. It relied on total rejection of Hinduism.
Which of the above is/are correct?
(a) 1 only
(b) 2 only
(c) Both 1 and 2
(d) None
Solution: (b)
Justification & Learning: When Ambedkar lead the Mahar movement, he rejected the Gandhian
approach relying on change of the heart of caste Hindu and of incorporating the Harijan in the
Sudra Varna; he decided to abandon Hinduism altogether and along with five lakhs Mahars
embraced Buddhism.
There was another movement - Shri Narayana Dharma Paripalana Yogam Movement - SNDP –
that had a different approach.
Narayana Guru (founder of SNDP) built a number of temples, which were declared open to all
castes.
He also simplified rituals regarding marriage, religious worship, and funerals. Narayana Guru
achieved a notable success in transforming the untouchable groups into a backward class.
He criticized Gandhi for his faith in Chaturvarna, which he considered the parent of the caste
system and untouchability. He gave a new slogan “one religion, one caste and one God for
mankind”. On the other hand, Mahars relied on the Buddhist social and ideological doctrines,
and thus statement 2 is false.

TRIBAL REVOLT
PEASANT MOVEMENTS

Q.538 With reference to the Satara Movement, 1943, consider the following
statements.
1. It was a Brahmin movement that sought to revive the caste system.
2. Dominated by peasants, the movement opposed landlordism.
3. In Maharashtra, it set up a parallel government with volunteer corps.
4. Due to its constructive work, it was supported at the national scale by Congress in the
later stages of the movement.
Select the correct answer using the codes below.
(a) 2, 3 and 4 only
(b) 2 and 3 only
(c) 1, 2 and 4 only
(d) 1 and 4 only
Solution: (b)
Justification: From the late nineteenth century, a non-Brahman movement, which opposed
the caste system and landlordism, had developed in Maharashtra.
This movement established links with the national movement by the 1930s.
In 1943, some of the younger leaders in the Satara district of Maharashtra set up a parallel
government ( prati sarkar), with volunteer corps (seba dals) and village units (tufan dals).
They ran people’s courts and organised constructive work. Dominated by kunbi peasants
and supported by dalits, the Satara prati sarkar functioned till the elections of 1946, despite
government repression and, in the later stages, Congress disapproval.

Q.539 Plantation workers of Assam opposed the Inland Emigration Act of 1859 because
1. It took away their right to free movement
2. It introduced 'free' emigration and unlicensed recruitment.
Which of the above is/are correct?
(a) 1 only
(b) 2 only
(c) Both 1 and 2
(d) None
Solution: (a)
Justification: Statement 1: Plantation workers were not permitted to leave the tea
gardens without permission and in fact they were rarely given such permission. When they
heard of the Non- Cooperation Movement, thousands of workers defied the authorities, left
the plantations and headed home.
They believed that Gandhi Raj was coming and everyone would be given land in their own
villages. Hence, in opposition to such acts, many participated in the movement.
Statement 2: This was through a later inland migration act.
The Inland Emigration Act of 1882, which introduced 'free' emigration and unlicensed
recruitment, also strengthened the penal contract system. Contemporary accounts testify to
the ruthlessness with which such power was exercised by the planters
All this reduced indentured wage labour employed on Assam plantations to forced labour or
unfree labour.

Q.540 With reference to Rampa Rebellion of 1922, consider the following statements.
1. It was led by Alluri Sitarama Raju.
2. It took the form of guerilla warfare ending with the British acceding forest rights to the
tribals.
3. It was the first organized tribal rebellion.
Select the correct answer using the codes below.
(a) 1 and 2 only
(b) 1 only
(c) 2 and 3 only
(d) 1 and 3 only
Solution: (b)
Justification: Statement 1: The Rampa Rebellion of 1922 was a tribal uprising, led by
Alluri Sitarama Raju in Godavari Agency of Madras Presidency, British India. It began in
August of 1922 and lasted until the capture and killing of Raju in May 1924.
The Rampa administrative area, situated in the hills of what are now the Godavari districts of
Andhra Pradesh,
Statement 2: It broke out in August 1922 and took the form of guerilla warfare, ending in
May 1924 with the capture and shooting of Raju
Statement 3: The tribal people had long felt that the legal system favoured zamindars
(estate landowners) and merchants, which had also resulted in the earlier Rampa Rebellion
of 1879. There were other rebellions as well, such as Santhal rebellion etc.
Learning: Tribals had traditionally been able to support their food requirements through
the use, in particular, of the podu system whereby each year some areas of jungle forest
were burned to clear land for cultivation.
The British Raj authorities had wanted to improve the economic usefulness of lands in
Godavari Agency, an area that was noted for the prevalence of malaria and blackwater
fever.
The traditional cultivation methods were greatly hindered when the authorities took control
of the forests, mostly for commercial purposes such as produce for building railways and
ships, without any regard for the needs of the tribal people.
The conflict later catapulted in the Rampa rebellion of 1922.

Q.541 Deccan Riots of 1875 had its roots in


(a) Forest enclosures and new forest laws by the British
(b) Opposition to the Indian Councils Act 1861
(c) Laying down of railways tracks
(d) Agrarian distress
Solution: (d)
Background & Learning: In 1875, peasants of Maharashtra in some parts of Pune,
Satara and Nagar districts revolted against increasing agrarian distress.
The Deccan Riots of 1875 targeted conditions of debt peonage (kamiuti) to moneylenders.
The rioters' specific purpose was to obtain and destroy the bonds, decrees, and other
documents in the possession of the moneylenders
he commercialization of agriculture under British land revenue policies burdened small
peasants by placing a premium on access to credit to finance productive investments in the
land.
Employing capital advanced by European merchants, local moneylenders obtained unlimited
title to the property and labor of their debtors; it gave them the "power to utterly ruin and
enslave the debtor.
During the 19th century, they used this power to control peasant labour, and not their land,
which was of little value without people to work it.
These changes in agriculture undermined the communal traditions which had been the basis
of Indian village life.
Aftermath: When the revolt spread in the Deccan, the Government of Bombay was
initially unwilling to see it as anything serious. But the Government of India, worried by the
memory of 1857, pressurised the Government of Bombay to set up a commission of enquiry
to investigate into the causes of the riots.
The commission produced a report that was presented to the British Parliament in 1878.

Q.542 Which of the following was/were NOT a causative factor behind the “Indigo or
Blue rebellion” in 1859?
1. The peasants were compelled to plant Indigo rather than the food Crops.
2. The peasants were provided loans called “dadon” for indigo planting which was at a very
high interest rate.
3. Acquisition of farm land by the British for their industrial projects
4. Restrictions on international trade of indigo
Select the correct answer using the codes below.
(a) 2 and 4 only
(b) 1 and 3 only
(c) 3 and 4 only
(d) 1 and 4 only
Solution: (c)
Justification: Indigo planting started in Bengal as early as 1777.
When the British Power expanded, the Indigo planting was emphasized because of a high
demand of the Blue Dye in Europe. Apart from the reasons mentioned above, others are:
Indigo farmers received very low returns for their crops.
The land under Indigo degraded the land for cultivation of any further crop.
The contract conditions under which Indigo planters kept the cultivators were harsh.
The loan made the people indebted and resulted in a rebellion.

Q.543 Bardoli Satyagraha (1928) was launched because


(a) The government refused to revise taxes despite floods and famine
(b) The big landlords of Bardoli had supported the British to usurp all farm land belonging to
the ryots
(c) The British extended irrigation facilities to all nearby villages except Bardoli without any
rational basis
(d) The government confiscated all crop produce of a farm land
Solution: (a)
Learning: In 1925, the taluka of Bardoli in Gujarat suffered from floods and famine, causing
crop production to suffer and leaving farmers facing great financial troubles.
However, the government of the Bombay Presidency had raised the tax rate by 30% that
year, and despite petitions from civic groups, refused to cancel the rise in the face of the
calamities.
In 1928, an agreement was finally brokered by a Parsi member of the Bombay government.
The Government agreed to restore the confiscated lands and properties, as well as cancel
revenue payment not only for the year, but cancel the 30% raise until after the succeeding
year.
The movement was eventually led by Vallabhbhai Patel, and its success gave rise to Patel
becoming one of the main leaders of the independence movement.

Q.544 The Bakasht land movement continued till 1945 till zamindari was abolished.
What were Bakasht lands?
(a) Lands confiscated by the government
(b) Lands claimed by the landlords from tenants who have held them for innumerable years
(c) Lands tilled by the small farmers but owned by the Zamindars on behalf of the
government
(d) Lands on which Zamindars received no rent
Solution: (b)
Learning: The Bakasht lands issue became a major ground of contention between the Bihar
Provincial Kisan Sabha and the Congress Ministry.
The Congress Ministry had initiated legislation for the reduction of rent and the restoration
of Bakasht lands.
Bakasht lands were those which the occupancy tenants had lost to zamindars, mostly during
theDepression years, by virtue of nonpayment of rent, and which they often continued to
cultivate as share-croppers.
But the formula that was finally incorporated in the legislation on the basis of an agreement
with the zamindars did not satisfy the radical leaders of the kisan Sabha.
The legislation gave a certain proportion of the lands back to the tenants on condition that
they pay half the auction price of the land. Besides, certain categories of land had been
exempted from the operation of the law.

Q.545 The initiative to organise peasants into Kisan Sabhas was taken by
(a) Moderates led by Ferozshah Mehta
(b) Extremists who separated from INC after Surat split 1907
(c) Members of Home Rule League
(d) A.O. Hume and associated founders of the INC
Solution: (a)
Learning: The Kisan Sabha Movement which led to the formation of All India Kisan Sabha
(AIKS) at the Lucknow session of the Indian National Congress had its roots in the
organization of peasants in Kisan Sabhas.
The initiative to organise peasants into Kisan Sabhas was taken by the active members of
Home Rule League in UP - Gauri Shankar Misra, Indra Narain Dwivedi supported by Madan
Mohan Malviya.
The UP Kisan Sabha, set up in 1918, had established 450 branches in 173 tehsils of the
province by mid-1919.
In the mid-1920, Baba Ramchandra emerged as the leader of peasants in Avadh and led a
few hundred tenants from Jaunpur and Pratapgarh districts to Allahabad and apprised
Jawaharlal Nehru of the conditions of the peasants.
Nehru made several visits to the rural areas and developed close contacts with the Kisan
Sabha Movement.

Q.546 Pabna agrarian leagues was established in East Bengal in 1873 to


(a) Resist against Zamindar’s efforts to charge high rents and evict tenants
(b) Reacquire the agricultural lands lawfully possessed by the local British residents
(c) Remove encroachments on forests, rivers and land by dikus
(d) Peacefully appeal to the government for abolishing farmer income tax
Solution: (d)
Learning: In East Bengal the peasantry was oppressed by zamindars through frequent
recourse to ejection, harassment, arbitrary enhancement of rent through ceases (abwabs)
and use of force.
The zamindars also tried to prevent them from acquiring the occupancy rights under the Act
of 1859.
In May 1873 an Agrarian League was formed in the Yusufzahi Pargana of Pabna district (East
Bengal).
Payments of enhanced rents were refused and the peasants fought the zamindars in the
courts. Similar leagues were formed in the adjoining districts of Bengal.
The main leaders of the Agrarian League were Ishan Chandra Roy, Shambu Pal and Khoodi
Mullah. The discontent continued till 1885 when the Government by the Bengal Tenancy Act
of 1885 enhanced the occupancy rights.

Q.547 The British government appointed the Deccan Riots Commission to


(a) Recovering land encroachment that have resulted due to public uprisings
(b) Act as an appellate court connected to 1857 mutiny
(c) Investigate into the causes of the Deccan uprising
(d) Examine law and order expenditure in the Deccan region
Solution: (c)
Justification & Learning: The Deccan peasants uprising 1875 was directed mainly against the
excesses of the Marwari and Gujarati money lenders.
Social boycott of moneylenders by the peasants was later transformed into armed peasant
revolt in the Poona and Alimadnagar districts of Maharashtra.
By June 1875 nearly a thousand peasants were arrested and the uprising completely
suppressed.
The Government appointed the Deccan Riots Commission to investigate into the causes of
the uprising.
The ameliorative measure passed was the Agriculturists Relief Act of 1879 which put
restrictions on the operations of the peasants land and prohibited imprisonment of the
peasants of the Deccan for failure to repay debts to the moneylenders.

Q.548 Eka Movement surfaced in Hardoi, Bahraich and Sitapur during the end of 1921.
The main reason behind the movement was
(a) High land rents
(b) Religious persecution
(c) Forced recruitment in British Indian army
(d) Untouchability
Solution: (a)
Learning: It was a peasant movement.
The main reason of movement was higher rent, which was generally higher than 50% of
recorded rent in some areas. Oppression of thekedhars who were entrusted to collect rent
and practice of share rent also contributed to this movement.
Small zamindars who were disenchanted with British Government due to heavy land
revenue demand were also a part of this movement.
Soon the leadership of Movement changed from Congress to Madari Pasi, a low caste leader
who was not inclined to accept non-violence. This led the movement losing contact with
nationalist class.
By 1922, due to severe repression of British the Eka Movement came to an end.

Q.549 The Indigo rebellion in 1859 was caused due to


(a) Indigo cultivators not receiving any loan and support from British planters
(b) Acquisition of cultivable Indigo land by British industrial projects
(c) Harsh agri-economic conditions under which Indigo cultivators were put by the British
planters
(d) Collapse of international prices of Indigo which resulted in large scale farmer suicides
Solution: (c)
Explanation & Learning: Indigo planting started in Bengal as early as 1777. It was first
planted by one British Louis Bonard. When the British Power expanded, the Indigo planting
was emphasized because of a high demand of the Blue Dye in Europe.
The peasants were compelled to plant Indigo rather than the food Crops.
The peasants were provided loans called “dadon” for indigo planting which was at a very
high interest rate.
Indigo farmers received very low returns for their crops.
The land under Indigo degraded the land for cultivation of any further crop.
The contract conditions under which Indigo planters kept the cultivators were harsh.
The loan made the people indebted and resulted in a rebellion.

WORKING CLASS MOVEMENT


Q.550 Co-founded by Lala Lajpat Rai, it is the oldest trade union federations in India. It is
(a) All India Trade Union Congress (AITUC)
(b) Bharatiya Mazdoor Sangh (BMS)
(c) The Centre of Indian Trade Unions (CITU)
(d) Hind Mazdoor Sangh (H.M.S.)
Solution: (a)
Learning: It was founded in 1920 in Bombay by
Lala Lajpat Rai, Joseph Baptista, N. M. Joshi, Diwan Chaman Lall and a few others and,
until 1945 when unions became organised on party lines, it was the primary trade union
organisation in India.
Since then, it has been associated with the Communist Party of India. AITUC is a founder
member of the World Federation of Trade Unions

PERSONALITIES

Q.551 She is widely remembered for hoisting the Indian National Congress flag at
the Gowalia Tank maidan in Bombay during the Quit India Movement, 1942 and
participated in public processions during the Salt Satyagraha. She is
(a) Begum Hazrat Mahal
(b) Annie Besant
(c) Kasturba Gandhi
(d) Aruna Asaf Ali
Solution: (d)
Learning: She became an active member of Congress Party after marrying Asaf Ali and
participated in public processions during the Salt Satyagraha.
In 1954, she helped form the National Federation of Indian Women, the women's wing of
CPI but left the party in 1956 following Nikita Khrushchev's disowning of Stalin.
She received India's highest civilian award, the Bharat Ratna, posthumously in 1997. She was
a member of the Congress Socialist Party

Q.552 Consider the following about the idea of Total Revolution as given by
Jayprakash narayan.
1. It demanded annihilation of British rule from India.
2. It desired social transformation on the lines of the ideals of the Sarvodaya.
Which of the above is/are correct?
(a) 1 only
(b) 2 only
(c) Both 1 and 2
(d) None
Solution: (b)
Justification: On his return home to Patna in 1975 after release from detention due to
acute illness, JP wrote a long letter to his followers explaining the circumstances of the Bihar
movement and committing the remaining years of his life to total revolution.
As per him, the objective of this movement was not merely to change the Government, but
also to change the society and the individual.
He laid out a plan for the movement; on how the revolution should move on the
propaganda for public education, and constructive programmes consisting of creating
consciousness about various social evils such as dowry system, caste conflicts, communalism
and untouchability.
Thus, JP had a very clear perspective of the social ideals and objectives when he took upon
the task of guiding the Indian masses for a concrete action.
JP strongly believed that for democracy to be a lively and effective instrument there is an
urgent need for a strong opposition, powerful public opinion, free and fearless press,
ideological and moral pressure from the academicians, and trade unions. He, in fact, advised
the people to revise their thinking and attitude towards democratic functioning in India.

Q.553 Consider the following statements about Madam Bhikaji Cama.


1. She co-founded the Paris Indian Society.
2. She has served as the president of Indian National Congress (INC).
3. She unfurled the National Flag at the International Socialist Conference in Stuttgart in
1907.
4. She founded the Indian Home Rule Society.
Select the correct answer using the codes below.
(a) 1 and 3 only
(b) 2, 3 and 4 only
(c) 3 only
(d) 1, 2 and 4 only
Solution: (a)
Justification: Statement 1: She co-founded the Paris Indian Society together with Singh
Rewabhai Rana and Munchershah Burjorji Godre.
Statement 2 and 4: She served as the private secretary of Dadabhai Naoroji, the president of
the British Committee of the Indian National Congress. Together with Naoroji and Singh
Rewabhai Rana, Cama supported the founding of Varma's Indian Home Rule Society in 1905.
Statement 3: In her appeal for human rights, equality and for autonomy from Great Britain,
she unfurled what she called the "Flag of Indian Independence

Q.554 With reference to British India, consider the following statements. Francis Buchanan
1. was a physician who visited India
2. was a surgeon to one of the Governor-General of India
3. undertook detailed surveys of the areas under the jurisdiction of the East India Company
Select the correct answer using the codes below.
(a) 1 only
(b) 2 and 3 only
(c) 1 and 3 only
(d) 1, 2 and 3
Solution: (d)
Justification: He served in the Bengal Medical Service (from 1794 to 1815).
For a few years he was surgeon to the Governor-General of India, Lord Wellesley.
During his stay in Calcutta (present-day Kolkata), he organised a zoo that became the
Calcutta Alipore Zoo; he was also in charge of the Botanical Gardens for a short period.
On the request of the Government of Bengal, he undertook detailed surveys of the areas
under the jurisdiction of the British East India Company.

Q.555 In 1815 he fell ill and returned to England. He is often called Buchanan-Hamilton.
“Seven Social Sins” is a list that was first coined in
(a) Gandhi’s speech in Banaras Hindu University (BHU)
(b) A political campaign involving the Indian National Congress led by Gokhale
(c) The book “My Experiments with Truth” by M.K. Gandhi
(d) None of the above
Solution: (d)
Justification: It's a common misconception that Gandhi was the one who used it first. He
published the same list in his weekly newspaper Young India on October 22, 1925.
But, it was actually uttered first in a sermon delivered in Westminster Abbey on March 20,
1925 by an Anglican priest named Frederick Lewis Donaldson.
He originally referred to it as the "7 Deadly Social Evils".
Gandhi later gave the same list to his grandson, Arun Gandhi, written on a piece of paper on
their final day together shortly before his assassination.

Q.556 With reference to Surendranath Banerjee, consider the following statements.


1. He was called the Indian Burke.
2. He founded the Indian Association to agitate for political reforms.
3. He had convened the Servants of India Society which merged with the Indian National
Congress.
4. He was the first Indian to become a Member of the British House of Commons.
Select the correct answer using the codes below.
(a) 1 and 2 only
(b) 1, 3 and 4 only
(c) 2 and 4 only
(d) 1, 2, 3 and 4
Solution: (a)
Justification: Surendranath Banerjee was called the Indian Burke. He firmly opposed the
Partition of Bengal.
He founded the Indian Association (1876) to agitate for political reforms. He had convened
the Indian National Conference (1883) which merged with the Indian National Congress in
1886.
Dadabhai Naoroji was known as the Grand Old Man of India. He is regarded as India’s
unofficial Ambassador in England. He was the first Indian to become a Member of the British
House of Commons.
Gopal Krishna Gokhale was regarded as the political guru of Gandhi. In 1905, he founded the
Servants of India Society to train Indians to dedicate their lives to the cause of the country.

Q.557 The title “Socrates of South Asia” is often attributed to


(a) Mahatma Gandhi
(b) Gopalkrishna Gokhale
(c) Pandit Nehru
(d) Periyar E.V. Ramaswamy
Solution: (d)
Learning: Periyar E.V. Ramaswamy was a great social reformer. In 1921, during the anti-
liquor campaign he cut down 1000 coconut trees in his own farm.
In 1924, he took an active part in the Vaikom Satyagraha. The objective of the Satyagraha
was to secure for untouchables the right to use a road near a temple at Vaikom in Kerala.
Subsequently in 1925, he started the “Self-Respect Movement”. The aims of the ‘Self -
Respect Movement’ were to uplift the Dravidians and to expose the Brahminical tyrany and
deceptive methods by which they controlled all spheres of Hindu life.
In 1938 at Tamil Nadu Women’s Conference appreciating the noble service rendered by
E.V.R. he was given the title “Periyar”.
UNESCO organisation praised and adorned him with the title “Socrates of South Asia” in
1970.
He attacked the laws of Manu, which he called the basis of the entire Hindu social fabric of
caste.
He founded the Tamil journals Kudiarasu, Puratchi and Viduthalai to propagate his ideals.

Q.558 G. Subramanya Aiyar founded which of the following publications?


1. The Hindu
2. Swadesamitran
3. Madras Express
Select the correct answer using the codes below.
(a) 1 and 2 only
(b) 2 and 3 only
(c) 3 only
(d) 1 and 3 only
Solution: (a)
Justification: He preached nationalism through the Madras Mahajana Sabha. He also
founded the The Hindu and Swadesamitran.
He was proprietor, editor and managing director of The Hindu from 20 September 1878 to
October 1898.
Subramania Iyer campaigned vehemently for reforms in Hindu society. He supported widow
remarriage and desired to abolish untouchability and child marriages.

Q.559 Consider the following about the philosophy of Shri Aurobindo Ghosh.
1. He gave the concept of the Integral Yoga system.
2. He propounded the doctrine of no-Brahman or no-God.
3. He refuted the theory of evolution.
Select the correct answer using the codes below.
(a) 1 only
(b) 1 and 2 only
(c) 1 and 3 only
(d) 2 and 3 only
Solution: (a)
Justification: Statement 1: Integral Yoga is a system of Yoga that synthesizes six branches
of classical Yoga philosophy and practice: Hatha, Raja, Bhakti, Karma, Jnana, and Japa Yoga.
His main literary works are The Life Divine, which deals with theoretical aspects of Integral
Yoga; Synthesis of Yoga, which deals with practical guidance to Integral Yoga; and Savitri: A
Legend and a Symbol, an epic poem.
Statement 2: Sri Aurobindo argues that divine Brahman manifests as empirical reality
through leela, or divine play.
Instead of positing that the world we experience is an illusion (maya), Sri Aurobindo argues
that world can evolve and become a new world with new species, far above the human
species just as human species have evolved after the animal species.
Statement 3: Sri Aurobindo believed that Darwinism merely describes a phenomenon of the
evolution of matter into life, but does not explain the reason behind it, while he finds life to
be already present in matter, because all of existence is a manifestation of Brahman.
He argues that nature (which he interpreted as divine) has evolved life out of matter and
then mind out of life.
All of existence, he argues, is attempting to manifest to the level of the supermind – that
evolution had a purpose.
He stated that he found the task of understanding the nature of reality arduous and difficult
to justify by immediate tangible results

Q.560 With reference to Dadabhai Naoroji, consider the following statements.


1. He was the first Indian to be elected to Parliament in Britain.
2. He was a founding member of the East India Association and London Indian Society.
3. He was a proponent of the 'drain theory'.
Select the correct answer using the codes below.
(a) 1 only
(b) 2 and 3 only
(c) 3 only
(d) 1, 2 and 3
Solution: (d)
Justification: Of Bombay Parsee origin, he was the first Indian to be elected to Parliament
in Britain. Naoroji travelled to Britain in 1885 as a business partner of Cama and Company.
A member of several businesses, he became Professor of Gujarati at University College,
London (1856-65). He had also been founder-editor of the journal Rast Goftar in Bombay in
1851.
He founded the London Zoroastrian Association in 1861. He was also founding member of
the East India Association and London Indian Society, and became vocal in promoting Indian
rights in regard to the ICS and trade.
Naoroji was an economist and proponent of the 'drain theory', building up a detailed
economic critique of British imperialism in India. He also established links with Irish MPs and
was one of the founders of the Indian National Congress in 1885 in Bombay.
His book Poverty and Un-British Rule in India brought attention to the draining of India's
wealth into Britain.

Q.561 Gandhiji was not in favour of the socialism propounded by Nehru, which emphasized
on large-scale production. Why?
(a) He believed that mass production would again lead to enslavement of India by
foreigners.
(b) He believed that post-independent India did not have the resources to go for large-scale
production
(c) He did not see the basic necessities of life being fulfilled by mass production.
(d) He feared that mass production would lead to greater exploitation of class by class and
urbanization
Solution: (d)
Justification: Gandhian economics places importance to means of achieving the aim of
development and this means must be non-violent, ethical and truthful in all economic
spheres.
In order to achieve this means he advocated trusteeship, decentralization of economic
activities, labour-intensive technology and priority to weaker sections.
The revival of the economy is made possible only when it is free from exploitation, so
according to Gandhi, industrialization on a mass-scale will lead to passive or active
exploitation of the people as the problem of competition and marketing comes in.
Gandhi believes that for an economy to be self-contained, it should manufacture mainly for
its use even if that necessitates the use of modern machines and tools, provided it is not
used as a means of exploitation of others.

Q.562 As per Netaji Subhas Chandra Bose’s view on freedom, freedom implies
1. Equal distribution of wealth
2. Emancipation from political bondage
3. Abolition of caste barriers
4. Destruction of communalism and religious intolerance
Select the correct answer using the codes below.
(a) 2 and 3 only
(b) 1, 2, 3 and 4
(c) 2, 3 and 4 only
(d) 1 and 4 only
Solution: (b)
Justification: As per him, “If we are to bring about a revolution of ideas we have first to hold
up before us an ideal which will galvanise our whole life. That ideal is freedom.
But freedom is a word which has varied connotations and, even in our country, the
conception of freedom has undergone a process of evolution. By freedom I mean all round
freedom, i.e., freedom for the individual as well as for society; freedom for the rich as well
as for the poor; freedom for men as well as for women; freedom for all individuals and for all
classes.
This freedom implies not only emancipation from political bondage but also equal
distribution of wealth, abolition of caste barriers and social iniquities and destruction of
communalism and religious intolerance.
This is an ideal which may appear Utopian to hard-headed men and women, but this ideal
alone can
appease the hunger in the soul.”
(Presidential Address to the Student’s Conference held at Lahore in 1929)

Q.563 The first Indian woman to hold a cabinet post in


1. Pre-independent India: Sarojini Naidu
2. Post-independent India: Amrit Kaur
Which of the above is/are correct?
(a) 1 only
(b) 2 only
(c) Both 1 and 2
(d) None
Solution: (b)
Justification: Statement 1: It was Vijaya Laxmi Pandit.
In 1937 she was elected to the provincial legislature of the United Provinces and was
designated minister of local self-government and public health. She held the latter post until
1939 and again from 1946 to 1947.
In 1946 she was elected to the Constituent Assembly from the United Provinces.
Following India's freedom from British occupation in 1947 she entered the diplomatic
service and became India's ambassador to the Soviet Union from 1947 to 1949
Between 1946 and 1968, she headed the Indian delegation to the United Nations. In 1953,
she became the first woman President of the United Nations General Assembly.
Statement 2: She was the first health minister of India and served for ten years in the
capacity. She was also a social activist and a member of the Constituent Assembly, the body
that framed the constitution of India.

Q.564 Consider the following statements.


1. Mahadev Govind Ranade
2. Found the Indian National Social Conference in the 18th Century to encourage
considerations of social problems on national scale.
3. Held clear views against child marriage and the Purdah system
4. Rejected entry into legislative councils of British India
Select the correct answer using the codes below.
(a) 1 only
(b) 2 and 3 only
(c) 1 and 2 only
(d) 2 only
Solution: (d)
Justification: He was one of the first members of the Prarthana Samaj, founded in 1867.
In 1887 (19th century, not 18th century) he founded the Indian National Social Conference,
whose meetings were concurrent with the annual Congress sessions.
In 1890 he inaugurated the Industrial Association of Western India, having come to the
conclusion that a constructive solution to India's problems lay in a vigorous policy of
industrial and commercial development.
He regularly used the INSC to voice views on social and economic reform at its annual
sessions.
He was a founding member of the Indian National Congress (INC) and owned several
designations as member of the Bombay legislative council, member of the finance
committee at the centre, and the judge of Bombay High Court.

Q.565 Consider the following about Madan Mohan Malaviya.


1. He took a vow not to join any legislative council.
2. He was the founder of Ganga Mahasabha.
3. He opposed the separate electorates for Muslims under the Lucknow Pact of 1916.
4. He was a President of the Indian National Congress (INC).
Select the correct answer using the codes below.
(a) 2, 3 and 4 only
(b) 4 only
(c) 1 and 3 only
(d) 1, 2, 3 and 4
Solution: (a)
Justification: Statement 1: He was a member of the Imperial Legislative Council from 1912
and when in 1919 it was converted to the Central Legislative Assembly he remained its
member till 1926.
Statement 2: He founded it at Haridwar in 1905. He was a member of the Hindu Mahasabha.
Statement 3: He was a moderate leader and opposed communal politics by the British. He
was an important figure in the Non-cooperation movement. However, he was opposed to
the politics of appeasement by Congress and its participation in the Khilafat movement.
Statement 4: Malaviya was the President of the Indian National Congress on two occasions
(1909, 1918). He left Congress in 1934.

Q.566 He founded the Swatantra Party, was one of the first recipients of India's highest
civilian award - Bharat Ratna - and lead the Salt Satyagraha in south India. He was?
(a) C. Rajagopalachari
(b) Khan Abdul Ghaffar Khan
(c) Maulana Abdul Kalam Azad
(d) E.V. Ramaswamy
Solution: (a)
Learning: Rajagopalachari was the last Governor-General of India. He also served as leader
of the Indian National Congress, Premier of the Madras Presidency, Governor of West
Bengal, Minister for Home Affairs of the Indian Union and Chief Minister of Madras state.
He also served as member of the Interim Government of 1946.
He joined the Indian National Congress and participated in the agitations against the Rowlatt
Act, joining the Non-Cooperation movement, the Vaikom Satyagraha, and the Civil
Disobedience movement. In 1930, Rajagopalachari risked imprisonment when he led the
Vedaranyam Salt Satyagraha in response to the Dandi March.

Q.567 In the Modern History of India, Aruna Asif Ali was associated with which of the following?
1. Quit India Movement
2. Salt Satyagraha
3. National Federation of Indian Women
Select the correct answer using the codes below.
(a) 1 and 2 only
(b) 1 and 3 only
(c) 2 and 3 only
(d) 1, 2 and 3
Solution: (d)
Justification: Statement 1: She is widely remembered for hoisting the Indian National
Congress flag at the Gowalia Tank maidan in Bombay during the Quit India Movement, 1942.
Statement 2: She became an active member of Congress Party after marrying Asaf Ali and
participated in public processions during the Salt Satyagraha.
Statement 3: In 1954, she helped form the National Federation of Indian Women, the
women's wing of CPI but left the party in 1956 following Nikita Khrushchev's disowning of
Stalin.

Q.568 As per the Gandhian notion of non-violence


1. Non-violence does not mean just refraining from causing physical harm, mental harm or
loss of livelihood
2. Helping someone in harming someone else or benefitting from a harmful act of violence
is also considered as violence
3. The practice of non-violence requires an element of conscious compassion
Select the correct answer using the codes below.
(a) 1 only
(b) 1 and 3 only
(c) 2 only
(d) 1, 2 and 3
Solution: (d)
Justification: We usually understand non-violence to mean non-injury. A non-violent act is
thought to be one that does not cause physical injury. Gandhi changed this meaning in two
fundamental ways.
For him non-violence meant not just refraining from causing physical harm, mental harm or
loss of livelihood. It also meant giving up even the thought of harming someone.
For him ‘causing’ did not mean doing the harm oneself. For Gandhi, “I would be guilty of
violence, if I helped someone in harming someone else or if I benefited from a harmful act.
In this sense Gandhi’s notion of violence was close to ‘structural violence’.The second major
change that Gandhi introduced was to give the idea of nonviolence a positive meaning.
Not causing harm was not enough. Ahimsa required an element of conscious compassion.
Gandhi was opposed to passive spiritualism. For him non-violence meant a positive and
active pursuit of well-being and goodness.
Therefore those who practise nonviolence must exercise physical and mental restraint under
the gravest provocation. Nonviolence is an extremely active force that has no room for
cowardice or weakness.
Gandhi in fact went to the extent of stating that if non-violence were inadequate to defend
oneself, then it would be better to resort to violence than take refuge in passivity in the
name of non- violence.

Q.569 Consider the following about the role and contributions of B.R. Ambedkar in modern
India:
1. He inspired the Dalit Buddhist Movement.
2. He was Independent India's first law minister.
3. He served as a Governor of Reserve Bank of India.
Select the correct answer using the codes below.
(a) 1and2only
(b) 2and3only
(c) 3 only
(d) 1 and 3 only
Solution: (a)
Justification: Statement 1: He spent his life advocating political rights and social freedom for Dalits. In
1956 he converted to Buddhism, initiating mass conversions of Dalits.
Statement 3: The Reserve Bank of India (RBI), was based on the ideas that Ambedkar presented to the
Hilton Young Commission. He, however, did not serve as its Governor.
Ambedkar was the first Indian to pursue a doctorate in economics abroad. He argued that
industrialisation and agricultural growth could enhance the Indian economy.

Q.570 Knownas‘AndhraKesari’,hedaredtheBritishtoopenfire uponhimduring thevisitof


Simon Commission to Madras in colonial India and later served as the first Chief Minister of
undivided Andhra Pradesh. Recently covered in news, he is?
(a) Tanguturi Parkasam
(b) E.V.R Naicker
(c) Alluri Sitharama Raju
(d) Narasimha Reddy
Solution: (a)
Learning: The Vice President of India, recently paid tributes to Tanguturi Parkasam on his birth
anniversary. This was published in PIB.
He became the first chief minister of the new Andhra state, created by the partition of Madras State
along linguistic lines.
Prakasam was arrested and jailed for more than three years for participating in the Quit India
movement of 1942.

Q.571 Rabindranath Tagore renounced his knighthood in response to


(a) Indian National Army (INA) agitations
(b) Jallianwalla Bagh incident
(c) Partition of Bengal
(d) Council Entry movement of Swarajists
Solution: (b)
Learning: Rabindranath was conferred Knighthood in 1915, the date of birth of 5th George
in recognition of the name he has established in India and Europe and of his genius as a
poet.
Rabindranath Tagore discarded this Title of Knighthood for the inhuman act of British
Government in Jallianwalabag in 1919. Gopal Krishna Gokhale refused to accept the same
title on political reason.
He was shocked at the extreme Brutality shown by the Britishers in the massacre.

Q.572 Consider the following statements.


Assertion (A): Lala Lajpat Rai was a stauch critic of Swami Dayananda Saraswati.
Reason (R): Lala Lajpat Rai held secular views and did not advocate any religious views or
participate in any such activity.
In the context of the above, which of these is correct?
(a) A is correct, and R is an appropriate explanation of A.
(b) A is correct, but R is not an appropriate explanation of A.
(c) A is incorrect, but R is correct.
(d) Both A and R are incorrect.
Solution: (d)
Justification: Lajpat Rai practised at Hissar and Lahore, where he helped to establish the
nationalistic Dayananda Anglo-Vedic School and became a follower of Dayananda Sarasvati,
the founder of the reformist Hindu society Arya Samaj. So, both A and R are incorrect.
He was popularly known as Punjab Kesari. He was part of the Lal Bal Pal trio.
He was also associated with activities of Punjab National Bank and Lakshmi Insurance
Company in their early stages.
He sustained serious injuries by the police when leading a non-violent protest against the
Simon Commission

You might also like